Far-1 Vol 2 Autumn 2023

You might also like

Download as pdf or txt
Download as pdf or txt
You are on page 1of 364

CAF-1

Financial Accounting
and Reporting I
Vol. II

Practice Manual Examination


Questions & Answers

Book Contains Translations of


QURANIC AYATS & AHADITHS,
Therefore Handle Carefully.
Table of Contents
Chapter Topic Page No.
1 IAS 16 & Impairment IAS 36 Part 1 01
2 IAS 36 Extra Questions 26
3 Reversal of Impairment Supplement 30
4 Supplement IAS 36 33
5 Exchange of Assets 39
6 Capital Maintenance 44
7 Supplement Frame Work 61
8 Extra Practice 63
9 Autumn 2020 80
10 Spring 2021 94
11 Cash Flow 105
12 Supplement IAS 7 132
13 Rectification of Error 141
14 A Supplement Rectification 161
15 Basic Earnings per Share 163
16 Diluted EPS 192
17 IAS 33 EPS Graded Questions 204
18 IAS 33 EPS Graded Answers 211
19 EPS Test 230
20 EPS Test Solution 232
21 Supplement IAS 33 239
22 Income and Expenditure Account 243
23 Accounting for NPO Final 276
24 Supplement NPO 299
25 Autumn 2021 305
26 Spring-2022 320
27 Autumn 2022 339
“If you learn from failure then it becomes like success rather than failure.”

Property Plant and Equipment (IAS-16) and Impairment of Assets


(IAS 36)
Definition of Property, Plant & Equipment [PPE]:
These are those tangible items that:
• Are held for use in business for production or supply of goods or for administrative purposes; and
• Are expected to be used during more than on accounting period.
 The asset are capitalized and then depreciated over their useful life.
 Examples of these assets may be;
• Land & Building
• Plant & Machinery
• Furniture & Fixtures
• Motor Vehicles
• Computer Equipment etc

Depreciation is the systematic allocation of the depreciable amount of an asset over its useful life.

Depreciable amount is the cost of an asset, or other amount substituted for cost, less its residual value.

Residual value of an asset is the current estimate of the amount that an entity would obtain from disposal of the
asset at the end of its useful life.

Useful life is:


(a) The period over which an asset is expected to be available for use by an entity; or
(b) The number of production or similar units expected to be obtained from the asset by an entity.

Carrying amount is the amount at which an asset is recognized after deducting any accumulated depreciation and
accumulated impairment losses. (Net book value (NBV) is a term that is often used instead of carrying amount)

Recognition
The cost of an item of PPE shall be recognised as an asset if, and only if:
(a) it is probable that future economic benefits associated with the item will flow to the entity; and
(b) the cost of the item can be measured reliably.

Spare parts and servicing equipment are usually carried as inventory and recognised in profit or loss as consumed.
However, major spare parts and stand-by equipment qualify as property, plant and equipment when an entity
expects to use them during more than one period.

As a practical expedient, immaterial items are not recognised as PPE even if they meet the definition criteria, for
example, staplers and calculators etc.

Page 1 of 25
1
Initial Measurement of an item of PPE
Initially an item of PPE is measured at cost. Cost components may be different depending upon whether the asset
is
→ Purchased locally from within Pakistan.
→ Imported from an outside country
→ Self-manufactured.

As a general rule;
“All amounts incurred to bring the asset into working condition as intended by management are added to the cost
of an item of PPE.”

Note: Points to remember for Depreciation


➢ Depreciation shall begin when the item of PPE is available for use (Irrespective of whether or not it is actually
being used)
➢ Depreciation shall cease when the asset is derecognized i.e. disposed of /written off.
➢ Depreciation shall not cease when the asset is temporarily idle unless usage methods of depreciation (e.g.
machine hour and production unit methods are used).
 Depreciation is charged as an expense unless asset is used to bring another asset into working condition. In
such a case depreciation of asset is capitalized to the cost of that other asset e.g. an equipment is used to
install a plant and machinery for a period of three months during the year. Three months depreciation
should be capitalized to cost of plant and machinery and remaining 9 months depreciation of equipment
should be expensed out.
 An item of property plant and equipment shall be measured at its cost.

Element of Cost [Para 15 to 20]


Amounts that should be Included In the initial cost of an asset are those that are necessary to bring the asset to a
location and condition that enables it to be used as intended by management. It is calculated as follows;
a) Purchase price less trade discount; plus import duties and non-refundable taxes (i.e. sales and Income)
b) Directly attributable cost
Examples of directly attributable cost
1) Cost of site preparation
2) Carriage inward
3) Installation and assembly cost
4) Professional fees and
5) Cost of testing that the asset Is functioning correctly(net off any proceeds made from the sale of items
produced during testing)
c) Initial estimates of future costs where the entity has the obligation to incur these costs [IAS-37].

Cost of dismantling, removing the asset and site restoration:


A company may be required to dismantle, remove the equipment and ‘clean up’ a location where it has been
working when production ceases.

This is often the case in industries where companies are only granted licenses to operate on condition that they
undertake to perform these activities. Such industries include, oil and gas, mining and nuclear power.

Page 2 of 25
2
Suppose a company has an obligation to perform these activities because it has agreed in a contract with the
relevant government of the country.

As the performance of these activities is agreed in a binding contract therefore a liability to dismantle and remove
the asset is recognized immediately after the installation of equipment (at the start of contract).
However as the liability is to be settled at the end of the contract, therefore liability is measured on the basis of
present value of expected future cash outflows.

Conclusion:
As the entity can’t obtain future economic benefits without agreeing to the clauses of the agreement therefore
second effect of this liability is capitalized to the cost of related assets.
Asset xxx
Liability to dismantle xxx
(At present value of expected future cash outflows)

Example of costs not part of an item of PPE


1) Abnormal wastages
2) Staff training e.g. training to operate newly acquired asset
3) Administrative expenses
4) Advertisement expenses
5) initial operating losses of business while the demand of output builds up
6) Relocation expenses(means cost of moving the asset to another location when it was working properly at a
place)

Q.1 A Limited bought a special bread-making plant on 1 January 2021, details of which follow:
Rupees
Purchase price (including sales tax of 14%) 570,000
Import duties - non-refundable 100,000
Installation costs 30,000
Fuel (incurred when transporting the plant to the factory) 45,000
Administration costs 10,000
Staff party to celebrate the acquisition of the new plant 14,000
Staff training 12,000
Testing to ensure plant fully operational before start of production 10,980
Proceeds from sale of samples and by-products made during testing 13,000
Advertising of the ‘special bread’ to be made by the new plant 50,000
Initial operating loss 35,000
Estimated costs of dismantling/ removal costs at the end of its useful life (future amount 27,000
payable of Rs 70,031 present valued at a discount rate of 10%)

Useful life is 10 years.


The company is registered person in sales tax (Means Company can take refund of sales tax paid).

Required:
Calculate the cost to be capitalized to the plant account.

Page 3 of 25
3
Depreciating the whole asset or the parts thereof:
When an item of PPE has more than one significant parts having different useful life and a significant cost, it is
more appropriate to depreciate each of these parts individually, rather than depreciating the asset as a whole.

Parts of an asset
Each part of an asset that has a cost that is significant in relation to the total cost of the item must be depreciated
separately. This means that the cost of an asset might be split into several different assets and each depreciated
separately.

Illustration: Cost
A company has purchased a new airplane for Rs 5,500 million.

The company has identified the following cost components and useful lives in respect of this airplane.
Rs. Million Useful lives
Engines 2,000 3 years
Airframe 1,500 10 years
Avionics 1,500 20 years
Fittings 500 5 years
Total cost 5,500

 Day to day servicing costs (also known as repair and maintenance cost) are expensed when incurred.

Subsequent expenditure
Expenditure relating to non-current assets, after their initial acquisition, should be capitalised if it:
• improves the asset (for example, by enhancing its performance or extending its useful life); or
• is for a replacement part (provided that the part that it replaces is derecognized).

A basic rule is that improvements are capitalized but repairs are expensed.

Major inspections
A company might only be allowed to operate some assets if those assets are subjected to regular major
inspections for faults.

The cost of such major inspections is recognized in the carrying amount of the asset if such costs benefit the
business for more than one accounting period.

Example: Major overhaul


A shipping company is required to put its ships into dry port every three years for an overhaul, at a cost of Rs.
3,000,000. The ships have a useful life of 20 years. A ship was purchased from a shipbuilder at a cost of Rs. 200
million.

Initial recognition
Ship will be recorded as an asset at a cost of Rs. 200 million.

Page 4 of 25
4
“Don’t let failure discourage you but take failure as guidance to achieve the success.”

End of year 3: When an overhaul is required.


The cost of the new overhaul is capitalised and added to the asset’s cost.

At the end of year 6, the cost of the old overhauling and accumulated depreciation of overhaul which would have
been equal to cost should be removed from the books. The cost of the new overhaul will be capitalized and to be
depreciated over its useful life.
This treatment is just like part replacement.

Impairment of Asset:
Asset is said to be impaired if its carrying amount exceeds recoverable amount.
• Asset can be tangible asset (IAS-16) or an intangible asset (IAS-38). In addition concept of impairment is
applicable to IAS 40 if cost model is used.
• Carrying Amount before impairment loss means:
For Assets at Cost Model For Assets at Revaluation Model
Cost - Revalued Amount -
Less Acc Depreciation (-) Less Acc Depreciation (-)
_ -__ __-__
Recoverable Amount: it is higher of:
• Fair value less cost to sell; and
• Value in use

Scenario 1
As on 30-6-2011, a machine measured at cost model has following carrying amount:
Cost 1,000,000
Less Acc Depreciation (350,000)
650,000

This machine is an asset from which we will obtain future economic benefits. There are two ways of obtaining
benefits from this machine; either:
a) Use the machine over remaining life and obtain production of goods; or
b) Immediately sell it and receive cash
• Suppose we have estimated as on 30-6-2011; that by using the machine over the remaining useful life we
can obtain benefit of Rs 700,000. In addition it is estimated that if the machine is sold on 30-6-2011
(today) we can obtain 800,000.
• What should we do; either use it or sell it. Obviously we should go towards higher figure; i.e 800,000. It
means our machine has a potential of Rs 800,000 benefit.
• The carrying amount is Rs 650,000 as on 30-6-2011; which means we are expecting a gain of Rs 150,000
(which should not be recorded immediately)
• Machine will remain at Rs 650,000 in statement of financial position as on 30-6-2011.

Page 5 of 25
5
Scenario 2
As on 30-6-2014. A machine measured at cost model has the following carrying amount:
Cost 500,000
Less Acc Depreciation (300,000)
200,000
Suppose benefit from use over remaining life is expected to be Rs 150,000. It is further estimated that we can
obtain Rs 70,000 if sold immediately.
• It means we can obtain maximum benefit of Rs 150,000 by using the machine in this case (which is higher
figure).
• However, even if we use the machine, this benefit is less than the carrying amount; which means we are
expecting a loss of Rs 50,000.
• This expected loss is called as impairment loss which should be recognized immediately at the reporting date
i.e 30-6-2014.

Impairment Loss 50,000


Acc impairment Loss 50,000
After impairment loss; the revised carrying amount will be
Cost 500,000
Less Acc Depreciation (300,000)
Less Acc Impairment loss (50,000)
Carrying Amount 150,000
• After impairment loss revised carrying amount is depreciated over remaining useful life.
 Value in use
Present value of future cash flows expected to be derived from an asset over its remaining useful life.

Calculation of value in use:


1) Estimate future cash inflows from continuing use of asset.
2) Estimate future cash outflows from continuing use of asset.
3) Estimate *net disposal proceeds at the end of its useful life (if any).
*Net disposal proceeds means ultimate sale proceeds at the end of remaining useful life less estimated cost of
disposal.
4) Calculate the present value of these figures.
5) Add the calculated present values. Total is called as value in use.
 Fair value less cost to sell (FV less CTS)
It is the price that would be received from selling an asset less estimated cost of disposal. (It is immediate net
sale proceeds from asset at the reporting date).

Page 6 of 25
6
Q.3 A machine has a carrying amount of Rs. 600,000 as on 31-12-2010. It is expected that the machine has a
remaining useful life of five years. The net cash flows generated from the use of that machine are
estimated as follows:
2011 2012 2013 2014 2015
Rs. Rs. Rs. Rs. Rs.
200,000 200,000 100,000 100,000 100,000

Net disposal proceeds at the end of 2015 are expected to be Rs 5,000.

Appropriate discount rate is 10% p.a.


Fair value of the machine is Rs 570,000 and costs to sell the machine is Rs. 22,000 as on 31-12-2010

Required: Calculate the impairment loss, if any, as on 31-12-2010.

Procedure for calculation of impairment loss:


 Assess at the end of each reporting period whether there is any *indication of impairment of an asset.
*Examples of indication of impairments.
a) Physical damage of the asset; or
b) Obsolescence of asset; or
c) Economic conditions of the country are deteriorating in which the company operates.
 If there is any indication then calculate *recoverable amount of the asset.
*Recoverable amount is higher of:
→ Value in use; and
→ FV less CTS
 Compare recoverable amount with carrying amount.
 If carrying amount exceeds recoverable amount then recognize the impairment loss as follows:
For a Non-revalued Asset → recognize in statement of profit or loss immediately.
For a Revalued asset → First adjust against revaluation surplus if any as a result of a previous revaluation.

If however, impairment loss is more than revaluation surplus recognize the difference immediately in statement of
profit or loss.

Impairment loss in case of Revalued Asset:


Plant purchased on 1-1-2007
Cost = 1,000,000
Useful Life = 20 years
Plant is carried at Revaluation Model for subsequent measurement.
Year end 31-12-2007
Carrying Amount 1000,000 – 50,000 = 950,000
Revalued Amount (FV) 1,500,000
Revaluation Surplus 550,000

Page 7 of 25
7
No indication of impairment loss on 31-12-2007
1,500,000 ÷ 19 550,000 ÷ 19
Depreciation (78,948) Surplus transferred to Retained (28,947)
Earnings
Carrying Amount 31-12-2008 1,421,052 Remaining Surplus 521,053

As on 31-12-2008
No indication of impairment loss
No revaluation as no material difference
Depreciation (78,948) (28,947)
Carrying Amount 31-12-2009 1,342,104 Remaining Surplus 492,106

As on 31-12-2009
No revaluation as no material difference
No indication of impairment loss
Depreciation (78,948) (28,947)
Carrying Amount 31-12-2010 1,263,156 Remaining Surplus 463,159

As on 31-12-2010
Suppose there is an indication of Impairment loss i.e Plant is now obsolete; therefore calculate the
recoverable amount.
Value in use 700,000
FV less CTS 600,000
Higher 700,000 which is now called as recoverable amount

Therefore impairment loss = 1,263,156-700,000 = 563,156


Accounting Entry
Revaluation Surplus (OCI) 463,159
impairment Loss (bal) (P.L) 99,997
Acc Impairment Loss 563,156

Final carrying amount as on 31-12-2010


Plant 1,500,000
Less Acc depreciation (78,948 x 3) (236,844)
Less Acc Impairment Loss _(563,156)_
_700,000_

After charging impairment loss, revised carrying amount; i.e 700,000 is depreciated over remaining useful life i.e
16 years.

Page 8 of 25
8
“Always wake up with a goal.”

Q.4 Scientific Pharma Limited (SPL) is a manufacturer of pharmaceutical products. In January 2010, one of its
plants suffered a major break down. It was repaired at a cost of Rs. 1.5 million but the production capacity
was reduced significantly. The plant was ready for production on June 30, 2010.

At that time the company’s engineers advised that the plant could be used at a reduced level for 3 years only. The
plant is expected to have a recoverable amount of Rs 19,227,000 as on 30-6-2010.

Other related information is as under:


1) The plant was imported at price of US$ 800,000. The payment was made at the time of shipment on July 1,
2000 at Rs. 52 per US$. Other charges including installation cost amounted to Rs. 7 million. Installation of the
plant was completed on December 31, 2000 and commercial production commenced from April 1, 2001.
2) The company uses straight line method of deprecation. Depreciation is charged from the month the asset is
available for use up to the month prior to disposal. At the time of purchase, the estimated useful life of the
plant was estimated at 15 years whereas the salvage value was estimated at Rs. 2.0 million.
3) Based on the report of a professional independent valuer, the plant was revalued on July 1, 2005 at Rs. 45
million. There was however, no change in estimated useful life of the plant.
4) The factory remained closed from April 1, to June 30, 2007 due to law and order situation.
5) The salvage value has not changed since it was first estimated at the time of purchase.

Required:
Prepare accounting entries for the year ended June 30, 2010. Give all the necessary calculations. (Ignore taxation)

Note: Points to remember for Depreciation


➢ Depreciation shall begin when the item of PPE is available for use (Irrespective of whether or not it is actually
being used)
➢ Depreciation shall cease when the asset is derecognized i.e disposed off /written off or fully depreciated
whichever is earlier.
➢ Depreciation shall not cease when the asset is temporarily idle unless usage methods of depreciation (e.g.
machine hour and production unit methods are used).

 Depreciation is charged as an expense unless asset is used to bring another asset into working condition. In
such a case depreciation of asset is capitalized to the cost of that other asset e.g an equipment is used to
install a plant and machinery for a period of three months during the year. Three months depreciation should
be capitalized to cost of plant and machinery and remaining 9 months depreciation of equipment should be
expensed out.

Page 9 of 25
9
Concept of Present value:

Present Value

Discounting Annuity

(1+i)-n 1-(1+i)-n

i=interest rate i

n= no of years

A company is required to pay 100,000 at the end of five years and suppose interest rate is 10%.Calculate PV.
=100,000 (1.1)-5
=Rs 62,092

A company is required to pay 100,000 at the end of each year for next five years.
− Interest rate = 10%
− Suppose today’s date is 1-1-2014

31-12-2014 100,000 x (1+0.1)-1 90,909


31-12-2015 100,000 x (1+0.1)-2 82,645
31-12-2016 100,000 x (1+0.1)-3 75,131
31-12-2017 100,000 x (1+0.1)-4 68,301
31-12-2018 100,000 x (1+0.1)-5 62,092
379,079

Important
Annuity 1-(1+i)-n
i

In order to simplify the above calculation of PV, we can use formula of annuity if there are more than one
instalments. However this formula can only be used if amounts are same and interval between the amounts is
constant.
100,000 [1-(1+0.1)-5]
0.1
= Rs 379,079

• Four annual instalments of Rs 71,465 payable at the end of each year. Interest rate is 16%.
= 71,465 [1-(1+0.16)-4]
0.16
= Rs 199,972

Page 10 of 25
10
Every success is based on the deeds so do great deeds for great success.”

• Four annual instalments of Rs 2,000 payable at the beginning of each year. Interest rate is 10%.

1-1-2014 2,000
1-1-2015 2,000

1-1-2016 2,000
1-1-2017 2,000

PV = 2,000 + 2,000 [1-(1.1)-3]


0.1
PV = Rs 6,974

Further discussion of impairment of assets:


IAS 36 applies to accounting for impairment of assets except for the followings:
• Inventories (IAS 2)
• Assets (means trade receivables) arising from contracts as per IFRS 15
• Investment properties that are measured at fair value (IAS 40)

Summary of the approach


Impairment of an asset should be identified and accounted for as follows:
(1) At the end of each reporting period, the entity should assess whether there are any indications that an
asset may be impaired.
(2) If there are such indications, the entity should estimate the asset’s recoverable amount.
(3) When the recoverable amount is less than the carrying value of the asset, the entity should reduce the
asset’s carrying value to its recoverable amount. The amount by which the value of the asset is written
down is an impairment loss.
(4) This impairment loss is recognised as a loss for the period.
(5) However, if the impairment loss relates to an asset that has previously been re-valued upwards, it is first
offset against any remaining revaluation surplus for that asset.
(6) Depreciation charges for the impaired asset in future periods should be adjusted to allocate the asset’s
revised carrying amount, minus any residual value, over its remaining useful life (revised if necessary).

Identifying impairment or possible impairment


An entity must carry out an impairment review when there is evidence or an indication that impairment may have
occurred. At the end of each reporting period, an entity should assess whether there is any indication that
impairment might have occurred. If such an indication exists, the entity must estimate the recoverable amount of
the asset, in order to establish whether impairment has occurred and if so, the amount of the impairment.

Indicators of impairment
The following are given by IAS 36 as possible indicators of impairment.

Page 11 of 25
11
When assessing whether there is an indication of impairment, IAS 36 requires that, at a minimum, the following
sources are considered:

External sources Internal sources


An unexpected decline in the asset’s Evidence that the asset is damaged or
market value. no longer of use to the entity.
Significant changes in technology, There are plans to discontinue or
markets, economic factors or laws restructure the operation for which the
and regulations that have an adverse asset is currently used.
effect on the company.
An increase in interest rates, affecting There is a reduction in the asset’s
the value in use of the asset. expected remaining useful life.
The company’s net assets have a There is evidence that the entity’s
higher carrying value than the expected performance is worse than
company’s market capitalization expected.
(which suggests that the assets are
over-valued in the statement of
financial position).

Internal indicators for impairment are generally refers to items under control of management while external
indicators are outside the control of management.

If there is an indication that an asset is impaired then it is tested for impairment. This involves the calculating the
recoverable amount of the item in question and comparing this to its carrying amount.

Measuring recoverable amount

It has been explained that recoverable amount is the higher of an asset’s:


• fair value less costs of disposal; and
• its value in use.

If either of these amounts is higher than the carrying value of the asset, there has been no impairment.

Measuring fair value less costs of disposal


Fair value is normally current market value of asset. If no active market exists, it may be possible to estimate the
amount that entity could obtain from disposal.

Direct selling costs normally include legal costs, taxes and costs necessary to bring the asset into a condition to be
sold (e.g. transaction taxes like Octori). However, redundancy and similar costs (for example, where a business is
reorganized following the disposal of an asset (means cost to reorganize the business after disposal of asset)) are
not direct selling costs.

Page 12 of 25
12
“You should work in a way that your work should encourage you to do more better.”

for example direct selling costs include:


• legal costs;
• costs of removal of the asset;
• Costs of delivery to customer
• costs incurred in bringing the asset to a saleable condition;
• Transaction taxes.
Example : recoverable amount – fair value less costs to sell
A company has an asset with the following details at 31 December 2013:

Expected selling price 200 000


Costs of delivery to potential customer 20 000
Legal costs involved in sale agreement 10 000
Required:
Calculate the fair value less costs to sell of the asset at 31 December 2013

Solution to example: recoverable amount – fair value less costs to sell

Expected selling price 200 000


Less the costs of disposal (20 000 + 10 000) 30 000
Fair value less costs to sell 170 000

Approach of calculating the recoverable amount:


• First calculate the fair value less cost to sell (it is generally easy)
• If fair value less cost to sell is higher than carrying amount then there is no need to calculate the value in use.
If it is not the case then calculate the value in use and then select the higher figure.
• If fair value less cost to sell is not available then calculate the value in use and take it as a recoverable amount.

Calculating the value in Use:


The following elements should be reflected in the calculation of an asset’s value in use:
• An estimate of the future cash flows the entity expects to derive from the asset
• Expectations about possible variations in the amount or timing of those future cash flows
• The time value of money (represented by the current market risk-free rate of interest)

Estimates of future cash flows must include:



cash inflows from the continuing use of the asset;

cash outflows that will be necessarily incurred to generate the cash inflows from continuing use of the asset;
and

net disposal proceeds at the end of the asset’s useful life.

Estimates of Cash outflows should include only future expenditure that is necessary to maintain the asset at the
standard of performance assessed immediately before the expenditure is made (means repair and maintenance).

Page 13 of 25
13
Cash flows to be excluded
Future cash flows are estimated based on the asset’s current condition. Care must therefore be taken not to
include the expected:
• cash inflows that relate to other assets, (since these will be taken into account when assessing the value in use
of these other assets);
• cash outflows that have already been recognized as liabilities (either as part of the asset or as an expense);
• cash inflows and outflows that relate to future expenditure that will ‘improve or enhance the asset.
• cash inflows and outflows from financing activities (means interest payments) (because the cash flows are
later discounted to present values using a discount rate that takes into account the time-value of money); and
• Cash flows in respect of tax receipts and tax payments (because the discount rate used to discount the cash
flows is a pre-tax discount rate).
• Non cash transactions.
• Cash flows that are expected to arise from a future restructuring to which entity is not yet committed(IAS 37)

Cash outflows that improve or enhance the performance


Until the entity incurs cash outflows that improve or enhance the asset’s performance, estimates of future cash
flows do not include the estimated future cash inflows that are expected to arise from increase in economic
benefits associated with cash outflows. Resultant outflows to improve the asset will be included in carrying
amount of asset when incurred.

The discount rate must be a pre-tax rate that reflects current market assessments of:

the time value of money; and

the risks specific to the asset for which the future cash flow estimates have not been adjusted.

Q.5 Property, plant and equipment as disclosed in the draft financial statements of Apricot Pakistan Limited (APL)
for the year ended 30 June 2018 include a plant having a carrying value of Rs. 610 million. The performance of the
plant has been deteriorating since last year which is affecting APL’s sales.

Following information/estimates relate to the plant for the year ending 30 June 2019:

Rs. in million
Inflows from sale of product under existing condition of the plant 250
Operational cost other than depreciation 25
Depreciation 170
Expenses to be paid in respect of 30 June 2018 accruals 8

Cost of increasing the plant’s capacity 60


Additional inflows (net) expected from the upgrade 40
Interest on loan 30
Maintenance cost 15
Tax payment on profits 18

Cash flows from the plant are expected to decrease by 15% each year from 2020 and onward. The plant’s residual
value after its remaining useful life of 3 years is estimated at Rs. 100 million.

Page 14 of 25
14
“Patience plays major role in our life, never let run out your patience.”

An offer has been received to buy the plant immediately for Rs. 570 million but APL has to incur the following
costs.
Rs. in million
Cost of delivery to the customer 45
Legal cost 10
Costs to re-organize the production process after 50
disposal of plant

Applicable discount rate is 9%.

Required:
Calculate the amount of impairment loss (if any) on plant for the year ended 30 June 2018

Answers:
A.1
Initial cost

Purchase price (excluding refundable sale tax: 570,000 x 100/114) (note 1) 500,000
Import duties - non-refundable 100,000
Installation costs 30,000
Fuel 45,000
Administration costs (note 2) 0
Staff party (note 2) 0
Staff training (note 2) 0
Testing 10,980
Proceeds from sale of samples and by-products (13,000)
Advertising 0
Initial operating losses (note 3) 0
Estimated cost to dismantle and remove the plant at end of useful life (note 4) 27,000
Debit to the asset account 699,980

Note 1: since the company is registered under sale tax act therefore sale tax paid is refundable and should not be
capitalised.

Note 2: these costs are excluded since they are not costs directly and necessarily associated with bringing the asset
to a location and condition enabling it to be used as intended by management.

Note 3: these costs are excluded because they are incurred after the asset was brought to a location and condition
that enabled it to be used as intended by management.

Page 15 of 25
15
Note 4: Since these costs will only be paid for at a future date, the credit entry will be to a liability account: a
certain degree of uncertainty exists regarding the Rs 70,031 of future costs, so the liability is classified as a
provision.

A.3
Carrying Amount = 600,000
Calculation of recoverable amount:
Value in use (W-1) = 555,736
FV less CTS (W-2) = 548,000

Higher of is Rs 555,736. After selection of higher figure, that figure is called as recoverable amount. As carrying
amount is Rs 600,000 therefore difference of Rs 44,264 is impairment loss which should be immediately
recognized as follows

Impairment loss 44,264


Accumulated Impairment loss 44,264

After impairment loss, revised carrying amount i.e 600,000 – 44,264 = 555,736 is depreciated over remaining
useful life.

W-1 Calculation of value in use


2011 200,000 (1.1)-1 = 181,818
2012 200,000 (1.1)-2 = 165,289
2013 100,000 (1.1)-3 = 75,131
2014 100,000 (1.1)-4 = 68,301
2015 105,000 (1.1)-5 = 65,197
555,736

W-2 FV less CTS


Fair value 570,000
Less: Cost to sell (22,000)
548,000

A.5 Scientific Pharma Limited

Page 16 of 25
16
Journal entries for the year ended June 30, 2010
Debit Credit
Rs.000 Rs.000
30.06.2010 Repair and maintenance expenses 1,500
Account payable / Bank 1,500
(Repair cost of major break down of the plant)

30.06.2010 Depreciation expense (45,000-2,000)/10.5 years 4,095


Accumulated depreciation 4,095
(Depreciation expense for the year)
30.06.2010 Revaluation surplus (10,380/10.5) 989
Retained earnings 989
(Incremental depreciation credited to retained earnings)

30.06.2010 Revaluation surplus (OCI) W 5,296


Accumulated impairment loss 5,296
(Impairment of plant due to break down)

Working
Purchase Price (800,000 x 52) 41,600,000
Installation Charges 7,000,000
Total cost 48,600,000
Depreciation till 30-6-2005 (13,980,000)
(48,600 – 2,000) x 4.5 /15
Carrying Amount 34,620,000
Revalued Amount 45,000,000
Revaluation Surplus as on 30-6-2005 10,380,000

Plant was ready for intended use as on 31 Dec, 2000 but production started on 1 April, 2001.

As per IAS-16 depreciation commenced when asset is available for use from 1 Jan, 2001.
Depreciation till 30-6-2010 Transfer of Surplus till 30-6-2010
(45,000,000 – 2,000,000) ÷ 10.5 10,380,000 ÷ 10.5
4,095,238/annum 988,571/annum
4,095,238 x 5 = (20,476,190) 988,571 x 5 = (4,942,855)
(From 1-7-2005 to 30-6-2010) From 1-7-2005 to 30-6-2010

Carrying Amount as on 24,523,810 Remaining Surplus as on 5,437,145


30-6-2010 30-6-2010
(45.000.000 – 20,470,190) (10,380,000 – 4,942,855)
Recoverable Amount 19,227,000

Impairment Loss 5,296,810

Page 17 of 25
17
As surplus is more than the impairment loss; therefore whole amount of impairment loss can be adjusted
against surplus.

Note: If an asset remains idle temporarily, it is still depreciated unless machine hour method or production
units method is used.

A.5 Computation of impairment of plant


Carrying value 610
Less : Recoverable amount
Value in use (W-1) 537
Fair value less cost of sell (W-2) 515

Higher of above 537


Impairment 73

W-1: Value in use 2019 2020 2021 Total


--------------- Rs. in million ---------------
Inflows from sale of product 250
Maintenance cost (15)
Operational cost other than dep. (25)

Cash flows – Undiscounted 210 178.50 151.73


PV of cash flows at 9% 193 150 117 460
PV of residual value (100 (1.09)-3) 77 77
Total value in use 537

W-2: Fair value less cost to sell Rs. in million


Fair value of machine 570
Cost to sell

Cost of delivery to the customer (45)


Legal cost involved in sale agreement (10)
515

Page 18 of 25
18
“Spend your life by serving the people and doing good deeds.”

Self-Test Questions
Q.1 Dominant Fertilizers has two plants. Following information is available for the purpose of impairment
testing:
(i) The remaining useful life of both plants is expected to be 3 years.
(ii) The fair values and written down values of the plants as on 31 December 2012 were as follows:

Incremental
WDV Fair value
Plants selling costs
Rupees in million
P-1 220 210 7
P-2 160 150 4

(iii) Expected cash flows from each plant in next three years are as follows:

P-1 P-2
Rs. in million
Annual inflows 105 55
Annual outflows 11 5
Sale proceeds at the end of year 3 8 3
Disposal costs at the end of year 3 2 1

(iv) Present value factor, based on a discount factor of 10%, for year 1, year 2 and year 3 are 0.909, 0.826 and
0.751 respectively.

Required: Compute impairment (if any) on each plant.

Q.2 On 1 January 2013 Delta acquired a specialized machine for its production department. The available
information is as follows:
Rupees
List price of machine 9,200,000
Freight charges 263,000
Electrical installation cost 245,000
Staff training for use of machine 351,000
Pre-production testing 193,000
Purchase of a three-year maintenance contract 528,000
Estimated residual value 175,000

Trade discount on list price 5%

Estimated life (in machine hours) 12,000

Page 19 of 25
19
Machine hours used during the years ended 31 December 2013, 2014 and 2015 were 2000, 3200 and 1400
respectively.

On 1 January 2015 Delta decided to upgrade the machine by adding new components at a cost of Rs. 1,753,000.
This upgrade led to a reduction in the production time per unit of goods being manufactured by the machine. The
upgrade also increased the estimated remaining life of the machine at 1 January 2015 to 8,000 machine hours and
its estimated residual value to Rs. 350,000.

Required:
For the years ended 31 December 2013, 2014 and 2015, compute the relevant amounts to be included (under each
head) in the statement of profit or loss and statement of financial position.
(15)

Q.3 On 1 January 2013 Delta Limited acquired a specialized machine for its production department. The available
information is as follows:
Rupees
List price of machine 9,200,000
Freight charges 263,000
Electrical installation cost 245,000
Staff training for use of machine 351,000
Pre-production testing 193,000
Purchase of a three-year maintenance contract 528,000
Estimated residual value 175,000

Trade discount on list price 5%

Estimated life 10 years

Delta Limited is using straight line method.

On 1 January 2015 Delta decided to upgrade the machine by adding new components at a cost of Rs. 1,753,000.
This upgrade led to a reduction in the production time per unit of goods being manufactured by the machine. The
upgrade increased its estimated residual value to Rs. 350,000. The company has now decided to use sum of years
digit method.

Required:
Prepare the following ledgers for the year ended 31.12.2013,2014 and 2015:
a) Machinery Account (7)
b) Accumulated depreciation account (7)

Page 20 of 25
20
“In difficult situations person needs help but not just advice.”

Q.4 ABA LIMITED


Aba Limited conducts its activities from two properties, a head office in the city centre and a property in the
countryside where staff training is conducted. Both properties were acquired on 1 April 2013 and had estimated
lives of 25 years with no residual value. The company has a policy of carrying its land and buildings at current
values. However, until recently property prices had not changed for some years. On 1 October 2015 the properties
were revalued by a firm of surveyors. Details of this and the original costs are:
Land Buildings

Rs. Rs.
Head office – cost 1 April 2013 500,000 1,200,000
– revalued 1 October 2015 700,000 1,350,000
Training premises – cost 1 April 2013 300,000 900,000
(property) – revalued 1 October 2015 350,000 600,000

The fall in the value of the training premises is due mainly to damage done by the use of heavy equipment during
training. The surveyors have also reported that the expected life of the training property in its current use will only
be a further 10 years from the date of valuation. The estimated life of the head office remained unaltered.

Note: Aba Limited treats its land and its buildings as separate assets. Depreciation is based on the straight-line
method from the date of purchase or subsequent revaluation.

Required: Prepare extracts of the financial statements of Aba Limited in respect of the above properties for the
year to 31 March 2016.

Solutions to self test questions:

A.1 Solution
P-1 P-2
Rs. in million
Fair Value as given 210.00 150.00

Less: Incremental Selling Cost (7.00) (4.00)


Fair value less incremental selling cost 203.00 146.00

Value in use (W-l) 238.19 125.80

Recoverable Value (Being higher of “value in use” and “fair


value less incremental selling cost”) 238.19 146.00

Written down value 220.00 (160.00)


Impairment loss - (14.00)

Page 21 of 25
21
Working

W-l: Value in use


P-1 P-2
Year 1 Year 2 Year 3 Year 1 Year 2 Year 3
Rs. in million
Expected cash inflows each year 105 105 105 55 55 55
Expected outflows per year (ID (ID (11) (5) (5) (5)
Sale proceeds - - 8 - - 3
Disposal costs - - (2) - - (1)
Net cash flows 94 94 100 50 50 52
Present value factor 0.909 0.826 0.751 0.909 0.826 0.751
Present value of net cash flows 85.45 77.64 75.1 45.45 41.3 39.05
Total value in use 238.19 125.8

A.2
Year ended 31-Dec-13 31-Dec-14 31-Dec-15
--------------- Amount in Rs. ---------------

Operating expenses (W-3) (1,720,333) (2,646,934) (1,371,028)


[1,195,028+17
[1,544,333+176,000] [2,470,933+176,000] 6,000]
Administration expenses
(Staff training) (351,000) - -
31-Dec-13 31-Dec-14 31-Dec-15
Statement of financial position: --------------- Amount in Rs. ---------------
Non-Current Assets:
Property, plant and equipment (W-4) 7,896,667 5,425,733 5,983,705
Long term prepayments 176,000
Current Assets:
Short term prepayments 176,000 176,000

Not required for extra information:


Machine Account
1-1-2013 Cash 9,441,000
31-12-2013 c/d 9,441,000
1-1-2014 b/d 9,441,000
31-12-2014 c/d 9,441,000
1-1-2015 b/d 9,441,000
1-1-2015 Cash 1,753,000
31-12-2015 c/d 11,194,000

Page 22 of 25
22
“Don’t break heart of anyone but do your best to mend the broken hearts.”

Accumulated Depreciation Account


b/d -
Depreciation 1,544,333
c/d 1,544,333
b/d 1,544,333
Depreciation 2,470,934
c/d 4,015,267
b/d 4,015,267
Depreciation 1,195,028
c/d 5,210,295

A.3
Machine Account
1-1-2013 Cash 9,441,000
31-12-2013 c/d 9,441,000
1-1-2014 b/d 9,441,000
31-12-2014 c/d 9,441,000
1-1-2015 b/d 9,441,000
1-1-2015 Cash 1,753,000
31-12-2015 c/d 11,194,000

Accumulated Depreciation Account


b/d -
Depreciation 926,600
c/d 926,600 [9,441,000-175,000] / 10
b/d 926,600
Depreciation 926,600
c/d 1,853,200
b/d 1,853,200
Depreciation (working) 1,997,956
c/d 3,851,156

Page 23 of 25
23
Workings:
WDV on 01.01.2015 = [9,441,000 – (926,600 x 2)] = 7,587,800
Year
Year Factor
2015 1 8/36 [7,587,800 + 1,753,000] – 350,000 x 8/36 = 1,997,956
2016 2 7/36 [7,587,800 + 1,753,000] – 350,000 x 7/36 = 1,748,211
2017 3 6/36 [7,587,800 + 1,753,000] – 350,000 x 6/36 = 1,498,467
2018 4 5/36 [7,587,800 + 1,753,000] – 350,000 x 5/36 = 1,248,722
2019 5 4/36 [7,587,800 + 1,753,000] – 350,000 x 4/36 = 998,978
2020 6 3/36 [7,587,800 + 1,753,000] – 350,000 x 3/36 = 771,733
2021 7 2/36 [7,587,800 + 1,753,000] – 350,000 x 2/36 = 499,488
2022 8 1/36 [7,587,800 + 1,753,000] – 350,000 x 1/36 = 249,744
36

A. 4 Aba Limited

Aba Limited statement of profit or loss (extracts) – year to 31 March 2016

Note: workings in brackets are in Rs.000 Rs. Rs.


Depreciation: head office – 6 months to 1 October
2015
(1,200/25 x 6/12) 24,000
– 6 months to 31 March
2016
(1,350/22.5 (W1) x 6/12) 30,000
–––––––
54,000
–––––––
Depreciation: training premises
– 6 months to 1 October
2016
(900/25 x 6/12) 18,000
– 6 months to 31 March
2016
(600/10 x 6/12) 30,000
––––––––
48,000
––––––––
Revaluation loss (W2) 210,000
––––––––
258,000
––––––––

Page 24 of 25
24
Statement of financial position (extracts) as at
31 March 2016 Rs. Rs.
Non-current assets
Land and buildings – head office (700 + 1,350 – 30) 2,020,000
– training premises (350 + 600 –30) 920,000
––––––––
2,940,000
––––––––
Revaluation reserve
Head office land (700 – 500) 200,000
Building (1,350 – 1,080 – 6,000) (W1)) 264,000
Training premises land (350 – 300) 50,000
––––––––
514,000

Working:
Transfer to realised profit (270/22.5 (W1) x 6/12
(6,000)
––––––––
514,000
Workings
(W1) The date of the revaluation is two and a half years after acquisition. This means the remaining life of the
head office would be 22.5 years. The carrying value of the head office building at the date of revaluation is
Rs. 1,080,000 i.e. its cost less two and a half years at Rs. 48,000 per annum (Rs. 1,200,000 – Rs. 120,000).

(W2) Revaluation loss: the carrying value of training premises at date of revaluation is Rs. 810,000 i.e. its cost less
two and a half years at Rs. 36,000 per annum (Rs. 900,000 – Rs. 90,000). It is revalued down to Rs. 600,000
giving a loss of Rs. 210,000. As the land and the buildings are treated as separate assets the gain on the land
cannot be used to offset the loss on the buildings.

Page 25 of 25
25
“The journey of millions of kilo meters begins with one step.”

Extra Practice [Impairment of Assets]


EXAMPLE 01: HUSSAIN ASSOCIATES LTD
The assistant financial controller of the Hussain Associates Ltd has identified the matters below which he believes may
indicate impairment of one or more assets:

Hussain Associates Ltd owns and operates an item of plant that cost Rs. 640,000 and had accumulated
depreciation of Rs. 400,000 at 1 October 2015. It is being depreciated at 12½% on cost.

On 30 Sep 2016 the plant was damaged when a factory vehicle collided into it. Due to the unavailability of replacement
parts, it is not possible to repair the plant, but it still operates, albeit at a reduced capacity. It is also expected that as a
result of the damage the remaining life of the plant from the date of the damage will be only two years.

Based on its reduced capacity, the estimated present value of the plant in use is Rs. 150,000. The plant has a current
disposal value of Rs. 20,000 (which will be nil in two years’ time), but Hussain Associates Ltd has been offered a trade-
in value of Rs. 180,000 against a replacement machine which has a cost of Rs. 1 million (there would be no disposal
costs for the replaced plant). Hussain Associates Ltd is reluctant to replace the plant as it is worried about the long-
term demand for the product produced by the plant. The trade-in value is only available if the plant is replaced.

Required; discuss the above situation with respect to extracts from the statement of financial position and
statement of profit or loss of Hussain Associates Ltd for the year ended 30 September 2016.

Answer:
The plant had a carrying amount of Rs. 240,000 (640,000-400,000) on 1 October 2015.

The depreciation on the plant from 1 October 2015 to 30 Sept 2016 would be Rs. 80,000 (640,000 x 12.5% giving a
carrying amount of Rs. 160,000 at the date of impairment. An impairment test requires the plant’s carrying amount
to be compared with its recoverable amount. The recoverable amount of the plant is the higher of its value in use of Rs.
150,000 or its fair value less costs to sell.

If Hussain Associates Ltd trades in the plant it would receive Rs. 180,000 by way of a part exchange, but this is conditional
on buying new plant which Hussain Associates Ltd. is reluctant to do. A more realistic amount of the fair value of the
plant is its current disposal value of only Rs. 20,000.

Thus the recoverable amount would be its value in use of Rs. 150,000 giving an impairment loss of Rs. 10,000 (Rs.
160,000 – Rs. 150,000). Thus extracts from the financial statements for the year ended 30 September 2016 would
be:

Page 1 of 4
26
Statement of financial position
Non-current assets Rs.
Plant 150,000
Statement of profit or loss
Plant depreciation 80,000
Plant impairment loss 10,000

EXAMPLE 02:
Sky-Line Limited (SL) operates a 4 Star Hotel facility in Muree. The hotel was constructed at a cost of Rs.300 million, 5
years back and it is depreciated on a straight-line basis (total useful life of 15 years and residual value of 20%). There
are indications that the property is not performing as expected due to;
(a) opening of a competing hotel nearby,
(b) a significant drop in number of tourists to the area because of terrorism.

There is a 40% probability that the hotel will generate net cash flows of Rs.40 million per annum and 60% probability
that the cash flows would only be Rs.20 million per annum.

The property’s net operating income is Rs.30 million which is at the rate of 15% of its market value. 5% of the
proceeds from sale would be expended in closing the deal.

Appropriate discount rate is 10%

Required: Calculate the impairment loss:

Answer
• Calculate the carrying value of property that is cost less accumulated deprecation
• Measurement of recoverable amount that is higher of
➢ fair value less cost to sell
➢ value in use (future cash flows discounted by discount rate)
• Compare the carrying value with the recoverable amount to arrive at the impairment loss The detailed
calculation is as under:
Carrying value of asset = Rs.300 million – [5 x (300 – 60(300 x 20%)) / 15 years]
= Rs.220 million

Recoverable amount is the higher of fair value less cost to sell and value is use
Fair value less cost to sell
= Rs.30 m / 15% =200 million
= Rs.200 million – (200x5%)=190million
Value in use = Present value of net cash flows discounted at 10% for 10 years [remaining useful life]
= (Rs.40 m x 0.4 + Rs.20 m x 0.6) x [1- (1.10)-10]/ 10%] + 60 (1.1)-10
= Rs.195 m
Recoverable amount = Rs.195 million
Impairment loss = Rs.25 million [220-195]

Page 2 of 4
27
“Confidence is the main secret of success.”

EXAMPLE 3: PREMIER LIMITED (PL)


Question: Premier Limited (PL) owns a plant which has a carrying amount of Rs.248 million as at 1 April 2019. It is
being depreciated at 12½% per annum on a reducing balance basis.

The plant is used to manufacture a specific product which has been suffering a decline in salesdue to obsolescence.
PL has estimated that the plant will be retired from use on 31 March 2023.

The estimated net cash flows from the use of the plant and their present values are:

Net cash Present


flows values
Rs. In millions
Year to 31 March 2021 120 109.2
Year to 31 March 2022 80 66.4

Year to 31 March 2023 52 39

252 214.6

On 1 April 2020, PL had an alternative offer from the competitor to purchase the plant forRs.200 million.

Required: Calculate the impairment loss as on 31.03.2020.

Answer:
At 31 March 2020
Recoverable amount is the higher of value in use [PV of future net cash flows (Rs.214.6 million) and fair value less
costs of disposal (Rs.200 million)].
Carrying amount = Rs.217 million [248 m – (248 m x 12·5%)] Impairment loss = Carrying amount – Recoverable
amount
= Rs.217 million – Rs.214.6 million
= Rs.2.4 million

EXAMPLE 04: NAVEED LIMITED


Question: Naveed Limited has an item of plant which has a carrying value of Rs.1, 800,000 as atthe end of the year
December 2020. It has undergone an impairment review and the following estimates were produced:

Fair value of plant = Rs.1, 400,000Costs to sell 2% of selling price

Page 3 of 4
28
Revenue and associated costs per annum for remaining useful life:
(Assume all cash flows occur at the end of the year).

Revenue Costs
2021 Rs.960,000 Rs.240,000
2022 Rs.880,000 Rs.220,000
2023 Rs.700,000 Rs.290,000

The plant has an estimated residual value of Rs.50, 000.

A discount rate of 10% is applicable to investments equivalent in risk to this plant.

Required:
Calculate the impairment loss if the appropriate discount rate is 10%.

Answer:

Cash flows
2021 2022 2023
--------------- Amount in Rs. --------------
Revenue 960,000 880,000 700,000

Costs (240,000) (220,000) (290,000)


Net Cash Inflow 720,000 660,000 410,000
Discount factor 0.909 0.826 0.751
Present Value 654,545 545,457 308,037

Residual Value (50,000 x 0.751) = 37,566


Value in use = 654,545 + 545,457 + 308,037 + 37,566 = Rs.1, 545,605
Fair value less cost to sell = Rs.1, 400,000 – 2% of Rs.1.4 million = Rs.1, 372,000Recoverable amount = Rs. 1,545,605
Carrying value = Rs.1, 800,000
Impairment loss = Rs.254, 395

Page 4 of 4
29
Impairment Supplement
Reversal of an impairment loss:
A company must make an assessment at the end of each reporting period as to whether a previously recognized
impairment should be decreased or may no longer exist. If the loss no longer exists, it is reversed subject to the following
guidance:

While reversal of an impairment loss, the carrying amount of an asset must not be increased above the lower of:
❑ its recoverable amount ; and
❑ the carrying amount that would have been determined (net of depreciation) had no impairment loss been
recognised for the asset in prior periods.

A reversal should be:


❑ recognised immediately in profit or loss(if asset is measured at cost model); unless:
Example 1.
❑ the original impairment was charged to the revaluation surplus, in which case the reversal should be credited
to the revaluation surplus.
❑ If the original impairment loss was charged to Revaluation Surplus and Profit or Loss Account, the impairment
loss charged to profit or loss shall be reversed first and remaining amount shall go towards revaluation
surplus.

Depreciation charges for future periods should be adjusted to allocate the asset’s revised carrying amount, minus
any residual value, over its remaining useful life.

Example 1: Reversal of impairment loss under Cost Model


1st January 2021
A plant was purchased at a cost of Rs. 100,000 and is being depreciated over 10 years on a straight-line
basis. Depreciation is Rs. 10,000 per annum.

31st December 2022


The asset has a carrying amount of Rs. 80,000 (Rs. 100,000 – (2 years x Rs. 10,000).
There are indications that the asset is impaired and its recoverable amount is estimated at Rs.64,000. The
resultant impairment loss of Rs. 16,000 is recognised.

From 2023 onwards the carrying amount of the asset after the recognition of the impairment loss (Rs. 64,000)
will be depreciated over the remaining useful life of 8 years resulting in an annual depreciation charge of Rs.
8,000 (64000 ÷ 8).

31st December 2024


The carrying amount of the asset is Rs. 48,000 (Rs. 64,000 – (2 years x Rs. 8,000).
There are indications that the impairment loss might have decreased and the company estimates the
recoverable amount of the asset to be Rs. 70,000.

Page 1 of 3
30
However, the carrying amount of the asset cannot be increased above the lower of:
(i) Carrying Amount today had there been no impairment i.e. Rs.60,000 (Rs. 100,000 – (4 years x Rs.
10,000); and
(ii) Recoverable Amount today i.e. Rs.70,000

Hence, the reversal will be made to such extent that asset will be increased to Rs.60,000 Therefore, a
reversal of Rs. 12,000 (Rs.60,000 – Rs.48,000) is recognized.

From 2025 onwards the carrying amount of the asset after the recognition of the reversal of the
impairment loss (I.e. Rs.60,000) will be depreciated over the remaining useful life of 6 years resulting in an
annual depreciation charge of Rs. 10,000 [60,000 ÷ 6].

Example 2: [Reversal of impairment loss under revaluation model]


1st January 2021
A plant was purchased at a cost of Rs. 100,000 and is being depreciated over 10 years on a straight-line
basis. Depreciation will be Rs. 10,000 per annum.

31st December 2021


Assuming the asset is revalued to Rs. 121,500 from its carrying amount of Rs.90,000 (resulting into Rs.31,500
as Revaluation Surplus). Remaining useful life is 9 years and depreciation will be Rs.13,500 per annum
[121,500 ÷ 9] from 2022 onwards.

31st December 2023


The asset has a carrying amount of Rs.94,500 (Rs. 121,500 – (2 years x Rs. 13,500).

Remaining amount of Revaluation Surplus is now Rs.24,500 (Rs.31,500 – (2 years x Rs.3,500(31500/9)


transferred to Retained Earnings).

There are indications that the asset is impaired and its recoverable amount is estimated at Rs.65,100. The
resultant impairment loss of Rs.29,400 (94,500 – 65,100) is recognized out of which Rs.24,500 is charged
to available balance of Revaluation Surplus and remaining Rs.4,900 to Profit or Loss Account.
Rev.Surplus 24,500
Imp.loss 4,900
Acc. imp loss 29,400

The carrying amount of the asset after the recognition of the impairment loss is Rs.65,100 which will be
depreciated over the remaining useful life of 7 years resulting in depreciation charge of Rs.9,300per annum
[65,100 ÷ 7] form 2024 onwards.

31st December 2025


The carrying amount of the asset is Rs.46,500 (Rs.65,100 – (2 years x Rs.9,300). There are indications that
the impairment loss might have reversed and the company estimates the recoverable amount of the asset
to be Rs.55,000.

Page 2 of 3
31
The carrying amount of the asset cannot be increased above the lower of:
(i) Carrying Amount today had there been no impairment i.e. 67,500(121,500 – (4 x Rs.13,500); and
(ii) Recoverable Amount today i.e. Rs.55,000

Hence, the reversal will be made to such extent that asset will be increased to Rs.55,000 .Therefore, a reversal
of Rs.8,500 (Rs.55,000 – Rs.46,500) is recognized. This reversal of impairment will first adjust the previous
impairment loss recognized in Profit or Loss Account and remaining amount (if any) will go to Revaluation
Surplus as follows
:
(as in case of reversal of revaluation loss.)
Reversal of impairment loss 8500

Reversal of loss 4,900 Cr. Rev.surplus 5,000


Extra dep 1,400 Dr.
[10,000 – 9,300] x 2
Reversal of impairment loss (P.L) 3,500 Cr.

Accumulated impairment loss 8,500


Reversal of impairment loss (P.L) 3,500
Rev.surplus 5,000

Page 3 of 3
32
IAS 36
Scope [IAS 36: 2 & 5]
IAS 36 is not applicable to various assets because IFRSs applicable to those assets have mechanism to ensure that the
relevant asset is not overstated, for example:
• Inventories are measured at lower of cost and net realisable value (IAS 2)
• Investment property measured at fair value.

However, IAS 36 is applicable to certain assets including, but not limited to:
• Property, plant and equipment (IAS 16)
• Investment property measured using cost model (IAS 40)
• Intangible assets (IAS 38)

IAS 36 is also applied to assets that are carried at revalued amount and it provides following guidance in this
regard:
If the disposalcosts The recoverable amount of the revalued asset is necessarily close to, or greater than, its
are negligible revalued amount.
In this case, after the revaluation requirements have been applied, it is unlikely that the
revalued asset is impaired and recoverable amount need not be estimated.

If the disposalcosts The fair value less costs of disposal of the revalued asset is necessarily less than its fair value.
are not negligible Therefore, the revalued asset will be impaired if its value in use is less than its revalued
amount.
In this case, after the revaluation requirements have been applied, an entity applies IAS 36 to
determine whether the asset may be impaired.

⯈ Example:
Consider the following three independent scenarios related to revalued assets:

Scenario (a) (b) (c)


Carrying amount i.e., fair value Rs. 500,000 Rs. 500,000 Rs. 500,000
Costs of disposal Negligible Negligible Negligible
Value in use Rs. 600,000 Rs. 480,000 Rs. 100,000

Required: STICKY NOTES


Discuss whether IAS 36 needs to be applied and calculate the impairment loss (if any).

⯈ Answer:

There is no need to apply IAS 36 and even if we do, no impairment loss would arise.

Scenario (a) (b) (c)


Carrying amount i.e., fair value Rs. 500,000 Rs. 500,000 Rs. 500,000
Fair value less costs of disposal Rs. 500,000 Rs. 500,000 Rs. 500,000

Page 1 of 6

33
Value in use Rs. 600,000 Rs. 480,000 Rs. 100,000
Recoverable amount (higher) Rs. 600,000 Rs. 500,000 Rs. 500,000
Impairment loss (RA > CA) Nil Nil Nil

Example:
Consider the following three independent scenarios related to revalued assets:

Scenario (a) (b) (c)


Carrying amount i.e., fair value Rs. 500,000 Rs. 500,000 Rs. 500,000
Costs of disposal Rs. 50,000 Rs. 50,000 Rs. 50,000
Value in use Rs. 600,000 Rs. 480,000 Rs. 100,000

Required:
Discuss whether IAS 36 needs to be applied and calculate the impairment loss (if any).

Answer:
IAS 36 needs to be applied, however, there may or may not arise an impairment loss.
Scenario (a) (b) (c)
Carrying amount i.e., fair value Rs. 500,000 Rs. 500,000 Rs. 500,000
Fair value less costs of disposal Rs. 450,000 Rs. 450,000 Rs. 450,000
Value in use Rs. 600,000 Rs. 480,000 Rs. 100,000
Recoverable amount (higher) Rs. 600,000 Rs. 480,000 Rs. 450,000
Impairment loss (RA > CA) Nil Rs. 20,000 Rs. 50,000

Page 2 of 6
34
SPOTLIGHT

Reversal of impairment loss [IAS 36: 110 & 117 to 120]

⯈ Example :
An asset was purchased for Rs. 100,000 on 1 January 2021 with 10 years useful life and nil residual value.
Depreciation is charged on straight line basis.

On 31 December 2021, Depreciation of Rs. 10,000 was recorded. On this date, the recoverable amount of asset was
determined to be Rs. 72,000 and therefore impairment loss of Rs. 18,000 was recognised.

On 31 December 2022, the depreciation of Rs. 8,000 [i.e., Rs.72,000 / 9 years] was recorded and asset now has
carrying amount of Rs. 64,000.

On this date, the recoverable amount has been estimated at Rs. 95,000.

Required: Journal entry to record the reversal if entity uses cost model.
⯈ Answer:
Journal entry

SPOTLIGHT
Debit Rs. Credit Rs.
Date Particulars 000 000

31.12.2022 Accumulated impairment losses 16


Reversal of loss (PL) 16

Note: The recoverable amount of asset exceeds its carrying amount by Rs. 31,000 reversal of impairment loss would
be restricted to only Rs. 16,000, because, had the asset been never impaired its carrying amount would have been
Rs. 80,000 (i.e., 100,000 – (10,000 x 2 years)), therefore, asset’s carrying amount can be increased from Rs. 64,000
to Rs. 80,000 (i.e., by Rs. 16,000). Alternatively, Rs. 16,000 may be calculated as Rs. 18,000 loss previously recognised
less depreciation decrease of Rs. 2,000 (i.e., 10,000 – 8,000).

STICKY NOTES
⯈ Example :
An asset was purchased for Rs. 100,000 on 1 January 2021 with 10 years useful life and nil residual value.
Depreciation is charged on straight line basis.

On 31 December 2021, Depreciation of Rs. 10,000 was recorded. On this date, the recoverable amount of asset was
determined to be Rs. 72,000 and therefore impairment loss of Rs. 18,000 was recognised.

On 31 December 2022, the depreciation of Rs. 8,000 [i.e., Rs.72,000 / 9 years] was recorded. On this date, the
recoverable amount has been estimated at Rs. 95,000.

Required:
Journal entry to record the reversal if entity uses revaluation model.

Page 3 of 6

35
Answer:
Journal entry

Date Particulars Debit Rs. Credit Rs.


000 000
31.12.2022 Accumulated impairment losses 31
Reversal of loss (PL) 16
Gain on revaluation (OCI) 15

Note: The recoverable amount of asset exceeds its carrying amount by Rs. 31,000 reversal of impairment loss in
profit or loss would be restricted to only Rs. 16,000, because, had the asset been never impaired its carrying amount
AT A GLANCE

would have been Rs. 80,000 (i.e., 100,000 – (10,000 x 2 years)), therefore, asset’s carrying amount can be increased
from Rs. 64,000 to Rs. 80,000 (i.e., by Rs. 16,000). Alternatively, Rs. 16,000 may be calculated as Rs. 18,000 loss
previously recognised less depreciation decrease of Rs. 2,000 (i.e., 10,000 – 8,000).

The additional increase of Rs. 15,000 (from Rs. 80,000 to Rs. 95,000) shall be recognised in OCI as gain on revaluation
because entity uses revaluation model.

Example:
On 1 January 2011 Entity Q purchased for Rs.240,000 a machine with an estimated useful lifeof 20 years and an
estimated zero residual value. Depreciation is on a straight-line basis.

The asset had been re-valued on 1 January 2013 to Rs.250,000, but with no change in useful life at that date.

On 1 January 2014 an impairment review showed the machine’s recoverable amount to be Rs.100,000 and its
remaining useful life to be 10 years.

Required: Calculate:
a) The carrying amount of the machine on 31 December 2012 and hence therevaluation surplus arising on 1
January 2013.
b) The carrying amount of the machine on 31 December 2013 (immediately before theimpairment).
c) The impairment loss recognised in the year to 31 December 2014.
d) The depreciation charge in the year to 31 December 2014.
Answer:

Part (a)

Carrying amount of the machine & revaluation Rs.


Cost 240,000
Accumulated depreciation (2 × (240,000 ÷ 20 years)) (24,000)
Carrying amount 216,000
Revaluation gain / surplus (balancing) 34,000
Revalued amount on 1 January 2013 250,000

Page 4 of 6
36
Part (b)
When the asset is revalued on 1 January 2013, depreciation is charged on the revalued amount over its remaining
expected useful life.

Carrying amount of the machine 31 December 2013 Rs.


Cost/revalued amount 250,000
Accumulated depreciation (Rs.250,000 ÷ 18 years) (13,889)
Carrying amount 236,111

Part (c)
On 1 January 2014 the impairment review shows an impairment loss of Rs. 136,111 (Rs.236,111– Rs.100,000).
SPOTLIGHT

An impairment loss of Rs.32,111 (Rs.34,000 – Rs.1,889) will be taken to other comprehensiveincome (reducing the
revaluation surplus for the asset to zero).

The remaining impairment loss of Rs.104,000 (Rs.136,111 – Rs.34,000) is recognised in thestatement of profit
or loss for 2014.

Part (d)
2014 depreciation charge is Rs.10,000 (Rs.100,000 ÷ 10 years).
SPOTLIGHT

Example:
AT
STICKY

On 1 January 2021, Jazib Limited (JL) purchased an asset at a cost of Rs. 100,000 and it is being depreciated over 10
A GLANCE

years on straight line basis. JL uses cost model for measurement of property, plant and equipment.
NOTES

On 31 December 2022, there are indications of impairment and recoverable amount has been estimated at Rs.
64,000.

On 31 December 2024, there are indications of reversal of impairment and recoverable amount has been estimated
at Rs. 70,000.
STICKY NOTES

Required:
Journal entries (by elimination of accumulated depreciation) for:
a) impairment loss on 31 December 2022
b) reversal of impairment loss on 31 December 2024

Page 5 of 6
37
Answer:
Journal entries

Date Particulars DebitRs. CreditRs.

31 Dec 2022 Impairment loss (PL) W2 16,000


Accumulated impairment loss (PPE) 16,000
31 Dec 2024 Accumulated impairment loss (PPE) W4 12,000
Reversal of impairment loss (PL) 12,000

W2: Impairment loss Rs. 64,000 – (100,000 – 20,000) = Rs. 16,000

W3: Depreciation Rs. 64,000 / 8 years x 2 years = Rs. 16,000

W4: Reversal of impairment loss: (Rs. 100,000 – 100,000/10 x 4 years) – (Rs. 64,000 – 16,000) = Rs. 12,000.

Page 6 of 6
38
“Don’t stress nor consider your life as burden, no matter what.”

Exchange of Non-Monetary Asset


1. If fair value (i.e. its cost) of only new asset received is available or is more reliably measureable then measure
the new asset at its fair value.
2. If fair value of new asset received as well as fair value of old asset given up both are reliable measureable or
only fair value of old asset given up is reliably measureable, then measure the new asset received at fair value
of asset given up plus cash paid minus cash received (if any)

Example 1:
Book value of old machine 8,000 (comprising of cost of 12,000 and accumulated depreciation of 4,000).
Fair value of machine given up = 6,000
List price of new machine = 16,000

Trade in allowance for old machine = 9,000 (benefit of old asset)

(It may not be the same as fair value of old machine because supplier may be charging an inflated price for his new
machine).

Cash paid (16,000 – 9,000) = 7,000

Required:
Prepare accounting entry to record gain / loss on exchange

Answer:
New machine (6,000 + 7,000) 13,000
Acc. Depreciation 4,000
Loss on exchange (P&L) 2,000
Old machine 12,000
Cash 7,000

3. If fair value of both the asset received or given up are not reliably measureable or are not available then
measure the new asset at carrying amount of asset given up plus cash paid minus cash received.

Page 1 of 5
39
Comprehensive Example:
Following information pertains to three exchange transactions relating to fixed assets:
--------- Rs. in million ---------
1 2 3
Cash received/(paid) 1.1 (2.1) 2
Assets given-up:
Original cost 10.3 12.4 14.5
Book value 6.4 7.3 3.4
Estimated fair value 8.5 6.6 4.6
Assets received:
Estimated fair value 7.1 9.0 4.1

Additional information:
i. In case of transaction (i), fair values of both assets are reliably measurable.
ii. In case of transaction (ii), fair value of the asset received is clearly more evident.
iii. In case of transaction (iii), fair value of neither asset is reliably measurable.

Required:
Prepare accounting entries in respect of the above exchange transactions. (06)

Answer
(i)
New asset (8.5-1.1) 7.4
Cash 1.1
Old asset 6.4
Gain (P.L) 2.1

(ii)
New asset 9.0
Loss (P.L) 0.4
Cash 2.1
Old asset 7.3

(iii)
New asset (3.4+2) 5.4
Cash 3.4
Old asset 2.0

Note: if exchange lacks commercial substance, i.e. when exactly similar assets are exchanged then measure the
new asset at the carrying amount of the given up plus cash paid or minus cash received.

Page 2 of 5
40
Practice question
Q.1 Following information pertains to plant and machinery of Alpha Enterprises (AE):
(i) As at 1 January 2018, balances of cost and accumulated depreciation amounted to Rs. 12,700,000
and Rs. 6,240,000 respectively.
(ii) On 1 April 2018, an old machine having fair value of Rs. 340,000 was exchanged for a new machine.
The balance of the purchase price was paid through a cheque of Rs. 680,000. The list price of the
new machine was Rs.1,130,000. The old machine had been acquired for Rs. 870,000 on 1 September
2015.
(iii) On 1 February 2018, a plant having a list price of Rs. 10,000,000 was acquired. A trade discount of
5% was allowed on the list price. The plant was ready for use on

1 August 2018 after incurring the following costs:

Rs. in '000
Freight charges 660
Consultant fees 540
Installation and testing 600
Administration and other general overheads 160
Staff training 120
Opening ceremony 100
2,180
(iv) On 31 October 2018, another machine was sold for Rs. 334,000. It was acquired on 1 January 2015
and had a net book value of Rs. 512,000 on 1 January 2018. A cost of Rs. 25,000 was incurred on its
disposal.

(v) AE depreciates plant and machinery at 20% per annum using the reducing balance method

Required:
Prepare following ledger accounts pertaining to the plant and machinery for the year ended 31 December
2018:

(a) Cost (06)

(b) Accumulated depreciation (06)

(c) Assets disposal (04)

Page 3 of 5
41
A.1 Alpha Enterprises

Plant and machinery - Cost


Date Description Rs. in '000 Date Description Rs. in '000
1-Jan-2018 Balance 12,700 1-Apr-2018 Assets disposal 870
1-Apr-2018 Bank 680 31-Oct-2018 Assets disposal (W-2) 1,000
Disposal 340
1,020
1-Aug-2018 Cash (W-4) 11,300 31-Dec-2018 Balance 23,150
25,020 25,020

Accumulated depreciation - Plant and machinery


1-Apr-2018 Assets disposal (W-1) 376 1-Jan-2018 Balance 6,240
31-Oct-2018 Assets disposal (W-2) 573 31-Dec- Depreciation exp. (W-3) 2,292
2018
31-Dec-2018 Balance 7,583
8,532 8,532

Assets disposal - Plant and machinery


1-Apr-2018 Old machine 870 1-Apr-2018 Acc. depreciation (W-1) 376
31-Oct-2018 Cost 1,000 1-Apr-2018 New machine 340
31-Oct-2018 Acc. depreciation (W-2) 573
31-Oct-2018 Cash (334-25) 309
31-Dec-2018 Loss on disposal (P.L) 272
1,870 1,870

W-1: Accumulated depreciation – Machine exchange Rs. in '000


Depreciation for 2015 870×0.2×4÷12 58
Depreciation for 2016 (870–58)×0.2 162
Depreciation for 2017 (870–58–162)×0.2 130
Accumulated depreciation upto 31-12-2017 (WDV = 870-350=520) 350
Depreciation for 2018 (870–350)×0.2×3÷12 26
376

Page 4 of 5
42
W-2:Accumulated depreciation – Machine sold
1-1-2015 100
x 20% (20)
31-12-2015 80
x 20% (16)
31-12-2016 64
x 20% (12.8)
31-12-2017 51.2
512,000/51.2 x 100
Cost = 1,000,000
Acc.dep on 31-12-2017=1,000,000-512,000=488,000
+ 512,000 x 20% x 10/12=85,333
Acc.dep on 10-10-2018=573,333

W-3 : Depreciation for the year


Opening WDV (12,700 – 6,240) 6,460
Less:Opening WDV of Disposal ( 512 + 520) (1,032)
5,428
x 20% 1086
+depreciation on Disposals (26+85) 111
+depreciation on Additions
1,020 x 20% x 9/12 153
11,300 x 20% x 5/12 942
Total depreciation 2,292

W-4 : Cost of the plant :


Rs.in ‘’000’’
Purchase price of the plant 10,000 × 0.95 9,500
Other directly related cost (660 + 540 + 1,800
600)
11,300

Page 5 of 5
43
“Stay close to anything that reminds you of ALLAH”

Concepts of Capital Maintenance


It simply means at least Capital/Equity at the beginning of the period should be equal to capital/Equity at the
end of the period. E.g. Rs. 10 millions.

It means companies should only pay dividends out of profits to maintain the capital.

Equity or capital = Total assets – Total liabilities

Discussion of capital account

There are two main concepts of capital maintenance:


❑ A financial concept of capital;
❑ A physical concept of capital.

Example:
• A Company commenced business on 01.01.2010 with 100,000 capital.
• It purchased 20,000 units of inventory at 5 per unit on 01.01.2010 at a cost of Rs. 100,000.
• It sold all inventory during the year for Rs. 120,000 in cash
• Therefore it resulted into profit of Rs. 20,000.
• If this profit is taken out in the form of drawings/dividends then it means closing equity is equal to the
equity at the start of the business, i.e. 100,000 (which means financial capital is maintained)
• However let’s assume that inventory cost 5.4 per unit at the end of the period due to inflation. Entity can
only buy 18,519 units (100,000/5.4) with 100,000, which means company has not maintained its operating
capacity and therefore physical capital is not maintained.
• Physical capital would have been maintained had the closing equity be equal to 20,000 units x 5.4
=108,000 (which would require profit of Rs.8,000 being retained in equity and therefore dividend should
not exceed 12,000)

Comparing the two concepts


In practice, almost all entities use financial capital maintenance, but both concepts can provide useful
information.

Financial capital maintenance is likely to be the most relevant to investors (external users) as they are
interested in maximising the return on their investment and therefore its purchasing power.

Physical capital maintenance is likely to be most relevant to management and employees (internal users) as
they are interested in assessing an entity’s ability to maintain its operating capacity. This is particularly true for
manufacturing businesses, where management may need information about the ability of the business to
continue to produce the same or a greater volume of goods.

Page 1 of 17
44
1. Financial concept of capital:
In this concept we simply compare whether the closing equity is equal to or greater than the opening equity in
the statement of financial position.

It also has two sub divisions:


a. Financial capital maintenance (money terms). Under this concept, an entity makes a profit when its closing
equity (net assets) exceeds its opening equity (net assets) (without any inflation adjustment). It is most
commonly used in financial accounting.
b. Financial capital maintenance (real terms). Under this concept, an entity makes a profit when it’s closing
equity exceeds opening equity re measured by using general inflation rate (after adjusting for capital raised
or distributed). This is achieved by a simple double entry.

Adjustment to maintain opening equity

Credit
Statement of profit or loss/profit X

Inflation reserve X

This entry simply means transferring profits to equity to maintain the same purchasing power as the
entity had at the start of the period.

2. Physical concept of capital in this concept it is seen that whether at the end of the period we have resources to
acquire same quantity of similar physical assets (e.g. inventories, property, plant and equipment, intangible assets
etc) which we had at the beginning of the period.

This concept simply compares the closing equity with opening equity re measured using specific rates of inflation
that apply to the individual components of the net assets of the business. Again, this is achieved by the same
simple double entry
Adjustment to maintain opening equity

Credit

Statement of profit or loss/profit X

Inflation reserve X
.

This entry simply means transferring profits to equity to maintain the same operating capacity as the entity
had at the start of the period.

Page 2 of 17
45
“No matter how hurt you are, You will always find comfort with ALLAH”

The following example should help you to understand this.

Example: Capital maintenance concepts


X Limited commenced business on 1 January with a single item of inventory which cost Rs.10, 000.
During the year it sold the item for Rs.14, 000 (cash).
During the year general inflation was 5% but the inflation specific to the item was 10%.

Profit is calculated under each concept in the following ways.


Inventory 10,000
Capital 10,000
Cash 14,000
Sale 14,000
Cost of sale 10,000
Inventory 10,000

Financial capital maintenance (Money terms)


Income statement:
Sale 14,000
COS (10,000)
G.P 4000

Statement of financial position


N.C.A -
C.A
Cash 14,000
Equity
Capital 10,000
Profit 4,000 14,000
14,000

Financial capital maintenance (real terms)


Income statement
Sale 14,000
COS (10,000)
4,000
Inflation reserve (10,000 x 5%)[opening capital x general rate of inflation] (500)
Remaining profit 3,500

Page 3 of 17
46
Statement of financial position
N.C.A -
C.A 14,000
14,000
Equity
Opening capital 10,000
Inflation reserve 500
10,500
Profit 3,500
14,000
Liability -
14,000
10,000 x 5% = 500
Profit 500
Inflation reserve 500
[Like general or specific reserve]

Capital maintenance physical


Income statement
Sale 14,000
COS (10,000)
Gross profit 4,000
Inflation reserve(10,000 x 10%)[opening capital x specific rate of inflation] (1,000)
Remaining profit 3,000

Statement of financial position


N.C.A -
C.A
Cash 14,000
14,000
Equity
Opening capital 10,000
Inflation reserve 1,000
11,000
Net Profit 3,000
14,000

10,000 x 10% = 1,000


Profit 1,000
Inflation reserve 1,000

Page 4 of 17
47
“When Love is for the sake of ALLAH, It never dies.”

Commentary on example:
Money financial capital maintenance
Under historical cost accounting, the profit is Rs.4, 000. If the business paid this out as a dividend it would have
Rs.10, 000 left.

Rs.10, 000 is the opening equity expressed as a number of units of currency. This means that the company
would have maintained its equity expressed as a number of units of currency. However, inflation in the period
has caused the purchasing power of the currency to decline. This means that Rs.10, 000 no longer has the same
purchasing power that it had a year ago. The company has not maintained its capital in real terms.

Real financial capital maintenance


To maintain its equity in real terms the company would have to ensure that it had the same purchasing power
at the year-end as it had at the start. Inflation was 5% so the company would need Rs.10, 500 at the year-end in
order to have the same purchasing power as it had at the start of the year. The company can achieve this by
transferring Rs.500 from profit and loss into an inflation reserve. Profit would then be reported as Rs.3, 500.

If the business paid out Rs.3, 500 as a dividend it would have Rs.10, 500 left. This is not enough t o buy the
same asset that it had at the start of the year. The asset has been subject to specific inflation of 10% therefore
the company would need Rs.11, 000 at the year-end in order to buy the same asset.

This means that the company would not have the same capacity to operate as it had a year ago.

Physical capital maintenance


To maintain its equity in physical terms the company would have to ensure that it had t h e same ability to
operate at the year-end as it had at the start. In other words it would need to have Rs.11, 000. The company
can achieve this by transferring Rs.1, 000 from profit and loss into an inflation reserve. Profit would then be
reported as Rs.3, 000. Therefore if operating capacity is to be maintained, then company should only pay a
dividend of 3,000.

Summary of capital maintenance:


financial capital financial capital Physical capital
maintenance( Money maintenance (Real terms) maintenance
terms)
Inflation adjustment No Yes; based on general Yes; based on specific
of opening capital inflation rate inflation rate
Also referred to as Historical cost accounting Constant purchasing Current cost accounting
power accounting
Most relevant to Investors (as they are Investors (as they are Management and
interested in maximizing interested in maximizing employees(as they are
return on investment) return on investment) interested in assessing
entity’s ability to maintain
operating capacity)
MCQs number 9 to 18

Page 5 of 17
48
Concepetual framework for financial reporting
Purpose of conceptual framework [conceptual framework SP 1.1 and SP 1.5]
The Conceptual Framework for Financial Reporting (Conceptual Framework) describes the objective of, and
the concepts for, general purpose financial reporting.

The purpose of the Conceptual Framework is to assist:


• IASB to develop IFRSs that are based on consistent concepts;
• preparers to develop consistent accounting policies when no Standard applies to a particular transaction
or other event, or when a Standard allows a choice of accounting policy; and
• all parties to understand and interpret the Standards.

The Conceptual Framework contributes to the stated mission of the IFRS Foundation and IASB i.e. to develop
Standards that bring transparency, accountability and efficiency to financial markets around the world.
The Conceptual Framework provides the foundation for Standards (IASs and IFRSs) that:
• contribute to transparency by enhancing the international comparability and quality of financial
information, enabling investors and other market participants to make informed economic decisions.
• strengthen accountability by reducing the information gap between the providers of capital and
management. IFRSs and Conceptual Framework are also source of information for regulators.
• contribute to economic efficiency i.e. the use of a single, trusted accounting language derived from
Standards based on the Conceptual Framework lowers the cost of capital and reduces international
reporting costs.

Contents of conceptual framework [conceptual framework SP 1.2 and SP 1.4]


The Conceptual framework is divided into eight chapters, namely:
Chapter 1 — The Objective Of General Purpose Financial Reporting
Chapter 2 — Qualitative Characteristics Of Useful Financial Information
Chapter 3 — Financial Statements And The Reporting Entity
Chapter 4 — The Elements Of Financial Statements
Chapter 5 — Recognition And Derecognition
Chapter 6 — Measurement
Chapter 7 — Presentation And Disclosure
Chapter 8 — Concepts Of Capital And Capital Maintenance

Status of conceptual framework [conceptual framework 4.1 and 4.2]


The Conceptual Framework is not a Standard and nothing in the Conceptual Framework overrides any
Standard (IASs or IFRSs) or any requirement in a Standard.

Elements of financial statements


The elements of financial statements defined in the conceptual framework are:
• assets, liabilities and equity, which relate to a reporting entity’s financial position; and
• income and expenses, which relate to a reporting entity’s financial performance

Page 6 of 17
49
These elements are linked to the economic resources, claims and changes in economic resources and claims
and are explained as under:
Item discussed Elements Definition or description
Economic resource Asset A present economic resource controlled by the entity as a result of
past events.
An economic resource is a right that has the potential to produce
economic benefits.
Claim Liability A present obligation of the entity to transfer an economic resource
as a result of past events.
Equity The residual interest in the assets of the entity after deducting all its
liabilities.
Changes in economic Income Increases in assets, or decreases in liabilities, that result in increase
resources and claims, in equity, other than those relating to contributions from holders of
reflecting financial equity claims.
performance Expenses Decreases in assets, or increases in liabilities, that result in decreases
in equity, other than those relating to distributions to holder of
equity claims.
Other changes in - Contributions from holders of equity claims, and distributions to
economic resources them.
and claims - Exchanges of assets or liabilities that do not result in increases or
decreases in equity.

Information to be useful for decision making [conceptual framework 2.4]


If financial information is to be useful, it must be relevant and faithfully represent what it purports to
represent. The usefulness of financial information is enhanced if it is comparable, verifiable, timely and
understandable.

It means information must have certain characteristics in order for it to be useful for decision making. The IASB
Conceptual Framework describes:
• fundamental qualitative characteristics; and
• enhancing qualitative characteristics

Fundamental qualitative characteristics [conceptual framework 2.5]


The fundamental qualitative characteristics are
• relevance; and
• faithful representation

Relevance [conceptual framework 2.6 and 2.7]


Relevant financial information is capable of making a difference in the decisions made by users. Information
may be capable of making a difference in a decision even if some users choose not to take advantage of it or
are already aware of it from other sources.

Page 7 of 17
50
The relevance of information is affected by its materiality. Information is material if omitting it or misstating it
could influence decisions that users make on the basis of financial information about a specific reporting
entity.

Faithful representation (True and fair view) [conceptual framework 2.12 and 2.13]
Financial reports represent economic phenomena in words and numbers. To be useful, financial information
must not only represent relevant phenomena, but it must also faithfully represent the substance of the
phenomena that it purports to represent. Although, in many circumstances, the substance of an economic
phenomenon and its legal form are the same, an accountant should be careful to identify when this might not
be the case.

To be a perfectly faithful representation, a depiction would have three characteristics. It would be:
• complete,
• neutral and
• free from error.

Of course, perfection is seldom, if ever, achievable. The objective is to maximise those qualities to the extent
possible.

Enhancing qualitative characteristics [conceptual framework 2.23]


The qualitative characteristics that enhance the usefulness of information that is relevant and a faithful
representation are:
• comparability;
• verifiability
• timeliness; and
• understandability

Comparability [conceptual framework 2.25,2.26 and 2.29]


Comparability enables users to identify and understand similarities in, and differences among, items.
Information about a reporting entity is more useful if it can be compared with similar information about other
entities and with similar information about the same entity for another period or another date.

Consistency is related to comparability but is not the same. Consistency refers to the use of the same methods
for the same items, either from period to period within a reporting entity or in a single period across entities.
Consistency helps to achieve the goal of comparability.

Verifiability [conceptual framework 2.30 and 2.31]


This quality helps to assure users that information faithfully represents the economic phenomena it purports
to represent. Verifiability means that different knowledgeable and independent observers could reach
consensus that a particular depiction is a faithful representation. Quantified information need not be a single
point estimate to be verifiable. A range of possible amounts and the related probabilities can also be verified.
Verification can be direct or indirect.
• Direct verification means verification through direct observation, e.g. by counting cash or inventory.
• Indirect verification means checking the inputs to a model, formula or other technique and recalculating
the outputs using the same methodology. For example, the carrying amount of inventory might be verified

Page 8 of 17
51
by checking the inputs (e.g. costs) and recalculating the closing inventory using the same assumption (e.g.
FIFO).

Timeliness [conceptual framework 2.33]


This means having information available to decision-makers in time to be capable of influencing their
decisions. Generally, the older the information is the less useful it is.

Understandability [conceptual framework 2.34 to 2.36]


Information is made understandable by classifying, characterising and presenting it in a clear and concise
manner. Some phenomena are inherently complex and cannot be made easy to understand, however,
excluding the relevant information is not justified in such circumstances.

Financial reports are prepared for users who have a reasonable knowledge of business and economic activities
and who review and analyse the information diligently.

Cost constraint on useful information [conceptual framework 2.]


Cost is a pervasive constraint on the information that can be provided by financial reporting. Reporting
financial information imposes costs, and it is important that those costs are justified by the benefits of
reporting that information.39 to 2.41

The benefits obtained from financial information should exceed the cost of obtaining and providing it.
Information should not be provided if the cost is not worth the benefit.

Recognition criteria [conceptual framework 5.6,5.7 and 5.11]


Only items that meet the definition of an asset, a liability or equity are recognised in the statement of financial
position. Similarly, only items that meet the definition of income or expenses are recognised in the
statement(s) of financial performance.

However, not all items that meet the definition of one of those elements are recognised. Not recognising an
item that meets the definition of one of the elements makes the statement of financial position and the
statement(s) of financial performance less complete and can exclude useful information from financial
statements. On the other hand, in some circumstances, recognising some items that meet the definition of one
of the elements would not provide useful information.

An asset or liability is recognised only if recognition of that asset or liability and of any resulting income,
expenses or changes in equity provides users of financial statements with information that is useful, i.e. with:
• relevant information about the asset or liability and about any resulting income, expenses or changes in
equity; and
• a faithful representation of the asset or liability and of any resulting income, expenses or changes in
equity.

Items that fail to meet the criteria for recognition should not be included in the financial statements. However,
some of these items may have to be disclosed as additional details in a note to the financial statements.

Page 9 of 17
52
Recognition links elements of financial statements [conceptual framework 5.3]

Measurements of elements of financial statements [conceptual framework 6.1 and 6.2]


Elements recognised in financial statements are quantified in monetary terms. This requires the selection of a
measurement basis. A measurement basis is an identified feature, for example, historical cost, fair value or
fulfilment value, of an item being measured.

Applying a measurement basis to an asset or liability creates a measure for that asset or liability and for
related income and expenses. Consideration of the qualitative characteristics of useful financial information
and of the cost constraint is likely to result in the selection of different measurement bases for different assets,
liabilities, income and expenses.

Historical cost [conceptual framework 6.4 to 6.8]


Historical cost measure provides monetary information about assets, liabilities and related income and
expenses, using information derived, from the price of the transaction or other event gave rise to asset or
liability.

The historical cost of an asset, when it is acquired or created is the value of the cost incurred in acquiring or
creating the asset, comprising the consideration paid to acquire or create the asset. The historical cost of a
liability when it is incurred is the value of consideration received to incur liability.

The historical cost of an asset is updated overtime to depict, if applicable:


o The consumption of part of the economic resource that constitute the asset depreciation or amortization;
o Payments received that extinguish part or all of the asset (any receiveable);
o The effect of events that cause part or all the historical cost of the asset to be no longer
recoverable(impairment);

Page 10 of 17
53
o Accrual of interest to reflect any financing component of the asset.

The historical cost of a liability is updated over time to depict, if applicable:


• fulfilment of part or all of the liability (payment);
• the effect of events that increase the value of the obligation (estimate change); and
• accrual of interest to reflect any financing component of the liability.

Current value [conceptual framework 6.10 to 6.11]


Current value measures provide monetary information about assets, liabilities and related income and
expenses, using information updated to reflect conditions at the measurement date. Because of the
updating, current values of assets and liabilities reflect changes, since the previous measurement date, in
estimate of cash flows and other factors reflected in those current values. Unlike historical cost, the current
value of an asset or liability is not derived, from the price of the transaction or other event that gave rise to the
asset or liability.

Current value measurement bases include:


o Fair value
o Value in use for assets and fulfillment value for liabilities
o Current cost

Fair value [conceptual framework 6.14 to 6.16]


Fair value is the price that would be received to sell an asset, or paid to transfer a liability, in an orderly
transaction between market participants at the measurement date.

Fair value reflects the perspective of market participants—participants in a market to which the entity has
access. The asset or liability is measured using the same assumptions that market participants would use when
pricing the asset or liability if those market participants act in their economic best interest.

In some cases, fair value can be determined directly by observing prices in an active market. In other cases, it
is determined indirectly using measurement techniques, for example, cash-flows-based measurement
techniques reflecting all the following factors:
o estimates of future cash flows
o possible variations.
o The time value of money
o The price for bearing the uncertainty inherent in the cash (a risk premium or risk discount).
o Other factors; e.g liquidity, if market participants would take those factors into accounts in the
circumstances.

Value in use (for assets) and fulfillment value (for liabilities) [conceptual framework 6.17 to 6.20]
Value in use is the present value of the cash flows that an entity expects to derive from the use of an asset and
from its ultimate disposal. Fulfillment value is the present value of the cash that an entity expects to be obliged
to transfer as it fulfills a liability.

Page 11 of 17
54
Because value in use and fulfillment value are based on future cash flows they don’t include transaction cost
incurred on acquiring an asset or taking on a liability. However, value in use and fulfillment value include the
present value of any transaction cost that entity expects to incur on the ultimate disposal of the asset or on
fulfilling the liability.

Value in use and fulfillment value reflect entity specific assumptions rather than assumptions by market
participants.

Value in use and fulfillment value cannot be observed directly and or determined using cash flow based
measurement techniques. Value in use and fulfillment value reflect the same factors described for fair value
earlier, but from an entity specific perspective rather than from a market participant perspective.

Current cost [conceptual framework 6.21 to 6.22]


The current cost of an asset is the cost of an equivalent asset at the measurement date comprising the
consideration that would be paid at the measurement date plus the transaction cost that would be incurred
at that date. The current cost of a liability is the consideration that would be received for an equivalent liability
at the measurement date that would be incurred at that date. Current cost, like historical cost is an entity
value; it reflects prices in the market in which the entity would acquire the asset or would incur the liability.
Hence it is different from fair value in use and fulfillment value, which are exit value. However, unlike historical
cost, current cost reflects conditions at the measurement date.

In some cases, current cost cannot be determined directly by observing prices and in an active market and
must be determined directly by other means. For example, if prices are available only for new asset the
current cost of a used asset might need to be estimated by adjusting the current price of a new asset to reflect
the current age and condition of the asset held by the entity (depreciated replacement cost).

Example: Measurement Bases


Question: Adeel Limited (AL) owns a machine which it purchased two years ago for Rs. 200,000. The
accumulated depreciation on the machine to date is Rs. 80,000 based on 5 years life using straight line
method.

The machine could be sold in the market for Rs. 100,000 but there would be dismantling costs of Rs. 10,000.
The cash flows from the existing machine are estimated to be Rs. 50,000 for the next two years followed by Rs.
40,000 in the last year. Relevant discount rate is 10%.

To replace the machine with a new version would cost Rs. 220,000.

Required:
Measure the machine using different measurement bases for AL using the above information.

Page 12 of 17
55
Answer:
Historical cost Rs.
Cost 200,000
Less: Accumulated depreciation (80,000)
120,000

Fair value
The fair value is market value (exit price) of Rs. 100,000 without deducting cost to sell of Rs. 10,000.

Value in use Rs.


Year 1 Rs. 50,000 x 1.1-1 45,455
Year 2 Rs. 50,000 x 1.1-2 41,322
Year 3 Rs. 40,000 x 1.1-3 30,053
116,830

Rs.
Current cost
Cost of new asset 220,000
Less: Accumulated depreciation* Rs. 220,000 / 5 x 2 years (88,000)
132,000
*The replacement cost is of new machine and needs to be adjusted for two years usage.

Practice questions:
Example 01:
❑ A manufacturing unit valuing Rs. 5 million, owned and controlled by the Company
❑ A fleet of trucks valuing Rs 100 million, controlled by another company
❑ A highly skilled workforce, getting an annual compensation of Rs. 12.5 million

Required: Which of the above assets will be recognized in the financial statements of a company in
accordance with the recognition criteria?

Answer:
The assets will be recognized in the financial statements of company in accordance with conceptual
framework as under:
❑ A manufacturing unit valuing Rs. 5 million, owned and controlled by the Company
❑ The fleet of truck will not be recognized because it is not controlled by the entity.
❑ Similarly, workforce will not be recognized by the entity because there is no certainty about the probability
of future economic benefits from workers as they can quit the entity at any time.

Example 02: Carrie starts in business on 1 January Year 1. Carrie’s sole shareholder contributed capital of Rs.
1,000. Carrie purchased one item of inventory for Rs. 1,000 and sold that inventory for cash of Rs. 1,400. At the
end of Year 1 the replacement cost of the same item of inventory is Rs. 1,100. General inflation during the year
was 7%.

Page 13 of 17
56
Required
Calculate the profit for the year and set out a summary statement of financial position as of 31 December Year
1 under the following capital maintenance concepts.
(a) Physical capital maintenance
(b) Financial capital maintenance
i. Historical cost accounting
ii. Constant purchasing power accounting

Answer:
(a) PhysicalCapital
Maintenance
(b) Financial CapitalMaintenance

(i) Historicalcost ii)Constant


accounting Purchasing
power
accounting

Profit for the year Rs. Rs. Rs.


Sales 1,400 1,400 1,400
Cost of sales (1,000) (1,000) (1,000)
Inflation adjustment
- Specific (1,100 – 1,000) (100) - -
- General (1,000 x 7%) - - (70)
------------- ------------- -------------
Profit 300 400 330
------------- ------------- -------------
Balance sheet as at 31 December Year 1
Cash at bank 1,400 1,400 1,400
------------- ------------- -------------
Share capital (1,000 + 100) (1,000 + 70) 1,100* 1,000 1,070*
Reserves 300 400 330
------------- ------------- -------------
1,400 1,400 1,400
------------- ------------- -------------

Tutorial note
Share capital at the year end is restated under the physical capital maintenance concept for an increase in
specific price changes and under Constant Purchasing Power accounting for general price changes. This is the
other side of the entry to the inflation adjustments in the statement of profit or loss.

Page 14 of 17
57
Example 03:
Question: Read the following statements:
A. In case of conflict between requirements of conceptual framework and IFRS, the requirements of
conceptual framework shall prevail.
B. Conceptual framework is not an International financial reporting standard (IFRS)
C. HR related cost is recognized as an asset in the financial statements since economic benefit is probable
from human resource
D. Internally generated goodwill is recognized as asset and measured at fair value in the financialstatements
E. When economic benefits arise over several accounting periods, and the association with income can only
be decided in broad terms, expenses should be recognized in profit and loss of each accounting period on
the basis of systematic and rational allocation procedure
F. When an item of expenditure is not expected to provide any future economic benefit, it is recognized as
an asset in the financial statements
G. In fair value method, assets are measured at the amount that would be paid to purchase the same or a
similar asset currently

Required:
Analyse the above statements as true or false along with reasons for the selected answer.

Answer:
A. False. In case of conflict between requirements of conceptual framework and IFRS, the requirements of
IFRS shall prevail being an established principle that specific law requirements prevail over general law
requirements.
B. True. Conceptual framework provides foundation for the IFRSs
C. False.HR related cost can never be capitalized as it does not meet the definition criteria of asset
“controlled by the entity”
D. False. Internally generated goodwill can never be recognized as it does not meet one of the basic
recognition criteria i.e. “The item should have a cost or value that can be measured reliably”
E. True, because of matching principle
F. False. For any item to be recognized as an asset, it must be probable that an item shallprovide future
economic benefits to the entity.
G. False. In current cost method assets are measured at the amount that would be paid to purchase the same
or a similar asset currently

Example 04:
Question: ABC received Rs. 160,000 in cash on 20 December 2004 from RM in return for having provided
financial advice during the 2004 financial year.

Required:
(a) Explain, with reference to the relevant definitions, which elements should possibly be recognized in the
2004 financial year.
(b) Briefly identify whether and/ or how your answer would change if the cash received had been received for
financial advice to be provided in the 2005 financial year.

Page 15 of 17
58
Answer:
Part (a)
The cash received meets the definition of an asset i.e. present resource now controlled by the entity and entity
may spend it as it may wish. Services have already been provided, therefore, there is no obligation (no change
in liability). It shall be recognised as an income.

An asset and an income shall be recognised in year 2004.

Part (b)
The cash received meets the definition of an asset i.e. present resource now controlled by the entity and entity
may spend it as it may wish. Services have not been provided and there is present obligation to provide
services, resulting in increase in liability. No income can be recognised as there is no equity increase.
An asset and a liability shall be recognised in year 2004.

Example 05:
Question: Read the following scenarios:
1. An amount paid to landlord totalling Rs.120,000 on 1st January 2012 against the rent for the year ended
31st December 2012. Year end of the entity is 30 June 2012.
2. An expenditure incurred on repairs and maintenance of plant amounting Rs.300,000.
3. There has been legal dispute between the entity and its customer and company expects the outflow of
Rs. 200,000 in order to settle the dispute.
4. Entity purchased goods costing Rs. 20,000 for trading purposes and the same was sold for Rs. 25,000.

Required:
Which of the above, would be recognized as expense &/or asset in the financial statements of a company in
accordance with the criteria given in conceptual framework.

Answer:
1. Increase in asset (advance rent: Future benefits) Rs. 60,000 and decrease in asset (Cash) Rs. 120,000
resulting in net decrease in equity is Rent expense (Rs. 60,000).
2. Decrease in asset (Cash) Rs. 300,000 and no increase in other assets (unless increase in present
resources) resulting in net decrease in equity is Repair expense (Rs. 300,000).
3. Increase in liability (obligation to settle) Rs. 200,000 and no increase in any assets resulting in net
decrease in equity is Expense (Rs. 200,000).
4. When entity purchased inventory, it was a present economic resource and recognised as an asset.
When sold, it becomes expense (cost of sales) due to decrease in assets resulting in decrease in equity.

Example 06:
Question: Read the following scenarios
1. Advance received from customer amounting Rs. 50,000 against the goods to be delivered after 6 months
2. Services provided to ABC and Co. on credit amounting Rs.30,000.
3. Account Receivables already written off in previous years amounting Rs. 30,000 were received during the
year.

Page 16 of 17
59
Required:
Which of the above, would be recognized as income &/or liability in the financial statements of a company in
accordance with the criteria given in conceptual framework.

Answer:
1. Increase in asset (Cash) Rs. 50,000 and also an increase in liability (obligation to deliver) Rs. 50,000 and
there is no income as no increase in equity.
2. Increase in asset (Right to receive) Rs. 30,000 and no increase in liability (services already provided) and
resulting net increase in equity Rs. 30,000 recognised as income.
3. Increase in asset (cash) Rs. 30,000 but no decrease in asset (RA was already written off) resulting in Net
increase in equity is Income.

Page 17 of 17
60
Derecognition [Conceptual Framework: 5.26]
Derecognition is the removal of all or part of a recognized asset or liability from an entity’s statement of financial
position. Derecognition normally occurs when that item no longer meets the definition of an asset or of a liability:
a) for an asset, derecognition normally occurs when the entity loses control of all or part of the recognized asset;
and
b) for a liability, derecognition normally occurs when the entity no longer has a present obligation for all or part
of the recognized liability.

For example, when an item of property, plant and equipment is sold or destroyed, it will be derecognized.
Similarly, a liability will be derecognised when it is paid or settled otherwise.

Measurement bases for assets and liabilities – summary


Measurement Definitions Characteristics
bases
Historicalcost Asset: The consideration paid to acquire or Derived from past transaction /event.
create the asset plus transaction costs. Reflects conditions existing at the time of
Liability: The consideration received to incur acquisition.
or taken the liability minus transaction costs. Entry value.
Transaction costs at time of disposal are not
relevant.
Fair value Asset: The price that would be received to Derived using information updated to reflect
sell an asset in an orderly transaction conditions at the measurement date.
between market participants at the Reflects market-participant assumptions.Exit value.
measurement date. Transaction costs are not relevant on
Liability: The price that would be paid to acquisition as well as on disposal.
transfer a liability in an orderly transaction
between market participants at the
measurement date.
Value in useor Asset: The present value of the cash flows, or Derived using information updated to reflect
Fulfilmentvalue other economic benefits, that an entity conditions at the measurement date.
expects to derive from the use of anasset and Reflects entity specific assumptions. Exit value.
from its ultimate disposal. Transaction costs on acquisition are not relevant,
Liability: The present value of the cash, or however, present value of transaction costs on
other economic resources, that an entity ultimate disposal/transfer are included in
expects to be obliged to transfer as it fulfils a calculation.
liability.
Current cost Asset: The cost of an equivalent asset that Derived using information updated to reflect
would be paid at the measurement date plus conditions at the measurement date.
the transaction costs. Reflects prices in market in which entity would
Liability: The consideration that would be acquire the asset or incur a liability.
received for an equivalent liability at the Entry value.
measurement date minus the transaction Transaction costs at time of disposal are not
costs. relevant.

Page 1 of 2
61
Measurement of equity [Conceptual Framework: 6.87 & 6.88]
The total carrying amount of equity (total equity) is not measured directly. It equals the total of the carrying amounts
of all recognized assets less the total of the carrying amounts of all recognized liabilities (i.e., equity = total assets –
total liabilities).

Because the general-purpose financial statements are not designed to show an entity’s value, the total carrying
amount of equity will not generally equal:
• the aggregate market value of equity claims on the entity;
• the amount that could be raised by selling the entity as a whole on a going concern basis; or
• the amount that could be raised by selling all of the entity’s assets and settling all of its liabilities.

Page 2 of 2
62
“There is nothing heavier in the scales than good character”

Additional Practice Questions


1 Question
MK Corporation Limited, an entity listed in Pakistan Stock Exchange is in the business of manufacturing and
sale of yarn products. Company year-end is December. Below is the relevant information given?

Opening balances as at January 01, 2018


Description Rs.

Opening Share Capital (at par value of Rs. 10 per share) 25,000,000

Share Premium 7,500,000

Opening General Reserves 750,000

Opening RE 18,250,000

Revaluation Surplus 1,500,000

Following events have taken place in year 2018 and 2019:


1. On March 31, 2018, Company issued Right shares for Rs. 20 per share. Right shares were issued in the
proportion of 1 right share against 5 ordinary shares held.
2. Board of Directors of the Company approved Interim dividend of Rs. 2.25 per share for the half year
ended June 30, 2018.
3. Annual profit for the year ended December 31, 2018 is Rs. 10,250,000.
4. The Board of Directors of the Company recommended annual dividend of Rs. 4.25 per share on
February 15, 2019, which was duly approved by the Shareholders on March 21, 2019.
5. The Board of Directors approved Bonus Shares of 20% of the outstanding shares on June 30, 2019
which were duly credited in Shareholders account on August 31, 2019.
6. Board of Directors of the Company approved Interim dividend of Rs. 1.25 per share for the third
quarter ended September 30, 2019.
7. Annual profit for the year ended December 31, 2019 is Rs. 12,500,000.
8. The Board of Directors of the Company recommended annual dividend of Rs. 5 per share on February
15, 2020, which was duly approved by the Shareholders on March 21, 2020.
9. The Company has a policy to transfer 5% of the Annual Profit to General Reserves.
10. Company revalued fixed assets on December 31, 2017 resulting in Revaluation Surplus of Rs.
1,500,000. Remaining useful life of the Asset is 10 years and Company has a straight line method for
Depreciation.

Required: Statement of Changes in Equity for the year ended December 31, 2018 and 2019.

Page 1 of 17
63
2 Question
HMK Corporation Limited, an entity listed in Pakistan Stock Exchange is in the business of manufacturing and
sale of Cars. Company year-end is December. Below is the relevant information given?

Opening balances as at January 01, 2018


Description Rs.

Opening Share Capital (at par value of Rs. 10 per share) 100,000,000

Share Premium 50,000,000

Opening General Reserves 5,000,000

Opening RE 55,000,000

Following events have taken place in year 2018 and 2019:


1. On February 28, 2018, the Company issued Initial Public Offering, thereby offering 5 million shares for Rs.
25 each. All shares were subscribed and company received the subscription money by March 31, 2018.
2. The Board of Directors of the Company recommended annual dividend for the year ended December 31,
2017 of Rs. 5 per share on February 15, 2018, whereas Shareholders only approved Rs. 4 per share on March
31, 2018.
3. Board of Directors of the Company approved Interim dividend of Rs. 1 per share for the third quarter ended
September 30, 2018.
4. Annual profit for the year ended December 31, 2018 is Rs. 130,250,000.
5. The Board of Directors of the Company recommended annual dividend of Rs. 4.25 per share on February
15, 2019, whereas Shareholders approved Rs. 6 per share on March 31, 2019.
6. The Board of Directors approved Bonus Shares of 10% of the outstanding shares on June 30, 2019 which
were duly credited in Shareholders account on August 31, 2019.
7. Board of Directors of the Company approved Interim dividend of Rs. 1.5 per share for the third quarter
ended September 30, 2019.
8. Annual profit for the year ended December 31, 2019 is Rs. 175,000,000.
9. The Board of Directors of the Company recommended annual dividend of Rs. 5.5 per share on February 15,
2020, which was duly approved by the Shareholders on March 31, 2020.
10. The Company has a policy to transfer 5% of the Annual Profit to General Reserves.

Required: Statement of Changes in Equity for the year ended December 31, 2018 and 2019.

Page 2 of 17
64
“Forgive others as quickly as you expect Allah (God) to forgive you.”

IAS 16: PROPERTY, PLANT AND EQUIPMENT


1 Question
Abbas Limited (AL) is engaged in the business of manufacturing near the Karachi-Hyderabad Motorway. Its
Property, Plant and Equipment comprises of land and buildings, plant and machinery, and equipment and
fittings.

Details for the period up to 30 June 2018 are as follows:


1. The balances of the Property, Plant and Equipment as at 30 June 2018 are given below:

Assets Gross Carrying Amount Accumulated Depreciation (Rs.


(Rs. Million) Million)

Land 12 N/A

Buildings 125 38

Plant and Machinery 500 300

Equipment 100 36

2.The relevant information pertaining these assets is given below:

Assets Depreciation Method Subsequent Measurement Model

Land N/A Fair Value

Buildings Straight-line Cost

Plant and Machinery Units of Production Cost

Equipment Written down value @ 20% Cost

3. Abbas Limited uses proportionate policy to depreciate its Property, Plant and Equipment.
4. All of the plant and machinery pertains to factory use whereas all the equipment pertains to office use.
However floor areas occupied by factory and office are in the ratio 60:40 respectively.
5. The equipment was purchased on 1 July 2016. No disposals and acquisitions took place in the period up to 30
June 2018.
6. Until 30 June 2018, 12,000 units had been produced by Abbas Limited in its factory. The plant and machinery
does not have any residual value. No additions or disposals of plant and machinery took place till this date.
7. The buildings were acquired on 1 July 2014 with a residual value of Rs. 11 million. No additions and disposals
took place till 30 June 2018.

Page 3 of 17
65
8. The land had actually cost Rs. 15 million on the date of its acquisition.
9. It is assumed that value of land and buildings is spread evenly across the area occupied.

The following information pertains to the year ended 30 June 2019:


1. On 1 July 2018, land was revalued to Rs. 20 million. The value was determined by an independent firm M/s
Ashfaq & Co. Chartered Accountants.
2. This year, 5,000 units were produced in the factory of AL.
3. On January 1, 2019, AL disposed 25% of its area comprising of land and buildings at a price of Rs. 90 million.
The portion of land was sold at its fair value as determined on 1 July 2018. The legal costs of drafting transfer
agreements were Rs. 0.1 million. It is assumed that this disposal will not affect the proportion of areas
occupied by factory and office.
4. Further equipment costing Rs. 60 million was acquired on 1 November 2018.
5. In the meeting of its board of directors, it was decided to open a new factory premises near Lahore-Islamabad
motorway. An expenditure of Rs. 20 million was spent of the construction of the factory on 1 December 2018,
financed by a loan obtained from the bank at the rate of 12% per annum. The construction had not been
completed at the end of the year.
6. Moreover, the directors also made a contract with M/s Uni Power& Co. to purchase plant and machinery
worth Rs. 35 million once the construction of factory building is completed.

Required:
a) Prepare journal entries to record the revaluation of land and disposal of land and buildings.
b) Prepare the disclosure under IAS 16 in relation to Property, Plant and Equipment in the notes to the published
accounts for the year ended 30 June 2019.

2 Question
Games Limited (GL) commenced a business of preparing and burning video game CDs on 1 July 2015.

The following information pertains to the year ended 31 March 2016:


1) GL purchased 30 computers on the date of commencement of business at a cost of Rs. 20,000 each, purely for
the task of burning CDs. The management of GL estimates that since the computers are subject to obsolescence,
more of its benefit can derived in its early life therefore use reducing balance method. The total useful life at
the date of acquisition was estimated to be 4 years and residual value was estimated to be Rs. 4,802 for each
computer.GL decided to adopt historical cost model for subsequently measurement of computers.
2) GL purchased an office building at the date of start of business worth Rs. 3 million. GL decided to adopt fair
value model due to fluctuations in property prices. 80% of the building is occupied by computer labs, whereas
20% is used by administrative and selling departments. The useful life is estimated to be 10 years at the date of
acquisition with no residual value, and the economic benefits are expected to be derived evenly over its useful
life. At the end of the year, the fair value of office buildings was assessed to be Rs. 3,237,500.
3) GL also purchased fittings for its administrative and selling departments, costing Rs. 120,000 on 1 July 2015. It
is to be depreciated over 10 years using the straight-line method, with no residual value.
4) GL made a contractual commitment with Al-Karim Computers to purchase 6 computers of Rs.20,000 each to be
delivered at GL’s premises on 1 May 2016.

Page 4 of 17
66
The following information pertains to the year ended 31 March 2017:
1) The computers were delivered at the GL’s premises by Al-Karim Computers at the said date. It was decided to
use the same method and same rate to depreciate these computers. However, no further space was utilised by
the computer labs.
2) At the end of the year, the fair value of office building was assessed to be Rs. 2 million. At the year-end GL
mortgaged entire building with JS Bank to obtain a loan worth Rs. 1.75 million for prospective investments in
other divisions.
3) Fittings with a cost of Rs. 30,000 were disposed of for Rs. 22,000 on 1 January 2017. The Suzuki

Driver was paid Rs. 1,000 to transfer the fittings to customer’s premises.

The fair values of the office building were determined byan independent firm M/s Hafeez Yasir Chartered
Accountants& Co. Moreover, GL uses proportionate policy to depreciate its assets.

Required:
(a) Prepare the disposal account to record the sale of fittings on 1 January 2017.
(b) Prepare the disclosure under IAS 16 in relation to Property, Plant and Equipment in the notes to the published
accounts for the year ended 31 March 2017 (comparatives are required).

ANSWERS
PREPARATION OF FINANCIAL STATEMENTS

1 Answer
Share Share General Retained Revaluation
Description Capital Premium Reserves Earnings Surplus Total

Opening Balance 25,000,000 7,500,000 750,000 18,250,000 1,500,000 53,000,000


Issuance of Right
Shares
(25,000,000 / 10 x 1 / 5)
x 10 5,000,000 5,000,000 - - - 10,000,000
Interim Dividend
(25,000,000 +
5,000,000)/ 10 x 2.25 (6,750,000) (6,750,000)

Profit for the year 10,250,000 10,250,000


Transfer
(5% x 10,250,000) 512,500 (512,500) -

Transfer of Rev Surplus


to RE
(1,500,000 / 10) 150,000 (150,000) -
Balance at the end of
the year 30,000,000 12,500,000 1,262,500 21,387,500 1,350,000 66,500,000

Page 5 of 17
67
Annual Dividend
(30,000,000 / 10 x 4.25) (12,750,000) (12,750,000)
Bonus Shares
(30,000,000 x 20% ) 6,000,000 (6,000,000) -
Interim Dividend
(36,000,000 / 10 x 1.25) (4,500,000) (4,500,000)

Profit for the year 12,500,000 12,500,000


Transfer
(12,500,000 x 5% ) 625,000 (625,000) -

Transfer of Rev Surplus


to RE
(1,500,000 / 10 ) 150,000 (150,000) -
Balance at the end of
the year 36,000,000 12,500,000 1,887,500 10,162,500 1,200,000 61,750,000

Page 6 of 17
68
“Allah does not burden a soul beyond that it can bear.” Quran 2:286
2 Answer

Page 7 of 17
69
PROPERTY, PLANT AND EQUIPMENT

1. Solution (a):
Date Particulars Dr. Cr.
Rs. 000 Rs.000

2018

July 1, Land 8,000


Revaluation Surplus 5,000
Reversal of Revaluation Loss (Other Income) (15-12) 3,000
(20 - 12)
2019

Jan 1, Cash (90m - 0.1m) 89,900


Accumulated Depreciation – Buildings (W1) 10,687.5
Buildings (125 x 25%) 31,250
Land (20 x 25%) 5,000
Gain on Disposal (Other Income) 64,337.5

Jan 1, Revaluation Surplus 1,250


Retained Earnings 1,250
(5,000 x 25%)= 1,250 [transfer on disposal]

(W1) Rs. 000

At the start of year (38 × 25%) 9,500


During the year [(125 – 11) × 25% ÷ 12* x6/12] 1,187.5

Accumulated Depreciation of Buildings Disposed 10,687.5

*38 /4 (01.07.2014 to 30.06.2018) = 9.5 per annum


(125 - 11) / x = 9.5
X = 114/ 9.5 =12

Page 8 of 17
70
Solution (b)

Abbas Limited
Notes to Financial Statements
For the year ended 30 June 2019
Property, Plant and Equipment:

Abbas Limited uses the following subsequent measurement bases to value its Property, Plant and Equipment, and
methods to calculate its depreciation.
Subsequent
Depreciation Useful Life/Residual
Assets Measurement
Method Value/Rate

Fair Value
N/A N/A
Land
Useful life of 12 years with a Cost less Accumulated
residual value of 8.8% (11 / 125) of
Straight-line cost.
Buildings Depreciation
(W1)
Cost less Accumulated
Units of
Plant and Rs. 25,000 per unit (W2)
Depreciation
production
Machinery
Cost less Accumulated
Written down
Rate of 20%
Equipment Depreciation
value

Furthermore, Abbas Limited uses proportionate policy to depreciate its assets.

Page 9 of 17
71
Schedule of Movement in Property, Plant and Equipment

For the year ended 30 June 2019 (in Rs. 000)

Plant and Total


Land Buildings Equipment
Machinery

Cost / Revalued amount


12,000 125,000 500,000 100,000 737,000
At 1 July 2018
- - - 60,000 60,000
Acquisitions
8,000 - - - 8,000
Revaluations
(5,000) (31,250) - - (36,250)
Disposals(10,687.5)
15,000 93,750 500,000 160,000 768,750
At 30 June 2019

Accumulated Depreciation:
- 38,000 300,000 36,000 374,000
At 1 July 2018 125,000
(5,000 x
- 8,312.5(W4) 25,000) 20,800(W5) 154,112.5
Depreciation charge for the
- - - - -
Year
- - -
Revaluation
Disposals (10,687.5)
- 35,625 425,000 56,800 517,425
At 30 June 2019

Carrying Amount at 30 15,000 58,125 75,000 103,200 251325


June 2019

Carrying Amount at 1 July 12,000 87,000 200,000 64,000 363,000


2018

An amount of expenditure of Rs. 20 million was incurred on the construction of a factory near Lahore-Islamabad
Motorway on 1 December 2018. This amount was capitalised as capital work-in-progress.

A further borrowing costs of Rs. 1.4 million (W6) were capitalised in respect of interest on loan obtained from the
bank to finance this project.

Page 10 of 17
72
A contract was made with M/s UniPower& Co. to purchase plant and machinery worth Rs. 35 million once the
construction of factory building is completed.

The following depreciations are either made part of cost of goods manufactured or operating expenses in statement
of profit or loss:

Assets Factory Operating Expense Total


Rs.000 Rs.000 Rs.000

4,987.5 3,325 8,312.5


Buildings (60%) (40%)

125,000 - 125,000
Plant and Machinery

- 20,800 20,800
Equipment

129,987.5 24,125 154,112.5


Total

Revaluation Disclosures:
(i) The revaluation of land took place on 1 July 2018. The value was determined by an
independent firm M/s Ashfaq& Co. Chartered Accountants.
(ii) The carrying amount of land had the revaluation not taken place:

Rs. million

15
At 1 July 2018
(3.75)
Disposals during the year (15 x 25%)
11.25
At 30 June 2019

Page 11 of 17
73
(iii) Revaluation Surplus
Rs. million

-
At 1 July 2018
5
Revaluation of land
(1.25)
Transfer to retained earnings (5 x 25%)
3.75
At 30 June 2019

A further reversal of revaluation loss of Rs. 3 million was reversed during the year.

(W1) (125 – 11) /38 x4 = 12 years

(W2)
300,000,000 / 12,0000 =Rs. 25,000 per unit

(W4) Depreciation during the year Rs. 000

Disposals [(125-11) x 25% /12 x 6/12] 1,187.5


During the year [(125 – 11) × 75% ÷ 12] 7,125

Depreciation Expense 8,312.5

(W5) Depreciation during the year Rs. 000

Acquisitions (60 × 0.2 × 8/12) 8,000


Remaining Assets (100 - 36)x 20% 12,800

Depreciation Expense 20,800

(W6) 20 million x 12% x 7


/12 = 1.4 million

Page 12 of 17
74
2 Solution (a):

30,000/10 x 9/12 =2,250+30,000/10 x 9/12 =2,250=4,500

Solution (b):
Games Limited

Notes to the financial statements


For the year ended 31 March 2017
Property, plant and equipment:
Games Limited (GL) uses the following subsequent measurement bases to value its Property, Plant and Equipment,
and methods to calculate its depreciation.
Subsequent
Depreciation
Assets Rate
Measurement
Method

Fair Value
Straight-line 10%
Buildings
Cost less Accumulated
Written-down
30% (W1)
Computers Depreciation
value

Cost less Accumulated


Straight-line 10%(useful life 10years)
Fittings Depreciation

Furthermore, Games Limited uses proportionate policy to depreciate its assets.

Page 13 of 17
75
Games Limited

Schedule of Movement in Property, Plant and Equipment

For the year ended 30 March 2016 (in Rs.)

Buildings Computers Fittings


Cost / Revalued amount
At start of year - - -
Acquisitions 600,000
3,000,000 (20,000x30) 120,000
Revaluations 462,500 - -
Elimination (225,000) - -
Disposal - - -
At end of year 3,237,500 600,000 120,000

Accumulated depreciation - - -
At start of year - - -
Depreciation charge for the
year (W-2) 225,000 135,000 9,000
Elimination (225,000) - -
Disposal - - -
At end of year - 135,000 9,000
Carrying amount at start of
year - - -
Carrying amount at end of
year 3,237,500 465,000 111,000

For the year ended 31-03-2017


Cost / Revalued amount
At start of year 3,237,500 600,000 120,000
Acquisitions - 120,000 -
Revaluations (887,500) - -
Elimination (350,000)
Disposal - - (30,000)
At end of year 2,000,000 720,000 90,000

Accumulated depreciation - - -
At start of year - 135,000 9,000
Depreciation charge for the
year (W-2) 350,000 172,500 11,250

Page 14 of 17
76
Elimination (350,000) - -
Disposal - - (4,500)
At end of year - 307,500 15,750
Carrying amount at start of
year 3,237,500 465,000 111,000
Carrying amount at end of
year 2,000,000 412,500 74,250

The entire office building was mortgaged with JS Bank on 31 March 2017, to obtain a loan worth Rs. 1.75 million for
prospective investments in other divisions.

No contractual commitments were made during the year ended 31 March 2017 to purchase Property, Plant and
Equipment.

A contract was made with Al-Karim Computers during the year ended 31 March 2016 to purchase 6 computers of
Rs. 20,000 each to be delivered on 1 May 2016.

The following depreciations are either made part of inventory or expensed out in statement of profit or loss:

Assets 2017 2016

OperatingEx OperatingE
Factory pense Total Factory xpense Total
Rs. Rs. Rs. Rs. Rs. Rs.

280,000 70,000 350,000 180,000 45,000 225,000


Buildings (80%) (20%) (80%) (20%)
172,500 - 172,500 135,000 - 135,000
Computers
- 11,250 11,250 - 9,000 9,000
Fittings
452,500 81,250 533,750 315,000 54,000 369,000
Total

Revaluation Disclosures:

(iv) The revaluations of office buildings took place on 31 March 2017 and 31 March 2016 respectively. The fair values
of the office building were determined by an independent firm M/s Hafeez Yasir Chartered Accountants & Co.

Page 15 of 17
77
(v) The carrying amount of buildings had the revaluation not taken place:

2017 2016
Rs. Rs.
Cost:
3,000,000 -
At start of year
- 3,000,000
Acquisitions
- -
Disposals
3,000,000 3,000,000
At end of year

Accumulated Depreciation:
225,000 -
At start of year
300,000 225,000
Depreciation charge for the year (3,000/10)
- -
Disposals

2017 2016
Rs. Rs.

525,000 225,000
At end of year

Carrying Amount at year start 2,775,000 -

Carrying Amount at year end 2,475,000 2,775,000

(vi) Revaluation Surplus

2017 2016
Rs. Rs.

462,500 -
At start of year
- 462,500
Revaluation of Buildings (Surplus)
Transfer to Retained Earnings (462,500/9.25) (50,000) -

Page 16 of 17
78
Revaluation building (loss) (412,500)
- 462,500
At end of year

Furthermore, a revaluation loss of Rs. 475,000 was recognized at 31 March 2017.


(W1)
4 4,802
𝑟 =30%𝑟 = 1 − √
20,000

Accumulated depreciation - Building 350,000


Building 350,000
(Reversal of prior year depreciation)
Surplus on revaluation of fixed assets(OCI) 412,500
Revaluation loss(PL) 475,000
Building 887,500

(W2) 2017 2016

3,000,000 ÷ 10 ×
9
12
Buildings 3,237,500 ÷ 9.25 = 350,000
*Remaining useful life = 225,000

Rs.
Acquisitions (120,000 ×30% × 11/12) 6,000,000 × 30%
33,000 9⁄12
= 135,000
Computers

Remaining [(600,000 - 135,000) × 139,500


30%]
Total 172,500

Rs.

Disposals (30,000 × 10% × 9/12) 2,250


Fittings 120,000 ÷ 10 ×9⁄12
Remaining (90,000 × 10%) 9,000 = 9,000
Total 11,250

Page 17 of 17
79
“He Knows what is in every Heart – Surah Mulk {67:13}”

Autumn 2020
Q.2
Ratios are computed by using numerical values from financial statements to gain meaningful information about an
entity. However, due to inherent limitations of ratio analysis, it may not reflect the correct financial situation.

Required:
Briefly explain any four limitations of ratio analysis.
(06)

Q.3
On 1 July 2014, Indus Pharma Limited (IPL) received a government grant of Rs. 280 million to setup a plant in an
under-developed rural area. The grant is repayable in full if the conditions attached to the grant are not met for a
period of five years from the date of commencement of the production. At the inception, it was highly probable
that IPL would comply with the conditions for the required period.

IPL incurred total cost of Rs. 630 million on plant and it started production on 1 January 2015. Useful life of
the plant was estimated at 7 years. IPL deducted government grant in arriving at the carrying amount of the asset.

In January 2019, IPL showed its inability to comply with the conditions attached to the grant and regulatory
authority issued a notice to IPL for repayment of the grant in full. Accordingly, the grant was repaid by IPL.

In view of repayment of the grant, IPL carried out an impairment review of the plant on 31 December 2019. Net
annual cash inflows for the remaining life of the plant have been estimated at Rs. 90 million and Rs. 80 million for
2020 and 2021 respectively. These cash inflows are net of annual interest and maintenance cost of Rs. 10 million
and Rs. 6 million respectively for both years. Applicable discount rate is 12%.

On the date of impairment review, the existing plant can be sold in the local market for Rs. 160 million.
Estimated cost of disposal would be Rs. 5 million.

Required:
Prepare journal entries for the year ended 31 December 2019 in respect of the above information. (Show all
necessary workings. Narrations are not required)

(08)

Page 1 of 14
80
Q.4
Select the most appropriate answer from the options available for each of the following Multiple Choice
Questions.
i. Which of the following statements is correct about financial statements based on historical cost in times
of rising prices?
(a) Profits will be overstated and assets will be understated
(b) Assets will be overstated
(c) Profits as well as assets will be understated
(d) Depreciation will be overstated
(01)
ii. Under IAS 40 Investment property’, which of the following disclosures is NOT required to be made under
cost model?
(a) Fair value of the property
(b) Depreciation method
(c) Reconciliation of carrying amounts at the beginning and end of a period
(d) Residual value of the property
(01)

iii. Which of the following would cause negative net cash flow from operating activities?
(a) Decrease in depreciation expense
(b) A substantial investment in fixed assets
(c) A significant increase in credit sales
(d) Repayment of a long-term loan
(01)

iv. Alpha Club’s financial year ends on 31 December. Following information pertain to its members'
subscription:
Rupees
Subscription received in 2018 for 2019 180,000
Subscription received in 2019 for 2018 90,000
Subscription received in 2019 for 2019 1.400.000
Subscription received in 2019 for 2020 200,000
Subscription for 2018 outstanding as on 31 December 2018 150,000
Subscription for 2019 outstanding as on 31 December 2019 325,000

Subscription income for the year ended 31 December 2019 is:

(a) Rs. 1,845,000


(b) Rs. 1,705,000
(c) Rs. 1.905,000
(d) Rs. 1,665,000
(02)

Page 2 of 14
81
“The Dunya (World) is not the resting place, it is the testing place.”

v. A company has current ratio and quick ratio of 2.0 and 0.8 respectively. If the company uses its positive
cash balance to pay a creditor, it will:
(a) increase current ratio as well as quick ratio
(b) increase current ratio and decrease quick ratio
(c) have no effect on current ratio as well as quick ratio
(d) decrease current ratio as well as quick ratio

vi. Which of the following would increase gearing ratio?


(a) Issuance of shares at premium
(b) Issuance of shares at discount
(c) Issuance of bonus shares
(d) Declaration and payment of cash dividend

vii. Which of the following statements is correct in the context of capitalisation of borrowing costs?
(a) If funds have been arranged from various general borrowings, the amount to be capitalised is based
on the weighted average cost of borrowings
(b) Capitalisation always commences as soon as expenditure for the asset is incurred
(c) Capitalisation always continues until the asset is brought into us?
(d) Capitalisation always commences as soon as borrowing costs are incurred

Q.6 Statement of financial position of Taxila Limited (TL) as on 30 June 2020 is as follows:

Assets 2020 2019 Equity & liabilities 2020 2019


Rs. in million Rs. in million
Property, plant and equipment 1,619 1,200 Share capital (Rs. 100 each) 1,200 800
Investment property 290 120 Share premium 290 150
Inventories 205 180 Retained earnings 260 90
Trade receivables 342 291 Revaluation surplus 215 200
Prepayments and other receivables 14 20 Long-term loans 367 445
Short-term investments 60 48 Trade and other payables 144 120
Cash and bank balances 24 6 Current portion of
long-term loans 78 60
2,554 1,865 2,554 1,865

Additional information:
(i) Equipment having fair value of Rs. 240 million was acquired by issuing 2 million shares.
(ii) As a result of revaluation carried out on 30 June 2020, property, plant and equipment was
increased by Rs. 80 million out of which Rs. 35 million was credited to profit and loss account.
(iii) During the year, fully depreciated items of property, plant and equipment costing Rs. 36
million were sold for Rs. 8 million out of which Rs. 3 million is still outstanding.
(iv) Depreciation on property, plant and equipment for the year amounted to Rs. 290 million.
(v) An investment property was acquired for Rs. 180 million. TL applies cost model for subsequent

Page 3 of 14
82
measurement of its investment property.
(vi) Financial charges for the year amounted to Rs. 45 million. Trade and other payables include
accrued financial charges of Rs. 12 million (2019: Rs. 17 million).
(vii) Short-term investments amounting to Rs. 35 million are readily convertible to cash (2019: Rs. 20
million). Investment income for the year amounted to Rs. 6 million.

Required:
Prepare TL’s statement of cash flows for the year ended 30 June 2020 in accordance with the
requirements of IFRS. (17)

Q.8 Following information pertain to property, plant and equipment of Harappa

Industries Limited (HIL) for the year ended 30 June 2020:

(i) Balance as on 30 June 2019


Cost/revalued Accumulated Revaluation Depreciation Useful
Assets
amount depreciation surplus method life/rate
----------- Rs. in '000 -----------

Land* 100,000 - - - Infinite


Buildings 70,000 14,000 16,000 Straight line 20 years
Plant 180,000 60,000 - Straight line 15 years
Vehicles 8,800 4,000 - Reducing 20%
balance

*An amount of Rs. 12 million had been charged to profit or loss upon previous revaluation

(ii) On 30 June 2020, the revalued amounts of the land and buildings were assessed by Smart Consultant
at Rs. 120 million and Rs. 35 million respectively.
(iii) Setting up of a new plant was commenced on 1 July 2019 and substantially completed on 29 February
2020. The plant was available for use on 1 April 2020 and immediately put into use. Useful life of the
plant was estimated at 10 years. Details of the cost incurred are as under:

Description Payment date Rs.in '000


1st payment 1 August 2019 12,000

2nd payment 1 October 2019 48,000

3rd payment 29 February 2020 48,000

4th payment 31 July 2020 12,000


120,000

The cost of the plant was financed through an existing running finance facility with a limit of Rs. 200 million
carrying mark-up of 12% per annum. A government grant of Rs. 20 million related to the plant was received
on 1 January 2020.

Page 4 of 14
83
“Kindness is a mark of faith, and whoever is not kind has not faith.”

(iv) One of the vehicles had an engine failure on 1 January 2020 and its engine had to be sold as scrap for
Rs. 0.1 million. The vehicle had been acquired on 1 January 2018 at a cost of Rs. 2.5 million. 40% of
the cost is attributable to its engine. Though the engine of similar capacity was available at a cost of
Rs. 1.2 million, the old engine was replaced on 1 January 2020 with a higher capacity engine at a cost
of Rs. 1.8 million.
(v) HIL uses cost model for subsequent measurement of property, plant and equipment except for land and
buildings.
(vi) HIL accounts for revaluation on net replacement value method and transfers the maximum possible
amount from revaluation surplus to retained earnings on an annual basis.
(vii) HIL deducts government grant in arriving at the carrying amount of the asset.

Required:
In accordance with IFRSs, prepare a note on ‘Property, plant and equipment’ for inclusion in HIL’s financial
statements for the year ended 30 June 2020. (20)

(Comparatives figures and column for total are not required)

Answers
A.2 Limitations of ratio analysis:
(i) Historical
All information used in ratio analysis is derived from actual historical results. This does not mean
that the same results will carry forward into the future. However, ratio analysis can be used on pro
forma information and compare it to historical results for consistency.

(ii) Historical versus current cost


The information on the income statement is stated in current costs (or close to it), whereas many
elements of the balance sheet are stated at historical cost (which could vary substantially from
current costs). This disparity can result in unusual ratio results.

(iii) Inflationary effect


If the rate of inflation has changed in any of the periods under review, this can mean that the
numbers are not comparable across periods. For example, if the inflation rate was 100% in one
year, sales would appear to have doubled over the preceding year, when in fact sales did not
change at all.

(iv) Aggregation
The information in a financial statement line item that is used for a ratio analysis may have been
aggregated differently in the past, so that running the ratio analysis on a trend line does not
compare the same information through the entire trend period.

Page 5 of 14
84
A.3
Journal Entries:
Rs in ‘Millions’
Dr. Cr.
1-7-2014
Cash 280
Deferred Grant income 280
1-1-2015
Plant 630
Cash / Payable 630
1-1-2015
Deferred grant income 280
Plant 280
31-12-2015
Depreciation 50
Accumulated depreciation 50
[ 630 – 280 ] / 7 = 50
31-12-2016
Depreciation 50
Accumulated Depreciation 50
31-12-2017
Depreciation 50
Accumulated Depreciation 50
31-12-2018
Depreciation 50
Accumulated Depreciation 50
1-1-2019
Plant 280
Cash (Repayment of grant) 280
1-1-2019
Cumulative depreciation to be charged immediately
Depreciation 160
Accumulated depreciation 160
[ 280 / 7 x 4 ] = 160
31-12-2019
Depreciation 90
Accumulated Depreciation 90
[ 630 / 7 ] = 90

31-12-2019
Impairment Test:
Carrying Amount = 180
[630 – 50 x 4 – 160 – 90]
Or
[630 – (90 x 5)]

Page 6 of 14
85
“Sometimes the blessings are not in what ALLAH gives, but in what ALLAH takes away!”

Recoverable Amount:
Higher of:
FV less CTS 155
(160 – 5)
Value in Use (W) 161.04
161.04

Working: Value in use


[90 + 10] x ( 1 + 0.12 ) -1 = 89.29
[80 + 10] x ( 1 + 0.12 ) -2 = 71.75
Total = 161 .04

Impairment Loss :
[180 – 161.04] = 18.96

Impairment Loss ( P.L ) 18.96


Accumulated Impairment Loss 18.96

A.4
i. a) Profits will be overstated and assets will be understated
ii. b) Residual value of the property
iii. c) A significant increase in credit sales
iv. e) Rs. 1,905,000 (W.1)
v. f) Increase current ratio and decrease quick ratio (W.2)
vi. g) Declaration and payment of cash dividend (W.3)
vii. h) If funds have been arranged from various general borrowings, the amount to be capitalised is based on
the weighted average cost of borrowings

W.1:
Subscription A/c
b/d 150,000 b/d 180,000
Cash 90,000
Bad debts 60,000
Income 1,905,000 (150,000 – 90,000)
Remaining receivable of 2018

Cash 1400,000
Cash 200,000
c/d 200,000 c/d 325,000

Page 7 of 14
86
W.2: For Example
Current Asset Current Liability Quick Assets Current Liabilites
200,000 100,000 80,000 100,000
Existing ratios 2 1 0.8 1
If creditors are (50,000) (50,000) (50,000) (50,000)
paid in cash
150,000 50,000 30,000 50,000
Revised ratios 3 1 0.6 1

W.3:
(a) Issuance of shares at premium (Equity will increase)
(b) Issuance of shares at discount (Equity will increase)
(c) Issuance of bonus shares (No effect of equity)
(d) Declaration and payment of cash dividend (Equity will decrease and therefore gearing ratio will
increase)

A.6
Taxila Limited
Statement of cash flows for the year ended 30 June 2020
Rs. in million
Cash flows from operating activities
Profit before tax (as no tax) 140
Adjustments for:
Depreciation on property, plant and equipment 290
Depreciation on investment property 120 + 180 + 290 10
Gain on disposal of property, plant and equipment (8)
Revaluation gain (35)
Interest expense 45
Investment income (6)
Operating profit before working capital changes 436
Changes in working capital:
Increase in inventory (25)
Decrease in prepayments and other receivables 9
Increase in trade receivables (51)
Increase in trade and other payables 29
Cash generated from operations 398
Interest paid (50)
Net cash flows from operating activities 348

Cash flows from investing activities


Purchase of property, plant and equipment (389)
Purchase of investment property (180)
Proceeds from disposal of property, plant & equipment 5
Disposal of short term investment (As no other information) 3

Page 8 of 14
87
Investment income recieved 6
Net cash flows used in investing activities (555)

Cash flows from financing activities


Proceeds from issue of shares at premium (200 + 100) 300
Repayment of long term loan (60)
Net cash flows from financing activities 240
Net increase in cash and cash equivalents 33
Cash and cash equivalents at beginning of the year 26
Cash and cash equivalents at the end of the year 59

Workings:

Cash and Cash equivalents:


2020 2019
ST. Investment 35 20
Cash & Bank 24 6
59 26

PPE
b/d 1,200
SC 240 Disposal -
R.S 45 Depreciation 290
Reversal of loss(P.L) 35
Cash 389 c/d 1,619

Inventories
b/d 180
Increase 25
c/d 205

Investment Property
b/d 120
Cash 180 Dep 10
c/d 290

Page 9 of 14
88
Trade Receivables
b/d 291
Increase 51
c/d 342

Prepayments & others


b/d 20
Disposal 3 Decrease 9
c/d 14

S.T Investment
b/d 28
(48-20)
Disposal 3
c/d 25
(60-35)

S.C
b/d 800
PPE 200
(2 x 100)
Cash 200
c/d 1,200

S.P
b/d 150
PPE 40
(240-200)
Cash 100
c/d 290

R.E
b/d 90
PAT 140
R.E 30
c/d 260

Page 10 of 14
89
“If Jannah is your dream, HOLD TIGHT to your deen!”

Revaluation Surplus
b/d 200
R.E 30
PPE 45
c/d 215

Trade Payable
b/d 103
(120 - 17)
Increase 29
c/d 132
(144 - 12)

Loan
b/d 505
(445 + 60)
Cash 60
c/d 445

Financial Charges Payable


b/d 17
Cash 50
Expense 45
c/d 12

Receivable 3
Cash 5
PPE (36 - 36) 0
Gain (P.L) 8

Page 11 of 14
90
A. 8
Harappa Industries Limited
Notes to Financial Statements
For the year ended 30-06-2020
Property, Plant and Equipment:
Land Building Plant Vehicles
-------- Rs. In”000” ---------
Gross Carrying Amount:
Opening balance 100,000 70,000 180,000 8,800
Additions - - 103,240 (W-1) 1,800
Disposals - - - (1,000)
Elimination - (17,500) (W-5) - -
Revaluation 20,000 (W-6) (17,500) (W-5) - -
Closing balance 120,000 35,000 283,240 9,600
Accumulated depreciation:
Opening balance - 14,000 60,000 4,000
Depreciation - 3,500 14,581 (W-2) 1,068 (W-3)
[70,000 / 20]
Disposal - - - (352) (W-3)
Elimination - (17,500) - -
Closing balance - - 74,581 4,716
Carrying Amount 120,000 35,000 208,659 4,884

Disclosures:

Land Building Plant Vehicles


Measurement Basis Revaluation Revaluation Model Cost Model Cost Model
Model
Useful life/Rate - 15 Years 15/10 years 20%
Method of Depreciation - Straight Line Straight Line Reducing Balance

The last revaluation was performed by Smart Consultants on 30-06-2020, an independent firm of valuer. Had the
land and building been at cost model, carrying amount would have been on 30-06-2020
112 million (100 + 12) and 37.5 million (1.1) respectively.

(1.1)
16,000 / 16 x 20 = 20,000 should be surplus on 1-7-2015 (four years ago) therefore WDV on that date would be
70,000 – 20,000 = 50,000.

Afterwards WDV should be depreciated over remaining useful life (As original cost and total life is not available).
50,000 / 20 x 5 = 12,500
So 50,000 – 12,500 = 37,500

Page 12 of 14
91
“You will die the way you lived – Prophet Muhammad SAW”

W-1) Cost of New Plant:


Cost [12,000 + 48,000 + 48,000 + 12,000] 120,000
Less: Government Grant 20,000
100,000

Capitalization of borrowing cost:


12,000 x 12% x 7/12 (From Aug to Feb) 840
48,000 x 12% x 5/12 (From Oct to Feb) 2,400
Total 3,240
Total Cost of plant 103,240

W-2) Depreciation of Plant:


Opening balance [180,000 / 15] 12,000
On new plant [103,240 / 10 x 3/12] 2,581
Total 14,581

W-3) 01-01-2020 for Vehicle:

Vehicles
b/d 8,800 1-1-2020
Disposal (2.5 x 40%) 1,000
1-1-2020
Cash 1,800
c/d 9,600

Accumulated Depreciation
1-1-2020 b/d 4,000
Disposal 352
Depreciation 1,068
c/d 4,716

Accumulated Depreciation of Engine disposed on 1.1.2020:


Cost (2,500 x 40%) [1-1-2018] 1,000
x 20% x 6/12 (100)
30-06-2018 900
x 20% (180)
30-06-2019 720
x 20% x 6/12 (72)
1-1-2020 648
Accumulated Depreciation (100 + 180 + 72) = 352

Page 13 of 14
92
Depreciation:
Opening WDV (8,800 – 4,000) 4,800
Less: Opening WDV of Disposal (720)
4,080
x 20% 816
+ 720 x 20% x 6/12 72
+ 1,800 x 20% x 6/12 180
1,068

W-4) Revaluation Surplus – Building:


Opening balance [It is not the surplus on the date of last revaluation] 16,000
Incremental Depreciation [Transfer of surplus] [ 16,000 / 16 (w-4.1)] (1,000)
Remaining Surplus on 30-06-2020 15,000

(W-4.1)
Depreciation per Annum [70,000 / 20 = 3,500]
No. of years completed [14,000 / 3,500 = 4 upto 30-06-2019]
Remaining life on 1-7-2019 [20-4] = 16

W-5) Revaluation of buildings on 30-06-2020


Carrying amount [70,000 – 14,000 – 3,500] 52,500
Fair value 35,000
Revaluation Loss 17,500

Dr. Cr.
Accumulated Depreciation [14,000 + 3,500] 17,500
Building 17,500
Revaluation Surplus (OCI) 15,000
Reversal of Loss (P.L) 2,500
Building 17,500

W-6) Revaluation of Land:


Dr. Cr.
Land 20,000
Reversal of Loss (P.L) 12,000
Revaluation Surplus (OCI) 8,000

Page 14 of 14
93
Q.4 Select the most appropriate answer from the options available for each of the following Multiple Choice
Questions (MCQs).
(i) Which of the following future cash flows should NOT be included in the calculation of value in use of an
asset?
(a) Cash flows on maintaining the asset’s performance
(b) Cash flows on enhancing the asset’s performance
(c) Cash flows from continuing use of the asset
(d) Cash flows from disposal of the asset (01)

(ii) When an impairment review is carried out, an impaired asset is measured at:
(a) fair value less cost to sell (b) value in use
(c) cost (d) recoverable amount (01)

(iii) Which of the following would be an external indicator that an asset of an entity may be impaired?

(a) Increase in central bank discount rates


(b) Decline in economic performance of an asset
(c) Physical obsolescence of an asset
(d) Future restructuring plan of an asset (01)

(iv) Which of the following is NOT a measurement base for assets as referred in theConceptual
Framework?

(a) Value in use (b) Fulfilment value


(c) Current cost (d) Fair value (01)

(vi) An entity recognises revenue over time if:

(a) entity’s performance does not create an asset with an alternative use
(b) entity’s performance creates an asset whose control will be transferred at the endof contract

(c) customer simultaneously receives and consumes the benefit provided by theentity’s performance

(d) entity has an enforceable right to payment for performance completed to-date (01)

(vii) An entity made a profit of Rs. 550,000 for the year 2020 based on historical cost accounting principles. It had
opening capital of Rs. 1,500,000. During 2020, specific prices indices increased by 15% while general price indices
increased by 10%. How much profit should be recorded for 2020 under physical capital maintenance concept?

(a) Rs. 325,000 (b) Rs. 400,000


(c) Rs. 467,500 (d) Rs. 495,000 (01)

Page 1 of 11
94
(viii) In order to survive in the long run, a business must generate positive net cash flowfrom:

(a) investing activities


(b) operating activities
(c) financing activities
(d) both (a) and (b) (01)

Q.5 A fire broke out in the office of Moderna Sports Club (MSC) and burnt all the accounting records. The accountant
was able to retrieve a burnt copy of financial statements of MSC for the year ended 31 December 2020. However, few
information (as indicated by capital alphabets) were unreadable. The retrieved copy is as follows:

Balance sheet as on 31 December 2020


Funds and liabilities Rs. in '000 Assets Rs. in '000
2020 2019 2020 2019
General fund: Fixed assets - net 1,403 1,300
Opening balance A 1,586 Members’ subscription 270 158
Excess of income over expenditure B C Misc. supplies 13 10
Tuck-shop rent E 37
Tennis court fund 260 200 Advance salaries 18 15
Bank F 530
Liabilities:
Members’ subscription 20 25
Salaries 52 41
Utilities 25 D
Annual sports event 10 -

Income and expenditure account for the year ended 31 December 2020
Expenditure Rs. in '000 Income Rs. in '000
Salaries G Members’ subscriptions 919
Utilities 221 Tuck-shop rent 252
Misc. supplies H Donation - sports equipment 70
Members’ subscription written off 12 L M
Annual sports event expenses I Annual sports event - contribution R
J K Annual sports event – entrance fee S
Disposal of fixed assets 8
Repair and maintenance 40
Excess of income over expenditure B

Page 2 of 11
95
Receipts and payments account for the year ended 31 December 2020
Receipts Rs. in '000 Payments Rs. in '000
Opening balance 530 Salaries 560
N O Fixed assets 92
Tennis court fund P Annual sports event 180
Contribution for annual sports event 49 Misc. supplies 132
Entrance fee - annual sports event 86 Utilities 214
Sale of fixed assets 21 Repair and maintenance Q
Tuck-shop rent 248 Construction of tennis court 131
Scrap sale 15 Closing balance F

Required: Determine the missing information as indicated by capital alphabets.

(Redrafting of Financial Statements is not required) (18)

Q.7
You have recently joined as the finance manager of Corv Limited (CL). While reviewing the draft financial
statements for the year ended 31 December 2020 prepared by the junior accountant, you have
noted the following:

(i) In January 2020, Government allotted an industrial plot to CL at a prime location subject to
the condition that CL will establish a factory. CL constructed the factory building which was
available for use on 1 October 2020. Due to delay in recruitmentof key factory employees, the
production activities will commence on 15 March 2021.
The accountant has not recorded the land as it was given free of cost. While the factory
building is still appearing in capital work in progress as production activities will commence on
15 March 2021. (06)

(ii) CL acquired a three story building on 1 March 2020. CL uses the ground floor for itsmarketing
department while remaining two floors were in excess of CL’s need and therefore were
rented out. The first floor was rented out on 1 June 2020 and the second floor was rented
out on 1 December 2020.
The accountant has recorded the building as property, plant and equipment. The
depreciation on ground, first and second floors has been computed from 1 March
2020, 1 June 2020 and 1 December 2020 respectively. (05)

(iii) CL is constructing a power generation plant for its factory. The project started on 1
February 2020 and would complete on 30 November 2021. The work remained suspended
for 3 months. The project is financed through long term loan, acquired specifically on 1
January 2020. The unutilized amount of loan is kept in a separate saving account.

The accountant has deducted income of separate saving account from full year’s interest
on loan and presented the net amount as finance cost in the statement of profit or loss.
(05)

Page 3 of 11
96
The accounting policy of CL is to carry land and building at fair value (wherever permitted by IFRS).

Required:
Discuss how the above issues should be dealt in the financial statements of CL for the year
ended 31 December 2020 in accordance with the requirements of IFRSs.

Q.8 Sputnik Sea Limited (SSL) runs a cruise business across oceans. Following information inrespect of one of
SSL’s cruise ship is available:
(i) SSL bought a cruise ship on 1 March 2018. After completing all the requiredformalities, the ship
was ready to sail on 1 April 2018.
(ii) Details regarding components of the ship are as under:

Cost Estimated residual


Component (Rs. in million) Useful life value (Rs. in
million)
Engine 840 50,000 hours 40
Body 535 25 years 35
Dry-docking (overhaul) 60 5 years -

(iii) On 1 May 2019, the ship suffered an accident which damaged its body. Repair work took 2 months
and costed Rs. 26 million. The repair work did not change useful life and residual values of the
components.
(iv) The average monthly sailing of the ship during the last three years are as under:

Year Hours
2018 360
2019 480
2020 600
(v) SSL uses revaluation model for subsequent measurement. SSL accounts for revaluation on net
replacement value method and transfers the maximum possible amount from the revaluation surplus
to retained earnings on an annual basis.
(vi) The revalued amounts of the ship as at 31 December 2019 and 2020 were determined as Rs. 1,400
million and Rs. 1,000 million respectively. Revalued amounts are apportioned between the
components on the basis of their book values before the revaluation.

Required:
Prepare necessary journal entries to record the above transaction from the date of acquisition of the ship to
the year ended 31 December 2020. (17)

Page 4 of 11
97
A.4
(i) (b) Cash flows on enhancing the asset’s performance
(ii) (d) Recoverable amount
(iii) (a) Increase in central bank discount rates
(iv) (b) Fulfilment value
(v) (c) Customer simultaneously receives and consumes the benefit provided by theentity’s
performance
(vii) (a) Rs. 325,000[550,000 - (1,500,000x15%)]
(viii) (b) Operating activities

A.5
Balance sheet as on 31 December 2020
Funds and liabilities Rs. in '000 Assets Rs. in '000
2020 2019 2020 2019
General fund: (w-1) Fixed assets - net 1,403 1,300
Opening balance 1,766 1,586 Members’ subscription 270 158
Excess of income over expenditure 62 180 Misc. supplies 13 10
1,828 1,766 Tuck-shop rent (w-9) 41 37
Tennis court fund (w-2) 260 200 Advance salaries 18 15
Bank 450 530
Liabilities:
Members’ subscription 20 25
Salaries 52 41
Utilities(w-4) 25 18
Annual sports event 10 -
2,195 2,050 2,195 2,050

Income and expenditure account for the year ended 31 December 2020
Expenditure Rs. in '000 Income Rs. in '000
Salaries (w-7) 568 Members’ subscriptions 919
Utilities 221 Tuck-shop rent 252
Misc. supplies (w-8) 129 Donation - sports equipment 70
Members’ subscription written off 12 Scrap sale 15
Annual sports event expenses (W5) 190 Annual sports event - contribution 49
DEP. (w-6) 161 Annual sports event – entrance fee 86
Disposal of fixed assets 8
Repair and maintenance 40
Excess of income over expenditure (w- 62
1)
1256 1,256

Page 5 of 11
98
Receipts and payments account for the year ended 31 December 2020
Receipts Rs. in '000 Payments Rs. in '000
Opening balance 530 Salaries 560
Membership subscriptions (w- 790 Fixed assets 92
3)
Tennis court fund (w-2) 60 Annual sports event 180
Contribution for annual sports event 49 Misc. supplies 132
Entrance fee - annual sports event 86 Utilities 214
Sale of fixed assets 21 Repair and maintenance 40
Tuck-shop rent 248 Construction of tennis court 131
Scrap sale 15 Closing balance 450
1,799 1,799

Page 6 of 11
99
WORKINGS:
w-1) w-2)
General fund Tennis court fund
b/d 1586 b/d 200
Surplus 180 Cash 60
c/d 1,766 c/d 260
b/d 1,766
Surplus 62
c/d 1828 w-4)
Utilities payable
Cash 214 b/d 18
w-3)
Subscription A/C Exp 221
b/d 158 b/d 25
Income 919 Bad debts 12 c/d 25
Cash 790
c/d 20 c/d 270
w-6)
w-5) Fixed Assets - WDV
Annual spots event payable b/d 1300 Disp. 29
Cash 180 b/d -
Exp 190 Donation 70 Dep. 161
c/d 10
Cash 92
w-7)
Cash 131 c/d 1403
Advance + salaries payable
b/d 15 b/d 41
w-8) Misc. supplies
Cash 560
Exp 568 b/d 10
Cash 132
c/d 52 c/d 18 Exp 129

c/d 13

w-9) w-10)
Tuck shop rent receivable Disposal A/C
b/d 37 Cash 248 Cash 21
Asset 29

Income 252 Loss 8


c/d 41

Page 7 of 11
100
A.7
(i) The accounting treatment adopted by accountant for not recording land is incorrect. Allotment of land by
Government is a transfer of a non-monetary asset and should be considered as a government grant. Such non-
monetary grant may be recorded at fair value or at a nominal value. As per CL’s policy, fair value of the land should
be assessedand reported in the financial statements under the head property, plant and equipment (PPE). The grant
was made subject to construction of factory so the resulting deferred income should be recognized in income on a
systematic basis over the useful life of the factory building.

The factory building should also be transferred from capital work in progress to PPE account as the building is
available for use on 1 October 2020. Further depreciation should also be charged from same date i.e. 1 October 2020
(for three months)

(ii) The accounting treatment adopted by accountant to record complete building under PPE head is incorrect
(assuming portions can be sold separately). Two floors which have been leased/rented separately so should be
accounted for as investment property (under IAS-40). While ground floor used by marketing department should be
recorded as property, plant and equipment under IAS 16 and depreciated over its useful life.

As per CL policy, investment property should be recorded at fair value and changes in fair value should be taken to
statement of profit or loss. Any depreciation already charged on these floors should be reversed.

If however portions cannot be sold separately then entire building should be classified as investment property as
significant portion (2 floors out of 3 floors) is on rent, under IAS-40. The whole building should then be recorded at
fair value and changes in fair value should be taken to statement of profit or loss. In such a situation whole
depreciation should be reversed.

(iii) The accounting treatment adopted by accountant to expense out borrowing cost is incorrect as some borrowing
cost is eligible for capitalization. Power generation plant falls under definition of qualifying asset as its construction
involves substantial period.

Construction of the power plant is financed through specific borrowing so actual borrowing cost incurred less
temporary investment income on the borrowings would be capitalized. However, the borrowing cost will be
capitalized from the date when construction actually started i.e. 1 February 2020 rather than 1 January 2020.
Further, the capitalization of borrowing costs should be suspended and charged to the statement of profit or loss
during the three months when work was suspended.(if considered as necessary suspension)

In the statement of profit or loss, borrowing cost on loan and interest income earned from saving account should be
presently separately (related to the months of January, three months of suspension).

Page 8 of 11
101
A.8
Sputnik Sea Limited
General Journal Rs. in million
Date Description Debit Credit
01-03-2018 Cruise ship (840+535+60) 1,435.00
Bank 1,435.00

31-12-2018 Depreciation expense (W-1) 75.84


Accumulated depreciation - Cruise ship 75.84

01-05-2019 Repair cost 26.00


Bank 26.00

31-12-2019 Depreciation expense (W-1) 108.80


Accumulated depreciation - Cruise ship 108.80

31-12-2019 Accumulated depreciation - Cruise ship 184.64


Cruise ship 75.84+108.80 184.64

31-12-2019 Cruise ship 149.64


Revaluation surplus 149.64

31-12-2020 Depreciation expense (W-1) 165.82


Accumulated depreciation - Cruise ship 165.82

31-12-2020 Revaluation surplus 165.82–147.20 (W-2) 18.62


Retained earnings 18.62

31-12-2020 Accumulated depreciation - Cruise ship 165.82


Cruise ship 165.82

31-12-2020 Revaluation surplus (W-3) 131.02


Revaluation loss (W-3) 103.16
Cruise ship 1,234.18–1,000 234.18

Page 9 of 11
102
W-1: Rs. in million
Date Description Engine Ship's Body Dry Docking Total
1/3/2018 Acquisition cost 840.0 535.0 60.0 1,435
31/12/18 Depreciation expense (51.84) (15.0) (9.0) (75.84)
(840– (535– 60÷5×9/12
40)÷50,000×(360×9) 35)÷25×9÷12
31/12/19 Depreciation expense (76.80) (20.0) (12.0) (108.80)
(840–40) ÷ 50,000 × (535– 60÷5
(480×10) 35)÷25

(12 – 2)
Repair
months
31/12/19 Carrying value 711.36 500.0 39.0 1,250.36
Revaluation surplus 85.13 59.84 4.67
(Bal.) 149.64
31/12/19 Revalued amount 796.49 559.84 43.67 1,400.00
[711.36/1250.36 [500/1250.36 [39/1250.36
x1400] x1400] x1400]
31/12/20 Depreciation expense (129.81) (22.57) (13.44) (165.82)
(796.49–40) ÷ (559.84– 43.67÷(5–
(50,000–3240 35)÷(25–1.75) 1.75)
–4,800)×(600×12)
31/12/20 Carrying value 666.68 537.27 30.23 1,234.18
31/12/20 Revaluation loss (126.50) (101.95 (5.74) (234.18)
)
31/12/20 Revalued amount 540.18 435.33 24.49 1,000
[666.68/1234.18 [537.27/1234.1 [30.23/1234.18
x1000] 8 x1000] x1000]

W-2:
31.12 .2020 Transfer of surplus to retained earnings:

Engine: 85.13/(50,000 – 3,240 – 4,800) x (600 x 12) = 14.61


Ships body: 59.84/(25 – 1.75) = 2.57
Dry Dockings: 4.67/(5 – 1.75) = 1.44

Total 18.62

Page 10 of 11
103
W-3
31 .12. 2020 Adjustment of revaluation loss:
Revaluation Transfer of Remaining Revaluation Adjustment Revaluation loss
surplus on surplus on surplus on loss as on against (P.L)
31.12.2019 31.12.2020 30.6.2020 31.12.2020 surplus(OCI)
Engine 85.13 14.61 70.52 126.50 70.52 55.98
Ships 59.84 2.57 57.27 101.95 57.27 44.68
body
Dry 4.67 1.44 3.23 5.74 3.23 2.51
Dockings
Total 149.64 18.62 131.02 234.18 131.02 103.17

Page 11 of 11
104
“Avoid sin but whenever you commit a sin, say “Astaghfirullah (I ask Allah for forgiveness).”

Statement of Cash Flow: (IAS-7)


While preparing statement of financial position and statement of comprehensive income we use accrual basis of
accounting. In accrual basis of accounting;

 Incomes are recorded when they are earned whether or not received; and
 Expenses are recorded when they are incurred whether or not paid.

However in a statement of cash flow information to users are provided on the basis of cash concept.

Therefore always consider cash flow statement just like a receipt & payment account in a statement form.

 Receipt and payment A/c is a combined cash and bank A/c. e.g
Receipts and Payments
b/d 10,000 b/d 2,000 (Bank O/D)
Receipts 500,000 Payments 400,000
(Bank OD) c/d 12,000 c/d 120,000

A simplified cash flow statement from this receipt and payment account can be prepared as follows:
Receipts 500,000
Payments (400,000)
Net cash flow 100,000
Opening Balance (10,000 – 2,000) 8,000
Closing Balance (120,000 – 12,000) 108,000

Important points to remember:


• If there is no opening and closing balances of debtors then sale should be equal to receipt.
• If there is no opening and closing balances of creditors then purchase is equal to payment.
• If there is no opening and closing balances of stock then purchase is equal to cost of sales in a trading
organization.
• If there is no opening and closing balances of prepaid expenses or expenses payable then it means expense is
equal to payment.
• If there is no opening and closing balances of dividend payable then it means dividend declared is equal to
dividend paid.

Example:
Opening balance of advance tax is 70,000
Opening balance of income tax payable is 100,000

Page 1 of 27
105
Current tax expense for the period 800,000
Closing balance of advance tax is 50,000
Closing balance of income tax payable is 120,000

Required:
Calculate the tax paid during the period.

Solution
Advance Tax + Tax Payable
b/d 70,000 b/d 100,000
Cash (Bal) 760,000 Tax expense 800,000
c/d 120,000 c/d 50,000

Conclusion
Always prepare separate ledger of every expense payable or prepaid expense however;

If there is no breakup of expense into prepaid expense ledger and expense payable ledger then we can prepare a
combined ledger of prepaid expense and expense payable to calculate balancing figure of payment.
Format of Cash Flow Statement

Cash flow from operating activities X

Cash flow from investing activities X

Cash flow from financing activities. X

Net cash inflow (or outflow) during the period X

Cash and cash equivalents at the beginning of the period X

Cash and cash equivalents at the end of the period X

Important Definitions:

Financing Activities: Those activities which results into changes in equity and borrowing of business.

Investing Activities: Acquisition and disposal of *non-current assets and those investment not included in cash
and cash equivalents.

*Non-current Assets include Property, plant and equipment, intangible assets, capital work in progress and long
term investments etc.

Operating Activities Principal revenue producing activities of the business; e.g Receipts from customers, payment
to suppliers, payments for expenses etc.

Page 2 of 27
106
“Allah is sufficient for believers.”

Example
Identify operating, Investing and financing activities from the following:
a) Cash received from debtor
b) Cash received from sale of goods
c) Cash received from sale of fixed assets
d) Cash received from issue of shares
e) Cash repayments of amount borrowed
f) Dividend received
g) Dividend paid
h) Wages paid to employees
i) Interest paid
j) Interest received
k) Purchase of fixed assets
l) Acquisition of fixed asset by issue of shares
m) Cash advances to suppliers
n) Taxes paid
o) Depreciation charge for the year
p) Cash paid to purchase goods
q) Loan given by the company to its subsidiary company

List of Non-cash Transactions


• Revaluation surplus/Loss
• Amortization
• Bad debts expenses/ Bad debt written off etc.

Cash flows between cash and cash equivalents are not shown in statement of cash flows. e.g. cash deposited into
or withdrawn from bank (because they are cancelled out)

Components of cash & cash equivalents


Cash in hand -
Cash at bank -
*Short term investment -
Bank overdraft (-)
Total of cash & cash equivalents -
Definitions:

Cash: comprises of cash in hand and demand deposits (means cash at bank)

Cash equivalents: means short term highly liquid investments that are readily convertible into cash.

*Short term investments in cash flow statement means those investments having a maturity date of 3 months or
less from the date of acquisition (i.e date of investment)

Page 3 of 27
107
For example
• Amount investment in bank on 15-6-2015 is Rs 1,000,000
• Statement of Financial Position date is 30-6-2015
• Encashment date (i.e maturity date) is 15-12-2015

It is a short term investment for the purpose of preparing statement of financial position but for cash flow
statement it is not a cash equivalent investment. Had the maturity date is upto 15-9-2015, then this short term
investment should be classified as cash equivalent for the purpose of cash flow statement.

Presentation of Cash Flow from Operating Activities


 
Direct Method Indirect Method

Direct Method (Format) Assumed Figures


Statement of cash flows: direct method Cash flows from operating activities Rs.

Cash receipts from customers xxx


Cash payments to suppliers (xxx)
Cash payments to employees (xxx)
Cash paid for other operating expenses (if any) (xxx)
Cash generated from operations xxx
Taxation paid (Xxx)
Interest charges paid (Xxx)
Net cash flow from operating activities xxx

Q.1 The following data relates to Shahnawaz Sports (Private) Limited for the year ended June 30,

2007 2006
Particulars
Rs. Rs.
Inventory 230,000 185,000
Prepaid expenses 14,000 16,000
trade debtors 52,000 30,000
Cash 15,000 38,000
Accounts payable 39,000 44,000
Income tax payable 5,000 4,000
Sales 500,000
Cost of sales 310,000
Operating expenses 80,000
Interest expenses 21,000
Taxation 30,000
Depreciation - included in operating expenses 6,000
included in cost of goods sold 6,000
Profit before tax 89,000

Page 4 of 27
108
Required:
What will be the net cash flow from operating activities for the year ended 30.06.2007 as under?
(i) Direct method
(ii) Indirect method

Answers of practice questions


A.1 i) Direct Method
Shahnawaz Sports (Pvt) Limited
Statement of Cash Flow
For the year ended June 30,2007

Cash flow from Operating Activities:


Cash received from debtors 478,000
Cash paid to suppliers (354,000)
Expenses paid (72,000)
Cash generated from Operations 52,000
Interest paid (21,000)
Taxes Paid (29,000)
Net cash from Operating Activities 2,000

Debtors Accounts Payable


b/d 30,000 Cash 478,000 Cash 354,000 b/d 44,000
Sales 500,000 c/d 52,000 c/d 39,000 Purchase 349,000

Inventory
Prepaid Expense
b/d 185,000 Cost of sales (310-6) 304,000
b/d 16,000 Expenses 74,000
Purchase 349,000 c/d 230,000
cash 72,000 c/d 14,000
Taxation Payable
Cash 29,000 b/d 4,000
c/d 5,000 Tax 30,000
ii) Indirect Method
Shahnawaz Sports (Pvt) Limited
Statement of Cash Flow
For the year ended June 30,2007
Profit before tax 89,000
Depreciation 12,000
Interest Expense 21,000
Operating Profit before working capital 122,000
Changes in working capital:
Prepaid Expense 2,000
Inventory (45,000)
Debtors (22,000)
Accounts Payable (5,000)

Page 5 of 27
109
Cash generated from operations 52,000
Interest Paid (21,000)
Tax Paid (29,000)
Net cash from operating Activities 2,000

Extra practice:
Q.1 Following is the abridged balance sheet of Platinum International Traders Limited as at June 30, 2006.

Rs. 000
Particulars
2006 2005
Property, plant and equipment 10,500 10,000
Intangible assets 1,200 1,000
Short term investments 750 500
Stocks 1,290 1,000
Trade receivables (net of provisions) 450 400
Advances and deposits 200 200
Cash and bank balances 160 100
Bank overdraft 400 200
Creditors and other liabilities 500 400
Provision for taxation 2,250 1,500
Net Assets 11,400 11,100
Net Equity and Surplus on Revaluation of Fixed assets 11,400 11,100

The following additional information is also available:


(i) Accounting profit after tax is Rs. 3,000,000.
(ii) Income tax paid during the year amounted to Rs. 1,000,000.
(iii) Depreciation for the year is Rs. 750,000.
(iv) Provision for doubtful debts at the end of the year amounted to 5% of closing balance of trade
receivables. The opening balance of the provision was NIL.
(v) 48,000 shares were issued during the year at a premium of Rs. 2.50 per share. Face value of the
company’s share is Rs. 10.
(vi) Markup paid during the year amounted to Rs. 500,000.
(vii) An old machine having book value of Rs. 500,000 was exchanged with a new machine costing Rs.
950,000. The company paid Rs. 350,000 as the difference. No other fixed asset was sold during the year.
(viii) Intangibles are amortized at 35% of book value.
(ix) The company revalues its major fixed assets periodically. A revaluation carried out during the year
showed that the value of fixed assets had declined by Rs. 3,300,000. The amount was charged against
surplus recorded in earlier years.

Required:
Prepare a cash flow statement under ‘indirect method’ in accordance with IAS-7 for the year ended 30.06.2006.
Also submit necessary workings.

Page 6 of 27
110
Note:
1. If only statement of financial position is given in a question of statement of cash flow then question cannot
be solved by using direct method.
2. If the figure of debtors given in statement of financial position is net of allowance for doubtful debts then
prepare separate workings for gross debtors and allowance for doubtful debts. From allowance a/c find out
bad debts for the period and difference in gross debtors should be presented in working capital changes (if
indirect method is used).
3. In a question of cash flow statement, if a statement of financial position and/or statement of comprehensive
income is given then it means all accounting entries would have been correctly made and recorded.
4. Short term investments can be classified as cash and cash equivalents or can be presented in investing
activities. Do not include short term investments in cash and cash equivalents unless maturity date is given or
it is mentioned in question that short term investments are highly liquid investments readily convertible into
cash.

Q.2 One of your clients has contacted you to prepare cash flow statement as per the requirements of IAS-7
and has provided you the following information.
2005 2004
Rupees in ‘000’
Cash and bank 21,750 17,000
Trade & other receivable 17,000 13,400
Stocks 14,000 12,000
Investments-(Non-current) - 4,000
Building 28,000 35,000
Equipments 40,000 20,000
Intangible Asset 5,000 6,250
Provision for doubtful debts 3,000 4,500
Accumulated depreciation-Equipments 3,500 6,000
Accumulated depreciation-Building 8,500 12,000
Creditors 12,000 10,000
Dividend payable - 6,000
. Current maturity of long term loans 3,000 4,000
Long term loans 33,000 29,000
Issued, subscribed & paid up capital 40,000 28,000
Un-appropriated profit 22,750 8,150

Additional data relating to the accounts for the year ended June 30, 2005 is as follows:
• Equipment that had cost Rs. 11 million and was 40% depreciated at the time of disposal was sold for Rs. 2.5
million.
• Investments were sold at Rs 2.5 million above their cost.
• Rs. 12 million of the long term loan was settled by issuing, 1,200,000 ordinary shares of Rs.10 each.
• Cash dividend of Rs. 6.0 million was paid on September l, 2004.
• A long term loan of Rs. 16 million was obtained to Finance the purchase of equipment.
• On July 1, 2004 a portion of the building was completely destroyed by fire. Insurance claim of Rs. 15 million
was received from the insurance company. The additions to the building during the year amounted to Rs.
10.5 million and depreciation provided during the year was Rs 2.0 million.

Page 7 of 27
111
• Interest and income taxes paid during the year were Rs. 2 million and Rs. 5 million respectively.

Required:
Prepare cash how statement for the year ended June 30, 2005.

Q.3 Following are the extracts from income statement of Quality Enterprises (QE) for the year ended 31
December 2015 and its statement of financial position as al that date, together with some additional information:
Income statement for the year ended 31 December 2015
Rs. in '000
Profit from operations 6,402
Other income 1,357
Interest expense (100)
Profit before tax 7,659
Income tax expense (1,376)
Profit for the year 6,283

Statement of financial position as at 31 December 2015


2015 2014 2015 2014
Equity and liabilities Assets
— Rs. in '000 — --- Rs. in '000 ---
Non-current assets
Owner's capital 14,219 10,703 Property, plant and equipment 19,628 11,845
Un-appropriated profit 10,652 6,697 Investments 7,645 6,498
27,273 18,343
Revaluation surplus 2,676 1,911
10%bank loan 6,000 -
Current liabilities Current assets
Trade and other payables 3,337 4,953 Inventories 4,642 3,073
Income tax payable 1,300 994 Trade and other receivables 2,273 3,865
Bank overdraft - 27 Cash and bank 3,996 4
4,637 5,974 10,911 6,942
38,184 25,285 38,184 25,285

Additional information:
(i) During the year, movements in property, plant and equipment include:
• Depreciation amounting to Rs. 5,280,000.
• Machinery having a carrying amount of Rs. 2,481,000 was sold for Rs. 3,440,000.
• Factory building was revalued from a carrying amount of Rs. 5,963,000 to Rs. 8,000,000.
• An office building which had previously been revalued, was sold at its carrying amount of Rs.
2,599,000.
(ii) The owner of QE withdrew Rs. 300,000 per month. The amounts were debited to un-appropriated
profit.
(iii) Trade debts written off during the year amounted to Rs. 200,000. The provision for bad debts as at
31 December 2015 was Rs. 400,000 (2014: Rs. 550,000).

Page 8 of 27
112
“May Allah forgive us and guide us all to right path.”

(iv) The interest on bank loan is payable on 30th June every year. The bank loan was received on 1
November 2015. Interest for two months has been accrued and included in trade and other
payables.
(v) Other income includes investment income of Rs. 398,000. As at 31 December 2015, trade and other
receivables included investment income receivable amounting to Rs. 96,000 (2014: Rs. 80,000).

Required:
Prepare a statement of cash flows for Quality Enterprises for the year ended 31 December 2015, using the
indirect method. (18)

Q.4
Following are the extracts from the financial statements of Universal Limited (UL) for the year ended 30 June
2017:
Statement of Financial Position as on 30 June 2017
2017 2016 2017 2016
Assets Equity & Liabilities
Rs. in 000 Rs. in 000
Property, plant and 158,500 120,000 Share capital (Rs. 10 each) 175,000 150,000
equipment
Stock in trade 58,000 45,000 Retained earnings 54,434 21,500
Trade receivables 68,000 56,000 Revaluation Surplus 10,000 -
Cash 47,934 48,000 Debentures 18,000 26,000
Interest payable 1,000 2,500
Trade payables 42,000 39,000
Accrued liabilities 20,000 18,000
Unearned maintenance 2,000 4,000
Provision for taxation 10,000 8,000
332,434 269,000 332,434 269,000

Statement of profit or loss for the year ended 30 June 2017


Rs. in ‘000’
Sales 273,000
Cost of sales (187,500)
Gross profit 85,500
Operating expenses (46,766)
Other income 11,200
Profit before interest and tax 49,934
Interest expense (2,000)
Profit before tax 47,934
Tax expense (15,000)
Profit after tax 32,934

Page 9 of 27
113
Additional information:
(i) 60% of sales were made on credit.
(ii) UL maintains a provision for doubtful receivables at 6%. During the year, trade receivables of Rs. 7 million
were written off.
(iii) Depreciation expense for the year was Rs. 22.5 million. 70% of the depreciation was charged to cost of
sales.
(iv) Other income comprises of:
• Gain of Rs. 3 million on disposal of vehicles for Rs. 12 million;
• Maintenance income of Rs. 8.2 million

Required: Prepare UL’s statement of cash flows for the year ended 30 June 2017 using direct method.

Summary of discussion about dividend


It is a distribution of profits to owners (shareholder). If dividend is given as a percentage then multiply the rate
with the amount of share capital (nominal value of share capital) which is at the date of declaration of dividend to
calculate the amount of dividend. A company may pay dividend either in cash or in form of shares.
Dividend
Cash Dividend (Either Final or Interim) Bonus Shares (Either Final or Interim)
When Declared Dividend XXX
Dividend(Retained Earnings) XXX Share Capital A/c XXX
Dividend Payable A/c XXX

When Paid
Dividend Payable A/c XXX
Cash/ Bank A/c XXX

• Bonus shares are distributed from retained earnings and if no balance of retained earnings then from
share premium.
• Dividend is recognized on the date of declaration.
• If the dividend is declared after reporting date but before the authorization of the financial statements it
is disclosed in the financial statement to which it relates.
• Dividend is adjusted against retained earnings in the statement of changes in equity,
• If the date of declaration is not given, then:
1. For interim dividend assume that dividend is declared during the accounting period.
2. For final dividend assume declared after the reporting date.
• If nature of dividend (means whether final or interim) is not given then assume final dividend.
• If type of dividend (means cash or bonus) is not available then assume cash dividend.

Page 10 of 27
114
“Repent and seek forgiveness from Allah. Allah forgives all sins.”

Answers
A.1
Platinum International Traders
Statement of Cash Flow
For the year ended 30-6-2006
Rs 000 Rs 000
Cash flow from operating activities
Profit before tax (3,000 + 1,750) 4,750
Depreciation 750
Bad debts 24
Interest Expense 500
Amortization of Intangibles 646
Gain on exchange of machine (100)
Profit from operations before working capital changes 6,570
Working capital changes
Increase in stock in trade (290)
Increase in trade receivable (74)
Increase in creditors 100
Cash generated from operations 6,306
Income tax (1,000)
Interest paid (500)
Net cash flow from operating activities 4,806
Cash flow from investing activities
Property, Plant and equipment (4,100 + 350) (4,450)
Intangible assets (846)
Short term investment (750-500) (250)
Net cash used in investing activities (5,546)

Cash flows from financing activities


Cash received from issuance of share (48,000 x 12.5) 600
Net cash flow from financing activities 600
Net decrease in cash and cash equivalents (140)
Cash and cash equivalents at the beginning of the period (100)
Cash and cash equivalents at the end of the period (240)

Working
W.1 2006 2005
Note for cash and cash equivalent 160 100
Cash and bank balance (400) (200)
Bank overdraft (240) (100)

Page 11 of 27
115
W.2 Rs 000
Profit before tax
Profit after tax 3,000
Income tax expenses (W-3) 1,750
Profit before tax 4,750

W.3
Provision for taxation
Cash 1,000 b/d 1,500
c/d 2,250 Tax expense (bal) 1,750
3,250 3,250

W.4
Machinery Disposal
Fixed Asset (book value) 500 Fixed Asset (950-350) 600
Profit & Loss (Gain) 100
600 600

W.5
Fixed Assets
b/d 10,000 Depreciation 750
Disposal 600 Disposal 500
Cash (bal) 4,100 Revaluation Surplus 3,300
Cash (machine) 350 c/d 10,500
15,050 15,050

W.6
Intangible Assets
b/d 1,000 Amortization 646
(1,200÷65% x 35%)
Cash (bal) 846 c/d 1,200
1,846 1,846

W.6
Net Equity & Surplus on Revaluation of Fixed Asset
b/d 11,100
Revaluation Deficit 3,300 Profit for the year 3,000
c/d 11,400 Cash (shared issue) (480+120) 600
16,450 16,450

Page 12 of 27
116
“Don’t be sad nor grieve. Allah is with us.”

A.2
CLIENTS
Rs. 000
Cash flow from operating activities
Profit before taxation (14,600 + 5,000) 19,600
Adjustments for:
Depreciation: Equipment 1,900
Depreciation: Building 2,000
Gain on investment 2,500
Loss on sale of equipment (W3) 4,100
Amortization of intangible asset 1,250
Interest expense 2,000
Gain on receipt of insurance claim (W4) (3,000)
Reversal of provision for doubtful debts (1,500)
Operating profit before working capital changes 23,850
Working capital changes:
Increase in trade and other receivables (3,600)
Increase in stock (2,000)
Increase in creditors 2,000
Cash generated from operations 20,250
Interest paid (2,000)
Income tax paid (5,000)
Net cash generated from operating activities 13,250
(31,000)
CASH FLOW FROM INVESTING ACTIVITIES 2,500
Purchase of equipment (W1) (10,500)
Sale proceeds of equipment 15,000
Purchase of building (W2) 6,500
Insurance claim received
Receipts from sale of investments
Net cash used in investing activities (17,500)
CASH FLOW FROM FINANCING ACTIVITIES
Long term loan received 16,000
Repayment of long term loan (1,000)
Dividend paid (6,000)
Net cash generated from financing activities 9,000
Net increase in cash and cash equivalents 4,750 17,000
Cash and bank at beginning of year
Cash and bank at end of the year 21,750

Page 13 of 27
117
Workings
W-1
Equipment
b/d 20,000
Cash 31,000 Disposal 11,000
c/d 40,000
W-2
Building
b/d 35,000
Cash 10,500 Disposal 17,500
c/d 28,000
W-3
Disposal - Equipment
Equipment 11,000 Cash 2,500
Acc depreciation 4,400
Loss 4,100
W-4
Disposal - Building
Building 17,500 Cash 15,000
Gain 3,000 Acc Depreciation 5,500
W-5
Acc Depreciation- Building
Disposal 5,500 b/d 12,000
c/d 8,500 Depreciation 2,000
W-6
Acc Depreciation- Equipment
Disposal 4,400 b/d 5,000
c/d 3,500 Depreciation 1,900

Retained Earnings
b/d 8,150
c/d 22,750 Profit after tax (bal) 14,600

Long Term Loan


Share capital 12,000 b/d 33,000
Cash 1000 Cash 16,000
c/d 36,000 Depreciation 1,900

Page 14 of 27
118
“If you want that Allah would love you then obey Allah.”

Ans.3 Quality Enterprises:


Statement of Cash Flow
For the year ended 31-12-2015
Cash Flow from Operating Activities:
Profit before tax 7,659,000
Investment income (398,000)
Interest expense 100,000
Depreciation 5,280,000
Bad debts (200 – 150) 50,000
Profit on disposal (959,000)
Profit before working capital changes 11,732,000

Working Capital Charges:


Inventories (3,073 – 4,642) (1,569,000)
Debtors 1,558,000
Trade and other payables (1,716,000)
Cash generated from operations 10,005,000
Tax paid (1,070,000)
Net cash from operating activities 8,935,000
Cash Flow from Investing Activities:
Purchase of PPE (16,106,000)
Sale of PPE (3,440 + 2,599) 6,039,000
Investment income received 382,000
Purchase of investment (7,645 – 6,498) (1,147,000)
Net cash flow from investing activities (10,832,000)
Cash Flow from Financing Activities:
Receipt from bank loan 6,000,000
Additional capital 3,516,000
Drawings (3,600,000)
Net cash flow from financing activities 5,916,000
Net cash flow 4,019,000
Opening balance (23,000)
Closing balance 3,996,000
Workings:
Components of Cash and Cash Equivalents:
2015 2014
Cash and bank 3,996,000 4,000
Bank overdraft (27,000)
3,996,000 (23,000)

Page 15 of 27
119
PPE
b/d 11,845 Depreciation 5,280
R.S 2,037 Disposal 2,481
Cash 16,106 Disposal 2,599
c/d 19,628
R.S
R.E (balance) 1,272 b/d 1,911
PPE 2,037
c/d 2,676

R. Earnings
Drawing 3,600 b/d 6,697
Profit after tax 6,283
R.Surplus 1,272
c/d 10,652
Capital
b/d 10,703
Cash 3,516
c/d 14,219
Trade Debtors
b/d (3,865 – 80 + 550) 4,335 Bad debts 200
Decrease 1,558
c/d [2,273-96+400] 2,577
Provision
b/d 550
Bad debts 150
c/d 400

Interest Receivable
b/d 80
Income 398 Cash 382
c/d 96
Trade payable
Decrease 1,716 b/d 4,953
c/d (3,337 – 100) 3,237

Interest Payable
b/d --
Expense 100
c/d 100

Page 16 of 27
120
Don’t take prayer (Salat) as burden. Prayer gives us relief.

Tax Payable
b/d 994
Cash 1,070 Exp. 1,376
c/d 1,300

Disposal
PPE 2,481 Cash 3,440
Gain 959

Disposal
PPE 2,599 Cash 2,599

A.4
Universal Limited
Statement of Cash Flow
For the year ended 30-6-2017
Cash Flow from Operating Activities:
Cash receipt from customer [273,000 × 40% + 144,034] 253,234
Receipt from maintenance services (W) 6,200
Payment to suppliers (W) (181,750)
Payment for expenses (W) (30,250)
Cash generated from operations 47,434
Interest paid (3,500)
Tax paid (13,000)
Net cash from operation activities 30,934
Cash Flow from Investing Activities:
Receipt from disposal 12,000
Purchase of PPE (60,000)
Net cash from investing activities (48,000)
Cash Flow from Financing Activities:
Receipt from issue of shares 25,000
Repayment of debentures (8,000)
Net cash from financing activities 17,000
Net cash flow (66)
Cash and cash equivalents at the beginning of the period 48,000
Cash and cash equivalents at the end of the period 47,934

Page 17 of 27
121
Workings:
Cash and Cash Equivalents:
2017 2016
Cash 47,934 48,000
PPE
b/d 120,000 Disposal 9,000
R.S 10,000 Depreciation 22,500
Cash 60,000
c/d 158,500
Stock
b/d 45,000 Cost of sales 171,750
[187,500 – 22,500 × 70%]
Purchases 184,750
c/d 58,000
Provision A/c
b/d 35,761
Expenses 766
c/d 4,340
Trade Receivables
b/d [56,000 + 3,574] 59,574 Bad debts 7,000
Sales 163,800 Cash 144,034
c/d [68,000 + 4,390] 72,340
Share Capital
b/d 150,000
Cash 25,000
c/d 175,000
R/Earnings
b/d 21,500
PAT 32,934
c/d 54,434
R. Surplus
b/d --
PPE 10,000
c/d 10,000
Debentures
b/d 26,000
Cash 8,000
c/d 18,000

Interest Payable
b/d 2,500
Cash 3,500 Expenses 2,000
c/d 1,000

Page 18 of 27
122
Prayer purifies our hearts and brings mercy of Allah to us.”

Trade Payable
b/d 39,000
Cash 181,750 Purchases 184,750
c/d 42,000

Accrued Liability
b/d 18,000
Cash 30,250 Expenses
[46,766 – 7,000 – 22,500 × 30% - 766] 32,250
c/d 20,000
Unearned maintenance Income
b/d 4,000
Income 8,200 Cash 6,200
c/d 2,000
Disposal
Cash 12,000
PPE 9,000
Gain (bal) 3000
Provision for Tax
b/d 8,000
Cash 13,000 Expenses 15,000
c/d 10,000

A.4
Cash Flow From Operating Activities: (by using Indirect Method)
Rs. in ‘000’
Profit before tax 47,934
Depreciation 22,500
Bad debts [7,000 + 766] 7,766
Gain on disposal (3,000)
Interest expense 2,000
Profit before working capital charges: 77,200

Working Capital Changes:


Stock (13,000)
Debtors (19,766)
Trade payable 3,000
Accrued expenses 2,000
Unearned Income (2,000)
Cash Generated from Operations 47,434

Page 19 of 27
123
Trade receivable
b/d 59,574 Bad debts 7,000
Net increase 19,766
c/d 72,340

Extra practice questions


Q. 1 Following information pertains to Nadir Limited:

Extract from statement of profit or loss for the year ended 31 December 2017

Rs. in ‘000
Profit before taxation 8,955
Taxation (2,945)
Profit after taxation 6,010

Extract from statement of financial position as on 31 December 2017


2017 2016 2017 2016
Equity and liabilities Assets
---- Rs. in ‘000 ---- ---- Rs. in ‘000 ----
Share capital 12,400 10,000 Property plant &
Share premium 1,400 - equipment – net 21,400 15,800
Retained earnings 13,450 12,440 Current assets:
Surplus on revaluation 4,000 - Stock-in-trade 5,600 5,750
Non-current liabilities: Trade receivables – net 6,840 4,446
Long-term loans 4,100 5,000 Other receivables 2,385 800
Current liabilities: Cash & bank 2,355 3,204
Trade payables 1,900 1,400
Accruals & other payables 680 660
Tax liability 650 500
38,580 30,000 38,580 30,000

Other information:

(i) Shares issued during the year were as follows: 10% bonus shares in March 2017.
Right shares in July 2017.
(ii) During the year, a plant costing Rs. 9,500,000 and having a book value of Rs. 5,200,000 was disposed of for
Rs. 4,800,000 of which Rs. 1,800,000 are still outstanding.
(iii) Depreciation for the year amounted to Rs. 7,350,000.
(iv) Financial charges for the year amounted to Rs. 1,100,000. Accrued financial charges as on 31 December
2017 amounted to Rs. 112,000 (2016: Rs. 48,000).
(v) Provision for doubtful trade receivables is maintained at 5%.

Page 20 of 27
124
“May Allah accept our prayers and forgive our sins’”

Required:
Prepare statement of cash flows for the year ended 31 December 2017, in accordance with
IAS 7 ‘Statement of Cash Flows’ using indirect method. (15)

A.1
Nadir Limited
Statement of Cash Flow
For the year ended 31-12-2017
Cash flow from operating activities:
Profit before tax 8,955
Loss on disposal 400
Depreciation 7,350
Interest expense 1,100
Bad debts 126
Profit before working capital changes 17,931
Changes in working capital:
Trade payable 500
Accrues expenses (44)
Stock 150
Debtors (2,520)
Other receivables 215
Cash generated from operating activities 16,232

Interest paid (1,036)


Tax paid (2,795)
Net cash flow from operating activities 12,401

Cash flow from investing activities:

Receipt from disposal of plant (4,800 – 1,800) 3,000


Purchase of PPE (14,150)
Net cash flow from investing activities (11,150)

Cash flow from financing activities:

Receipt from 2,800


Dividend paid (4,000)
Repayment of loan (900)
Net cash flow from financing activities (2,100)

Net cash flow (12,401 – 11,150 – 2,100) (849)


Opening cash and cash equivalent 3,204

Page 21 of 27
125
Closing cash and cash equivalents 2,355

Workings:
Components of cash and cash equivalents:
2017 2016
Cash and Bank 2,355 3,204

Share Capital
b/d 10,000
Cash 1,400
Bonus share 1,000
(10,000 x 10%)
c/d 12,400

Share Premium
b/d -
Cash 1,400
c/d 1,400

Retained Earnings
Dividend 1,000 b/d 12,440
(Bonus Share)
Cash 4,000 PAT 6,010
c/d 13,450

Revaluation Surplus
b/d -
PPE 4,000
c/d 4,000

Loan
b/d 5,000
Cash 900
c/d 4,100

Trade Payable
b/d 1,400
500
c/d 1,900

Page 22 of 27
126
“Prayer is the key of success. Prayer will work for us If we obey Allah.”

Other Payable
b/d 612
(660 – 48)
Cash 44
c/d 568
(680 – 112)

Allowance
b/d 234
(4,446/95
x5
Bad debts 126
c/d 360
(6,840/95 x 5)

Tax Payable
b/d 500
Cash 2,795 Tax expense 2,945
c/d 650

Debtors
b/d 4,680
(4,446/95 x 100)
2,520
c/d 7,200
(6,840/95 x 100)

PPE
b/d 15,800 Disposal 5,200
R. surplus 4,000 Depreciation 7,350
Cash 14,150
c/d 21,400

Disposal
PPE 5,200 Cash 3,000
Receivable 1,800
Loss 400

Stock

Page 23 of 27
127
b/d 5,750
150
c/d 5,600

Other Receivables
b/d 800
Disposal Outstanding 1,800 215
c/d 2,385

Interest Payable
b/d 48
Cash 1,036 Expense 1,100
c/d 112

Q.2 Junior Accountant of Drum Limited has prepared the following statement of cash flows for the year ended 31
December 2018:

Statement of cash flows


Rs. in '000
Cash flows from operating activities
Increase in retained earnings 1,360
Increase in dividend payable 200
Increase in net trade receivables (100)
Increase in interest accrued 50
1,510
Cash flows from investing activities
Increase in land and building (2,600)
Increase in equipment (1,550)
Decrease in inventory 400
Decrease in tax payable (60)
(3,810)
Cash flows from financing activities
Increase in share capital and premium 2,350
Decrease in long term loan (1,000)
Increase in trade and other payables 600
1,950
Decrease in cash balance during the year (350)
Opening cash balance 450
Closing cash balance 100

Junior Accountant informed you that he has taken the difference of opening and closing balances of each balance
sheet item and classified each difference as either operating, investing or financing cash flows. He further

Page 24 of 27
128
informed that the statement is tied up with the cash balances appearing in the balance sheet. He has ignored the
following information:
(i) Depreciation on building and equipment amounted to Rs. 480,000 and Rs. 810,000 respectively.
(ii) During the year, an equipment costing Rs. 560,000 and having a book value of Rs. 310,000 was sold for Rs.
440,000.
(iii) Provision for doubtful debts was increased by Rs. 140,000.
(iv) Dividend amounting to Rs. 700,000 was paid during the year.
(v) Interest and tax expenses for the year amounted to Rs. 378,000 and Rs. 650,000 respectively.
(vi) Trade and other payables as at 31 December 2018 included Rs. 950,000 for purchase of land and building.

Required:
Prepare statement of cash flows for the year ended 31 December 2018, in accordance with IAS 7 ‘Statement of
Cash Flows’ using indirect method. (14)

Ans. 2 Drum Limited


Statement of cash flows for the year ended 31 December 2018
Rs. in ‘000
Cash flows from operating activities
Profit before tax 1,360+700+200+650 2,910
Adjustment for:
Depreciation 480+810 1,290
Gain on disposal (130)
Increase in provision of doubtful debts 140
Interest expense 378
4,588
Working capital change
Decrease in inventory 400
Increase in trade receivables –100–140 (240)
Decrease in trade payable 600–950 (350)
(190)
Cash generated from operations 4,398
Interest paid –378+50 (328)
Tax paid –650–60 (710)
3,360

Cash flows from investing activities


Purchase of land and building –2,600–480+950 (2,130)
Purchase of equipment –1,550–810–310 (2,670)
Disposal of equipment 440
(4,360)

Cash flows from financing activities

Page 25 of 27
129
Issuance of shares 2,350
Loan repaid (1,000)
Dividend paid (700)
650
Decrease in cash during the year (350)
Opening cash 450
Closing cash 100

Equipment WDV
b/d - Disposal 310
Cash 2,670 Dep 810
c/d 1,550

Land & Building WDV


b/d - Dep 480
Payable 950
2,130 c/d 2,600
Dividend Payable
Cash 700 b/d -
c/d 200 Dividend 900

Payable
350 b/d -
c/d 600 Building 950

Retained earnings
Dividend 900 b/d -
c/d 1,360 PAT 2260

Debtor – Net
b/d -
c/d 100

Debtor – Gross
b/d -
240 c/d 240

Allowance
b/d -
c/d 140 140

Interest payable
328 b/d -

Page 26 of 27
130
c/d 50 Exp 378

Tax payable
710 b/d 60
c/d - Exp 650

Inventories
b/d 400 400
c/d -

S.C & S.P


b/d -
c/d 2,350 Cash 2,350

Loan
1,000 b/d 1,000
c/d -

PBT 2,910
Tax 650
PAT 2,260

Page 27 of 27
131
IAS 7
Cash equivalents are short‑ term, highly liquid investments that are readily convertible to known amounts of cash
and which are subject to an insignificant risk of changes in value.

Not all investments are cash equivalents. An investment normally qualifies as a cash equivalent only when it has a
short maturity of, say, three months or less from the date of acquisition. Equity investments are excluded from cash
equivalents unless they are, in substance, cash equivalents, for example, redeemable preference shares acquired
within a short period of their specified maturity date.

⯈ Example:
Which of the following items are cash equivalents:
i. Investment in ordinary shares of another entity. The shares are traded on Pakistan StockExchange.
ii. Prize bonds issued by Government of Pakistan
iii. Government treasury bills with specified two months’ maturity (to be encashed intospecified amount)
iv. Trade receivables
v. Plant and machinery
vi. Inventory

⯈ Answer:
i. Not cash equivalent. Although these shares are readily convertible into cash yet there issignificant risk of changes
in value.
ii. Cash equivalent.
iii. Cash equivalent.
iv. Not cash equivalent. These are not investments.
v. Not cash equivalent. Not readily convertible into known amount of cash.
vi. Not cash equivalent. Not readily convertible into known amount of cash.

Cash flows are inflows and outflows of cash and cash equivalents.

Cash flows exclude movements between items that constitute cash or cash equivalents.

⯈ Example:
Whether the following transactions shall be reported as cash flows?
i. Cash in hand deposited in bank account
ii. Cash withdrawn from bank to be held as cash in hand
iii. Sale or purchase of prize bonds
iv. Investment in government treasury bills with 60 days’ maturity

⯈ Answer:
None of the transactions shall be reported because cash flows exclude movements between items
that constitute cash or cash equivalents.

Page 1 of 9
132
⯈ Example:
Calculate the total amount of cash and cash equivalents from the following data:

Rs.
Cash in hand 200,000
Cash at bank (demand deposit) 400,000
Cash at bank (term deposit 3 months) 100,000
Cash at bank (term deposit 2 years) 300,000
Short term investment in T bills (1 month) 50,000
Investment in prize bonds 30,000
Trade receivables 80,000
Prepaid rent 60,000
Bank overdraft 8,000

Cash and cash equivalents Rs.


⯈ Answer:
Cash in hand 200,000
Cash at bank (demand deposit) 400,000
Cash at bank (term deposit 3 months) 100,000
Short term investment in T bills (1 month) 50,000
Investment in prize bonds 30,000
Bank overdraft (8,000)
772,000

Disclosure [IAS 7: 45 to 48]


An entity shall disclose the components of cash and cash equivalents and shall present a reconciliation of the
amounts in its statement of cash flows with the equivalent items reported in the statement of financial position.

⯈ Example:
The following data has been extracted from statement of financial position of Kashif SoftwareLimited (KSL):

2023 2022 2023 2022


Current assets Rs. m Rs. m Current liabilities Rs. m Rs. m
Inventory 145 140 Short term loan 190 180
Trade receivables 195 180 Trade payables 185 170
Marketable securities* 5 8 Accrued expenses 13 17
Cash at bank 7 - Bank overdraft - 9
Cash in hand 2 3

*Cash equivalents

Page 2 of 9
133
Required: Prepare the disclosure note of components of cash and cash equivalents along with reconciliation with
equivalent items presented in statement of financial position for KSL.
⯈ Answer:

Rs. m
Net increase in cash and cash equivalents 12
Cash and cash equivalent at beginning of period 2
Cash and cash equivalent at end of period 14

2023 2022
Cash and cash equivalents Rs. m Rs. m
Marketable securities* 5 8
Cash at bank 7 -
Cash in hand 2 3
Bank overdraft - (9)
14 2
An entity shall also disclose, together with a commentary by management, the amount of significant cash and cash
equivalent balances held by the entity that are not available for use. For example, cash and cash equivalent balances
held at a branch that operates in a country where exchange controls or other legal restrictions are such that the
balances are not available for general use by the entity in Pakistan.

Interest, dividend and taxation [IAS 7: 31 to 34]


Cash flows from interest and dividends received and paid shall each be disclosed separately. Each shall be classified
in a consistent manner from period to period as either operating, investing or financing activities.
Each of these items may be classified in either of the following two ways:

Item Suggested classification Alternative classification


Interest paid (expensed or Operating because it is deducted in Financing because they are costs of
capitalised in accordance determining profit or loss. obtaining financial resources.
with IAS 23)
Interest received Investing because they represent Operating because it is deducted in
returns on investment. determining profit or loss.
Dividend received Investing because they represent Operating because it is deducted in
returns on investment. determining profit or loss.
Dividend paid Financing because they are costs of Operating in order to assist users to
obtaining financial resources. determine the ability of entity to pay
dividends out cash generated from
operations.

Page 3 of 9
134
Non-cash transactions [IAS 7: 43]
Investing and financing transactions that do not require the use of cash or cash equivalents shall be excluded from
a statement of cash flows. Examples include conversion of debt to equity, bonus issue of shares and revaluation
and/or impairment of non-current assets.

Such transactions shall be disclosed elsewhere in the financial statements in a way that provides all the relevant
information about these investing and financing activities.

⯈ Example:
Identify the following transactions as operating, investing, financing activity or otherwise for the purpose of
preparing statement of cash flows:
i. Cash received from customers
ii. Cash sales
iii. Cash proceeds from disposal of PPE
iv. Right issue of shares
v. Dividend paid
vi. Salaries paid to employees
vii. Interest paid
viii. Interest received by a trading entity on some investment
ix. Repayment of loan
x. Taxes paid
xi. Purchase of a patent and software
xii. Advance paid to supplier
xiii. Depreciation
xiv. Bonus issue of shares
xv. Impairment loss on a plant


Answer: i. Operating
ii. Operating
iii. Investing
iv. Financing
v. Financing or operating
vi. Operating
vii. Operating or financing
viii. Investing
ix. Financing
x. Operating
xi. Investing
xii. Operating
xiii. Non-cash transaction
xiv. Non-cash transaction
xv. Non-cash transaction

Page 4 of 9
135
USEFULNESS OF CASH FLOW INFORMATION
Benefits of cash flow information [IAS 7: 4 & 5]
A statement of cash flows (alongwith other financial statements) enables users to evaluate:
a) the changes in net assets of an entity,
b) its financial structure (including its liquidity and solvency); and
c) its ability to affect the amounts; and
d) timing of cash flows in order to adapt to changing circumstances and opportunities.

Historical cash flow information is used as an indicator of the amount, timing and certainty of future cash flows
a) in checking the accuracy of past assessments of future cash flows;
b) in examining the relationship between profitability and net cash flow and the impact of changing prices; and
c) in comparability of the reporting of operating performance by different entities because it eliminates the effects
of using different accounting treatments for the same transactions and events.

Classification wise usefulness [IAS 7: 13, 16 & 17]


Cash flows are classified by operating, investing and financing activities as each of these classifications provide useful
information from different perspective:

Cash flows arising from Usefulness

Operating activities It is a key indicator of the extent to which the operations of the entity have generated
sufficient cash flows to repay loans, maintain the operating capability of the entity, pay
dividends and make new investments without recourse to external sources of
financing. Information about the specific components ofhistorical operating cash flows
is useful, in conjunction with other information, in forecasting future operating cash
flows.
Investing activities These cash flows represent the extent to which expenditures have been made for
resources intended to generate future income and cash flows.

Financing activities It is useful in predicting claims on future cash flows by providers of capital to the

STICKY NOTES
entity.

Comparison of usefulness of cash flow information with profit or loss


Businesses must have sufficient cash; otherwise they cannot survive.
• A business may be incurring losses but may still survive if it has sufficient cash or has access to other liquid
assets.
• A profitable business may not survive if it is unable to pay its obligations when they fall due, because it does not
have enough cash or access to other liquid assets.

The fundamental purpose of being in business is to generate profit, as this will increase the owners' wealth.
Profitability relates to the long-term performance of the business and indicates that over the long term a business will
generate cash. In the short term, the business' viability is determined by its ability to generate cash. However, as a
statement of profit or loss is prepared on an accrual basis, the profit for the year is unlikely to correlate with the
movement in the company's bank balance. Therefore, understanding cash inflows and outflows is important.

Page 5 of 9
136
Each financial statement, individually and in combination with other financial statements and other information,
provides useful information that helps users of financial statements to make informed decisions. A balance between
profitability and liquidity (cash balance) is required, a huge cash balance does not usually indicate good management
as this could have been invested to earn more profits. In particular, following points should be considered:
• The amount and composition of net assets of an entity changes due to income and expenses (statement of profit
or loss) and cash flows (statement of cash flows). Both statements are relevant but provide different aspects of
information.
• Many decision-making models and valuation models rely on present value of the future cash flows generated
by an entity e.g., NPV and IRR. Historical cash flow information can be useful to check the accuracy of past
assessments and development of future assessments.
• Profitability is an important performance measure, and this information is provided by statement of profit or
loss, liquidity information is also important, and this information is provided by statement of cash flows in
conjunction with statement of financial position.
• Cash flows are necessary to survive in short term but in long term business must be profitable to survive. Entities
often forego short term benefits for long term major benefits e.g. sales on credit usually earns higher profit
margin as compared to cash sales.
• The cash flow is not affected by different accounting policies and estimates, and this makes cash flow
information more comparable, and consequently, less vulnerable to manipulation.

Example:
Following is the draft balance sheet of XYZ Limited as at 31 December 2014 which was prepared by its accountant:
Assets Rs. in million Equities and liabilities Rs. in million
Leasehold land – cost 250 Capital 1,000
Leasehold land – accumulated (200) Accumulated profit 1,816
amortisation
Building – cost 1,000 Long term bank loan 200
Building – accumulated (500) Trade payables 228
depreciation
Machinery – cost 1,750 Income tax payable 85
Machinery – accumulated (1,150) Accrued interest 13
depreciation
Long term deposit 70
Stocks 910
Account receivables – net of 361
provision
Cash and bank 851
3,342 3,342

Page 6 of 9
137
Additional information:
i. Profit before tax and income tax expenses for the year amounted to Rs. 275 million and Rs. 13 million
respectively.
ii. Balances as at 31 December 2013 were as under:

Rs. in million
Stock 703
Account receivables – net of provision 418
Cash and bank 243
Trade payables 150
Income tax payable 80
Long term deposit 70

The company follows a policy of maintaining provision for bad debts equal to 5% ofaccount receivables.

iii. The bank loan was obtained on 1 January 2014 and carries interest @ 9% per annum.
iv. XYZ uses straight line method for depreciation. Rates of depreciation are as under:

Leasehold land 2%
Building 5%
Machinery 10%

Full month’s depreciation is provided in the month of acquisition, but no depreciation is charged in
the month of disposal. Depreciation for the year 2014 has already been provided.
On review the CFO has discovered the following:
• A machine with list price of Rs. 50 million was purchased on 1 January 2014. An amount of Rs. 30
million had been paid in cash whereas Rs. 20 million wereadjusted against trade-in of a machine
costing Rs. 40 million and having a bookvalue of Rs. 25 million. The transaction was recorded by
debiting the plant andmachinery account by Rs. 30 million i.e. the net amount paid to the supplier.
• One of the company's customers became bankrupt during the year. Rs. 5 million out of total debt
of Rs. 25 million were recovered from him. Balance has to be written off.

Required: Prepare a statement of cash flow as at 31 December 2014.

Page 7 of 9
138
XYZ Limited
Statement of cash flows for the year ended 31 December 2014

Answer

Cash flows from operating activities Rs. m


Cash generated from operations (Note 1) 451
Interest paid [18 + 0 – 13] (5)
Income taxes paid [13 + 80 – 85] (8)
Net cash inflow from (used in) operating activities 438

Cash flows from investing activities


Cash paid in exchange of machines (30)
Net cash outflow from (used in) investing activities (30)

Cash flows from financing activities


Loans obtained 200
Net cash inflow from (used in) financing activities 200

Net increase in cash and cash equivalents 608


Cash and cash equivalent at the beginning of the year 243
Cash and cash equivalent at the end of the year 851

Note 1: Cash generated from operations (indirect method) Rs. m


Profit before tax W1 253
Adjustments:
Depreciation W2 228
Loss on disposal [25 - 20 trade-in value] 5
Bad debts 20
Reversal of doubtful debts expense W3 (4)
Finance costs 18
Operating profit 520
Working capital changes:
Inventories [910 – 703] (207)
Trade receivables [(361 + 19 W3) – (418 + 22 W3)] 60
Trade payables [228 – 150] 78
451

Page 8 of 9
139
W1: Corrected profit before tax Rs. m
Profit as given 275
Loss on disposal [25 book value - 20 Trade-in value] (5)
Depreciation reversal [(20 Addition - 40 Disposal) x 10% 2
Bad debts to be written off [25 - 5] (20)
Decrease in doubtful debt allowance [20 x 5%] 1
253

W2: Depreciation Rs. m


On leasehold land [250 x 2%] 5
On Building [1,000 x 5%] 50
On Machinery [1,750 x 10%] 175
Reversal of depreciation [(20 Addition - 40 Disposal) x 10% (2)
228

W3: Doubtful debts expense (reversal) Rs. m


Closing provision [361 / 95 x 5] 19
Opening provision [418 / 95 x 5] (22)
Decrease (3)
Further decrease due to adjustment W1 (1)
Total reversal in PL (4)

Page 9 of 9
140
Rectification Of Errors
Types of errors:
1) Those errors that do not cause a difference In trail balance agreement.
2) Those errors that do cause a difference in trail balance.

Main Heads of accounts [types of accounts]


• Assets
• Liabilities
• Income
• Expenses
• Capital

1 Errors that do not cause any difference in trail balance include:

Sr # Reasons of errors Rectifying entry


Error of omission:
When financial information completely omitted for Debtors Account 1,000
recording in the books of original entry e.g. Sale invoice of Sales Account 1,000
1
Rs. 1000 issued before closing date but not recorded In the
books of original entry (Sales Journal)

Errors of Commission:
When correct accounting effect (Dr/Cr) is given in the Computer a/c 5,000
wrong accounting head but the main head [means type of Furniture a/c 5,000
2 account] remains correct E.g. Purchase of computer Rs.
5,000 for office was wrongly debited to the furniture
account (Both accounts relate to the assets main head)

Error of Principle:
When correct accounting effect (Dr/Cr) is given in the Computer a/c 5,000
wrong accounting head as well as the wrong main head Salary a/c 5,000
[means type of account]. E.g. Purchase of computer Rs.
5,000 for office was wrongly debited to the salary account,
3
(both accounts belong to the different main head like
computer Is Asset and salary is Expense).
This error will also cause a different in financial
performance and financial position (Asset = Owner’s equity
+ Liability).

Error of Original entry (transposition error):


When the correct accounting entry is recorded in the Debtors a/c 270
4 books of accounts but the amount in both accounting Bad debt a/c 270
effects is wrong although the amount is same: E.g. Debtors
to be written off for Rs. 250 as bad debts were recorded in

Page 1 of 20
141
the correct accounts but the amount was posted in both
accounts was Rs. 520 causing a different of Rs. 270 on both
sides. It Is also known as transposition error

Compensating Error:
When sum of more than One errors contra the accounting
effect with each other. E.g. Sales were less credited with Capital A/c 200
Rs.260 and at the same time opening balance of capital Rent Expenses 60
account was brought forward with an amount that is Rs. Sales 260
200 more than the correct amount and Rent Expense
5 owing of Rs.60 was although credited to rent payable
account but was not debited to the rent expense a/c.

Complete reversal of entry


Means correct accounts and amounts but sides (debit and Cash 200,000
6
credits) reversed. For example; obtained a loan of Rs. Loan 200,000
100,000 from bank recorded as a reverse entry

Possible reasons for Difference in Trial Balance


When sum of debit column of trial balance exceeds sum of credit column the difference named as suspense
account will appear in credit column of trial balance and vice versa (unless errors are corrected).

These types of errors are:


1. Casting error:
Incorrect addition in any account, i.e. under casting or overcasting.

2. Balancing error:
Opening balance not brought down or brought down on wrong side or with wrong amount.

3. Extraction error:
The ledger balance omitted or placed in trial balance at wrong side or with incorrect amount.

4. Posting error:
Part of the transaction not posted or transaction posted with incorrect amount or posting to wrong side of the
account.

5. Error of part omission:


A debit entry has been made but no corresponding credit entry or vice versa i.e. single sided entry.

6. Error of different amounts:


Debit and credit entries have been made but at different amounts.

Page 2 of 20
142
Sr# Reasons of errors Rectifying entry
Under/over casting of a ledger a/c maintained in main ledger
e.g.
• Sales income account was over casted by Rs. 200 Sales A/c 200
1
• Sales Income Account Was under Casted by Rs. 500 Suspense a/c 200
Suspense a/c 500
Sales a/c 500
Omission of a balance from trail balance e.g. balance of a bad
2 debts account Rs. 700 is not appearing in the trail balance. Bad debt a/c 700
Suspense a/c 700
Balance representing an account appearing in trail balance with
less or excess amount e.g. Building a/c balance c/f Rs. 700,000 Building a/c 630,000
3
wrongly appearing in the Trail balance with Rs. 70,000. Suspense a/c 630,000

An account was given debit effect instead of credit effect Suspense a/c 1600
(causing difference with double amounts) e.g Sales of Rs. 800 on Sales a/c 1600
4
credit was correctly debited in debtors a/c but was also debited
in sale a/c mistakenly
An account was given credit effect instead of debit effect
(causing difference with double amounts) e.g Sales of Rs. 800 on Debtor a/c 1,600
5
credit was correctly credited in sales a/c but was also credited in Suspense a/c 1,600
debtors a/c mistakenly
Single accounting effect either Dr or Cr was recorded /posted in
the books of account e.g. sales of Rs.800 on credit was posted in Debtor a/c 800
6
sales a/c only. Suspense a/c 800

One of the accounting effects was given with wrong amount e.g. Debtor a/c 720
7 sales of Rs. 800 on credit was correctly credited in sales a/c but Suspense a/c 720
was wrongly debited to debtors a/c with Rs.80 only.

CORRECTION OF ERRORS

1. GRANT
The accountant of Grant Company has prepared a trial balance, but has found that the total of debit balances is
Rs.864,600 and the total of credit balances is Rs.862,150.
On investigation, he discovers the following errors in the book-keeping:
(1) Total purchases in the period were recorded at Rs.100 below their correct value, although the total value
of trade payables was correctly recorded.
(2) Total telephone expenses were recorded at Rs.1,000 above their correct amount, although the total
value of the amounts payable was correctly recorded.
(3) Purchase returns of Rs.550 were recorded as a debit entry in the sales returns account, but the correct
entry had been made in the trade payables control account.
(4) Equipment costing Rs.2,000 had been recorded as a debit entry in the repairs and maintenance account.

Page 3 of 20
143
(5) Rental expenses of Rs.5,490 were entered incorrectly as Rs.5,940 in the expense account but were
entered correctly in bank account in the ledger.
(6) Bank charges of Rs.200 have been omitted entirely from the ledger.

Required:
Prepare journal entries for the correction of the errors.

Open a suspense account. Record the appropriate corrections in the suspense account, so that the balance on
this account is eliminated.

2. EASTERN PRODUCTS

The following errors and adjustments were discovered:

(i) An invoice of Rs. 3,700 was debited to purchases but the goods were received after year-end and
were not included in the closing inventory.
(ii) Store equipment costing Rs. 8,100 and having a book value of Rs. 3,600 was sold for Rs. 2,500. Cash
was debited and store equipment was credited by Rs. 2,500. No other entries were made.
(iii) A cheque of Rs. 1,850 received from a customer was dishonoured on June 25, 2013 but no entry
was made in the books. Cash there against was received after year-end.
(iv) Purchase of office equipment costing Rs. 15,200 was entered in the purchases account.
Depreciation on office equipment is provided at the rate of 10%.
(v) A purchase invoice of Rs. 197 was debited to the supplier account as Rs. 917.
(vi) Purchase returns book was under-casted by Rs. 650.
(vii) The opening balance of furniture account was brought forward as Rs. 18,300 instead of Rs.13,800.
Depreciation on furniture is provided at the rate of 10%.
(viii) A balance of Rs.730 in the debtor account is to be offset against a balance of Rs. 880 in the creditor
account.

The profit for the year was calculated at Rs. 956,180 before the correction of above errors..

Required:
(a) Prepare journal entries to adjust the above items.
(b) Recalculate the net profit for the year.

Page 4 of 20
144
Answers:
1. GRANT

Transaction Debit Credit

Rs. Rs.
1 Purchases 100
Suspense account 100
Correction of error: purchases under-stated by Rs.100.
2 Suspense account 1,000
Telephone expenses 1,000
Correction of error: telephone expenses over-stated by Rs.1,000.
3 Suspense account 1,100
Purchase returns 550
Sales returns 550
Correction of error. Purchase returns of Rs.550 incorrectly recorded as a debit entry in sales returns.

4 Equipment 2,000
Repairs and maintenance 2,000
Correction of error. Equipment purchase costs incorrectly recorded as repairs and maintenance
expenses

5 Suspense account 450


Rent expenses 450
Correction of error: rent expenses over-stated by Rs.450.

6 Bank Charges 200


Bank account 200

Transaction omitted from the ledger.

Suspense account
Rs. Rs.
Telephone expenses 1,000 Opening balance 2,450
Purchase returns 550 (864,600 – 862,150)
Sales returns 550 Purchases 100
Rent expenses 450

2,550 2,550

Page 5 of 20
145
2. EASTERN PRODUCTS

(a) Debit Credit


1 Creditor account 3,700
Purchases 3,700
2 Accumulated Depreciation 4,500
Loss on sale of store equipment 1,100
Store equipment a/c 5,600
3 Debtors a/c 1,850
Bank 1,850
4 Office Equipment 15,200
Purchases 15,200
Depreciation expense 1,520
Accumulated depreciation – office
equipment 1,520

5 Suspense a/c 1,114


Creditor a/c (197 + 917) 1,114
6 Creditors 650
Purchase Returns 650
7 Suspense a/c 4,500
Furniture a/c 4,500
Accumulated Depreciation 450
Depreciation expenses 450
8 Creditor a/c 730
Debtors a/c 730

(b) Recalculation of net profit for the year


Net profit as per question 956,180

Add: Reversal of purchases 3,700


Equipment wrongly debited to purchases 15,200
Purchase returns 650
Reversal of depreciation on furniture 450
976,180
Less: loss on sale of store equipment (1,100)
Depreciation on office equipment (1,520)
Adjusted net-profit 973,560

Page 6 of 20
146
Extra practice questions
Q1. Draft income statement of Timothy Enterprises (TE) for the year ended 31 December 2017 shows gross
profit of Rs. 850,000 and net profit of Rs. 460,000.

During the review of the financial statements, following errors were noticed:

(i) An invoice of Rs. 3,700 was debited to purchases but the goods were received after year-end and
were not included in the closing inventory.

(ii) Transportation inward amounting to Rs. 2,000 was included in transportation outward.

(iii) The sub-total of a closing stock sheet had been carried forward as Rs. 21,830 instead of Rs. 21,380.

(iv) A receipt of Rs. 21,850 was credited to sales. The amount was received from a debtor as full and
final settlement of an outstanding balance of Rs. 23,000.

(v) Goods having sales value of Rs. 4,500 were used for office repairs. No entry has been made in the
books.

(vi) Purchase of office computer on 1 April 2017 amounting to Rs. 42,000 was entered in the purchase
account.

(vii) An item of furniture was sold on credit for Rs. 3,000 and entered in the sales day book. The book
value of this item was Rs. 5,000.

(viii) Purchase return amounting to Rs. 6,700 has been recorded as sales return.

(ix) The owner had withdrawn goods costing Rs. 4,680 for personal use. No entry has been made in the
books.

TE uses periodic inventory method. Goods are sold at cost plus mark up of 25%.
Depreciation on office computer is provided at the rate of 25%.

Required:
Compute the corrected gross profit and net profit for the year. (14)

Page 7 of 20
147
Ans.
Effect on Gross Profit
Given Profit 850,000 Cr
Purchases 3,700 Cr
Carriage Inwards 2,000 Dr
Cost of sales 450 Dr
Sales 21,850 Dr
Discount Allowed 1,150 Dr
Purchases 3,600 Cr
Purchases 42,000 Cr
Sales 3,000 Dr
Purchase return 6,700 Cr
Sale return 6,700 Cr
Purchases 4,680 Cr
Adjusted Gross Profit 888,930 Cr

Effect on Net Profit


Given Profit 460,000 Cr
Effect of change in gross profit [850,000 – 38,930 Cr
888,930]
Carriage Outward 2,000 Cr
Office Repairs 3,600 Dr
Depreciation 7,875 Dr
Loss 2,000 Dr
Adjusted Net Profit 487,455 Cr

Note:
Increase in closing stock increases the Gross Profit because of decrease in cost of sale while decrease in closing
stock decrease the gross profit because of increase in cost of sale.

Workings:
1 Creditor 3,700
Purchases 3,700
2 Carriage Inwards 2,000
Carriage outwards 2,000
3 Cost of sales 450
Stock 450
4 Sales 21,850
Discount Allowed(sales) 1,150
Debtor 23,000
5 Office Repairs 3,600
Purchases 3,600

Page 8 of 20
148
6 Office Computer 42,000
Purchases 42,000
Depreciation 7,875
Acc. Depreciation 7,875
7 Posted:
Debtor 3,000
Sale 3,000
Required:
Receivable 3,000
Loss 2,000
Furniture 5,000(WDV)
Rectifying:
Sale 3,000
Debtor 3,000
Receivable 3,000
Loss 2,000
Furniture 5,000(WDV)

8 Suspense 13,400
Purchase return 6,700
Sale Return 6,700
9 Drawings 4,680
Purchases 4,680

Q.2 Financial statements of Zeta Traders (ZT) for the year ended 30 June 2019 is under preparation. Following
information has been gathered in this respect:

(i) Trade receivables as at 30 June 2019:

Rupees
Trade receivables 2,500,000
Provision for doubtful debts (400,000)
Net trade receivables 2,100,000

It was noted that:


▪ an old outstanding balance of Rs. 250,000 which was written off previously was settled
during the year at 20% discount. The amount received was credited to trade receivables.
▪ purchase return amounting to Rs. 500,000 was mistakenly debited to trade receivables
instead of trade payables.
▪ Rana and Sons having a balance of Rs. 80,000 due for more than one year was declared
bankrupt and its balance needs to be written off.

Page 9 of 20
149
ZT maintains provision for doubtful debts:
▪ at 25% for balances outstanding for more than six months. As at 30 June 2019, such
balances are aggregated to Rs. 600,000 (excluding balance of Rana and Sons); and

▪ at 5% for the remaining balances

(ii) A cheque dated 25 June 2019 for Rs. 150,000 was received from an insurance company and deposited by
the owner in his personal bank account. The cheque was received in settlement of an inventory loss claim.
Actual inventory loss was determined at Rs. 180,000. No entries have been made for loss of inventory and
insurance claim.
(iii) The opening balance of accumulated depreciation was brought forward as Rs. 280,000 instead of Rs.
820,000. The error was tried to be corrected with the difference by crediting accumulated
depreciation and debiting depreciation expense.
(iv) Goods amounting to Rs. 350,000 received from a supplier on 30 June 2019 were included in the year-
end physical inventory count but recorded in purchases day book on 1 July 2019.
(v) Third party stock of Rs. 500,000 lying on ZT premises has been included in ZT’s year-end inventory.
(vi) ZT uses periodic inventory method.

Required:

a) Prepare adjusting / correcting entries for the year ended 30 June 2019. (Narrations are not required)
b) Compute the net effect of the above on ZT’s profit for the year ended 30 June 2019

A.2 (a) Zeta Traders


Adjusting/correcting entries
General journal
Description Debit Credit
------ Rupees ------
(i) Trade receivable (250,000 x 80%) 200,000
Bad debt 200,000
OR
Trade receivable (250,000 x 80%) 200,000
Discount allowed 50,000
Bad debts 250,000
Trade payable 500,000
Trade receivable 500,000
Bad debts 80,000
Trade receivable 80,000
Allowance for doubtful debts (W-1) 174,000
Bad debts 174,000

(ii) Abnormal loss 180,000


Purchases 180,000
Drawings 150,000

Page 10 of 20
150
Abnormal loss 150,000
(iii) Suspense (820,000–280,000) 540,000
Depreciation expense 540,000
(iv) Purchases 350,000
Trade payables 350,000
(v) Cost of sales 500,000
Closing inventory 500,000

Debtors
Unadjusted b/d 2,500,000 Creditors 500,000
Bad debts 200,000 Bad debts 80,000
c/d 2,120,000

Allowance
Allowance 174,000 Unadjusted b/d 400,000
c/d (W.1) 226,000

W-1
1.Specific allowance
600,000 x 25% 150,000
2.Closing debtors 2,120,000
Less balance on which specific allowance is created (600,000)
Remaining Debtors 1,520,000
3. General allowance @5% 76,000
4.Total allowance (150,000 + 76,000) 226,000

(b)
Increase / decrease in net profit
Recovery of old outstanding balance previously written off 200,000 Cr.
Bad debts 80,000 Dr
Bad debts 174,000 Cr
Loss of goods (180,000-150,000) 30,000 Dr
Purchases 180,000 Cr
Depreciation expense 540,000 Cr
Purchases 350,000 Dr
Cost of sales 500,000 Dr
Net effect on profit 134,000 Cr

Page 11 of 20
151
Test Questions Rectification of errors:
1.The accountant of BA Enterprises prepared a statement of comprehensive income for the year ended December
31, 2013 which showed gross profit of Rs. 1,050,000 and net profit of Rs. 650,000. The company sells goods at cost
plus mark-up of 20%.

The following errors/omissions were found on a detailed review of the financial statements.
1. Items not included in the statement of comprehensive income:
• Free samples costing Rs. 25,000 were sent to potential and regular customers.
• Goods costing Rs. 10,000 were taken by the owner for personal use and goods having sales value of Rs. 2,500
were used for office repairs.
• Unpaid salaries and transportation (inward) expenses payable, amounting to Rs. 20,000 and Rs. 10,000
respectively.
2. Old furniture items were sold on credit for Rs. 3,000 and entered in the sales day book. The bookvalue of
these items was Rs. 2,000.
3. Rs. 24,500 were paid to a creditor as full and final settlement of an amount of Rs. 25,000 and debited to
purchases.
4. The sales day book was overcast by Rs. 30,000.
5. An amount of Rs. 67,000 was carried forward in the purchase day book as Rs. 6,700.

Required:
Ascertain the correct amount of gross and net profit for the year.

Answer 1: BA Enterprises
Effect on Gross Profit
For the year ended 31-12-2013

Gross Profit ( Given ) 1,050,000 Cr.


Purchases 25,000 Cr.
Purchases 10,000 Cr.
Purchases 2,083 Cr.
Transportation Inwards 10,000 Dr.
Sales 3,000 Dr.
Purchases 24,500 Cr.
Discount received (Purchases) 500 Cr.
Sales 30,000 Dr.
Purchases 60,300 Dr.
Adjusted Gross profit 1,008,783 Cr.
Effect on Gross Profit:
Given Net Profit 650,000 Cr.
Effect of change in gross profit [ 1,050,000 – 1,008,783] 41,217 Dr.
Free samples 25,000 Dr.
Office Repairs 2,083 Dr.
Salaries 20,000 Dr.
Gain on disposal 1,000 Cr.
Adjusted Net Profit 562,700 Cr.

Page 12 of 20
152
Workings:
Dr. Cr.
1.
i) Free Samples [in operating expenses] 25,000
Purchases 25,000
ii) Drawings 10,000
Purchases 10,000
Office Repairs 2,083
Purchases 2,083
[ 2,500 / 120 x 100 ]
iii) Salaries 20,000
Payable 20,000
Transportation Inwards 10,000
Payable 10,000
2. Sales 3,000
Debtors 3,000
Receivable 3,000
Furniture 2,000
Gain (P.L) 1,000
3. Creditor 25,000
Purchases 24,500
Discount Received (purchases) 500
4.Sales 30,000
Debtor 30,000
5.Purchases 60,300
Creditor 60,300

2. The bookkeeper has produced the following statement of financial position at 31 December for Smetena’s
Newsagents.

Rs. Rs.
Non-current assets 72,208
Current assets
Inventory 18,826
Trade receivables 26,216
Drawings 8,260
Suspense account 3,830
Cash 700
57,832

130,040

Capital account 50,224

Loan – L Franks 12% 20,000


Trade payables 26,782

Page 13 of 20
153
Bank overdraft 14,634
Profit for year 18,400
130,040

Jan Smetena, the proprietor, is unhappy with the statement of financial position and asks you to revise it. You
discover the following.
1. The suspense account balance represents the difference on the trial balance.
2. The purchases day book total for October of Rs.4,130 was posted to the purchasesaccount as Rs.4,310
although the correct entry was made to the payables ledger control account.
3. Inventory sheets were overcast by Rs.2,000.
4. Cash should be Rs.110.
5. Fixtures and fittings account balance of Rs.4,600 has been omitted from the trialbalance.
6. Interest for a half year on the loan account has not been paid and no provision hasbeen made for it.

Required:
a) Show the journal entries to correct the above errors.
b) Write up the suspense account.
c) Draw up a revised statement of financial position at 31 December. Clearly show theadjustments to profit.

Answer:
Dr.bit Cr.
Suspense a/c 180
Purchases 180
(2) Correction of amount posted to purchases a/c arising from transposition error
Cost of Sales 2,000
Stock 2,000
(3) Correction of overcasting of Inventory-Sheets
Suspense a/c 590
Cash a/c 590
(4) Correction of overstatement of cash in hand
Fixtures & Fittings 4,600
Suspense a/c 4,600
(5) Correction of omission from the trial balance of fixtures & fittings
Interest a/c 1,200
Interest Payable 1,200
(6) Provision for interest due on loan not yet provided for [20,000 x 12% x 6/12]

Page 14 of 20
154
(b) Suspense account
Rs. Rs.
DIFFERENCE IN TB 3,830 Fixtures & Fittings omitted from TB (5) 4,600
PURCHASES (2) 180
CASH IN TB (4) 590

4,600 4,600

(c) Statement of financial Position at 31 December

Rs. Rs.
Non-Current Assets (72,208+4,600) 76,808
Current Assets :
Inventory(18,826-2,000) 16,826
Trade Receivables 26,216
Cash(700-590) 110
43,152
Total assets 119,960

Opening capital 50,224


Add: Net Profit (working below) 15,380
65,604
Less: Drawings (8,260)
57,344
Non-Current Liabilities
Loan – L Franks 20,000
Current Liabilities
Bank Overdraft 14,634
Trade Payables 26,782
Interest Payable 1,200

Total Capital and Liabilities 119,960


WORKING
Adjustment to Profit
Profit per draft statement of financial position 18,400
Less: Overstated closing inventory (3) (2,000)
Interest on loan a/c (6) (1,200)
Add: Overstated Purchases (2) 180
15,380

Page 15 of 20
155
Question 3
Following draft statement of financial position as on 31 December 2019 of Naltar Establishment (NE) is under
review:

Assets Rs. in ‘000 Equity & liabilities Rs. in ‘000


Fixed assets – net 22,590 Opening capital 32,240
Current assets: Net profit for the year 9,360
Stock 15,320 41,600
Trade receivables 19,730 Current liabilities:
Drawings 1,400 Trade payables 17,332
Cash & bank 3,850 Other payables 2,680
40,300 Suspense account 798
Discount received 480
21,290
62,890 62,890

The following matters are identified:


(i) Goods costing Rs. 5,800,000 received on 31 December 2019 were included in the year-end physical count.
However, these goods were recorded in purchases day book on 5 January 2020 on receipt of the invoice.
(ii) Year-end physical count sheets include a third party stock of Rs. 1,320,000.
(iii) Goods sold on credit at a trade discount of 10% were recorded at gross amount of Rs. 6,400,000.
(iv) An unidentified credit of Rs. 294,000 appearing in the bank statement was accounted for in the suspense
account. It was discovered that the credit was a settlement of an old outstanding balance previously written
off. The amount was net of 2% bank charges.
(v) A cheque of Rs. 500,000 issued by a customer as an advance was dishonored and returned by the bank on
31 December 2019. No entry was made on return of cheque.
(vi) Operating expenses include insurance premium of Rs. 900,000 paid during the year, out of which Rs.
200,000 pertain to owner's residential premises. The policy is valid up to 30 June 2020.
(vii) An up gradation of a plant to improve quality and efficiency was completed on 1 July 2019 at a cost of
Rs. 2,500,000. The cost was charged to repair and maintenance expense.
(viii) Total sales of 26 December 2019 as per sales day book was Rs. 167,000. This was posted in trade receivable
control account as Rs. 671,000. Trial balance was balanced by taking the difference to the suspense account.

Other information:
NE uses periodic stock method. The plant is depreciated at 20% using diminishing balance method.

Required:
Prepare corrected statement of financial position as on 31 December 2019. (17)

Page 16 of 20
156
A.3 Naltar Establishment
Statement of financial position as at 31 December 2019
Rs. in '000
Assets
Fixed assets 22,590+2,500–250 24,840
Current assets:
Closing stock 15,320–1,320 14,000
Trade receivables 19,730–640–504 18,586
Prepaid Insurance 350
Cash & bank 3,850–500 3,350
36,286
61,126

Equity & liabilities


Opening capital 32,240
Net profit for the year (W-1) 5,174
Drawings 1,400+200 (1,600)
Net equity 35,814
Current liabilities:
Trade payables 17,332+5,800 23,132
Other payables 2,680–500 2,180
25,312
61,126

W-1: Corrected net profit for the year Rs. in '000


Balances as per draft financial statements (given) 9,360 Cr.
Corrections:
(i) Purchases of Dec. 2019 recorded in Jan. 2020 (5,800) Dr.
(ii) Cost of Sales (1,320) Dr.
(iii) Goods sold recorded at gross of 10% trade discount (640) Dr.
(iv) Clearance of unidentified credit taken to suspense account:
- Bank charges (6) Dr.
- Recovery of previously written off balance 300 Cr.
(vi) Insurance premium paid pertains to:
- Owner 200 Cr.
- Insurance 350 Cr.
(vii) Upgradation cost of a plant charged to repair and maintenance:
- Repair and maintenance expense 2,500 Cr.
- Depreciation expense (2,500×20% x6/12) (250) Dr.
Discount received(already appearing in liabilities) 480 Cr.
Revised net profit 5,174

Page 17 of 20
157
W-2
Purchases 5,800
Creditor 5,800
Cost of sale 1,320
Sock 1,320
Sales 640
Debtor 640
[6,400 x 10%]
Bank 294 Bank 294 Suspense 294
Bank charges 6 Suspense 294 Bank Charges 6
(294/98 x 2)
Bad debts 300 Bad debts 300
(Required) (Posted) (Rectifying)
Advance from customer 500
Bank 500
Drawings 200
Operating expense 200
350
Prepaid Insurance
Operating expense 350
[700 x 6/ 12]
Fixed assets 2,500
Repair and maintenance 2,500
Depreciation 250
Acc. Depreciation 250
(2,500 x 20% x 6/12)
Debtor 167 Debtor 671 Suspense 504
Sales 167 Sales 167 Debtor 504
Suspense 167
(Required) (Posted) (Rectifying)

Suspense a/c
Rs. Rs.
Original balance 798 1,030
Bad debts 294
Debtors 504
798 798

Page 18 of 20
158
Rectification of errors:
Q. While reviewing the draft financial statements of Sky Electronics (SE) for the year ended 31 December 2017,
following errors have been identified:
I. Computers costing Rs.240,000 purchased on 1 September 2017 for office use were debited to purchases
account. SE depreciates computers at 20% per annum using straight line method.
II. Furniture costing Rs. 1,200,000 and having a book value of Rs. 670,000 as on 31 December 2017 had already
been sold on 1 November 2017. The proceeds of Rs. 700,000 were credited to sales. SE depreciates furniture
at 10% per annum using straight line method.
III. On 1 April 2017, SE rented-out one of its premises at an annual rent of Rs. 900,000 payable in advance. The
rent received was credited to income.
IV. Trade receivables include a balance of Rs. 180,000 which is irrecoverable but has not been written-off.
Further, a recovery of Rs. 96,000 against receivables written off in prior years was credited to trade
receivables. As per SE's policy, provision for doubtful receivables has already been made at 5% on year-end
balance.
V. A cheque of Rs. 192,000 was received after a discount of 4% from a customer. However, in the cash book, the
amount received was entered in the discount allowed column and the amount of discount was entered in the
bank column.

Required:
Prepare rectification entries to correct the above errors. (Narrations are not required)

Page 19 of 20
159
A. (a)
Sky Electronics

Debit Credit
Date Description
Rupees

(i) Fixed assets (Computers) 240,000


Purchases 240,000
Depreciation expense [240,000×0.2÷12×4] 16,000
Accumulated depreciation 16,000
(ii) Accumulated depreciation (1,200,000×0.1×2÷12) 20,000
Depreciation expense 20,000
Sales 700,000
Accumulated depreciation [(1,200,000-670,000) - 20,000] 510,000
Fixed assets-Furniture
1,200,000
Gain on disposal (Balancing figure) 10,000
(iii) Rent income (900,000÷12×3) 225,000
Unearned rent income 225,000
(iv) Bad debt expense 180,000
Trade receivables 180,000
Trade receivable 96,000
Bad debt expense/Bad debt recovered 96,000

Provision for doubtful (180,000-96,000)×5% 4,200


Bad debt expense
(Reversal of allowance) 4,200
*(v) Bank 192,000- (192,000÷0.96×0.04) 184,000
Discount allowed (sales) 184,000

Accounting entries for correction of the errors

* Required Posted
Bank 192,000 Bank 8,000
Disc. Allowed 8,000 Disc. Allowed 192,000
(192,000/96 x 4) Debtor 200,000
Debtor 200,000

Page 20 of 20
160
Rectification of Errors
Example:
Following information has been extracted from the draft financial statements of SmartEstablishment (SE) for the
year ended 31 December 2022:

Statement of profit or loss Rs. in '000


Revenue 3,500
Cost of sales (2,000)
Gross profit 1,500
Administrative expenses (800)
Selling expenses (550)
Operating profit 150
Other expenses (60)
Other income 200
Net profit 290

During the review following matters were identified:


i. Discounts received of Rs. 32,000 in January 2022 have been posted to the debit of the discounts allowed
account.
ii. Salaries of Rs. 29,000 paid in February 2022 have not been posted from the cash book.
iii. A remittance of Rs. 94,000 received from Tariq (a customer) in November 2022 has been posted to the credit of
Babar (another customer).
iv. In December 2022, SE took advantage of an opportunity to purchase a large quantity of stationery at a bargain
price of Rs. 20,000. No adjustments have been made in the accounts for the fact that three-quarters, in value,
of this stationery was in stock at year end.
v. A payment of Rs. 41,000 to Waqas (a supplier) in January 2022 has been posted in his personal account as Rs.
14,000.
vi. A remittance of Rs. 30,000 received from Naveed (a customer) in April 2022 has been credited to sales.

SE does not maintain control accounts for receivables or payables.

Required: Pass correcting entries and prepare corrected statement of profit or loss for the year ended 31 December
2022.

Page 1 of 2
161
Correcting entries

Sr. # Particulars Debit Credit


Rs. 000 Rs. 000
(i) Suspense account 64
Discount received 32
Discount allowed 32
(ii) Salaries expense 29
Suspense account 29
(iii) Babar (Receivable) 94
Tariq (Receivable) 94
(iv) Office supplies (stationery) stock 15
Office supplies expense [20 x ¾] 15
(v) Waqas (Payables) [41 – 14] 27
Suspense account 27
(vi) Sales 30
Naveed (Receivables) 30

Statement of profit or loss (Corrected) Rs. in '000


Revenue [3,500 + 32 (i) – 30 (vi)] 3,502
Cost of sales [2,000 – 32 (i)] (1,968)
Gross profit 1,534
Administrative expenses [800 + 29 (ii) – 15 (iv)] (814)
Selling expenses (550)
Operating profit 170
Other expenses (60)
Other income 200
Net profit 310

Page 2 of 2
162
Earnings per share (IAS 33)
Introduction
‘Earnings per share’ is a ratio used in the financial analysis of a set of financial statements. This ratio is, however,
so useful and popular that the standard, IAS 33, had to be developed to control the method of calculation
thereof. This standard sets out how to calculate:
• the numerator: earnings; and
• the denominator: the number of shares

Objective of IAS 33
The objective of IAS 33 is to set out principles for:

the calculation of EPS; and

the presentation of EPS in the financial statements.

The purpose of standardizing the calculation and presentation of EPS is to make it easier for the users of financial
statements to compare the performance of:

different entities in the same reporting period; and

the same entity for different reporting periods over time.

Scope of IAS 33
IAS 33 applies only to publicly-traded entities or those which are about to be publicly traded. A publicly-traded
entity is an entity whose shares are traded on a stock exchange.

Price earning (P/E) RATIO AND EARNINGS PER SHARE (EPS)


Earnings per share
Earnings are profits available for equity holders (ordinary shareholders). Earnings per share (EPS) is a measure
of the amount of earnings in a financial period for each equity (ordinary) share.

As its name implies, EPS is calculated as earnings divided by the number of ordinary shares in issue.

The price/earnings ratio


The price/earnings ratio (P/E ratio) is also a key stock market ratio. It is a measure of the company’s current
share price (market price) in relation to the EPS. The P/E ratio is calculated as follows:

Formula: Price earnings ratio

P/E ratio = Market value of share / Earnings per share


= 46 / 23 = 2 times

Interpretation: This ratio shows the payback period of investment.

EPS is used by investors as a measure of the performance of companies in which they have invested or might
possibly invest. Investors are usually interested in changes in a company’s EPS over time.

Page 1 of 29
163
EPS should therefore be calculated by all companies in a standard way, so that investors can obtain a reliable
comparison between the EPS and P/E ratios of different companies.

Definition of ordinary share


An ordinary share is an equity instrument that is subordinate to all other classes of equity instruments.

The ordinary shares used in the EPS calculation are those entitled to the residual profits of the entity, after
dividends relating to all other shares have been paid.

Types of shareholders
Ordinary shareholders
Ordinary shareholders buy a share in a company to earn dividends, (when this payment is considered prudent)
and for capital growth. These dividends fluctuate annually depending on profits and available cash reserves etc.
As the terms ‘ordinary’ and ‘preference’ implies, the ordinary shareholders have fewer rights than the
preference shareholders. By way of illustration, assume that a company with both preference and ordinary
shareholders is liquidated (wound up): the preference shareholders will have their capital returned first and only
if there are sufficient funds left over, will the ordinary shareholders have their capital paid out.

Preference shareholders
Preference shareholders have more rights than ordinary shareholders. Not only do they have preference on
liquidation, but they also have a fixed amount paid out each year in dividends (as opposed to ordinary
shareholders whose dividends are at the discretion of the entity and are largely dependant on profits and
available cash reserves). The rate of dividends paid out is based on the preference share’s rate (e.g. 10%). A
shareholder owning 1 000 preference shares with a par value of 2 each and a rate of 10% will expect dividends
of 200 per year (2 x 1 000 x 10%). The shareholder’s rights to dividends depend on whether his shares were:
• cumulative; or
• non-cumulative.

Cumulative preference shares indicate that if a dividend was not paid out in a particular year, (perhaps due to
insufficient funds), these arrear dividends must be paid out in the future to the holders of these shares when
funds become available. No dividend may be paid out to ordinary shareholders until the arrear preference
dividends have been paid. Non-cumulative preference shares are those where, if a dividend is not paid out in a
year, these unpaid dividends need never be paid.

There is a further variation with regard to preference shares; the shares may be:
• redeemable; or
• non-redeemable(Irredemable)

Redemption of a share involves the company returning the amount invested by the shareholder to this
shareholder at some stage in the future. Shares that are redeemable should be classified as a liability instead of
as equity, in which case the related dividends will be recognised as ‘finance charges’ in the statement of
comprehensive income instead of as ‘dividends’ in the statement of changes in equity (as in case of ordinary
shares)

Page 2 of 29
164
Point to remember:
As suggested already, some preference shares are recognised as liabilities rather than as equity and their
dividends are recognised as finance charges instead of as dividends. In these instances, even if the dividend has
not yet been declared as at the end of the reporting period, the dividend will be recognised as a finance charge
on accrual basis (just like interest expense on a loan)

There are two types of Earnings per share


1. basic earnings per share
2. diluted earnings per share

1. Basic earnings per share

Earnings
Number of shares
Detailed Formula: Basic EPS

Net profit (or loss) attributable to ordinary shareholders during a period


Weighted average number of ordinary shares in issue during the period

Important points to remember:


• This profit should be before deducting ordinary dividend (if any)
• In the event that the entity reports a loss instead of a profit, the earnings per share will simply be reported
as a loss per share instead.
• This ratio is different from dividend per share.

Basic earnings

Profit (or loss) for the period (Profit (or loss) after tax xxx
Less preference dividends related to irredeemable preference shares (based on (xxx)
Profit attributable to ordinary shareholders
the rate) xxx

Preference dividends are, in fact, not always deducted. Deciding whether or not to deduct the preference
dividends depends on whether the shares are cumulative or non-cumulative. The following guidelines should be
helpful when dealing with irredeemable preference shares:
• in respect of non-cumulative preference shares, deduct only the preference dividends that are declared in
respect of that period; and
• in respect of cumulative preference shares, deduct the total required preference dividends for the period
(in accordance with the preference share’s rate), whether or not these dividends have been declared.

It should be borne in mind that where the preference shares are classified as a liability in case of redeemable
preference shares, their dividends would be treated as finance costs, in which case these dividends would have
already been deducted in the calculation of ‘net profit for the period’. These must obviously not be deducted
again when calculating ‘earnings attributable to the ordinary shareholders’.

Page 3 of 29
165
Note: It is always assumed before the calculation of EPS that all accounting entries should have been correctly
made and recorded in the financial statements, unless there is any indication.

A. When there are only ordinary shares


If there are only ordinary shareholders, the entire profit or loss of the company belongs to the ordinary
shareholders (owners).
Example 1: ordinary shares only
A company has 10 000 ordinary shares in issue throughout 2011. The company earns a profit after tax of 100 000.

Required:
Calculate the basic earnings per ordinary share.
Answer:
Calculation of the earnings per ordinary share:

= Earnings
Number of ordinary shares

= 100 000
10 000

= 10 per share

B. When there are ordinary and preference shares


If there are both ordinary and preference shareholders, some of the profit for the year must be set aside for
the preference shareholders’ preference dividends.

Example 2: ordinary and preference shares


A company has 10 000 ordinary shares and 10 000 non-cumulative, non-redeemable 10% Rs. 2 each
preference shares in issue throughout 2011.
The company earns a profit after tax of 100 000.
The company declared the full 2011 dividends owing to the preference shareholders.
Required: Calculate the basic earnings per ordinary share.

Answer: Calculation of earnings attributable to ordinary shareholders:


Profit (or loss) for the year 100 000
Less fixed preference dividends (10 000 x 2 x 10%) (2 000)*
Earnings attributable to ordinary shareholders 98 000
*if this dividend would not have been declared then it would be ignored because preference shares are non
cumulative.
Calculation of the earnings per ordinary share:

= Earnings belonging to ordinary shareholders


Number of ordinary shares
= 98 000
10 000

= 9.80 per share

Page 4 of 29
166
Example 3: preference shares and preference dividends – equity versus liability

A company has 10 000 ordinary shares and 10 000 10% Rs. 2 each preference shares in issue
throughout 2012. The profit after tax was 100 000 in 2012.
Required:
Calculate the basic earnings in 2012, assuming that the preference shares are:
A) non-cumulative and non-redeemable (i.e. equity) and the dividend is declared.
B) non-cumulative and non-redeemable (i.e. equity) and the dividend is not declared.
C) cumulative and redeemable (i.e. liability) and the dividend is declared.
D) cumulative and redeemable (i.e. liability) and the dividend is not declared.

Answer: 3A Calculation of earnings attributable to ordinary shareholders:


Profit (or loss) for the year 100 000
Less preference dividends: 10 000 x 2 x 10% (2 000)
Earnings attributable to ordinary shareholders 98 000
3B Calculation of earnings attributable to ordinary shareholders:
Profit (or loss) for the year 100 000

3C Calculation of earnings attributable to ordinary shareholders:


Profit (or loss) for the year 100 000
Less preference dividends (see comment below) (0)
Earnings attributable to ordinary shareholders 100 000

Comment: Preference shares that are cumulative and redeemable are treated as liabilities. The
dividends on these preference shares are therefore recognized as interest. This dividend has therefore
already been deducted in calculating the profit for the period.

3D Calculation of earnings attributable to ordinary shareholders:


Profit (or loss) for the year 100 000
Less preference dividends (see comment below) (0)
Earnings attributable to ordinary shareholders 100 000

Comment: Preference shares that are cumulative and redeemable are treated as liabilities. The
dividends on these preference shares are therefore recognized as interest, irrespective of whether or
not the dividend has been formally declared. This dividend should therefore already been deducted on
accrual basis in calculating the profit for the period.

Example: Cumulative preference shares


In the year ended 31 December 2011, Entity G made profit after tax of Rs. 3,500,000.
Entity G has Rs. 1,000,000 10% cumulative irredeemable preference share capital in issue. (This
would entitle investors to receive a dividend of Rs. 100,000 (10% of Rs. 1,000,000)
Entity G had 1 million ordinary shares in issue throughout the year.

Entity G’s basic EPS for the year ended 31 December 2011 is calculated as follows:

Page 5 of 29
167
EPS = 3,500,000 – 100,000 / 1,000,000 = 3.4 per share

Note that Rs. 100,000 deducted above would be deducted irrespective of whether a dividend had
been declared or not. However, if these preference shares had been non-cumulative then the
dividend would have been deducted only in case of declaration by the company.
Basic number of shares
Overview
In the event that there was no movement of shares during the year, (i.e. the balance of shares at the
beginning of the year equals the balance of shares at year-end, say 10 000), then the denominator in
the earnings per share calculation is simply 10 000 shares.

If, however, there was movement in the number of shares during the year, then the number of shares
to be used in the calculation will need to be adjusted or weighted. The movement could result into an
increase or a decrease in the number of shares.

Increase in number of shares could come in the form of:


• issue for value (e.g. shares issued at their market price);
• issue for no value (e.g. bonus shares and share split); and
• combination issue (e.g. shares issued at less than their market value). This share issue is also
called as right shares.
Decreases in the number of shares could come in the form of:
• share buy-backs/repurchase of shares (a for-value reduction); and
• share consolidations (a not-for-value reduction).

Each of these types of movements will now be dealt with separately.

1.Issue for value (means shares issued at full market value)


Time apportionment to find weighted average shares
On 1 January 2020 a company had 5,000,000 ordinary shares in issue. On 1 April 2020, 1,000,000
new shares were issued.
On 1 July 2020 an extra 1,000,000 shares came into existence. On 1 November 2020 500,000 more
shares were issued.
Assume all issues were at full market price.
The weighted average number of shares is calculated as follows.
Weighted
Number of Time average
Date shares factor number
1 January 5,000,000 X 12/12 5,000,000
New issue on the 1 April 1,000,000 X 9/12 750,000
New issue on the 1 July 1,000,000 X 6/12 500,000
New issue on the 1 November 500,000 X 2/12 83,333

7,500,000 6,333,333

Page 6 of 29
168
Practice question 1
Company B has a financial year ending 31 December.

On 1 January 2013, there were 9,000,000 ordinary shares in issue.

On 1 May 2013, Company B issued 1,200,000 new shares at full market price.

On 1 October 2013, it issued a further 1,800,000 shares, also at full market

Total earnings in 2013 were Rs. 36,900,000.

Required
Calculate the EPS for the year to 31 December 2013.

SOLUTION
Weighted
Number of Time average
Date shares factor number
1 January 9,000,000 × 12/12 9,000,000
New issue on 1 May 1,200,000 × 8/12 800,000
New issue on 1 October 1,800,000 × 3/12 450,000

12,000,000 10,250,000
EPS = Rs. 36,900,000/10,250,000 = Rs. 3.6

2.Issue for no value (bonus shares and share split)


Issues for no value involve an entity effectively giving away shares. Examples of this include capitalization issues
(bonus issues or scrip issue) and share splits (will be discussed later on). Capitalization issues frequently occur
when a company has a shortage of cash with the result that shares are issued instead of paying cash dividends
to the shareholders.

Since there has been no increase in capital resources (there is no cash injection), an increase in profits cannot
be expected. If the earnings in the current year are the same as the earnings in the prior year and there is an
increase in the number of shares in this current year as a result of bonus shares or share split, the earnings per
share in the current year will, when compared with the earnings per share in the prior year, indicate
deterioration in the efficiency of earnings relative to the available capital resources. Comparability would thus
be jeopardized (adversely affected) unless an adjustment is made.

The adjustment made for an ‘issue for no value’ is made to the prior year, (note: an ‘issue for value’ is adjusted
for in the current year based on time). This adjustment has the effect that it appears that the shares issued in
the current year had already been in issue in the prior periods. This adjustment is not time weighted as well as
like issue for value.

Page 7 of 29
169
Example 8: issue for no value
A company had 10 000 ordinary shares in issue during the previous year. There was a capitalisation issue
of 10 000 ordinary shares during the current year. The earnings in the previous year were 20 000, and
thus the earnings per share in the previous year were 2 per share (20 000/ 10 000 shares). Earnings in
current year are 30,000.

Required:
Calculate EPS of current year.

Answer: Although the capital base doubled in the current year, the resources available to the entity
remained the same and thus the user could not reasonably expect an increase in profits.

The earnings per share in the current year will be disclosed at 1.50 per share (30 000/ 20 000 shares) and
the earnings per share in prior period presented will be restated: the prior period will disclose an EPS of 1
per share (20 000/ 20 000 shares (10,000 + 10,000)).

Point to remember:
Please note that the adjustment is not time-weighted and therefore issues for no value made during the
year, (either at the beginning or end of the year or at any other point), are all dealt with in the same way (by
increasing the current period shares and adjusting the prior period number of shares).

Share split (example can be Rs. 1,000 note divided into 10 notes of Rs. 100) is a situation where a share is
subdivided into two or more shares. E.g. denomination of share is reduced (e.g. Govt. has changed the
denomination of shares); means Rs. 10 each share is divided into 2 shares of Rs. 5 each etc. In EPS the treatment
of share split is similar to the treatment of bonus shares (as it is also a not for value issue).

Example 8A:

A company had 10 000 ordinary shares in issue on 1.01 2011. On 1 April 2012, there was a share split
whereby every 2 shares became 5 shares. On 1 June 2012, 12 000 shares were issued at market value of
5 per share. The basic earnings were 150 000 in 2011 and 261 250 in 2012.

Required:
Calculate the basic earnings per share for the years ended 31 December 2012 with comparatives. Also
calculate EPS of 2011.

Page 8 of 29
170
Answer:
Earnings per share: [2011]

150 000
10 000
= 15 per share
Earnings per share: [2012]
2012 2011

Earnings 261 250 150 000


Number of shares = 32 000 25 000

= 8.16 per share 6 per share


Workings:

Number of shares Actual Current year Prior year


(weighted) (adjusted)
Opening balance 10 000 10 000 10 000
Issue for no value (bal) [01.04] 15 000 15 000 15 000
[10,000/2x5] 25 000 25 000 25 000
Issue for value [01.06] 12 000 7 000(2) 0
Closing balance 37 000 (1) 32 000 25 000

Example 9: issue for no value after an issue for value (a share split with issue of shares with value )

A company had 10 000 ordinary shares in issue on 1.01 2011. On 1 April 2012, 12 000 shares were issued
at market value of 5 per share. On 1 June 2012, there was a share split whereby every 2 shares became 5
shares. The basic earnings were 150 000 in 2011 and 261 250 in 2012.

Required:
Calculate the basic earnings per share for the years ended 31 December 2012 with comparatives. Also
calculate EPS of 2011.

Answer:
Earnings per share: [2011]

150 000
10 000
= 15 per share
Earnings per share: [2012]

2012 2011

Earnings 261 250 150 000


Number of shares = 47 500 25 000

= 5.5 per share 6 per share

Page 9 of 29
171
Workings:

Number of shares Actual Current year Prior year


(weighted) (adjusted)
Opening balance 10 000 10 000 10 000
Issue for value [01.04] 12 000 9 000(2) 0
22 000 19 000 10 000
Issue for no value[01.06] 33 000 28 500(3) 15 000(4)
55 000 (1) 47 500 25 000

(1) 22 000 / 2 x 5 = 55 000

(2) 12 000 x 9 / 12 = 9 000


(3) 33 000 / 22 000 x 19 000 = 28 500
(4) 33 000 / 22 000 x 10 000 = 15 000

Practice question 2

Company D has a 31 December year end and had 2,000,000 ordinary shares in issue on 1 January 2012.

On 31 March 2012, it issued 500,000 ordinary shares, at full market price.

On 1 July 2012, Company D made a 1 for 2 bonus issue. In 2011, the EPS had been calculated as Rs. 30 per
share. In 2012, total earnings were Rs. 85,500,000.

Required
Calculate the EPS for the year to 31 December 2012, and the comparative EPS figure for 2011.

Solution:

Number of shares Actual Current year Prior year


(weighted) (adjusted)
Opening balance 2,000,000 2,000,000 2,000,000
Issue for value 31.3 500,000 375,000 (500,000 0
x9/12)

2,500,000 2,375,000 2,000,000


Issue for no value (bonus shares 1/2) 1,250,000 (1) 1,187,500 1,000,000
(2) (3)
3,750,000 3,562,500 3,000,000

Page 10 of 29
172
(1) 2,500,000 x 1 / 12 = 1 250 000
(2) 1 250 000 / 2 500 000 x 2 375 000 = 1 187 500
(3) 1 250 000 / 2 500 000 x 2 000 000 = 1 000 000

EPS in 2012 = Rs. 85,500,000/3,562,500 = Rs. 24 per share.

The 2011 EPS restated as: Rs. 60,000,000 / 3,000,000 = 20 per share
In 2011 EPS original will be: 60,000,000 / 2,000,000 = 30 per share
3.Combination issues (right shares)

A combination issue is one that involves an issue at less than market value. A right issue occurs when shares are
offered to existing shareholders at a specified reduced price, (less than the market price) in proportion to
existing shares.

The issue price of the new shares in a rights issue is generally below the current market price. This means that
they include a bonus element which must be taken into account in the calculation of the weighted average
number of shares.

While calculation we can subdivide it as: in effect an issue for value (i.e. some of the shares are assumed to have
been sold at full market value) and an issue for no value (some of the shares are assumed to have been given
away).
There are two methods of calculating the number of shares: one involves the use of a table (where the principles
are those used in the previous examples) and the other involves the use of formula. Both will give you the same
final answer.

Example 10: rights issue


A company had 10 000 shares in issue at the beginning of the current year (01.01.2012), and 3 months
before the year-end, the company had a rights issue of 1 share for every 5 shares held. The exercise (issue)
price was 4 per share when the fair value immediately before the rights issue was 5 per share (i.e. market
value cum rights). Earnings of prior period are 100 000.
All the shares offered in terms of this rights issue were taken up.
Required:
Calculate the weighted average number of shares in issue for the purposes of calculating earnings per
share in the financial statements for the year ended 31 December 2012. Alongwith comparative
Answer Table form:
• The number of shares issued in terms of the rights issue: 10 000/5 x 1 share = 2 000 shares
• The cash received from the rights issue: 2 000 shares x 4 = 8 000
• The number of shares that are issued may be split into those shares that are effectively sold and
those that are effectively given away:
Number
Shares sold (issue for value): proceeds/ market price cum rights = 8 000/ 5 1,600
Shares given away (issue for no value): [total shares issued – shares sold = [2 000 400
shares – 1 600 shares ]
2,000

Page 11 of 29
173
The weighted and adjusted average number of shares may then be calculated:

Number of shares (weighted & adjusted) Actual Current year Prior year
(weighted) (adjusted)
Balance: 1/1 10 000 10 000 10 000
Issue for value 1/10 1 600 400 0
(1 600 x 3/12)
11 600 10 400 10 000
Issue for no value 1/10 400 359* 345**

(400/ 11 600 (400/ 11 600 x


x 10400) 10 000)
Balance: 31/12 12 000 10 759 10 345

Solution to example 10: using the ‘formula approach’(if table cannot be used )

Theoretical ex-rights value per share (means market price of the share that ought to be in theory after the
right issue):

(Shares before right issue x Actual cum rights price) + (Right shares x issue price)
ights issue Number of shares in issue after the rights issue

= 10 000 shares x 5 + 2 000 shares x 4


10 000 + 2 000 000

= 58 000
12 000

= 4.833 per share


Adjustment factor:

Actual cum rights price


Theoretical ex-right value per share

= 5
4.833

= 1.0345
Number of shares:
Current period: (number of shares before the right issue x adjustment factor x fraction of the year before the
right issue + total shares after the right issue x fraction of the year after the right issue)
Prior period: (number of shares before the right issue x adjustment factor)

= Current year (10 000 shares x 1.0345 x 9/12 + 12 000 x 3/12) 10 759
= Prior year (10 000 shares x 1.0345) 10 345

Or last year EPS can be adjusted as: original EPS x Theoretical ex-right value per share/ Actual cum rights
price:100 000/10 000 =10 x 4.833 /5 = 9.67 or 100 000/10 345 = 9.67

Page 12 of 29
174
Comment: Notice that the current year calculation of the number of shares is weighted for the number of
months before the issue and after the issue, whereas the prior year is not weighted at all.

Practice question 3
Company F had 3 million ordinary shares in issue on 1 January 2017.
On 1 April 2017, it made a 1 for 2 rights issue of 1,500,000 ordinary shares at Rs. 20 per share.
The market price of the shares prior to the rights issue was Rs. 50.
An issue of 400,000 shares at full market price was then made on 1 August 2017.
In the year to 31 December 2017, total earnings were Rs. 17,468,750. In 2016 EPS had been reported as Rs. 3.5
(means prior period earnings were (3,000,000 x 3.5 = 10,500,000).

Required
Calculate the EPS for the year to 31 December 2017, and the adjusted EPS for 2016 for comparative purposes.

Solution:
The number of shares issued in rights issue: 1,500,000 shares
The cash received from the rights issue: 1,500,000 shares x 20 = 30,000,000
Number
Shares sold (issue for value): proceeds/ market price 30,000,000 / 50 600,000
Shares given away (issue for no value): total shares issued – shares sold = 900,000
1,500,000 shares – 600,000 shares
1,500,000
The weighted and adjusted average number of shares may then be calculated:

Number of shares (weighted & Actual Current year Prior year


adjusted) (weighted) (adjusted)
Balance: 1/1 3,000,000 3,000,000 3,000,000
Issue for value 1 / 4 600,000 450,000 0
(600,000 x9/12)
3,600,000 3,450,000 3,000,000
Issue for no value 1 / 4 900,000
862,500 750,000
(900 / 3,600 x 3,450) (900 / 3,600 x
3,000)
1/4 4,500,000 4,312,500 3,750,000
1/8 400,000 166,667 0
(400 x 5 / 12)

31 / 12 4,900,000 4,479,167 3,750,000

EPS for 2017: 17,468,750 / 4,479,167 = 3.9 per share


EPS for 2016 (Restated): 10,500,000 / 3,750,000 = 2.8 per share

Page 13 of 29
175
Example 11: various issues over three years
Numbers Ltd has a profit of 100 000 for each of the years 2013, 2014 and 2015. There are no preference
shares. On 1 January 2013, there were 1 000 ordinary shares in issue, all of which had been issued at 10
each equal to nominal amount, after which, the following issues took place:

• 30 June 2014: 1 000 ordinary shares were sold for 35 each (their market price);
• 30 September 2014: there was a capitalisation issue of 1 share for every 2 shares in issue
on this date;
• 30 June 2015: 2 000 ordinary shares were sold for 40 each (their market price); and
• 31 August 2015: there was a share split whereby every share in issue became 3 shares.
Required:
A) Journalise the issues for the years ended 31 December 2014 and 2015.
B) Calculate the basic earnings per share to be disclosed in the financial statements of Numbers Ltd for
the year ended 31 December 2015 (with comparative figures of 2014 and 2013)

C) Calculate the basic earnings per share as disclosed in the financial statements of Numbers Ltd for the
year ended 31 December 2014 (with comparative figures of 2013).

Solution to example 11A: journals

30/6/2014 Debit Credit


Bank (1,000 x 35) 35 000
Ordinary share capital 10 000
Share premium 25 000
Issue of 1 000 ordinary shares at 35 (market price)
30/9/2014
Dividend 10 000
Ordinary share capital 10 000
Capitalisation issue: (1 000 + 1 000) / 2 x 1 x 10

30/6/2015
Bank (2,000 x 40) 80 000
Ordinary share capital (2 000 x 10) 20 000
Share premium (2 000 x 30) 60 000
Issue of 2 000 ordinary shares at 40 (market price)
31/8/2015
There is no journal entry for a share split (the authorized and issued number of shares are simply
increased accordingly). However, remember that entry is made for bonus shares.

Page 14 of 29
176
Solution to example 11B:
EPS for 2013:
EPS = 100,000 / 1,000 = 100 per share

Actual Current year Prior year


(weighted) (adjusted)
Balance: 1/1 1,000 1,000
No movement - -
Balance: 31/12 1,000 1,000

EPS for 2014:


EPS 2014 = 100,000 / 2,250 = 44.44 per share
EPS 2013 [Restated]= 100,000 / 1,500 = 66.67 per share

Actual Current year Prior year


(weighted) (adjusted)
2014 2013
Balance: 1/1 1,000 1,000 1,000
Issue for value 30 / 06 1,000 500 0
(1,000 x 6/12)
2,000 1,500 1,000
Issue for no value 30 / 09 1,000 750 500
(bonus issue) (2,000 x 1/2) (1,000 / 2,000 x 1500) (1,000 / 2,000 x
3,000 2,250 1,000)
1,500

EPS for 2015:


EPS 2015 = 100,000 / 12,000 = 8.33 per share
EPS 2014 [Restated]= 100,000 / 6,750 = 14.81 per share
EPS 2013 [Restated]= 100,000 / 4,500 = 22.22 per share

Actual Current year Prior year Prior year


(weighted) (adjusted) (adjusted)
2015 2014 2013
Balance: 1/1/2015 3,000 3,000 2,250 1,500
Issue for value 30 / 06 2,000 1,000 0 0
(2,000 x 6/12)
5,000 4,000 2,250 1,500
Issue for no value 31 / 08 10,000 8,000 4,500 3,000
(share split) (10,000 / 5,000 x (10,000 / 5,000 (10,000 / 5,000
(5,000 /1x 3) 15,000 4,000)
12,000 6,750X 2,250) x 1,500)
4,500

Page 15 of 29
177
4.Share buy-backs
A share buy-back involves a decrease in the capital base of the entity through the entity repurchasing shares
from its shareholders. A buy-back involves a reduction of the capital base (fewer issued shares exist after the
buy-back) and a reduction in the money/ resources of the entity (the entity pays the shareholders for the shares).

Since the entity pays the shareholders for their shares, the share buy-back is a for-value reduction. The
treatment of a for-value reduction is very similar to that of a for-value issue with the exception that the number
of shares involved is subtracted rather than added.

Example 12: share buy-back


A company had 10 000 ordinary shares in issue during 2012.
There was a share buy-back:
• of 5 000 ordinary shares (at market price)
• 60 days before the end of the current year (year-end: 31 December 2013).
The basic earnings in 2012 were 20 000 and 17 000 in 2013

Required:
Calculate the earnings per share in 2013 along with comparatives.

Answer:

W1: Denominator: number of shares Actual Current year Prior period (Adjusted)
(weighted)
2013 2012

Opening balance: 1/1 10 000 10 000 10 000


Reduction for value: 2/11 (5 000) (822) 0
( 5 000 x 60 /365)

5 000 9 178 10 000

W2: earnings per share for inclusion in 20X3 financial statements

Basic earnings per share: 2013 Restated


2012

Basic earnings 17 000 20 000


Weighted average number of shares 9 178 10 000

1.85 2.00

Page 16 of 29
178
5.Share consolidation (Also called as reverse share split)
A share consolidation is the combination of 2 or more shares into 1. This transaction requires none of the entity’s
resources and thus it is treated as a not-for-value reduction. The treatment of a not-for-value reduction is very
similar to that of a not-for-value issue with the exception that the number of shares involved is subtracted rather
than added.

Example 13: share consolidation


A company had 10 000 ordinary shares in issue during 2012.
During 2013, the company consolidated its shares:
• such that every 2 shares were consolidated into 1 share.
• 60 days before the end of the current year (year-end: 31 December 2013).

The basic earnings in 2012 were 20 000 and 17 000 in 2013

Required:
Calculate the earnings per share in 2013 along with comparative.
Answer:
W1: Denominator: number of shares Actual Current year Prior period (Adjusted)
(weighted)
2013 2012
Opening balance: 1/1 10 000 10 000 10 000
Reduction for no value: 2/11 (5 000) (5 000) (5 000)
(5 000 x 10 000/ 10 000) (5 000 x 10 000/ 10 000)

5 000 5 000 5 000

W2: earnings per share for inclusion in 2013 financial statements

Basic earnings per share: 2013 2012


Basic earnings 17 000 20 000
Weighted average number of shares 5 000 5 000

3.40 4.00*
* The 2012 financial statements would have reflected earnings per share figure of 2 (20 000 / 10 000).

Extra discussion points to be considered:


Total earnings
The total earnings figure is the profit or loss from continuing operations after deducting tax and preference
dividends.

When there is a net loss, total earnings, and therefore, the EPS are negative.

Earnings from discontinued operations (means if a segment of business is closed. E.g. Dubai operations) are
dealt with separately. An EPS from any discontinued operations must also be disclosed separately.

Page 17 of 29
179
The total earnings figure must be adjusted for the interests of preference shareholders before it can be used in
EPS calculations.

Preference shares
Preference shares must be classified as either equity or liability

If a class of preference shares is classified as equity (means irredeemable preference shares), any dividend
relating to that share is recognized in equity. Any such dividend must be deducted from the profit or loss from
continuing operations.

If a class of preference shares is classified as liability (redeemable preference shares), any dividend relating to
that share is recognized as a finance cost in the statement of profit or loss. It is already deducted from the profit
or loss from continuing operations and no further adjustment need be made.

Rs. 3,000,000* Rs. 3 per share (EPS from

= =
1,000,000 continued operations)

Rs. 500,000 Rs. 0.5 per share (EPS from

= 1,000,000 = discontinued operations)

*preference dividend on preference shares classified as a liability should have already been deducted from profit
after tax.

Page 18 of 29
180
Example: Basic EPS
In the year ended 31 December 2011, Entity G made profit after tax of Rs. 3,500,000.

Of this, Rs. 3,000,000 was from continuing operations and Rs. 500,000 from discontinued operations.

It paid ordinary dividends of Rs. 150,000 and preference dividends of Rs. 65,000.

The preference shares were correctly classified as equity.

Entity G had 1 million ordinary shares in issue throughout the year.

Entity G’s basic EPS for the year ended 31 December 2011 is calculated as follows:
EPS =

Rs. 3,000,000 - Rs. 65,000 Rs. 2.935 per share

= =
1,000,000 (continued operations)

Rs. 500,000 Rs. 0.5 per share

= 1,000,000 = (discontinued operations)

As EPS from discontinued operations must also be disclosed but this does not have to be disclosed on the face
of statement of profit or loss. Instead it may be shown in notes to financial statements.

Retrospective adjustments
If the number of ordinary shares outstanding increases as a result of a capitalisation, bonus issue or share split,
or decreases as a result of a reverse share split (means share consolidation), the calculation of basic EPS for all
periods presented shall be adjusted retrospectively.

If these changes occur after the reporting period but before the financial statements are authorised for issue,
the per share calculations for those and any prior period financial statements presented shall be based on the
new number of shares.

The fact that per share calculations reflect such changes in the number of shares shall be disclosed.

In addition, basic earnings per share of all periods presented shall be adjusted for the effects of errors and
adjustments resulting from changes in accounting policies accounted for retrospectively.

Page 19 of 29
181
Important note: If both the announcement of shares and entitlement dates of shares are available, then use
entitlement date in the working of EPS.
Example: Home Dynamics Limited
Question: The following information pertains to the financial statements of Home Dynamics Limited (HDL),
a listed company, for the year ended 31 December 2016:
(i) Profit after tax for the year: Rs. in
million
Profit from continuing operations – net of tax 765
Profit from discontinued operations – net of tax 155
Profit after tax 920
(ii) Shareholders’ equity as on 1 January 2016 comprised of:
▪ 10 million ordinary shares of Rs. 10 each, having market value of Rs. 25 each.
▪ 4 million cumulative preference shares of Rs. 10 each entitled to a cumulative dividend at
10%.
(iii) On 31 March 2016, HDL announced 40% right shares to its ordinary shareholders at Rs. 25 per
share. The entitlement date of right shares was 31 May 2016. The market price per share
immediately before the announcement date and entitlement date was Rs. 28 and Rs. 32
respectively.
(iv) On 2 August 2016, HDL announced 20% bonus issue. The entitlement date of bonus shares was
31 August 2016.
(v) On 1 February 2017, the board of directors announced 20% cash dividend and 10% bonus issue
being the final dividend to the ordinary shareholders and 10% cash dividend for preference
shareholders.

Required:
Calculate basic earnings per share for inclusion in HDL’s financial statements for the year ended 31
December 2016. Show all relevant calculations.

Answer:
Basic earnings per share for the year ended 31 December 2016.
Rupees in millions
Continuing Discontinuing
operations operations
Profit attributable to ordinary shareholders 761 155
(765-4)(4 x 10 x
10%)
Weighted average number of shares in issue during the 16.65 16.65
year (W.1) [shares in millions]
Earnings per share 45.71 9.31
.

Page 20 of 29
182
Workings: [W.1] shares in 000
Actual Current year
(weighted)

Opening balance 1.1 10,000 10,000

Issue for value 31.05 (Note 1) 3,125 1,823


(4,000,000 x 1/4) (3,125 x 7/12)
13,125 11,823

Issue for no value 31.05 [note 1] 875 788


(875/13,125 x 11,823)

14,000 12,611
Issue for no value 31.8 2,800 2,522
(14,000 x 20%) (2,800/14,000 x 12,611)
Closing balance 31.12 16,800 15,133
Bonus issue after the year No entry 1,513 (10%)
end(Final) (10%)[no value] [1,680/16,800x15,133]

16,646
Note 1:
For value: 10,000 x 40% =4,000 x 25 = 100,000/32 = 3,125 shares
For no value: 4,000 – 3,125 = 875 shares

Practice Questions

AIRCON LTD
Mr Hamad, currently owns 20 million shares in Aircon Ltd. He recently received the published financial
statements of Aircon Ltd for the year ended 31 March 2016. Mr Hamad is not sure how the performance of the
company during the year will affect the market value of the entity’s shares but he is aware that the earnings per
share statistics are often used by analysts in assessing the performance of listed companies.

Extracts from these published financial statements and other relevant information are given below.

Statement of profit or loss for the period ended 31 March 2016

2016 2015
Rs.’m Rs.’m
Revenue 18,000 15,300
Cost of sales (11,340) (9,180)
Gross profit 6,660 6,120
Operating expenses (3,420) (3,240)

Page 21 of 29
183
Operating profit 3,240 2,880
Interest expense (540) (576)
Profit before tax 2,700 2,304
Taxation (846) (720)
Profit after tax 1,854 1,584

Statement of financial position as at 31 March 2016


2016 2015
Rs.’m Rs.’m Rs.’m Rs.’m
Intangible assets 5,400 -
Tangible assets 7,200 6,660
──── ────
12,600 6,660
Current Assets
Inventory 2,340 1,800
Receivables 2,700 2,160
Cash in bank 180 5,220 162 4,122
──── ──── ──── ────
17,820 10,782
──── ────
Capital and Reserves
Share Capital 2,700 900
Share Premium 4,860 900
Retained Earnings 1,620 1,206
──── ────
9,180 3,006

Current Liabilities
Trade Payables 3,060 2,160
Taxation 900 756
Bank Overdraft 1,080 1,260
──── ────

5,040 4,176
──── ────
14,220 7,182
15% Loan Note 3,600 3,600
──── ────
17,820 10,782
──── ────

Page 22 of 29
184
The following information is also relevant:
(i) The share capital of the company comprises Rs. 1 equity shares only.
(ii) On 1 October 2015, the company made a rights issue to existing shareholders of two new shares for every
one share held at a price of Rs.5.94 per share.
(iii) The market price of shares immediately before the rights issue was Rs. 6.30 per share.
(iv) No other changes took place in the equity capital of Aircon Ltd in the year ended 31 March 2016.

Required
(a) Compute EPS for the year and the comparative figures that will be included in the published financial
statements of Aircon Ltd for the year ended 31 March 2016.
(b) Using the extracts you have been provided with, write a report to Mr Hamad identifying the key factors
which led to the change in the EPS of Aircon Ltd since the year ended 31 March 2016.
(c) Comment on the relevance of the EPS statistics to shareholders.

Aircon Ltd

(a) Earnings Per Share

EPS for 2016: 1,854/1,817.5 (W 1) = 1.02 per share

EPS for 2015 [Restated]: 1,584/936 = 1.7 per share

Workings
• The number of shares issued in rights issue (900 / 1 x 2) 1,800 shares
• The cash received from the rights issue: 1,800 shares x 5.94 = 10,692

Shares sold (issue for value): proceeds/ market price [10,692 / 6.3] 1,697
Shares given away (issue for no value): total shares issued – shares sold = 1,800 103
shares – 1,697 shares
1,800

The weighted and adjusted average number of shares may then be calculated:

Note 1: Please remember that issues for value during the year require weighting of the number of shares
to take into account how long the extra capital was available to the entity.
Note 2: Please note that an issue for no value will not cause an increase in the profits and therefore, in
order to ensure comparability, the prior year shares are adjusted as if the issue for no value had occurred
in the prior year.

Number of shares (weighted & adjusted)


Actual Current year Prior year
(weighted) (adjusted)
Balance: 1/4/2015 900 900 900
Issue for value 1/10/2015 1,697 848.5 (1,697 x 6/12) 0
2,597 1,748.5 900

Page 23 of 29
185
Issue for no value 1/10/2015 103 69 36
(103/2,597 x 900) (103/2,597 x
900)
31/03/2016 2,700 1,817.5 936

(b) Report
To: Mr Hamad
From: Accountant
Date: 15 April 2016

Subject: Evaluating the changes in EPS of Aircon Ltd

Calaculation of EPS is based on data of income statement therefore the key factors which has led to changes in
the EPS of Aircon Ltd. are as follows:

Revenue has increased by Rs. 2,700 million which means 18% increase over the last year (2,700/15,300 x 100).
However, as the capital employed is increased by a higher amount of 6,174 therefore, asset turnover ratio
decreased from last year.

The gross profit ratio has declined which is an indication of increase in cost of sales.

There is an increase in operating expenses therefore operating profit ratio and net profit ratio remains
approximately constant.

Return on assets, return on capital employed and return on equity all indicate increase in total assets, capital
employed and equity but the profit has not increased proportionately.

Conclusion:
Assets are increased because of increase in intangible assets by Rs. 5,400 millions. The company has generated
cash flow from shares of Rs. 5,760 (7,560 – 1,800) which looks like invested in assets. Therefore, company can
generate increased profits in the future periods by utilizing the assets acquired during the period.
However, as these assets have yet to start generating income during the year, the EPS has therefore fallen from
Rs. 1.69 in 2015 to Rs. 1.01 in 2016.

Signed

Accountant

Page 24 of 29
186
APPENDIX TO THE REPORT

Relevant ratios

Total Asset Turnover Net Sales___


(Utilization of Total Assets) Avg. Capital Employed
(Answer is in times)
2016: 18,000 / 9,180 + 3,600 = 1.41 times
2015: 15,300 / 6,606 = 2.31 times

Gross profit Ratio Gross Profit x 100 =…...%


Net Sales
(Answer in %)
2016: 6,600 / 18,000 x 100 = 37 %
2015: 6,120 / 15,300 x 100 = 40 %

Operating Profit Ratio Operating Profit x 100


(Operating profit is Profit before interest & Tax) Net Sales
(Answer in %)
2016: 3,240 / 18,000 x 100 = 18 %
2015: 2,880 / 15,300 x 100 = 19 %

Expense Ratio
Operating Expenses x 100
Net Sales
(Answer in %)

2016: 3,420 / 18,000 x 100 = 19 %


2015: 3,240 / 15,300 x 100 = 21 %
Net Profit after tax Ratio
Profit After tax x 100
Net Sales
(Answer in %)

2016: 1,854 / 18,000 x 100 = 10 %


2015: 1,584 / 15,300 x 100 = 10 %

Return on Capital Employed (ROCE) Profit before interest & tax x100
Avg Capital Employed

(Answer is in %)
2016: 3,240 / 9,180 + 3,600 x 100 = 25 %
2015: 3,880 / 6,660 x 100 = 44 %

Page 25 of 29
187
Return on Equity PAT – *Preference Dividend x100
Equity= (Share capital + Reserve) Average Equity
(Answer is in %)
*If preference shares are non-redeemable

2016: 1,854 / 9,180 x 100 = 20.2 %


2015: 1,584 / 3,006 x 100 = 52.7 %
Average is only calculated if it can be for all given relevant periods.

Relevance of EPS to shareholders


• The EPS is used to compute the price earning (P/E) ratio, a major market indicator to determine how
successful a company has been operating. The price earning figure is a multiple of the EPS, where the
multiple represents the number of years’ earnings required to recoup the price paid for the share (pay back
period).
• Rising trend in EPS is a more accurate performance indicator than rising trend in profit after tax (because of
considering only profit from continuing operations).
• EPS is a measure of performance from the existing and potential investors’ perspective.
• EPS show the amount available to each ordinary shareholder thereby indicating the potential returns on
individual investment.
• EPS is used to compare the activities of two entities in the same industry (however EPS has some limitations
as well).

Extra practice questions:


Example 1 share issuance with share consolidation
At 1 January 20X5 an entity had 10,000,000 ordinary shares in issue. It issued 3,000,000 new shares on
30.04.20X5 for cash at full market value. On 31.08.20X5 it went through a share reconstruction by consolidating
the shares in issue, on the basis of one new share for two old shares. The year ned of the company is 31.12.

Required: Calculate the weighted average number of shares in issue during the year.

Solution:
Actual Current year
(weighted)
Opening balance 1.1 10,000,000 10,000,000
Issue for value 30.04 3,000,000 2,000,000
(3,000,000
x 8/12)
13,000,000 12,000,000
Issue for no value 31.08 (6,500,000) (6,000,000)
(share consolidation) (6,500,000/
(13,000,000 13,000,000
x 1/2) x 12,000,000)
Opening balance 31.12 6,500,000 6,000,000

Page 26 of 29
188
Example 2 cash and right issue
At 1 January 20X4 and 20X5 an entity had 14,000,000 ordinary shares in issue. The financial year end of the
entity is 31.12.20X5.It issued further shares as follows:
It issued 4,000,000 new shares on 01.04.20X5 for cash at full market value.
On 01.07.20X5 it made a right issue of one for six at 15 each when market price was 20 each.
A profit of 17,000,000 attributable to ordinary equity holders was reported for 20X5 and 14,000,000 for 20X4.

Required: calculate the earnings per share for 20X5 including comparative figure of 20X4.

Solution:
EPS [2005] = 17,000,000 / 18,796,296 = 0.90 per share
EPS [2004] [Restated] = 14,000,000 / 14,518,518 = 0.96 per share

Workings:
Actual Current year Prior year
(weighted) (adjusted)
Opening balance 1.1 14,000,000 14,000,000 14,000,000
Issue for value 1.4 4,000,000 3,000,000 0
(4,000,000 x 9/12)
18,000,000 17,000,000 14,000,000
Issue for value 1.7[Right shares(W)] 2,250,000 1,125,000 0
(2,250,000 x 6/12)
20,250,000 18,125,000 14,000,000
Issue for no value 1.7(W) 750,000 671,296 518,518
(750/20,250 x (750/20,250 x
18,125,000) 14,000,000)

Closing balance 31.12 21,000,000 18,796,296 14,518,518

Working :
Issued for value = 18,000,000 x 1/6 = 3,000,000 x 15 = 45,000,000/20 = 2,250,000 shares
Issued for no value = 3,000,000 – 2,250,000 = 750,000 shares

Example 3 cash and right issue


At 1 January 2016 there were 13,000 ordinary shares in issue. It issued further shares as follows:
It issued 8,000 new shares on 01.04.2016 for cash at full market value.
On 01.06.2016 it made a bonus issue of one new share for every three shares held.
On 01.11.2016 it issued 8,500 new shares at full market value.
Profit of 1,000,000 attributable to ordinary equity holders was reported for 2016 and 613,000 for 2015.

Required: calculate the basic earnings per share for the year ended31.12. 2016 and 2015(including comparative
figures).

Page 27 of 29
189
Solution:
For the year ended 2015:
EPS [2015] = 613,000 / 13,000 = 47.15 per share
For the year ended 2016:
EPS [2016] = 1,000,000 / 26,750 = 37.38 per share
EPS [2015] [Restated]= 613,000 / 17,333 = 35.37 per share

Workings:
Actual Current year Prior year
(weighted) (adjusted)

Opening balance 1.1 13,000 13,000 13,000


Issue for value 1.4 8,000 6,000 0
(8,000 x 9/12)

21,000 19,000 13,000


Issue for no value 1.6 7,000 6,333 4,333
(bonus issue) (21,000 x 1/3) (7/21 x 19,000) (7/21 x 13,000)
28,000 25,333 17,333
Issue for value 1.11 8,500 1,417 0
(8,500 x 2/12)

Opening balance 31.12 36,500 26,750 17,333

Example 4 cash and right issue


At 1 January 2016 there were 6,000 ordinary shares in issue. It issued further shares as follows:
It issued 5,000 new shares on 01.03.2016 for cash at full market value.
On 01.04.2016 it made a 2 for 3 rights issue at 5 per share. The market price of shares prior to the rights issue
was 15.
On 01.09.2016 it issued 12,000 new shares at full market value.
Profit of 200,000 attributable to ordinary equity holders was reported for 2016 and 70,000 for 2015.
Required: calculate the basic earnings per share for the year ended31.12. 2016 and 2015 (including
comparative figures).
Solution:
For the year ended 2015:
EPS [2015] = 70,000 / 6,000 = 11.67 per share
For the year ended 2016:
EPS [2016] = 200,000 / 20,363 = 9.82 per share
EPS [2015] [Restated] = 70,000 / 8,180 = 8.56 per share

Page 28 of 29
190
Workings:
Actual Current year Prior year
(weighted) (adjusted)

Opening balance 1.1 6,000 6,000 6,000


Issue for value 1.3 5,000 4,167 0
(5,000 x 10/12)

11,000 10,167 6,000


Issue for value 1.4[Right issue (W)] 2,444 1,833 0
(2,444 x 9/12)
13,444 12,000 6,000
Issue for no value 1.4 (W) 4,888 4,363 2,181
(4,888/13,444 (4,888/13,444
x 12,000) x 6,000)
18,332 16,363 8,181
Issue for value 1.9 12,000 4,000 0
(12,000 x 4/12)

Opening balance 31.12 30,332 20,363 8,180

Working :
Issued for value = 11,000 x 2/3 = 7,333 x 5 = 36,666/15 = 2,444 shares
Issued for no value = 7,333 – 2,444 = 4,888 shares

Page 29 of 29
191
Diluted EPS
A company might have potential ordinary shares in issue.

Potential ordinary share


Definition
A potential ordinary share is a financial instrument that may entitle its holder to ordinary shares (at some time in
the future).

Examples of potential ordinary shares include:


• Convertible bonds (debentures)
• Convertible preference shares
• Share options or warrants

1. Convertible bonds are fixed interest debt securities (means a loan) which give the holder the right to convert the
bond into ordinary shares of the company. The conversion (e.g after 3 years every Rs. 20 is convertible into 1 ordinary
share) takes place at a pre-determined rate and on a pre-determined date. If the conversion does not take place the
bond will run its full life and be redeemed in cash on maturity.

Once converted into ordinary shares, convertible securities cannot be converted back into loan.

2. Convertible preference shares have similar characteristics as convertible bonds.

3a. Share Options


Definition:
A share option gives the holder the right, but not the obligation to purchase the shares in future.
The holder of an option sometimes pays an amount in exchange for the option (Fee for option).
If the holder of the option takes up the option, this is called ‘exercising’ the option.

For example, let’s say an employee or any other investor pays a premium of Rs. 30 for the option to buy a share in
Honda limited for Rs. 200 in 3 months’ time. Rs. 200 is called the ‘strike price’(also called as Exercise price).

If the price of shares in Company is Rs. 250 in 3 months’ time then the holder of the option will exercise his right to
buy a share at Rs. 200. He could immediately sell that share for Rs. 250 in the open market making a profit of Rs. 50
(less the original option premium of Rs. 30)(net gain is 20).

However, if in 3 months’ time the market price of shares in company is only Rs. 180. In this case the holder of the
option will not exercise his option to buy for Rs. 200 as he can buy shares in Company at that time in the open market
for Rs. 180. In this case the option ‘lapses’ (i.e. is not exercised).

When the market price of a share is such that to exercise the option would enable the option holder to make a profit
this option is called ‘in the money’. If the market price is such that to exercise the option would lose money, the option
is said to be ‘out of the money’.

So in the previous example the option with a strike price of Rs. 200 is ‘in the money’ when the market price of the
shares is above Rs. 200, but ‘out the money’ when the market price of the shares is trading below Rs. 200.

Page 1 of 12
192
Example: On 1 January 2019, Marigold Enterprises (ME) purchased an option for Rs. 10,000 allowing ME to buy 5,000
shares of Aroma Limited (AL) at a price of Rs. 140 per share, during the next two months. On 12 February 2019, ME
purchased the shares at the agreed price when the market value of AL's shares was Rs. 180 per share.

Required: Briefly explain what is the meaning of in the money and out of the money options and link the terms to the
above scenario.

Ans. When the market price of the share is such that by exercising the option, the option holder makes a profit, the
option is said to be ‘in the money’.

When the market price of the share is such that by exercising the option, the option holder suffers a loss, the option
is said to be ‘out the money’.

By exercising the option, ME made a profit of Rs. 5,000 x 40 (180–140) = 200,000, therefore, the option is said to be
‘in the money’. However net gain will be Rs. 200,000-10,000 = 190,000.

3b.A warrant (a loan and share option to purchase company’s share in future) is similar to a convertible bond in that
the warrant allows the holder to buy shares at a set price i.e at exercise price (rather than convert the bond into
shares). The share option part of a warrant can be separated from the bond and traded on its own whereas a
convertible bond cannot be separated.

Impact of potential ordinary shares:


Potential ordinary shares do not impact calculation of basic EPS but diluted EPS might differ from basic EPS when
there are potential ordinary shares in existence.

If potential shares become actual ordinary shares, the earnings figure will be shared with a larger number of ordinary
shares. This might dilute the EPS. The literal meaning of ‘dilution’ is ‘reduction in strength. IAS 33 defines ‘dilution’
as follows:

Definition of dilution:
Dilution is a reduction in earnings per share resulting from the assumption that convertible instruments are
converted, or that options or warrants are exercised.

Overview
With respect to earnings per share, dilution would occur if the same earnings have to be shared amongst
more shares than are currently in existence. Many entities at year-end have potential shares outstanding,
which if converted into shares, would dilute the earnings per share. Diluted earnings per share shows the
lowest earnings per share possible assuming that potential ordinary shares are converted into ordinary
shares. The diluted earnings per share shows users the maximum potential dilution of their earnings in the
future (i.e. the worst case scenario) assuming the potential shares currently in existence are converted into
ordinary shares. It logically follows that diluted earnings per share can never be higher than basic earnings
per share.

Page 2 of 12
193
Objective of diluted EPS
The objective of diluted EPS is consistent with that of basic earnings per share, that is, to provide a measure of the
performance of each ordinary share taking into account dilutive potential ordinary shares outstanding during the
period.

Calculation of diluted EPS:


Always calculate the basic EPS before the diluted EPS and then consider adjustment if any, on basic earnings and
basic number of shares.

1.Share options
Options are granted to individuals allowing them to acquire a certain number of shares in the company at a
specified price per share (the strike price or exercise price) in the future. This is usually lower than the average
market price (fair value) of the share, which therefore encourages the option holder to buy the share.

When the option is exercised it results in both a ‘for value issue’ (relating to the cash received) and a ‘not for
value issue’ (relating to the bonus element, being the difference between what should have been received
based on the market price and what was received).

The possible conversion of options into ordinary shares will result into increase in number of shares.

The amount that would be received on exercise of the options is treated as cash received from selling shares at full
price with the remaining shares having been given away. The shares sold at full price are not considered to be dilutive
as any cash received would be invested to earn some return. It is only the free shares that are dilutive.

Example 1: options to acquire shares


2015
Profit before tax 800 000

Income tax expense (390 000)


Profit for the year 410 000

There are 200 000 ordinary Rs. 2 shares in issue.


During 2015 the company’s shares had an average market value of Rs.6.
The company’s directors hold option to purchase 100 000 shares, at a strike price of Rs. 2 each.

Required:
Calculate the basic and diluted earnings per share figures for 2015.

Page 3 of 12
194
Answer:

Basic earnings per share


2015
Basic Earnings = 410 000
Weighted number of shares outstanding 200 000

Basic earnings per share = 2.05

Diluted earnings per share (2015):

Diluted earnings 410 000


number of ordinary shares(W) 266 667

Diluted earnings per share = 1.5375

W.1
Weighted number of shares:
Basic number of shares 200,000
Not for value shares(W1.1) 66,667
Diluted number of shares 266,667

(W1.1) Diluted earnings per share:


Effective number of shares that would be sold
100 000 x 2 = 200,000/6 = 33,333 shares
100 000 – 33,333 = 66,667 shares (sold for no
value)
After that example 5

2.Potential ordinary shares other than share options (e.g. convertible bonds (debentures) or convertible
preference shares):
If such potential ordinary shares become actual ordinary shares, not only there would be change in number of shares
but earnings may also change as entity will no longer be paying interest on convertible bonds or dividend on
convertible preference shares.

For the purpose of calculating diluted EPS, an entity shall adjust profit or loss attributable to ordinary equity holders,
and the weighted average number of shares outstanding, for the effects of all dilutive potential ordinary shares.

The effect of a conversion will be:


◼ an increase in the expected earnings; and
◼ an increase in the number of shares on conversion.

Page 4 of 12
195
Example 2: convertible debentures
There are:
• 100 000 ordinary Rs. 2 shares in issue
• 200 000 Rs. 2 convertible debentures in issue (the conversion rate is: 1 ordinary share for each debenture).

Profit for the year ended 2015 was 279 000, which included finance costs on the convertible debentures of 30
000. Tax is levied at 30%

Required:
Calculate basic earnings and diluted earnings per share to be included in the statement of comprehensive
income for the year ended 31 December 2015.

Answer:
Basic earnings per share:

Basic earnings 279 000


Weighted number of ordinary shares in issue 100
= 000

Basic earnings per share 2.79


=

Convertible bonds
Interest saving resulting into increase in earnings = Rs. 30,000
Increase in tax increased earnings = Rs. 30,000 x 30% = Rs. 9,000
Incremental earnings = Rs. 30,000 – 9,000 = Rs. 21,000
Incremental shares = 200,000 shares(one share for each debenture)
Incremental EPS = Rs. 21,000 / 200,000 shares = Rs. 0.105

These are potentially dilutive since incremental EPS is less than basic EPS.
Diluted EPS Rs. 279,000 + 21,000
= = Rs. 1.00 per share
(2012) 100,000 + 200,000

If the holder of the instrument is faced with more than one conversion option, the entity (being the issuer of the
instrument) must use the most dilutive option in the diluted earnings per share calculation. For example, if the
holder of a debenture has the option to convert the debenture into an ordinary share or to redeem it for cash,
the entity must assume that the holder will choose the ordinary shares since this will increase the number of
shares and therefore decrease dilutive earnings per share.

Number of shares
The weighted average number of shares is increased, by adding the maximum number of new shares that would be
created if all the potential ordinary shares were converted into actual ordinary shares.

IAS 33 provides for use of most dilutive option when multiple conversion options are available.

Page 5 of 12
196
Example 3: Gold Limited
Question: Gold Limited (GL) has 12,000,000 ordinary shares and Rs. 4,000,000 5% convertible bonds in issue as at
31 December 2012, there have been no new issues of shares or bonds for several years.

The bonds are convertible into ordinary shares in 2013 or 2014, at the following rates:
▪ At 30 shares for every Rs. 100 of bonds if converted at 31 December 2013
▪ At 25 shares for every Rs. 100 of bonds if converted at 31 December 2014

Total earnings for the year to 31 December 2012 were Rs. 36,000,000. Tax is payable at a rate of 30% on profits.

Required:
Calculate basic EPS and diluted EPS for the year ended 31 December 2012.

Answer:
Basic EPS Rs.36,000,000
= = Rs. 3 per share
(2012) 12,000,000 shares

Convertible bonds
Interest saving resulting into increase in earnings = Rs. 4,000,000 x 5% = Rs. 200,000
Increase in tax on increased earnings = Rs. 200,000 x 30% = Rs. 60,000
Incremental earnings = Rs. 200,000 – 60,000 = Rs. 140,000
Incremental shares (maximum) = Rs. 4,000,000 / 100 x 30 shares = 1,200,000 shares
Incremental EPS = Rs. 140,000 / 1,200,000 shares = Rs. 0.1167

These are potentially dilutive since incremental EPS is less than basic EPS.

Diluted EPS Rs. 36,000,000 + 140,000


= = Rs. 2.74 per share
(2012) 12,000,000 + 1,200,000 shares

The additional number of shares is calculated on the assumption that they were in issue from the beginning of the
year or from the date of issue whichever is later.

If new convertibles are issued during the course of the year, the additional number of shares and the earnings
adjustment are included only from the time that the convertibles were issued.

Example 4: Silver Limited


Question: Silver Limited (SL) has 10,000,000 ordinary shares in issue on 1 January 2015. There has been no new issue
of shares for several years. However, the company issued Rs. 2,000,000 of convertible 6% redeemable preference
shares on 1 April 2015.

These are convertible into ordinary shares at the following rates:


▪ On 31 March 2018: 25 shares for every Rs. 100 of convertible redeemable preference shares
▪ On 31 March 2019: 20 shares for every Rs. 100 of convertible redeemable preference shares

In the financial year to 31 December 2015 total earnings were Rs. 40,870,000. Tax is at the rate of 30%, however
preference dividend is not deductible for tax purposes.

Page 6 of 12
197
Required:
Calculate basic EPS and diluted EPS for the year ended 31 December 2015.

Note: preference dividend is not tax deductible whether the preference shares are redeemable or irredeemable..

Answer:
Basic EPS Rs. 40,870,000
= = Rs. 4.09 per share
(2015) 10,000,000 shares

Convertible Preference shares


Preference dividend saving results into increased earnings = Rs. 2,000,000 x 6% x 9/12 = Rs. 90,000
Increase in tax on increase in earning = NIL (since there is no relief on preference dividend)
Incremental shares (maximum) = Rs. 2,000,000 / 100 x 25 shares x 9/12 = 375,000 shares
Incremental EPS = Rs. 90,000 / 375,000 shares = Rs. 0.24

These are potentially dilutive since incremental EPS is less than basic EPS.
Diluted EPS Rs. 40,870,000 + 90,000
= = Rs. 3.95 per share
(2015) 10,000,000 + 375,000 shares

Example 5: Bronze Limited(share options)


Question: Bronze Limited (BL) had total earnings during Year 2013 of Rs. 25,000,000. It has 5,000,000 ordinary shares
in issue.

There are outstanding share options on 400,000 shares, which can be exercised at a future date, at an exercise price
of Rs. 25 per share.

The average market price of shares in BL during Year 2013 was Rs. 40.

Required:
Calculate basic EPS and diluted EPS for the year ended 31 December 2013.

Answer:
Basic EPS Rs. 25,000,000
= = Rs. 5 per share
(2013) 5,000,000 shares

Share options
There is no effect of conversion of shares into ordinary shares on earnings. Only consider issue for no value in
number of shares.

For value: 400,000 options x 25 = 10,000,000 / 40 = 250,000 shares


For no value: 400,000 – 250,000 = 150,000 shares
Diluted EPS Rs. 25,000,000 + 0
= = Rs. 4.85 per share
(2013) 5,000,000 + 150,000 shares

Anti dilution:
Not all potential ordinary shares are dilutive, they may be anti-dilutive.

Page 7 of 12
198
Definition
Anti-dilution is an increase in earnings per share or a reduction in loss per share resulting from the assumption that
convertible instruments are converted, that options or warrants are exercised, or that ordinary shares are issued
upon the satisfaction of specified conditions.

When potential ordinary shares are anti-dilutive


When potential ordinary shares are anti-dilutive, they are disregarded in the calculation of diluted EPS. The following
summary is helpful:

Options and warrants


Anti-dilutive, when Options are ‘out of money’. [it means they will not be expected to convert into ordinary shares]
Options are ‘out of money’ when exercise price of the option exceeds market price. Nobody would pay an exercise
price of Rs. 100 for something worth only say Rs. 80.

Convertible bonds/debentures/redeemable preference shares


Incremental EPS = Interest or dividend for current period (net of tax) / ordinary shares on conversion
Anti-dilutive, when:
Incremental EPS > (greater) basic earnings per share

Convertible irredeemable preference shares


Incremental EPS = Dividend for current period / ordinary shares on conversion
Anti-dilutive, when:
Incremental EPS > basic earnings per share

Multiple dilutive instruments


Many companies have more than one type of dilutive instruments in issue. Some of these instruments will be
more dilutive than others. If you recall, the objective of dilutive earnings per share is to show the most dilutive
option or ‘worst case scenario’. In order to achieve this all instruments must be ranked (most dilutive to least
dilutive) and the correct combination that lowers dilutive earnings per share the most must be chosen. The
instrument that has the lowest incremental earnings per share is the most dilutive and is ranked first.

In this case, the diluted EPS is calculated in two steps:


1. Rank in order of Dilution.
2. Testing whether or not dilutive and select the most dilutive combination.

In the money options, which have no effect on earnings but do have an effect on the number of shares, will
thus have a zero incremental earnings per share and will always be the most dilutive instrument. (and therefore
always ranked first)

Page 8 of 12
199
Example 6: Olympics Limited
Question: The following information relates to Olympics Limited (OL) for the year ended 31 December 2015.
Number of ordinary shares in issue 2,000,000
Profit after rax Rs. 6,000,000
Average market price of shares during the year Rs. 80
Potential ordinary shares:
Options 600 ,000 options, with an exercise price of Rs. 60
1,000,000 7% convertible Each preference share is convertible in 2018 into ordinary shares at
preference shares of Rs. 10 the rate of 3 ordinary share for every 10 preference shares
each
4% convertible bonds of Each bond is convertible in 2019 into ordinary shares at the rate of
Rs. 5,000,000 20 new shares for every Rs. 100 of bonds.

Tax rate is 30%.

Required:
Calculate basic EPS and diluted EPS for the year ended 31 December 2015.

Note: if there is no information then assume that convertible preference shares are redeemable.

Answer:
Basic EPS Rs. 6,000,000
= = Rs. 3.00 per share
(2015) 2,000,000 shares
Diluted basic earnings per
share(W.1) 1.94

W.1) Step 1:
Ranking in order of
dilution Ranking:
Increase in earnings/Increase in 5,000,000x4%x70%/ 0.14
Convertible bonds shares 5,000,000/100x20 dilutive 2
Convertible Increase in earnings/Increase in 1,000,000x10x7%*/ 2.33
preference shares shares 1,000,000/10x3 dilutive 3
Options Note 1 0.00*
0/150 000 dilutive 1
*This will always be zero.

*preference dividend is not tax deductible.

Note 1:
For value: 600,000 options x 60 = 36,000,000 / 80 = 450,000 shares
For no value: 600,000 – 450,000 = 150,000 shares

Page 9 of 12
200
Step 2: Testing whether dilutive or
not
Basic earnings 6 000 000 3.00
Basic number of shares 2 000 000
Adjust for:
1. Options 6 000 000 + 0 6 000 000 2.79
2 000 000 + 150 000 2 150 000 Dilutive

2. Options &
convertible bonds 6 000 000 + 0 + 140 000 6 140 000 1.94
2 150 000 + 1 000 000 3 150 000 Dilutive

3. Options,
convertible bonds & 6 140 000 + 700 000 6 840 000 1.98
convertible preference shares 3 150 000 + 300 000 3 450 000 Anti-
dilutive
On the basis of above working our diluted EPS is 1.94 per share.

Presentation requirements [para 66]


An entity should present in the statement of profit or loss:
• the basic EPS and
• the diluted EPS

for the profit or loss from continuing operations.

The basic EPS and diluted EPS should be presented with equal prominence for all the periods presented (the current
year and the previous year). These figures are presented at the end of the statement of profit or loss.

If there is a discontinued operation, the basic EPS and diluted EPS from discontinued operation should be shown
either on the face of the statement of profit or loss or in a note to the financial statements. [para 68]

The basic and the diluted EPS should be presented, even if it is a negative figure (means even if it is a loss per share).

If diluted earnings per share is reported for at least one period, it shall be reported for all periods presented, even if
it equals basic earnings per share. If basic and diluted earnings per share are equal, dual presentation can be
accomplished in one line in the statement of comprehensive income [means by writing basic and diluted EPS
combined].

Disclosure requirements [para 70]


IAS 33 also requires disclosure in a note to the financial statements of the following:
• The total amounts used as the numerators (total earnings figures) to calculate the basic EPS and diluted EPS,
and a reconciliation of these numerator figures to the profit or loss for the period
• The total amounts used in the denominators (weighted average number of shares) to calculate the basic EPS
and diluted EPS, and a reconciliation of these two denominator figures to each other.

Page 10 of 12
201
Additional measure of EPS [para 73]
An entity may disclose, in addition to basic and diluted earnings per share, amounts per share using a reported
component of profit other than one required by IAS 33, for example, EPS based on operating profit or profit from
major segment of the business. In such case, the following requirements apply:
• such amounts shall be calculated using the weighted average number of ordinary shares determined in
accordance with IAS 33.
• Basic and diluted amounts per share relating to such a component shall be disclosed with equal prominence
and presented in the notes (not on face of profit or loss).
• An entity shall indicate the basis on which the numerator(s) is (are) determined, including whether amounts per
share are before tax or after tax.
• If a component of profit is used that is not reported as a line item in the statement of profit or loss, a
reconciliation shall be provided.

USEFULNESS OF EPS AS FINANCIAL INFORMATION


EPS is an important component of determining an entity’s P/E ratio i.e. calculated as market value per share divided
by earnings per share. P/E ratio is often used by investor in making stock market decisions. The reliable EPS means
reliable P/E ratio leading to better and informed decisions by investors.

Investors and their advisers pay close attention to an entity’s net profit for the period. However, profit for the period
can include large and unusual items and also the results of discontinued operations. This may make it volatile i.e.
liable to fluctuate rapidly up and down. Users can then find it difficult to assess trends in the profit figure or to use
the current year’s profit to predict an entity’s performance in future years.

Reasons as to why EPS is more reliable indicator of future performance or relevance of EPS to shareholders:
The trend (improvement or deterioration) in an entity’s published EPS figure can sometimes be a more reliable
indicator of future performance. There are a number of reasons for this.
• The standard version of both basic and diluted EPS is based on profit from continuing operations. This means
that the results of discontinued operations (which may distort total profit) are excluded.
• An entity may also choose to present one or more alternative versions of EPS. These normally exclude large or
unusual items so that EPS is based on ‘normal’ recurring earnings (e.g. gain or loss on disposal of non-current
assets).
• EPS measures an entity’s performance from the viewpoint of investors. It shows the amount of earnings
available to each ordinary shareholder (mean every shareholder can calculate how much he has earned on his
investment).
• Diluted EPS can provide an ‘early warning’ of any changes to an investor’s potential return on their investment
due to future share issues.

Limitations of earnings per share


EPS is probably the single most important indicator of an entity’s performance. It is a very useful measure when it is
used as the starting point for a more detailed analysis of an entity’s performance.

Page 11 of 12
202
However, EPS can have serious limitations:

Not all entities use the same accounting policies*. It may not always be possible to make meaningful
comparisons between the EPS of different entities.

EPS does not take account of inflation, so that growth in EPS over time might be misleading (means e.g
suppose 2016 profit figure is inflated as compared to 2015;so an increase in EPS in 2016 might only have
covered inflation impact)
• EPS measures an entity’s profitability, but this is only part of an entity’s overall performance. An entity’s cash
flow can be just as important as its profit (and more essential to its immediate survival).
• Diluted EPS is often described as an ‘early warning’ to investors that the return on their investment may fall
sometime in the future. However, diluted EPS is based on current earnings, not forecast earnings. This means
that it may not be a reliable predictor of future EPS.

One of the main problems with EPS can be the way that it is used by investors and others. Users often rely on EPS as
the main or only measure of an entity’s performance. Management know this and try to make EPS appear as high as
possible. They may attempt to manipulate the figure by using ‘creative accounting’ (e.g. fake recording of invoices).
They may also make decisions which increase EPS in the short term but which damage the entity in the longer
term.[e.g. to increase the profit credit sale is increased without properly scrutinising the customers]

*Accounting policies includes inventory valuation policy, property, plant and equipment valuation policy

Page 12 of 12
203
Extra Practice Questions
Question 1
You have recently been appointed the accountant of CP Products Limited. Your first assignment was to draw up the
financial statements of the company for the year ended 30 September 2014. This you have done, including earnings
and dividends per share.

The Managing Director, instead of praising you for your technical expertise as you expected, wants to know why you
changed last year’s number of shares when calculating the earnings per share for the comparative statement of
comprehensive income. He points out accusingly that last year’s statement of financial position reflects only 100,000
ordinary shares and that the 200,000 shares that you have reflected have only been in issue since half-way through
the current year. Furthermore, he wants to know why you included the profit of 80,000 made on the sale of
investments during the year. He believes that this should be excluded.

Required:
a) Explain to the managing director all the circumstances under which the previous year's comparative figures for
earnings per share should be restated. Give reasons why the restatement is necessary in each case.
b) Explain why the profit on the sale of investments was included in the amount of earnings used for the earnings
per share calculation.
c) Explain why the earnings per share figure is a better indicator of performance than
• dividends per share
• profit after tax

Question 1A
ROGER LIMITED
DRAFT RESULTS OF OPERATIONS
FOR THE YEAR ENDED 31. DECEMBER 2018
2018 2017

Profit before tax 750,000 730,000


Income tax expense (400,000) (300,000)
Profit for the period 350,000 430,000
Ordinary dividend declared (40,000) (30,000)
Preference dividend declared (32,000) (32,000)
Retained earnings for the year 278,000 368,000
Retained earnings - beginning of the year 568,000 200,000
Retained earnings - end year 846,000 568,000

Page 1 of 7
204
Additional information:
• The company's share capital at 31 December 2018 was as follows:
• 500,000 ordinary share capital at par value of 0.50 each.
• 200 000 8% non-cumulative non-redeemable preference shares of 2 each.
• On 30 September 2018, the company announced a rights issue of 1 ordinary share for
every 3 shares held at a price of 2.20. The market price at this date was 2.50. All the shareholders took up the
offer on this date. Prior to this date all shares issued were issued at par value.
• The normal tax rate for both years was 40%.

Required:
Prepare extracts from the statement of comprehensive income and statement of changes in equity of Roger
Limited in terms of International Financial Reporting Standards for the year ended 31 December 2018.

Comparatives and the earnings per shore note are required.

Question 2 [Basic EPS with errors]


Hubbard Limited’s bookkeeper drew up the following draft statement of comprehensive income for the year:
HUBBARD LIMITED
STATEMENT OF COMPREHENSIVE INCOME
FOR THE YEAR ENDED 30 JUNE 2016

2016 2015
Sales 500,000 400,000
Cost of sales (250,000) (200,000)
Gross profit 250,000 200,000
Other expenses (110,000) (103,000)
Profit on sale of plant 7,000 0
Interest received 3,000 3,000
Profit before tax 150,000 100,000
Income tax expense (40,000) (35,000)
Profit for the period 110,000 65,000

The following are extracts from the draft statement of changes in equity for the year ended: I

HUBBARD LIMITED
STATEMENT OF CHANGES IN EQUITY
FOR THE YEAR ENDED 30 JUNE 2016
Retained earnings
2016 2015

Opening balance 81,000 25,000


Profit for the period 110,000 65,000
Transfer to reserves (7,000) 0
Ordinary dividends (10,000) (5,000)

Page 2 of 7
205
Preference dividends (2,000) (4,000)
Closing balance 172,000 81,000

Additional information
• Before the 2016 financial statements were published, it was discovered that tax of 2015 was overstated by
3,500.
• The company had operated with an ordinary share capital of 100,000 (200 000 shares of Rs.0.5 each) for a
number of years. On 31 March 2015, 50,000 new shares were issued at a premium of Rs.10 each. On 1 January
2016, the directors decided to split the share capital into shares of Rs.0.25 in order to improve the shares’
marketability.
• The dividends paid to the ordinary shareholders were declared as follows:
2016 2015
31 December 4,000 -
30 June 6,000 5,000
Total 10,000 5,000
• The preference dividends in 2015 include Rs. 2,000 dividends owing in respect of 2014. A dividend was not
declared in 2014 as a loss was incurred in that year. (Therefore it means cumulative preference shares)

Required:
In so far as the information is available, prepare the statement of comprehensive income (including basic EPS and
its disclosures) and column of retained earnings in statement of changes in equity of Hubbard Limited for the year
ended 30 June 2016 in terms of International Financial Reporting Standards. Comparatives are required.

Question 3
Trini Limited operates in the retail sector and is listed on the PSE. The following extract of information is available
for its financial year ended 31 December 2018.
TRINI LIMITED
STATEMENT OF FINANCIAL POSITION
FOR THE YEAR ENDED 31 DECEMBER 2017
2017
Equity and Liabilities
Issued ordinary shares of Rs.2 par value each 1,000,000
Share premium 200,000
The correctly calculated net profit after tax amounted to Rs.3,220,000 for 2018 (2017: Rs 2,125,000)'

Additional information
On 30 April 2018, Trini Limited issued 125,000 shares at their market value of Rs.5 per share. Another issue of 30,000
shares took place on 30 November 2018 at their market value of Rs.7 per share. A further issue of 30,000 shares
took place on 20 January 2019 at their market value of Rs.7 per share.
Trini limited had a rights issue on 30 May 2018, the terms of which were as follows:
• One share was offered at an exercise price of Rs.3 for every 4 shares held on 30 May 2018, the market price
immediately before the issue was Rs.5 per share. All shares offered were taken up.

Page 3 of 7
206
On 31 October 2018, Trini Limited consolidated its shares such that every 5 shares were consolidated into 2 shares.
An ordinary dividend of Rs.275,000 on 30 December 2018. On 29 December 2017 the ordinary dividend declared
was Rs.200,000.

Required:
Disclose the earnings per share in the Statement of comprehensive income and in the related note to the financial
statements of Trini Limited for the year ended 31 December 2018 in accordance with IFRS. (Comparatives are
required)

Question 4
Sprog Limited had a profit for the year ended 2015 of Rs.20,000,000. Details regarding the company’s share capital
and potential share capital at 31 December 2015 are overleaf.
• There are 1,000,000,000 authorised ordinary shares (with a par value of Rs.4.50), of which 10% are in issue.
• There are 500,000 convertible debentures in issue. These debentures may be converted into ordinary shares in
a ratio of 100 ordinary shares for every 1 debenture held, (at the option of the debenture holder), on the 31
December 2018. Any debentures not converted at this date will be redeemed. Finance charges on these
debentures of Rs. 1,505,000 were incurred during 2015.
• There were no movements in share capital during 2015.
• No dividends were declared in 2015.
• Ignore Tax.

Required:
Disclose earnings per share in Sprog Limited’s statement of comprehensive income for the year ended 31 December
2015.

Question 5
Details of Laser Limited’s profits (or losses) for 2015 and 2014 are as follows:
• Profit for the year: Rs.125,000 (2014: loss of 50,000).

Details of Laser Limited’s share capital and potential share capital include the following:
• At 1 January 2014 there were 100,000 ordinary shares with a par value of Rs.1.75 in issue.
• On 30 November 2014, 12,000 ordinary shares were issued at a premium of Rs.0.25 per share. There have been
no other issues since 30 November 2014.
• On 31-12-2014 there are 25,000 options in issue entitling the option holder to 1 ordinary share for each option
at a strike price of Rs.2.00 per share, (the average market price of an ordinary share for 2015: Rs.2.75).

Other information includes:


• An interim ordinary dividend of Rs.0.04 per share was declared and paid on the 30 June 2015. On 15 December
2015 a final ordinary dividend of Rs.2,800 was declared.
• No dividends were declared in 2014 due to the loss made in 2014.
• Normal company tax is levied at 35%.

Required:
Disclose earnings per share for the year ended 31 December 2015 in the Laser Limited’s statement of comprehensive
income along with comparative.

Page 4 of 7
207
Question 6
Rebel Limited had the following draft statement of comprehensive income:
STATEMENT OF COMPREHENSIVE INCOME
FOR YEAR ENDED 31 DECEMBER 2015 (DRAFT)
2015 2014

Basic earnings per share 0.20 1.75


Basic diluted earnings per share 0.20 1.75
Dividends per share 0.05 0.00

The financial accountant of Rebel Limited resigned shortly before year end, leaving the bookkeeper to draw up the
draft annual financial statements. Numerous errors have been made by the bookkeeper. The errors are as follows:
• In both years, the irredeemable preference shareholders receive a fixed dividend of Rs.5,000 a year. This has
been ignored in the calculation of earnings per share.
• During further investigations, the bookkeeper revealed that no additional work had been done for dilutive
earnings per share as the bookkeeper was under the assumption that the calculations were highly complex and
that in their small business the figures would be the same as basic earnings per share.

In your investigations you came across the following working paper:

Actual 2015 Weighted 2015 Adjusted 2014


1 January 20,000 20,000 20,000
Rights issue: ‘for value’ portion 60,000 60,000 0
Sub-total 80,000 80,000 20,000
Rights issue: ‘not for value’ portion 20,000 20,000 0
31 December 100,000 100,000 20,000

At 1 January 2014 there were 20 000 ordinary shares in issue. The rights issue (which took place on the 30 June
2015) was on a ‘4 for 1’ basis, at a strike price of Rs. 1.50 per share. The market value per share immediately
before the rights issue was Rs.2.00.

The following potential ordinary shares are in issue:


Convertible debentures (convertible at the option of the debenture holders). These are convertible into 5,000
ordinary shares on 31 December 2017. If not converted, the debentures will be redeemed on 31 December 2017.
Finance costs of Rs.100 are incurred annually on these debentures. The convertible debentures were issued half way
through 2014.

No dividends were declared in 2014, but in the following financial year (early 2015, before the financial statements
were authorised and issued) a dividend of Rs.0.03 per share was declared. Other dividend declarations in 2015
included an interim dividend of Rs.0.05 per share (declared in June 2015) and a final dividend of Rs.0.02 per share
(declared in December 2015).

Ignore Tax

Required:
Recalculate the correct earnings per share figures and disclose them in the statement of comprehensive income of
Rebel Limited for the year ended 31 December 2015 (with comparatives)

Page 5 of 7
208
Question 7
The following relates to Early Morning Limited for the year ended 31 December 2015:
1. Profit for the year Rs.500,000 (2014: Rs.337,500). This profit includes a profit from a discontinued operation
(after tax) of Rs.52,500 (2014: Rs.0).
2. On 1 January 2014 there were 250 000 ordinary shares each with a par value of Rs.5.00 in issue. On the 30
September 2014 there was a rights issue on a basis of 1 ordinary share issued for every 5 already held at a price
of Rs.6.00 The market value of the ordinary shares immediately before the rights issue was Rs.7.50 per share.
On 31 May 2015 there was an issue of 50,000 ordinary shares at market price (Rs.5 per share).
3. There are 25,000 options in existence, each of which allows the holder to acquire four shares at a strike price of
Rs.10.00 per share. The average market price per ordinary share for 2014 and 2015 was 12.00. These options
were in existence throughout 2014 and 2015.
4. Preference shares in issue are convertible (at the option of the preference shareholders) into 500 ordinary
shares on 31 December 2017. If not converted, the preference shares will be redeemed on 31 December 2017.
Dividends of Rs. 1,000 are incurred annually on these preference shares (these have been correctly accounted
for as finance charges). The preference shares were in existence throughout 2014 and 2015.
5. Ignore tax.

Required:
Disclose earnings per share in the financial statements of Early Morning Limited for the year ended 31 December
2015. (with comparatives)

Question 8
L, G and M Limited is a listed company.
Details for their current reporting period is as follows:
The correctly calculated profit after tax for the year ended 31 December 2018 is Rs. 550,000 (2017: Rs. 400,000)
At 31 December 2017 the following were in issue:
1. 350,000 ordinary shares of Rs. 3 par value each
2. 300,000 10% redeemable, cumulative preference shares of Rs.3 each. The preference dividends for 2018 have
not yet been declared. The 2017 preference dividends were declared on 30 November 2017.
3. Options to acquire 40,000 ordinary shares in L, G and M limited after 30 November 2019 at a strike price of Rs.
6 per share. The average market price of the shares during 2017 and 2018 was Rs.10 per share

The following transactions took place in 2018:


• 1 January 2018 200,000 12% convertible debentures of Rs.2 each were issued. These debentures are convertible
on 30 September 2019 at the option of the debenture holders into L, G and M limited ordinary shares at a rate
of one ordinary share for two debentures. If not converted into ordinary shares they will be redeemed on 30
September 2019.
• On 31 March 2018, there was a capitalization issue of 2 ordinary shares for every 5 shares in issue. The
capitalization issue utilized current share premium as far as is possible so as to minimize the impact of this issue
on retained earnings.
• On 30 September 2018 50,000 ordinary shares were issued at their market value on that date. Another such
issue of 10,000 shares took place on 30 November 2018 at their market value on that date.
• On 20 October 2018, 30,000 ordinary shares were offered to existing shareholders. The offer was fully
subscribed for at Rs.7 per share on 31 October 2018, when the market price was Rs.10 per share(entitlement
date).
• The corporate tax rate is 30%

Page 6 of 7
209
Required:
Disclose the EPS in the statement of Comprehensive income and the notes to the financial statements of L, G and
M limited for the year ending 31 December 2018 in accordance with IFRS. (along with comparatives)

Question 9
The following information is for the year ended 30.06.2021:
Profit from continuing operations 16,400,000
Less dividends on preference shares (6,400,000)
Profit from continuing operations attributable to ordinary equity holders 10,000,000
Loss from discontinued operations (4,000,000)
Profit attributable to ordinary equity holders 6,000,000
Ordinary shares outstanding 2,000,000
Average market price of one ordinary share during year 75

Potential ordinary shares:


Options 100,000 with exercise price of 60
Convertible irredeemable 800,000 shares with a par value of 100 entitled to a cumulative dividend
preference shares of 8 per share. Each preference share is convertible to two ordinary
shares.
5% convertible bonds Nominal amount 100,000,000. Each Rs. 1,000 bond is convertible to 20
ordinary shares.
Tax rate 40%

Required: Calculate the basic and diluted Earnings per share for the year ended 30.06.2021.

Page 7 of 7
210
Solution 1
a) The following situations will result in comparatives for earnings per share to be restated:

A capitalisation issue or a share split (i.e. a not for value issue) and share consolidation
(i.e a not for value reduction)

A change in accounting policy or a correction of an error (As per IAS-8)

With a capitalisation issue or a share split and share consolidation no new cash resources are available to the
company. The number of shares increases resulting in a decrease in the EPS. Therefore, the comparative EPS must
be restated to ensure that comparability is not lost. This adjustment applies not only to the prior period but to the
figures of all previous periods that are presented as comparatives as well.

A change in accounting policy or a correction of an error gives rise to a prior year adjustment in terms of IAS 8.
With a prior year adjustment it is necessary to restate the previous year’s comparatives and the retained earnings
at the beginning of the prior year. If the adjustment has an impact on the ‘earnings’ used for the earnings per
share calculation, it will therefore be necessary to restate the earnings per share figure for the previous year.

(b) IAS 33 requires earnings per share to be based on basic earnings, which is defined as the profit or loss for the
period attributable to ordinary shareholders after deducting preference dividends. In terms of IAS 8, profit for the
period should include all items of income and expense recognised in a period. The profit on sale of investments is
an other income should be included in profit for the period and therefore should be included in basic earnings as
well.

(c) Dividends per share depends on the dividend payout policy of a company, and not necessarily on the size of its
profits. It is not possible to judge a company’s performance on its dividends declared. In new or expanding
companies, for example, it would be irresponsible to adopt too high a dividend payout ratio. A low dividend per
share in such cases would not necessarily reflect poor performance - management may just be retaining the profits
in order to re-invest in the business. Earnings per share, on the other hand, is based on profit earned by the
business regardless of whether such funds are being paid out to the owners or are being re-invested to increase
the value of the business.

Profit after tax on its own does not tell shareholders the extent of the return on their investment. For example if
two companies both reflect profit after taxation for the year of Rs.100,000 but company A has 1,000 shares and
company B has 2,000 shares, it cannot be said that a shareholder with one share in each of the companies has
earned the same amount on each investment, even though the profits earned by each company are the same. The
one share held in company A has yielded a Rs.100 return whereas the one share in company B has only yielded a
Rs.50 return once the profits have been shared out amongst the owners. It is therefore more meaningful to look at
earnings per share than at total earnings.

Page 1 of 19
211
Solution 1A

ROGER LIMITED

EXTRACTS FROM THE STATEMENT OF COMPREHENSIVE INCOME FOR THE YEAR ENDED 31 DECEMBER
2018
Notes 2018 2017

Profit before tax 750 000 730 000


Income tax expense (400 000) (300 000)
Profit for the year 350 000 430 000

Basic earnings per ordinary share


0.3832 0.5147

ROGER LIMITED

EXTRACTS FROM THE STATEMENT OF CHANGES IN EQUITY

FOR THE YEAR ENDED 31 DECEMBER 2018

Ordinary Preference Share Retained Total


shares shares premium earnings
Opening balance at 1/1/2017 375 000 400 000 0 200 000 975 000
(750,000 x 0.5)
Profit for the year 430 000 430 000
Preference dividends (32 000) (32 000)
Ordinary dividends (30 000) (30 000)
Opening balance at 1/1/2018
(W3) 375 000 400 000 0 568 000 1 343 000
Rights issue 125 000 425 000 550 000
250,000 x 0.5 = 125,000
250,000 x (2.2 – 0.5) = 425,000
Profit for the year 350 000 350 000
Ordinary dividends (40 000) (40 000)
Preference dividends (32 000) (32 000)
Closing balance at 31/12/2018 500 000 400 000 425 000 846 000 2 171 000

Page 2 of 19
212
ROGER LIMITED

EXTRACTS FROM THE NOTES TO THE FINANCIAL STATEMENTS FOR THE YEAR ENDED 31 DECEMBER
2018

Earnings per share

Basic earnings per share


The calculation of earnings per share is based on earnings of 318 000 (2017: 398 000) and a
weighted average of 829 897 ordinary shares (2017: 773 196) after adjusting for the rights issue on 30th
September 2018.

Reconciliation of profit to earnings 2018 2017

Profit/(loss) for the period 350 000 430 000


Preference dividends (32 000) (32 000)
Basic earnings 318 000 398 000

EPS 2018 2017


350,000 – 32,000 430,000 – 32,000
829,897 773,196

0.3832 0.5147

W2. Number of Shares (rights issue) - an alternative calculation


Actual Current weighted Prior Adjusted
2018 2018 2017
b/d 1-1-18 (W 3) 750,000 750,000 750,000
issue for value (30.9) 220,000 55,000 0
(220,000 x 3/12)
970,000 805,000 750,000
Issue for no value (30.9) 30,000 24,897 23,196
(30/970 x 805000) (30/970 x 750000)
Balance 21-12-2018 1,000,000 829,897 773,196

750,000/3 x 1 = 250,000 x 2.2/2.5 = 220,000 ( for value)


250,000 – 220,000 = 30,000 ( for no value)

Page 3 of 19
213
W3 Number of shares at beginning of the year

the number of shares at year-end = 500 000 / 0.50 = 1 000 000 shares let X = the number of shares in
issue at the beginning of the year then :

X + X/ 3 x 1 = 1 000 000 shares

1.3333X = 1 000 000

X = 1 000 000/1.3333 = 750 000 shares

check:

750 000 / 3 x 1 = 250 000 (issued through rights issue)

opening balance + rights issue = balance at year-end

750 000 + 250 000 = 1 000 000

Solution 2

HUBBARD LIMITED

EXTRACTS FROM THE STATEMENT OF COMPREHENSIVE INCOME FOR THE YEAR ENDED 30 JUNE 2016
Notes 2016 2015

Revenue 500 000 400 000


Cost of sales (250 000) (200 000)
Gross profit (2016: 7 000 + 250 000 200 000
Other income 3 000) 10 000 3 000
Other expenses (110 000) (103 000)
Profit before tax 150 000 100 000
Income tax expense (2016: 40 000 (40 000) (31 500)
2015: (35 000 – 3500)
Profit for the period 110 000 68 500

Earnings per share 10 0.2160 0.1565

Page 4 of 19
214
HUBBARD LIMITED
STATEMENT OF CHANGES IN EQUITY
FOR THE YEAR ENDED 30 JUNE 2016
Retained
earnings

Balance at 01.07.2014 25 000


Profit for the period [restated] 68 500
Ordinary dividend (5 000)
Preference dividend (4 000)
Balance at 30/6/2015 as restated 84 500
Profit for the period 110 000
Ordinary dividend ( 4,600 +
6,000) (10 000)
Preference dividend (2 000)
Transfer to reserves (7 000)
Balance at 30/6/2016 175 500

HUBBARD LIMITED
NOTES TO THE FINANCIAL STATEMENTS
FOR THE YEAR ENDED 30 JUNE 2016

Earnings per share

Basic earnings per share:


The calculation of earnings per share is based on earnings of 108,000 (2015: 66,500) and on the
weighted average of 500 000 shares in issue (2015: 425 000) after the share split on 1 January 2016.
Comparatives have been restated.

Reconciliation of profit to earnings 2016 2015

Profit for the period 110 000 68 500


Preference dividends (2 000) (2 000)
250
Basic earnings 108 000 66 500

Page 5 of 19
215
Workings of EPS:
2016 2015

110,000 – 2,000 66,500 – 2,000


500,000 425,000

0.216 0.1565
Prior period Error: There was an error in calculation of tax during the year 2015. Comparatives have been
appropriately restated.

The effect of the correction is as follows: 2015

Effect on the statement of comprehensive income

decrease in tax expenses 3,500


Increasee in profits 3,500

Effect on the statement of financial position

Decrease in tax payable 3 500


Increase in Retained
earnings 3 500

WORKINGS :

W.1 2016:
Actual 2016 2015
Current Prior
(weighted) (Adjusted)
b/d 250,000 250,000 212,500(W.2)
For no value (1.1) 250,000 250,000 212,500
(250/250
×212,500)
c/d 500,000 500,000 425,000
[250,000/1 ×2]

W. 2 2015
(2015) (2015)
Actual Current
(weighted)
b/d 200,000 200,000
For value (31.3) 50,000 12,500
(50,000×3/12)
c/d 250,000 212,500

Page 6 of 19
216
Solution 3

TRINI LIMITED

STATEMENT OF COMPREHENSIVE INCOME


FOR THE YEAR ENDED 31 DECEMBER 2018
2018 2017

Profit for the year 3 220 000 2 125 000


Basic earnings per share (W.2) 11.50 9.78

TRINI LIMITED

EXTRACTS FROM THE NOTES TO THE FINANCIAL STATEMENT

FOR THE YEAR ENDED 31 DECEMBER 2018

Earnings per share Basic earnings per share


The calculation of basic earnings per share is based on earnings of 3 220 000 (2017: 2 125 000)
and a weighted average of 279 900 (2017: 217 391) ordinary shares in issue during the year.

Reconciliation of earnings 2018 2017

Profit for the year 3 220 000 2 125 000


Basic earnings 3 220 000 2 125 000

Workings: W.1 2016:


Actual 2018 2017
Current Prior
(weighted) (Adjusted)
Balance 1/1/18 500 000 500 000 500 000
30 April for value

125 000 83 333 -


625 000 583 333 500 000
30 May rights issue for value 93 750 54 688 -
718 750 638 021 500 000
30 May rights issue for no value 62 500 55 480 43 478
781 250 693 501 543 478
31 Oct share consolidation
(bal.)
(781,250/5 x 2) (468 750 ) (416 101) (326 087)

Page 7 of 19
217
312 500 277 400 217 391
30 Nov for value issue 30 000 2 500 -
c/d [250,000/1 ×2] 342,500 279,900 217,391

625,000/4 x 1 x 3/5 = 93,750 [For Value]


625,000/4 = 156,250 – 93,750 = 62,500 [For Value]

W.2

Basic earnings per share 3 220 000 2 125 000


279 900 217391
= 11.50 = 9.78
Solution 4

SPROG LIMITED

EXTRACTS FROM THE STATEMENT OF COMPREHENSIVE INCOME FOR THE YEAR ENDED 31 DECEMBER
2015
2015

Profit for the year 20 000 000

Earnings per share:


Basic 20 000 000 ÷ (10% x 1 000 000 000) 0.20
Diluted basic (W-1) 0.14

Workings

W1: Dilutive earnings per share


Incremental EPS = 1,505,000/50,000,000* = 0.0301 Dilutive
*500,000/1 x 100

Revised earnings 20 000 000 + 1 505 000


Revised shares = 100 000 000 + 50 000 000

= 21 505 000 / 150 000 000


= 0.1434

Page 8 of 19
218
Solution 5

LASER LIMITED

EXTRACTS FROM THE STATEMENT OF COMPREHENSIVE INCOME FOR THE YEAR ENDED 31 DECEMBER
2015
2015 2014

Profit/ (loss) for the year 125 000 (50 000)

Earnings per share: (125,000/112,000)


Basic earnings (50,000/101,000) 1.116 (0.495)
(125,000/118,818)
Diluted basic (50,000/107,818) 1.052 (0.464)

Workings

W1: Basic earnings 2015 2014


Profit for the year 125 000 (50 000)
Preference dividends 0 0
Basic Earnings 125 000 (50 000)
W-2) 2014:

2014 Actual 2014 Current


(Weighted)
b/d 100,000 100,000
30.11 for value 12,000 1000
(12,000 x 1/12)
c/d 112,000 101,000

W.2.1) 2015. No Change in Shares:


2015 Actual 2015 Current 2014 Prior
(Weighted) (Adjusted)
b/d 112,000 112,000 101,000 (W.2)
c/d 112,000 112,000 101,000

W3: Dilutive number of shares


Options (bonus/ free portion) 25 000 – (25 000 x 1 x 2.00) ÷ 2.75 = 6 818

Dilutive number of shares 2014: (101 000 + 6 818*) = 107 818


2015: (112 000 + 6 818) = 118 818

*Options are in comparative year as on 31.12.2014.

Page 9 of 19
219
Solution 6

REBEL LIMITED
EXTRACTS FROM THE STATEMENT OF COMPREHENSIVE INCOME FOR THE YEAR ENDED 31 DECEMBER
2015
2015 2014
(15,000/62,500)
Basic earnings per share (30,000/25,000) 0.2400 1.2000
Basic diluted earnings per share (W4) 0.2237 1.0927

Workings:

W.1) Calculation of correct basic earnings


2015 2014
Incorrect basic earnings per
share Given 0.20 1.75
Incorrect number of shares Given 100,000 shares 20,000 shares
Incorrect basic earnings (0.2 x 100,000) (1.75 x 20,000) 20,000 35,000

W.2) Calculation of incorrect basic earnings


2015 2014

Incorrect basic earnings W-1 20,000 35,000


Preference shares dividends given (5,000) (5,000)
Correct basic earnings 15,000 30,000

W.3)
2015:
Actual Current 2015 Prior
(weighted) 2014(adjusted)
b/d 20,000 20,000 20,000 (W.3.1)

For value (30.6) 60,000 30,000(60,000 0


×6/12)
80,000 50,000 20,000

For no value (30.6) 20,000 12,500 5,000


(20/80 ×50,000) (20/80 ×20,000)
100,000 62,500 25,000
20,000/1×4= 80,000 × 1.5 = 120,000
120,000/2= 60,000 (for value)
80,000-60,000=20,000 (for no value)

Page 10 of 19
220
W.3.1 2014
Actual Current (weighted)
b/d 20,000 20,000
20,000 20,000

W-4) Dilutive earnings per share:


Whether dilutive or not
2014= [100 × 6/12] ÷ [5000 ×6/12]= 0.02 dilutive
2015= 100/5000= 0.02 dilutive

2015 2014
basic earning (15,000 + 100) 15,100 (30,000 + 30,050
(Financed costs saved) 100 x 6/12)
Dilutive number of shares (62,500 + 5,000) 67,500 (25,000 + 27,500
(Additional shares) 5,000 x
6/12)
Dilutive earnings per share (15,100 / 67,500) 0.2237 (30,500 / 1.0927
27,500)

Solution 7

EARLY MORNING LIMITED

EXTRACTS FROM THE STATEMENT OF COMPREHENSIVE INCOME FOR THE YEAR ENDED 31 DECEMBER
2015
2015 2014
Profit for the year 500 000 337 500

Earnings per share


Basic 500 000 / 329 166 337 500 / 268 966 1.5190 1.2548
Continuing
operations 447 500 / 329 166 337 500 /268 966 1.3595 1.2548
Discontinued
operations 52 500 / 329 166 0.1595
Diluted 500 000/ 345 833 337 500 / 285 633 1.4458 1.1816
Continuing operations 447 500 / 345 833 337 500 / 285 633 1.2940 1.1816

Discontinued operations 52,500/345,833 0.1518

Page 11 of 19
221
EARLY MORNING LIMITED
EXTRACTS FROM THE NOTES TO THE FINANCIAL STATEMENTS
FOR THE YEAR ENDED 31 DECEMBER 2015

Earnings per share


The calculation of the following earning per share are based on the following amounts
2015
Earnings Weighted Average number of
shares
Basic 500 000 329 166
Diluted basic 500 000 345 833

2014
Weighted Average number
Earnings of
shares
Basic 337 500 268 966
Diluted basic 337 500 285 633

Reconciliation of profit for the


year 2015 2014
to basic earnings
Profit for the year (after tax)

Less preference dividends ( No adjustment because classified as 500 000 337,500


liability) 0 0
500 000 337,500
Basic earnings

Reconciliation of earnings to diluted basic


earnings
2015 2014
Basic Basic
Earnings 500 000 337 500
Options 0 0
Preference shares
(not adjusted for as anti-dilutive)
Diluted basic earnings 500 000 337 500

Page 12 of 19
222
EARLY MORNING LIMITED
EXTRACTS FROM THE NOTES TO THE FINANCIAL STATEMENTS
FOR THE YEAR ENDED 31 DECEMBER 2015

Earnings per share continued …


Reconciliation of basic number of shares to diluted number of
shares
2015 2014
Basic number of shares 329 166 268 966
Options 16 667 16 667
Preference shares (not adjusted for because anti-dilutive)
Diluted number of shares 345 833 285 633

Workings

2015: Weighted average no. Of shares

Actual Current Prior 2014


weighted (adjusted)
(2015)
b/d 300,000 300,000 268,966
(W.2)
(31.5) For value 50,000 29,167 0
(50,000 ×7/12)
350,000 329,167 268,966

W. 2 2014:
Weighted average no. Of shares
Actual (2014)
Current
weighted
b/d 250,000 250,000
(30.9) for value (W) 40,000 10,000
(40,000×3/12)
290,000 260,000
(30.9) for no value (W) 10,000 8,966
(10,000/290,00
0 ×260,000)
c/d 300,000 268,966

Page 13 of 19
223
250,000/5 ×1=50,000×6= 300,000/7.5 =40,000[for value]
50,000-40,000 = 10,000[for no value]

W2: Ranking of dilutive instruments Ranking:

0.000
Options No change to earnings 0 / 16,667(w-2.1) Dilutive 1

2.0
Preference shares Dividend 1 000/500 Anti Dilutive 2

W.2.1)
25,000×4=100,000
100,000×`10=1,000,000
1,000,000/12 =83,333(for value)
100,000-83,333= 16,667(for no value)

Point to remember:
IAS 33 requires the dilutive (or anti-dilutive) effect of potential ordinary shares to be determined with reference to
basic EPS from continuing operations.

Of the total profit for the year, Rs. 52 500 (after tax) was earned from a discontinued operation. When testing to
determine whether the potential ordinary shares are dilutive, profit for the period excluding profit from
discontinued operations is used.

The actual calculation of diluted earnings per share will include profit from discontinued operations. The total
diluted earnings per share is then split into diluted earnings per share from continuing operations and diluted
earnings per share from discontinued operations (see extracts from the financial statements above).

W3: 2015 Test whether anti-dilutive


Basic earnings per share (500 000 -52 500) 447 500 = 1.3595
329 166

Options 447 500 + 0 447 500 = 1.2940 Dilutive


329 166 + 16 667 = 345 833
Options and Preference
shares 447 500 + 1 000 448 500 = 1.2950 Anti-
345 833 + 500 = 346 333 Dilutive

Page 14 of 19
224
W4: 2014 Test whether anti-dilutive
Based on shares of 2014 in 2015
Basic earnings per share 337 500 = 1.2548
268 966

Options 337 500 + 0 337 500 = 1.1816 Dilutive


268 966 + 16 667 = 285 633
Options andPreference
shares 337 500 + 1 000 338 500 = 1.1830 Anti-
285 633 + 500 = 286 133 Dilutive

Solution 8

L, G AND M LIMITED

EXTRACTS FROM THE STATEMENT OF COMPREHENSIVE INCOME FOR THE YEAR ENDED 30 SEPTEMBER
2013
2018 2017

Basic earnings / (loss) 550 000 (W.1) ÷ 514 952(W.2) ;400 000 (W.1) ÷497 861
per share (W.2) 1.068 0.8034
Diluted basic earnings 583 600 (W.3) ÷ 630 952 (W.4); 400 000 (W.3) ÷ 513 861
per share (W.4 and reconciliation) 0.925 0.778

Notes to Financial Statements


Earnings per share

The calculation of basic earnings per share is based on profit / (loss) of 550 000 (2017: 400 000) and

on the weighted average of 514 952 shares in issue (2017: 497 861) at the end of the year.

The calculation of diluted earnings per share is based on profit / (loss) of 583 600 (2017: 400 000) and on
the weighted average of 630 952 shares in issue (2017:513 861) at the end of the year.

Reconciliation of profit to earnings

2018 2017

Profit/(loss) for the period 550 000 400 000


Already
Preference dividends deducted from 0 0
profit
Basic earnings 550 000 400 000

Page 15 of 19
225
Basic earnings 550 000 400 000
Debentures finance
cost avoided (48,000 – 14,400) 33 600
Diluted basic
earnings 583 600 400 000

Reconciliation of basic number of shares to diluted shares


Basic share (w.2) 514 952 497 861
Not for value options (W.4.1) 16 000 16 000
Convertible debentures (200,000/2 x 1) 100 000 0
Diluted number of shares 630 952 513 861

1. Options were also as on 31-12-2017. Even if they are issued during the period they are adjusted
retrospectively as it is not for value.
2. Convertible debentures were issued at the beginning of the current year, i.e. as on 1-1-2018.
Therefore, no effect on 2017.

WORKINGS:
W.1 ) Basic Earnings
2018 2017

Profit After Tax 550,000 400,000


Finance cost on preference shares-redeemable 0 0
(No Adjustment)
Basic Earnings 550,000 400,000

W.2) Numbers of shares :


Actual Current Prior
2018 (weighted ) (adjusted)
b/d 350,000 350,000 350,000
(31.3) [issue for no value ] 140,000 140,000 140,000
[350,000/5 ×2]
490,000 490,000 490,000

(30.9) [issue for value ] given 50,000 12,500 0


(50,000 ×3/12)
540,000 502,500 490,000
(31.10)[for value(W-2.1)] 21,000 3,500 0
(21,000×2/12)
561,000 506,000 490,000
(31.10) [for no value (W-2.1)] 9,000 8,118 7,861
(9/561 (9/561 ×
×506,000) 490,0000
570,000 514,118 497,861

Page 16 of 19
226
(30.11) [for value ] given 10,000 834 0
(10,000×1/12)
c/d 580,000 514,952 497,861

W-2.1) 31.10.2018
30,000 ×7 = 210,000/10= 21,000 [For value]
30,000-21,000 = 9,000 [For no value]

W.3)Diluted Earnings:
2018 2017
Basic Earnings 550,000 400,000
Finance cost on convertible bonds[200,000 ×2×12%] 48,000 0*
Tax impact due to increase in profit [48,000×30%] (14,400) 0
33,600
Diluted Earnings 583,000 400,000
*Convertible bonds are not in 2017
**Redeemable preference shares are not convertible in this question therefore no impact on the diluted EPS.

W.4) Diluted EPS


(I)Ranking in order of dilution Rank
Options [0÷16,000(w-4.1)] 0 (dilutive)* 1
Convertible debanture [33,600÷100,000(200,000/2 ×1) 0.336 2
(dilutive)*
*In comparison is with basic EPS
W.4.1) 40,000 x 6 = 240,000 /10 = 24,000; [40,000 – 24,000 = 16,000]
(ii) Testing whether dilutive [2018]
Basic EPS [550,000÷514,952] 1.068
Adjustment For
Options [(550,000+0) ÷ (514,952+16,000=530,952)] 1.0359 dilutive
Option as well as convertible debantures
[(550,000+0+33,600) ÷ (514,952+16,000+100,000=630,952)] 0.925 dilutive
Lowest will be selected; i.e 0.925
Testing whether dilutive or not [2017]
Basic EPS [Restated] [400,000 ÷497,861] 0.8039
Adjustment For
Options [(400,000+0) ÷ [497,861+16,000 = 513,861] 0.778

Original diluted EPS in 2017:


(400,000 + 0)/ (350,000 +16000) = 1.09

Page 17 of 19
227
Answer 9
i) Basic EPS:
From continuing operations = 10,000,000/2,000,000 = 5 per share
From discontinuing operations = (4,000,000)/2,000,000 = (2) per share
Overall = 6,000,000/2,000,000 = 3 per share

ii)Calculation of diluted Earnings per share: (W1)


From continuing operations [13,000,000/4,020,000] 3.23
From discontinued operations [(4,000,000)/4,020,000] (0.99)
Overall [9,000,000/4,020,000] 2.24
W.1) Step 1
Ranking in order of dilution
Increase in Increase in number Earnings per RANK
earnings of ordinary shares incremental share
Options
Increase in earnings Nil
Incremental shares issued for no 20,000 0 1
considerations [100,000 – (100,000 x (dilutive)*
60/75)]
Convertible preference shares
Increase in profit 6,400,000
800,000 x 8
Incremental shares 2 x 800,000 1,600,000 4 3
(dilutive)*
5% convertible bonds
Increase in profit 3,000,000
100,000,000 x 0.05 x 60%
Incremental shares 100,000 x 20 2,000,000 1.5 2
(dilutive)*
*In comparison is with basic EPS from continuing operations:

The order in which to include the dilutive instruments is therefore:


1. Options
2. 5% convertible bonds
3. Convertible preference shares

Point to remember: IAS-33 requires the dilutive or antidilutive effect of potential ordinary shares to be determined
with reference to basic EPS from continuing operations. (Means all are dilutive compared with basic EPS of 5)
Of the total profit for the year, loss of 4,000,000 is from a discontinued operation. When testing to determine
whether potential ordinary shares are dilutive, profit for the period excluding profit from discontinued is used.
The actual calculation of diluted earnings per share will include profit from discontinued operations then total is
splitted into diluted EPS from continued and diluted EPS from discontinued operations.

Page 18 of 19
228
Step 2) Test whether Dilutive or Antidilutive:
Profit from continuing operations Ordinary Per share
attributable to ordinary equity holders shares
of the entity
As reported 10,000,000 2,000,000 5
Options - 20,000
10,000,000 2,020,000 4.95 Dilutive
5% Convertible bonds 3,000,000 2,000,000
13,000,000 4,020,000 3.23 Dilutive
Convertible preference share 6,400,000 1,600,000
19,400,000 5,620,000 3.45 Antidilutive

Because diluted earnings per share is increased when taking the convertible preference shares into account (from
3.23 to 3.45), the convertible preference are antidilutive and are ignored in the calculation of diluted earnings per
share. Therefore, diluted earnings per share for profit from continuing operation is 3.23:

Page 19 of 19
229
Question 1:
The profit after tax earned by AAZ Limited during the year ended December 31, 2007 amounted to Rs. 127.83 million.
The weighted average number of shares outstanding during the year were 85.22 million.

Details of potential ordinary shares as at December 31, 2007 are as follows:


• The company had issued debentures which are convertible into 3 million ordinary shares. The debenture holders
will exercise the option on December 31, 2009. If the debentures are not converted into ordinary shares they
shall be redeemed on December 31, 2009.The interest on debentures for the year 2007 amounted to Rs. 7.5
million.
• Preference shares issued in 2004 are convertible into 4 million ordinary shares at the option of the preference-
shareholders. The conversion option is exercisable on December 31, 2010. The dividend paid on preference
shares during the year 2007 amounted to Rs. 2.45 million.
• The company has issued options carrying the right to acquire 1.5 million ordinary shares of the company on or
after December 31, 2007 at a strike price of Rs. 9.90 per share. During the year 2007, the average market price
of the shares was Rs. 11 per share.

The company is subject to income tax at the rate of 30%.

Required:
a) Compute basic and diluted earnings per share.
b) Prepare a note for inclusion in the company's financial statements for the year ended December 31, 2007 in
accordance with the requirements of International Accounting Standards.
(18)
Question 2:
The following information relates to Afridi Industries Limited (AIL) for the year ended December 31, 2008:
i. The share capital of the company as on January 1, 2008 was Rs. 400 million of Rs. 10 each.
ii. On March 1, 2008, AIL entered into a financing arrangement with a local bank. Under the arrangement, all the
current and long-term debts of AIL, other than trade payables, were paid by the bank. In lieu thereof, AIL issued
4 million Convertible Term Finance Certificates (TFCs) having a face value of Rs. 100, to the bank. These TFCs are
redeemable in five years and carry mark up at the rate of 8% per annum. The bank has been allowed the option
to convert the TFCs on the date of redemption, in the ratio of 10 TFCs to 35 ordinary shares.
iii. On April 1, 2008, AIL issued 30% right shares to its existing shareholders at a price which did not contain bonus
element.
iv. During the year, AIL earned profit after tax amounting to Rs. 78 million. This profit includes a loss after tax for a
discontinued operation, amounting to Rs. 13 million.
v. The applicable tax rate is 35%.

Required:
Prepare extracts from the financial statements of Afridi Industries Limited for the year ended December 31, 2008
showing necessary disclosures related to earnings per share and diluted earnings per share. (Ignore corresponding
figures)

Page 1 of 2
230
Question 3:
The following information pertains to ABC Limited, in respect of year ended March 31,2010
Rs .in ‘000’
Profit for the year 13,000
Dividend paid during the year to ordinary shareholders 4,000
Dividend paid on 10% Cumulative preference shares for the year 2009 2,000
Dividend paid on 10% Cumulative preference shares for the year 2010 2,000
Dividend declared on 12% non cumulative preference shares for the year 2010 2,400
i. The dividend declared on the non-cumulative preference shares, as referred above, was paid in April 2010.
ii. The cumulative preference shares were issued at the time of inception of the company.
iii. The company had 10 million ordinary shares at March 31, 2009 of Rs. 10 each.
iv. 12% non-cumulative irredeemable preference shares having nominal value of Rs. 10 each are convertible into
one ordinary shares, on or before December 31, 2011.
v. 1.20 million right shares of Rs. 10 each were issued at a premium of Rs. 1.50 per share on October 1, 2009. The
market price on the date of issue was Rs. 12.50 per share.
vi. 20% bonus shares were issued on January 1, 2010.
vii. Due to insufficient profit no dividend was declared during the year ended March 31, 2009.
viii. The average market price for the year ended 31,2010 was Rs. 15 per share

Required: Compute basic and diluted earnings per share and prepare a note for inclusion in the consolidated
financial statements for the year ended March 31, 2010 (17)

Question 4:
The following information relates to Que Limited (QL) for the year ended 31 December 2011:
i. Issued share capital on 1 January 2011 consisted of 80 million ordinary shares of Rs. 10 each.
ii. Profit after tax amounted to Rs. 130 million. It includes a loss after tax from a discontinued operation, amounting
Rs. 40 million.
iii. On 30 September 2011, QL issued 20% right shares at a price of Rs. 11 per share. The market value of the share
immediately before the right issue was Rs. 12.50 per share.
iv. There are 25,000 share options in existence. Each option allows the holder to acquire 120 shares at a strike price
of Rs. 10 per share. The options have already vested and will expire on 30 June 2013. The average market price
of ordinary shares in 2011 was Rs. 12 per share.
v. QL had issued debentures in 2008 which are convertible into 6 million ordinary shares. The debentures shall be
redeemed on 31 December 2012. The conversion option is exercisable during the last six months prior to
redemption. The interest on debentures for the year 2011 amounted to Rs. 11 million.
vi. Preference shares issued in 2009 are convertible (at the option of the preference shareholders) into 4 million
ordinary shares on 31 December 2013. The dividend paid on preference shares during 2011 amounted to Rs.
5.75 million.
vii. The company is subject to income tax at the rate of 35%.

Required: Prepare extracts from the financial statements of Que Limited for the year ended 31 December 2011
showing all necessary disclosures related to earnings per share. (Ignore comparative figures).
(16)

Page 2 of 2
231
Daffodils Limited
(same question as in IAS – 8)
Basic earning per share :
Restated
2017 2016
Earnings 660.25 331.67
No. of shares 291.86(W-1) 255.04(W-2)

2.26 per share 1.30 per share

2017 2017 2106


Actual Current Prior
(weighted) (Adjusted)
b/d 1.1.2017 200 200 186.67(w.2)

1.4.2017 issue for no 20 20 18.67


value (10%) (20/200 X200) (20/200 X 186.67)
220 220 205.34
1.7.2017 issue for 30 15 0
value (right (30 x 6/12)
shares)(W-3)
250 235 205.34

1.7.2017 issue for no 20 18.85 16.43


value (20/250 X235) (20/250 X 205.34)
270 253.8 221.77
1.9.2017 issue for no 40.5 38.07 33.27
value (15%) (40.5/270 X 253.8) (40.5/270 X221.77)
310.5 291.86 255.04
(W-1) Weighted Average number of shares :

(W-2)
2016 2016
Actual Current weighted)
b/d 1.1.2016 160 160

1.5 issue for value (right shares but 40 26.67


no bonus element) (40×8/12)
200 186.67

(W-3) 1.7.2017
50 × 15/25 =30 [for value]
50-30 =20 [for no value]

Page 1 of 7
232
Answer 1:
AAZ Limited
Earning per share for the year end 31.12.2017
NOTE: convertible preference shares are assumed to be redeemable as there is no other information .
‘’Rs in Millions’’
Basic earning per share:
127.83
85.22
125.38
=1.5 per share
85.22

Dilutive Earning per share (W.1) = 1.46


AAZ Limited
Notes to Financial statements
For the year ended 31.12.2007

Reconciliation of earning used for basic EPS to diluted EPS :


Earnings used in basic EPS 127.83
Preference dividend 2.45
Earning used in diluted EPS 130.28

Reconciliation of weighted Average number of shares used for basic EPS to dilute EPS :
Weighted Average number of shares used for basic EPS 85.22
Shares option 0.15
convertible preference shares 4.00
Weighted Avg numbers of shares for dilute EPS 89.37

(W-1) Multiple potential ordinary shares :


Step 1 : Ranking
Incremental Earnings/Incremental ordinary shares
1. Debentures
7.5×70% 5.25
; = 1.75 Anti diluted
3 3
2. Convertible preference Shares
2.45
= 0.61 Dilutive
4

3. Options
0
= 0 dilutive
0.15∗

*[1.5 - (1.5 × 9.9/11)]


=0.15 [for no value]
Ranking:
1. Options
2. Convertible preference shares
3. Debentures

Page 2 of 7
233
Step 2 : Testing which combination is most dilutive
127.83
Basic earning per shares = 1.50
85.22
1. Option [127.83+0/85.22+0.15] ; [127.83/85.37] = 1.49 Dilutive
2. Option & Convertible preference Shares [127.83+0+2.45/85.22+0.15+4] ;
[130.28/89.37] = 1.46 Dilutive
3.Options , preference shares & debenture [127.83+0+2.45+5.25/85.22+0.15+4+3] ;
[135.53/92.37] =1.47 Anti Dilutive

Answer 2. Afridi Industries limited


Extracts of statement of Comprehensive Income For the year ended 31.12 2008
“Rs in Millions”

Continuing Operations:
Profit After tax (78+13) 91
Discontinuing Operations:
loss After tax (13)
Total profit After tax 78
Earnings per share:
Basic EPS:
Continued Operations 1.86
Discontinued Operations (0.27)
overall 1.59
Dilutive EPS:
Continued Operations 1.78
Discontinued Operations (0.21)
Overall 1.57

Workings:
Basic EPS:
Continued Operations (91/49) (W-1 =1.86
Discontinued Operations (13)/49 =(0.27)
Diluted EPS:
Continued Operations (108.33/60.67) (W-2) =1.78
Discontinued Operations (13)/60.67 = (0.21)

(W-1)
Actual Current(weighted)
b/d 40 40
1.4 For value (40 x 12 9(12×9/12)
30%)
52 49

Page 3 of 7
234
(W-2)
Only one potential Ordinary Share:
Incremental Earnings
Term Finance Certificate =
Incremental Ordinary Shares
10
4×100×8%× ×65% 17.33
12
= 4 10 ; = 1.49 Dilutive
×35× 11.67
10 12

Diluted EPS:
91
Basics EPS (from Continued Operations) = 1.86
49
[91+17.33] 108.33
Term Finance Certificate ; = 1.79(Dilutive)
[49+11.67] 60.67
Answer 3.
ABC limited
Basic and diluted EPS(W-4)
Earnings Attributable to Ordinary Shareholders
Weighted Avg number of shares outstanding
8600 (W−1)
= 0.67 per share
12,774 (w 2)
(W-1)
Profit After tax 13,000
Less: Cumulative preference dividend (2,000)
Less: Non-cumulative. preference dividend (2,400)
Profit Attributable to ordinary shareholders 8,600

(W-2) Weighted Avg. Number of shares:

Actual Current (Weighted)


b/d 10,000 10,000
1.10 For Value(W) 1,104 552
(1,100×6/12)
11,104 10,552
1.10 For No Value 96 91
(96/11,104 × 10552)
11,200 10,643
1.10 For No Value (20%) 2,240 2,129
(2,240/11,200×10,643)
13,440 12,772
1,200×11.5/12.5 = 1,104 [ For Value]
1,200-1,104 = 96 [For No Value]

(W-3) Convertible Non-Cumulative preference shares:


Preference dividend 2,400
Preference dividend rate 12%
Value of preference shares (2,400/12%) 20,000
No. of preference shares (20,000 ÷ 10) 2,000

Page 4 of 7
235
(W-4) Only one potential ordinary share:
2,400
Non- Cumulative preference shares = = 1.2 Anti-dilutive
2,000
Therefore, there is no expectation of dilution.
Notes to Financial statements:
Reconciliation of earnings used for basic EPS to diluted EPS:
Earnings used for basic EPS 8,600
Add: Preference dividend on convertible preference shares (W-4) -
Earnings used for diluted EPS 8,600

Reconciliation of weighted Avg. number of shares used for basic EPS to diluted EPS:
Weighted Avg number of shares used for basic EPS 12,774
Add: Convertible preference shares (W-4) -
Weighted Avg. number of shares used for diluted EPS 12,774

Answer 4:

Que Limited
Extracts of statement of Comprehensive Income;
For the year ended 31.12.2014
“ Rs. in Million “

Continued Operation :
Profit After tax (130 + 40) 170
Discontinued Operation:
Loss After tax (40)
Total profit After tax 130
Earning per share:
Basic EPS:
From Continued Operations (170.00 ÷ 85.22) 1.93
From discontinued Operations ((40) ÷ 85.22) (0.46)
Overall Basic EPS 1.47
Diluted EPS :
From Continued Operations (182.90 ÷ 95.72) 1.91
From discontinued Operations ( (40) ÷ 95.72) (0.42)
Overall Diluted EPS 1.49

Page 5 of 7
236
Reconciliation of earning used for basic EPS to diluted EPS:

Continued Discontinued
Operation Operation
Earning used for basic EPS 170 (40)
Finance cost on debenture ( net of tax ) 7.15 -

Preference dividend 5.75 -


Earning used for diluted EPS 182.90 (40)

Reconciliation of Weight Avg number of shares used for basic EPS to diluted EPS :

Weighted Avg number of shares used for basic EPS 85.22


Add : Share option 0.5
Add : Convertible debenture 6.0
Add : Preference dividend 4.0
Weighted Avg number of shares used for diluted EPS 95.72

Workings :

(W-1) Earnings for basic EPS


Continued Operations 170.00
Discontinued Operations ( Given) (40)

(W-2)

Actual Current (weighted)


b/d 80 80
30.9 Issue for value (w) 14.08 3.52 (14.08 x 3/12)
94.08 83.52
30.9 Issue for No value 1.92 1.7 (1.92/94.08 x83.52)
96.0 85.22

80 x 20% = 16 x 11/12.5 = 14.08 (For Value)


(16-14.08) = 1.92 (For No Value)

Page 6 of 7
237
Multiple Potential Shares :
Ranking: 𝑹𝒂𝒏𝒌𝒊𝒏𝒈
0
1.Options = 0 Dilutive (1)
0.5∗
10
*25,000 x 120= 3,000,000 x 2500,000 (For Value)
12
3,000,000 – 2,500,000 = 500,000 (For No Value) (0.5 in Million )
11 × 65% 7.15
Debentures ; =1.19 Dilutive (2)
6 6
5.75
Preference Shares = 1.44 Dilutive (3)
4

Testing :
170
Basic EPS (from continued operations) = 1.99 per share
85.22

170+0 170
1. Options ; = 1.98 Dilutive
85.22+ 0.5 85.72

170+0+7.15 177.15
2. Options and Debentures ; =1.93 Dilutive
85.22+0.5+6 91.72

170+0+7.15+5.75 182.90
3. Options, debentures and Preference shares ; = 1.91 Dilutive
85.22+0.5+6+4 95.72

Page 7 of 7
238
⯈ Example:
Kashif Construction Limited (KCL), a public listed company, is in the process of finalizing its accounts for the year
ended 31 December 2023. The following information is available:
(i) Share capital and reserves as at 1 January 2022 were as follows:

Rs. m
Ordinary share capital (Rs. 10 each) 900
Irredeemable preference share capital (Rs. 100 each) 360
Share premium 200
General reserve 150
Retained earnings 600

The preference shareholders are entitled to the cumulative preference dividend of 10% which becomes due on 31
December each year.

(ii) The details share issued for cash consideration are as follows:
• A right issue of 20% at Rs. 18 per share on 30 May 2022. Transaction costs of Rs. 3 million were also
incurred.
• A right issue of 1 for 8 shares already held at a premium of Rs. 11 per share on 10 June 2023.
Transaction costs of Rs. 4 million were also incurred.

(iii) The total comprehensive income for the years ended 30 June 2022 and 2023 are as follows:

2023 2022
Rs. m Rs. m
Profit after tax 400 350
Other comprehensive income
Gain (loss) on revaluation (4) 20
Total comprehensive income 396 370

Revaluation was carried on 1st January 2022 for the first time; the related assets had remaining useful life of 5 years.
Another revaluation of same assets was carried on 1st January 2023, however, there is no change in useful life. KCL
transfers revaluation surplus to retained earnings on annual basis.

(iv) The details of cash dividend and bonus issues of ordinary shares (as dividend) declaredand paid during the
three years are as follows:

Cash dividend Bonus


For the year ended Interim* Final** Interim* Final**
31 December 2021 5% - - 10%
31 December 2022 - 10% 15% -
31 December 2023 15% - - 7.5%
*in August with dispatch of half-yearly accounts

Page 1 of 4
239
** In April when annual general meeting of shareholders was held

(v) KCL follows a policy of transferring 10% of its profit after tax to general reserve.

Required:
Prepare statement of changes in equity for the year ended 30 June 2023, along withcomparatives. (total column
is not required)

Answer:
Kashif Construction Limited Statement of changes in equity
For the year ended 31 December 2023

Ordinary Preference Share General Retained Revaluation


share share premium reserve earnings surplus
capital capital
Rs. m Rs. m Rs. m Rs. m Rs. m Rs. m
Balance as on 1 Jan 2022 900 360 200 150 600 -
Bonus dividend 10% (Final 2021) 90 (90)
Right issue 20% 198 158.4
Transaction costs on issue (3)
Interim bonus dividend 15% 178.2 (178.2)
Total comprehensive income
Profit for the year 350
Other comprehensive income 20
Transfer [20/5] 4 (4)
Transfer to general reserve 10% 35 (35)
Preference dividend 10% x 360 (36)
Balance as on 1 Jan 2023 1,366.2 360 355.4 185 614.8 16
Cash dividend 10% (Final 2022) (136.62)
Right issue (1 for 8) 170.78 187.85
Transaction costs on issue (4)
Interim cash dividend 15% (230.55)
Total comprehensive income
Profit for the year 400
Other comprehensive income (4)
Transfer [(16-4)/4] 3 (3)
Transfer to general reserve 10% 40 (40)
Preference dividend 10% (36)
Balance as on 31 Dec 2023 1,536.98 360 539.25 225 574.63 9

Page 2 of 4
240
Example:
AT A GLANCE

The profit after tax earned by AAZ Limited during the year ended December 31, 2007 amounted to Rs. 127.83
million. The weighted average number of shares outstanding during the year was 85.22 million.

Details of potential ordinary shares as at December 31, 2007 are as follows:


• The company had issued debentures which are convertible into 3 million ordinary shares. The debenture holders
can exercise the option on December 31, 2009. If the debentures are not converted into ordinary shares they
shall be redeemed on December 31, 2009. The interest on debentures for the year 2007amounted to Rs. 7.5
million.
• IrredeemablePreference shares issued in 2004 are convertible into 4 million ordinary shares at the option of the
preference shareholders. The conversion option is exercisable on December 31, 2010. The dividend paid on
preference shares during the year 2007 amounted to Rs. 2.45 million.
• The company has issued options carrying the right to acquire 1.5 million ordinary sharesof the company on or after
December 31, 2007 at a strike price of Rs. 9.90 per share. During the year 2007, the average market price of the
shares was Rs. 11 per share.

The company is subject to income tax at the rate of 30%.

Required:
Compute basic and diluted earnings per share and prepare disclosure note in accordance with IAS 33 for the year
ended December 31, 2007 (comparatives are not required).

⯈ Answer:
STICKY NOTES

Basic EPS (2007) = Rs. 127.83 – 2.45 million = Rs. 1.471 per share
85.22 million shares

Ranking order

Incremental Incremental shares(m) Incremental


earnings (Rs. EPS (Rs.)
m)
Convertible debentures 5.25 3 1.75
[7.5m x 70%] Anti-dilutive
Convertible preferenceshares 2.45 4 0.61
Dilutive
Options - 0.15 Nil
[(1.5m x [(11 – 9.90) / 11)] Dilutive
Options are most dilutive (Rank 1) and then are convertible preference shares (Rank 2).
Convertible debentures will be ignored in calculation since they are anti-dilutive.

Page 3 of 4

241
Calculation of diluted EPS

EarningsRs. Shares EPS Impact


Rs.
Basic EPS 125.38 85.22 1.471
Options (1) 0 0.15
125.38 85.37 1.469 Dilutive
Convertible pref. shares (3) 2.45 4
127.83 89.37 1.430 Dilutive
Notes to the financial statements for the year ended 31 December 2007

AT A GLANCE
Earnings per share 2007
Earnings Rs. m
Profit after tax 127.83
Less: Preference dividend (2.45)
Earnings for calculation of basic EPS 125.38
Add: Effect of conversion of preference shares 2.45
Add: Effect of share options -
Earnings for calculation of diluted EPS 127.83

Weighted average number of shares Shares m


Shares for calculation of basic EPS 85.22
Add: Free shares under options 0.15
Add: Conversion of preference shares 4
Shares for calculation of diluted EPS 89.37

Page 4 of 4
242
Income and Expenditure Account
Many organizations exist, not in order to make profits. E.g.:
a) Clubs and societies
b) Charitable organizations
c) Trusts
d) NGOs
e) Hospitals

Terminologies used in non profit organizations:


a. Statement of Comprehensive Income Income and Expenditure account
b. Profit Surplus
c. Loss Deficit
d. Capital/Equity of business Equity Fund/ General fund / Accumulated fund
e. Receipt and Payment account Means a combined cash +bank a/c

Non-Profit Organization:
Primary purpose of these organization is to provide services rather than to make profit.
e.g.: charitable hospitals

Sources of Income:
▪ Membership fee or subscription fee
▪ Investment income from surplus funds
▪ Donation
▪ Legacy
▪ Supporting activities e.g.: sales of medicines, functions, cafeteria
▪ Life membership fee

Statement of Comprehensive Income income and expenditure account


Income and expenditure account
Expenses : Incomes:

Surplus (excess of income) OR Deficit (excess of expense)

Income and expenditure account can also be presented in statement form.


Capital Capital fund, General fund, Accumulated fund

Page 1 of 33
243
In the questions of income and expenditure accounts a subscription account is usually required to be prepared,
which looks like as follows:
(related to members) Subscription account (Just like debtor control account)
b/d (receivable from members) 15,000 b/d (advance from members) 500
Cash 1,000
Bank 15,000
Subscription Income (Bal.) 19,000 Bad debt 2,000
Discount allowed 500

c/d (closing advance) 10,000 c/d (closing receivable) 25,000


44,000 44,000

Example: Subscription account


Question: At 31 March 2016 a cricket club had membership subscriptions in arrears amounting toRs. 48,000 and
had received Rs. 12,000 subscriptions in advance.

During the year to 31 March 2017 the club received Rs. 624,000 including 26 memberships forthe year to 31
March 2018 at Rs. 1,200 per annum.

At 31 March 2017 16 members owed subscriptions of Rs. 1,200 each.

Half of the members who were in arrears at the end of the previous period still had not paid by 31March 2017. It
was decided to write these amounts off.

Required: How the above transactions would be recorded in the subscription’s ledger account for
the year to 31 March 2017?

Answer:
Subscriptions Account
Rs. Rs.
Balance b/d: Balance b/d:
Members in arrears 48,000 Advance payments 12,000
Cash 624,000
Membership fees for the
year (to I&E) 600,000 Bad debts (1/2 x 48,000) 24,000
Balance c/d: Advance Balance c/d: Members in
payments (26 × 1,200) 31,200 arrears (16 × 1,200) 19,200
679,200 679,200

Page 2 of 33
244
Q. 1 Following is the Receipt and Payment Accounts of Sehat club for the year ended 30 June 2011:
Receipts and Payments Account For the year ended 30 June 2011
Receipt Rupees Payment Rupees
Opening balance 15,000 Salaries 63,500
Subscription 201,000 Rent 34,000
Entrance fees 63,000 Travelling expenses 1,500
Donations ' 38,000 Printing and stationery 1,000
Interest 16,000 General charges 2,500
Receipt on disposal of furniture 500 Periodicals 500
Investments 200,000
Closing balance 30,500
333,500 333,500

The club’s balance sheet as on 30 June 2010 was as follows:


Balance Sheet As on 30 June 2010
Liabilities Rupees Assets Rupees
General Fund 172,500 Furniture-net 40,000
Liabilities: Rent 11,000 Sports equipment – net 20,000
Salaries 17,500 Investments 100,000
Subscription receivable 15,000
Interest receivables 11,000
Bank balance 15,000
201,000 201,000
Other details for the year ended 30 June 2011 are as follows:
(i) Furniture purchased on 1 July 2009 costing Rs. 4,000 was disposed off on 1 January 2011 at a scrap value
of Rs. 500.
(ii) On 1 July 2010, furniture having written down value of Rs. 6,000 was traded-in with new furniture having
fair value of Rs. 6,700.
(iii) Depreciation is charged on diminishing balance basis at 20% on furniture and 15% on sports equipment.
(iv) Sports equipment worth Rs. 12,000 were received at year end as donation.
(v) Following amounts are receivable /outstanding as at 30 June 2011:
Rs.
Subscription receivable 8,000
Entrance fee receivable 3,000
Salaries outstanding 4,000
Rent outstanding 2,000

Required:
Prepare an income and expenditure account of Sehat Club for the year ended 30 June 2011 and its balance sheet
on that date.

Page 3 of 33
245
2. GILTAN GOLF CLUB
The treasurer of the Giltan Golf Club has prepared the following receipts and payments account for the year ended
31 March 2016.
Rs.(000) Rs.(000)
Balance at 1 April 2015 682 Functions 305
Subscriptions 2,930 Repairs 146
Functions 367 Telephone 67
Sale of land 1,600 Extension of club house (building) 600
Bank interest 60 Furniture 135
Bequest (legacy) 255 Heat and light 115
Sundry income 46 Salary and wages 2,066
Sundry expenses 104
Balance at 31 March 2016 2,402

5,940 5,940

(a) Subscriptions received included Rs.65,000 which had been in arrears at 31 March 2015 and Rs. 35,000
which had been paid for the year commencing 1 April 2016.
(b) Land sold had been valued in the club's books at cost Rs.500,000.
(c) Accrued expenses
31 March 2015 31 March 2016
Rs.(000) Rs.(000)
Heat and light 32 40
Wages 12 14
Telephone 14 10
58 64

(c) Depreciation is to be charged on the original cost of assets appearing in the books at 31 March 2016 as
follows:

Buildings 5%
Fixtures and fittings 10%
Furniture 20%
The following balances are from the club's books at 31 March 2015:
Rs.(000)
Land at cost 4,000
Buildings at cost 3,200
Buildings allowance for depreciation 860
Fixtures and fittings at cost 470
Fixtures allowance for depreciation 82
Furniture at cost 380
Furniture allowance for depreciation 164
Subscriptions in arrears (including Rs.15,000 irrecoverable - 80
member had emigrated)

Page 4 of 33
246
Subscriptions in advance 30

Required:
Prepare an income and expenditure account for the year ended 31 March 2016 and a Statement of financial
position as at that date.

Q. 3 The following balances have been obtained from the books of Gulshan Cricket Club:
June 30, 2007 June 30, 2008
Building 6,024,000 6,438,150
Furniture 3,012,000 2,710,800
Books 1,129,500 1,246,950
Sports equipment 1,807,200 1,595,200
Investments - 436,000
Advance subscription 86,000 92,000
Prepaid expenses 122,000 176,000
Expenses payable 186,900 207,600
Subscriptions receivable 326,000 357,000
Cash 1,204,800 1,586,500

The following information is also available in respect of year ended June 30, 2008:
i. Depreciation for the year has been credited directly to the asset accounts. The rates of depreciation are as
follows:
Building 5%
Furniture and books 10%
Sports equipment 20%
ii. The club had 600 members on June 30, 2008. No fresh members were admitted during the year but 10
members left the club on January 1,2008. Subscription per member is Rs. 500 per month.

Required:
(a) Summary of receipts and payments made during the year ended June 30,2008.
(b) Income and Expenditure Account for the year ended June 30,2008.
(c) Statement of financial position as on June 30,2008.

Page 5 of 33
247
ANSWERS
A.1 Sehat Club
Income and Expenditure Account For the ye.ir ended 30 June 2011
Expenditure Amount (Rs.) Income Amount (Rs.)
Salaries (63.5+4-17.5) 50,000 Subscriptions (201+8-15) 194,000
Rent (34+2-11) 25.000 Entrance fees (63+3) 66,000
Travelling expenses 1,500 Donation 38,000
Printing and stationary 1,000 Interest(16-11) 5,000
General charges 2,500 Gain on trade in of furniture 700
Periodicals 500
Depreciation on furniture 7,820
Depreciation on sports equipment 3,000
Loss on furniture disposed off (2,880- 2.380
500)
Excess of income over expenditure 210,000
303,700 303,700

Sehat Club
Balance sheet As on 30 |une 2011
Liabilities Rupees Assets Rupees
General fund: Furniture 30,000
Opening balance 172,500 Sports equipment (20-3+12) 29,000
Add excess of income over expense 210,000 Investments (100+200) 300,000
382,500 Subscription receivable 8,000
Deferred contribution Equipment 12,000
Liabilities : Entrance fee receivable 3,000
Salaries payable 4,000 Bank balance 30,500
Rent payable 2,000
400,500 400,500

Furniture
b/d 40,000 Disposal (4,000-800-320) 2,880
Addition 01.07.2010 6,700 Assets exchanged (01.07.2010) 6,000
Depreciation expense 7,820*
c/d 30,000
46,700 46,700

*Depreciation:
(40,000 – 3,200 – 6,000) x 20% = 6,160
3,200 x 20% x 6/12 = 320
6,700 x 20% = 1,340
7,820

Page 6 of 33
248
Disposal
Furniture 2,880 Cash 500
Loss (bal) 2,350
2,880 2,850

Salary payable
Cash 63,500 b/d 17,500
c/d 4,000 Expense (bal) 50,000
67,500 67,500

Rent payable
Cash 34,000 b/d 11,000
c/d 2,000 Expense(bal) 25,000
36,000 36,000

Entrance fee receivable


b/d - Cash 63,000
Income (bal) 66,000 c/d 3,000
66,000 66,000

Interest receivable
b/d 11,000 Cash 16,000
Income(bal) 5,000 c/d -
16,000 16,000

Equipment
b/d 20,000 Depreciation (20,000 x 15%) 3,000
Donation (30-06-2011) 12,000 c/d 29,000
32,000 32,000

2. GILTAN GOLF CLUB


Income and expenditure account for Giltan Golf Club for year ending 31 March 2016
Income Rs.(000) Rs.(000)
Functions income 367
Sale of land (1,600 - 500) 1,100
Bank interest 60
Bequest 255
Sundry income 46
Subscriptions (W1) 2,860
4,688
Expenditure
Bad debts 15
Functions expenses 305
Repairs 146

Page 7 of 33
249
Telephone (67 - 14 + 10) 63
Heat and light (115 - 32 + 40) 123
Salaries and wages (2,066 - 12 + 14) 2,068
Sundry expenses 104
Depreciation – building 190
Depreciation – furniture 103
Depreciation - fixtures and fittings 47 (3,164)
Surplus for the year 1,524

Giltan golf club: Statement of financial position as at 31 March 2016


Non-current assets Cost Accumulated Carrying
depreciation Amount
Rs.(000) Rs.(000) Rs.(000)
Land (4,000 - 500) 3,500 - 3,500
Buildings (W3) 3,800 (1,050) 2,750
Fixtures and fittings (W4) 470 (129) 341
Furniture (W5) 515 (267) 248
8,285 (1,446) 6,839
Current assets
Bank 2,402
9,241
Accumulated Fund (W2) 7,618
Surplus for the year 1,524
9,142
Current liabilities
Accruals 64
Subscriptions in advance 35
9,241
Workings

W1 Subscriptions account
Rs.(000) Rs.(000)
Subs in arrears b/d 80 Subs in advance b/d 30
Income and expenditure 2,860 Bank 2,930
Subs in advance c/d 35 Bad debts 15
2,975 2,975

W2 Opening statement of affairs 2015


Assets Rs (000)
Bank 682
Subscriptions in arrears 80
Land 4,000
Buildings (3,200 - 860) 2,340
Fixtures (470 - 82) 388
Furniture (380 - 164) 216

Page 8 of 33
250
Liabilities 7,706
Accruals (58 + 30) (88)
7,618

W3 Buildings
Cost Acc. Depreciation
Rs.(000) Rs.(000)
Balance b/d 860
3,200
Extension to club house 600
Depreciation (3,800 x 5%) 190
3,800 1,050

W4 Fixtures and fittings


Cost Acc. Depreciation
Rs.(000) Rs.(000)
Balance b/d 470 82
Depreciation (10% x 470) 47
470 129
W5 Furniture
Cost Acc. Depreciation
Rs.(000) Rs.(000)
Balance b/d 380 164
Additions 135
Depreciation (20% x 515) 103
515 267
Telephone payable account
Rs.(000) Rs.(000)
cash 67 b/d 14
Expense (bal) 63
c/d 10
77 77
Heat and light payable
Rs.(000) Rs.(000)
cash 115 b/d 32
Expense (bal) 123
c/d 40
155 155
Telephone payable account
Rs.(000) Rs.(000)
Cash 2,066 b/d 12
Expense (bal) 2,068
c/d 14
2,070 2,070

Page 9 of 33
251
A. 3 Gulshan cricket club
(a) Receipt and payment account for the year ended June 30, 2008
b/d 1,204,800 Building (W-4) 753,000
Book (W-5) 256,000
Subscription (W-2) 3,605,000 Shop equipment (W-6) 186,800
Investment 436,000
Expenses (Bal.) 1,591,500
c/d 1,586,500
4,809,800 4,809,800

(b) Income and expenditure account for the year ended June 30, 2008
Expenses (W-8) 1,558,200
Depreciation: Subscription Income (W-1) 3,630,000
Building 338,850
Furniture 301,200
Books 138,550
Sports equipment 398,800
Surplus (bal) 894,400
3,630,000 3,630,000

(c) Gulshan cricket club


Statement of financial position as on June 30,2008.
Non current assets:
Building 6,438,150
Furniture 2,710,800
Books 1,246,950
Sports equipment 1,595,200
Investments 436,000 12,427,100
Current assets:
Prepaid expenses 176,000
Subscriptions receivable 357,000
Cash 1,586,500 2,119,500
14,546,600
Equity and liabilities:

Capital fund 13,352,600*


Surplus 894,400 14,247,000
Current liabilities:
Advance subscription 92,000
Expenses payable 207,600 299,600
14,546,600
*Note: Opening capital fund: 6,024,000 + 3,012,000 +1,129,500 + 1,087,200 – 86,000 +122,000 -186,900 + 326,000
+ 1,204,800 = 13,352,600

Page 10 of 33
252
Workings:
W-1) Subscription income
[600 x 500 x 6] + [610 x 500 x 6]

=3,630,000

(W-2) Subscription account


b/d 326,000 b/d 86,000
Subscription income 3,630,000 Cash (bal) 3,605,000
c/d 92,000 c/d 357,000
210,000 210,000

(W-4) Building account


b/d 6,024,000 Depreciation (6,438,150/95x5) 338,850
Cash(bal) 753,000 c/d 6,438,150
6,777,000 6,777,000

(W-5) Books
b/d 1,129,500 Depreciation (1,246,950/90x10) 138,550
Cash(bal) 256,000 c/d 1,246,950
1,385,500 1,385,500

(W-6) Sports Equipment


b/d 1,807,200 Depreciation (1,595,200/80x20) 398,800
Cash(bal) 186,800 c/d 1,595,200
1,994,000 1,994,000

(W-7) Furniture
b/d 3,012,000 Depreciation (2,710,800/90x10) 301,200
c/d 2,710,800
3,012,000 3,012,000

(W-8) Expenses prepaid + Expenses payable


b/d 122,000 b/d 186,900
Cash 1,591,500 Expenses (bal) 1,558,200
c/d 207,600 c/d 176,000
1,921,100 1,921,100

Page 11 of 33
253
Extra practice questions of Income and expenditure account:
Q. 1
Seaview Club started its operations on 1 February 2015. Sponsor of the club contributed Rs. 50 million towards
general fund for the start of operations and placed the amount in the bank. Following is the receipts and payments
summary for the period from 1 February 2015 to 31 December 2015:
Receipts Rs. In ‘000’ Payments Rs. In ‘000’
Sponsor’s contribution 50,000 Furniture & fixtures 1,200
Joining fees 20,800 Van 1,500
Subscription from members 29,952 Salaries 1,000
Sale of beverages 1,500 Rent 3,600
Utilities 570
Insurance 120
Repairs and maintenance 275
Purchase of beverages 1,367
Advance for plot of land 65,000
Balance 27,620
102,252 102,252
Additional information:
(i) The joining fee for award of membership is Rs. 50,000. Annual subscription is Rs. 24,000. All new members
pay three years subscription in advance. The memberships were awarded as follows:
Month March June September December
No. of members 112 98 101 105
(ii) The club sells beverages at a gross profit margin of 20%. All sales are billed in the first week of the next
month and the payment is received in the same month. Sale of beverages during December 2015 amounted
to Rs. 150,000.
(iii) 25% of total purchases of beverages made during the year remained unsold at year-end.
(iv) Salaries are paid on the first day of next month. The amount of salaries includes an advance amounting to
Rs. 10,000 paid to an employee on 1 December 2015. the advance is repayable on 1 February 2016.
(v) Rent for three years was paid in advance on 1 February 2015.
(vi) Presently the club is operating on rental premises. However, a plot of land has been purchased on which
construction would commence shortly. Title of land would be transferred after completion of legal
formalities.
(vii) Payments for utilities include security deposit paid to utility companies amounting to Rs. 20,000. Utility bills
are paid on the 7th day of the next month.
(viii) Insurance premium was paid on 1 February 2015 covering a period of 12 months.
(ix) Repairs and maintenance include an advance of Rs. 100,000 paid to a contractor for construction of a
parking shed. Repair bills amounting to Rs. 7,000 were outstanding at year-end.
(x) Furniture & fixtures and van were purchased on 1 February 2015. Depreciation on these assets is to be
charged at 10% and 20% respectively.

Required:
Prepare statement of financial position as at 31 December 2015 and income & expenditure account of Seaview
Club for the period ended 31 December 2015. (20)

Page 12 of 33
254
Question 2
The accountant of Leisure Club was terminated on account of charges of fraud on 31 December 2016 and Mr.
Emad has been appointed in his place. Emad has gathered the following information in respect of the year ended
31 December 2016:
(i) The club has 3,300 members and the membership fee is Rs. 10,000 per annum. The fee payable by each
member becomes due on the first day of the quarter in which he became a member. The fee received in
each quarter was as follows:
Quarter First Second Third Fourth
Subscription received (Rs.) 9,900,000 8,250,000 5,500,000 9,350,000
Last year the fee was Rs. 9,000 per annum. However, the number of members was the same.
(ii) A summary of the bank account for the year is shown below;
Deposits Rupees Withdrawals Rupees
Balance as at 1 Jan. 2016 3,700,500 Insurance 175,000
Cash deposited into bank 37,848,500 Rent and rates 4,200,000
Written off amount recovered 1,860,000 Utilities 4,365,000
Disposal of fixed assets 750,000 Freehold land purchased 17,000,000
Members subscription received Cash withdrawals from bank 6,120,000
directly in bank account 19,800,000 Payment to creditors 18,155,000
Repairs and maintenance 700,000
Exercise equipment 7,350,000
Balance as at 31 Dec. 2016 5,894,000
63,959,000 63,959,000
(iii) Amounts paid from petty cash were as follows:
Rupees
Salaries 2,300,000
Sundry expenses 640,000
(iv) The club has a tuck shop which earns a profit margin of 20% of sales. All sales of tuck shop are made on
cash. During the year, stock costing Rs. 500,000 was destroyed by fire.
(v) The opening WDV of fixed assets was Rs. 28,000,000. Exercise equipment was purchased on 1 October
2016. Fixed assets having opening WDV of Rs. 800,000 were disposed off on 31 March 2016. Fixed assets
are depreciated @ 20% under the reducing balance method.
(vi) The opening and closing balances of cash in hand were Rs. 300,000 and Rs. 25,000 respectively.
(vii) The following balances have been extracted through a scrutiny of the available records:
2016 2015
Rupees
Creditors 3,330,000 2,500,000
Prepaid rent 175,000 168,000
Stock- tuck shop 2,500,000 2,300,000

Required:
(a) Determine the amount of loss incurred by the club due to fraud committed by the previous accountant.
(09)
(b) An income and expenditure account for the year ended 31 December 2016. (05)
(c) Statement of financial position as at 31 December 2016. (06)

Page 13 of 33
255
Answers to extra practice questions:
A.1
Seaview Club
Income & Expenditure Account
For the period ended 31 December 2015
Expenditure Rs. In ‘000 Income Rs. In ‘000
Salaries and wages (1,000 – 10 + 99) 1,089 Joining fees (Given) 20,800
Rent (3,600/3 × 11/12) 1,100 Subscription income (W-1) 4,630
Utilities (570 – 20 + 55) 605 Profit on sale of beverages (W-2) 330
Insurance (120/12 × 11) 110
Repairs and maintenance (275–100+7) 182
Depreciation expense
(1,200×10%×11/12+1,500×20%11/12(110+275) 385
Excess of income over expenditure 22,289
25,760 25,760

Seaview Club
Statement of Financial Position
As at 31 December 2015
Assets Rs. In ‘000
Non-Current Assets
Land Advance 65,000
Furniture & fixtures (1,200 – 110) 1,090
Van (1,500 – 275) 1,225
Advance for parking shed 100
Long term deposits – Utility 20
Long term prepayment – Rent 1,300
68,735

Current Assets
Stock (W-2) 440
Debtors for beverages (credit sale) 150
Advance & prepayments (W-3) 1,220
Bank 27,620
29,430
Total Assets 98,165

Page 14 of 33
256
General Fund & Liabilities Rs. In ‘000
General fund 50,000
Excess of income over expenditure 22,289
72,289

long term advance (W-1) 15,338


Current Liabilities
Creditors (1,760 – 1,367) 393
Accrued expenses (7 + 55 + 99) 161
Advance subscription (W-1) 9,984
10,538

Total General Fund & Liabilities 98,165

W-1: Subscription Income


Subscription for 3 years is Rs. 72,000 so subscription for 1 year is Rs. 24,000 or Rs. 2,000 per month.
Month No. of No. of Months Subscription Income for the Advance subscription
members year income
A B A × B × 2,000 A × (36 – B) × 2,000
-------------------- Rupees -----------------------
March 112 10 2,240,000 5,824,000
June 98 7 1,372,000 5,684,000
September 101 4 808,000 6,464,000
December 105 1 210,000 7,350,000
4,630,000 25,322,000
Less: short term [(112+98+101+105)×24,000] (9,984,000)
Long term 15,338,000

or
Months
March 112 × 24,000 = 2,688,000 × 3 = 8,064,000
June 98 × 24,000 = 2,352,000 × 3 = 7,056,000
September 101 × 24,000 = 2,424,000 × 3 = 7,272,000
December 105 × 24,000 = 2,520,000 × 3 = 7,560,000
29,952,000
[29,952,000 – 4,630,000] = 25,322,000

W-2: Beverage Sale Results Rs. In ‘000’


Sales (1,500 + 150) 1,650
Less: Cost of sales
Purchases (1,320/0.75) 1,760
Closing stock (1,760 × 25%) (440) 1,320
330

Page 15 of 33
257
Debtors
b/d -- Cash 1,500
Sales 1,650
c/d 150
Payable
b/d --
Cash 1,367 Purchases 1,760
c/d 393
Stock
b/d -- Cost of sales 1,320
Purchases (bal) 1,760 (1,650 / 100 x 80)
c/d (1,320 / 75 x 25) 150

A.2
(a) Determination of Amount of Loss incurred due to fraud [it means a Cash Account in a Statement form to
find out the missing cash]

Opening balance 300,000


Receipts:
Collections from members [(3,300 × 10,000) – 19,800,000] 13,200,000
Bank withdraws 6,120,000
Tuck shop sales (W) 22,856,250
42,176,250
Payments:
Salaries (2,300,000)
Sundry Expenses (640,000)
Cash Deposited in Bank (37,848,500)
(40,788,500)
Closing balance that should have been 1,687,750
Closing Cash – Actual (Given) 25,000
Difference – Loss due to fraud 1,662,750
(b) Income and Expenditure Account
Incomes:
Subscription Income (W) 31,817,500
Income from tuck shop [22,856,250 – 18,285,000] 4,571,250
Other income – bad debts recovered 1,860,000
38,248,750
Expenditures:
Salaries (2,300,000)
Insurance (175,000)
Rent expense (4,193,000)
Utilities (4,365,000)
Repair & Maintenance (700,000)
Depreciation (W) (5,847,500)
Sundry Expenses (640,000)

Page 16 of 33
258
Loss on disposal (10,000)
Loss of inventory due to fire (500,000)
Loss due to fraud [from (a)] (1,662,750)
(20,393,250
Surplus 17,855,500

(c) Statement of Financial Position


Non-Current Assets:
Fixed Assets – WDV 45,742,500
Current Assets:
Stock 2,500,000
Prepaid rent 175,000
Cash at bank 5,894,000
Cash in hand 25,000
54,336,500
Fund and Liabilities:
Accumulated Fund – Opening [168 + 2,300 + 28,000 + 300 + 3,700.5 – 2,500 – 10,642.5] 21,326,000
Add: Surplus 17,855,500
39,181,500
Liabilities:
Creditors 3,330,000
Unearned subscription Income (W) 11,825,000
54,336,500
Workings:
(W-1) Subscription Income:
Opening balance of unearned subscription [11,825,000/10,000 × 9,000] 10,642,500
Add: Receipt for the year (3,300 × 10,000) 33,000,000
Less: Closing unearned subscription (W – 1.1) (11,825,000)
Subscription Income 31,817,500

(W-1.1) Closing Balance:


Quarter 1 --
Quarter – 2 [8,250,000 × 3/12] 2,062,500
Quarter – 3 [5,500,000 × 6/12] 2,750,000
Quarter – 4 [9,350,000 × 9/12] 7,012,500
11,825,000
(W-2) Creditors
Bank 18,155,000 b/d 2,500,000
Purchases 18,985,000
c/d 3,330,000

Page 17 of 33
259
(W-3) Prepaid Rent

b/d 168,000 Expense 4,193,000


Bank 4,200,000
c/d 175,000
(W-4) Stock
b/d 2,300,000 Loss 500,000
Cost of sales 18,285,000
Purchases 18,985,000
c/d 2,500,000

Sales from cost of sales:


18,285,000
× 100 = 22,856,250
80

(W-5) Fixed Assets Account


b/d 28,000,000 31-3 Disposal (W) 760,000
1-10 Bank 7,350,000 Depreciation (W) 5,847,500
Bank (Land) 17,000,000
c/d 45,742,500
(W-6) Disposal Account
Fixed Asset 760,000 Bank 750,000
Loss 10,000

Depreciation for the year:


[28,000,000 – 800,000] × 20% = 5,440,000
800,000 × 20% × 3/12 = 40,000
7,350,000 × 20% × 3/12 = 367,500
5,847,500
WDV of Disposal:
Opening WDV = 800,000
Depreciation [800,000 × 20% × 3/12] = (40,000)
760,000

Page 18 of 33
260
Extra practice questions
Q.1 Violin Family Club was formed in 2016. Following are the details of assets and liabilities of the club
as on 31 December 2017:

Assets Rs. in '000 Liabilities Rs. in '000


Subscription in arrears: Bank overdraft 181
2016 15 Subscription in advance for 2018 45
2017 90 Accrued electricity 23
Advance rent 24 Canteen wages 11
Canteen stock 215 Canteen creditors 118
Snooker tables 960
Furniture & equipment

Additional information:
(i) Some of the balances as on 31 December 2018 are as follows:
(ii)
Assets Rs. in '000 Liabilities Rs. in '000
Subscription in arrears for 2018 30 Accrued electricity 35
Canteen stock 247 Canteen creditors 142

(iii) Break-up of the subscription received during 2018 is as follows:

Related to year Rs. in '000


2017 60
2018 920
2019 75

The club’s management has decided to write-off the remaining subscription in arrears
relating to the year 2016 and 2017.
(iv) A scheme was introduced in 2016 under which a person is awarded life time membership
upon payment of Rs. 120,000. Life memberships received in the years 2016, 2017 and
2018 were 5, 8 and 6 respectively. Life memberships are credited to ‘Life Membership
Fund’ upon receipt and are transferred to income equally over 10 years, starting from the
year of admission.
(v) The club operates a canteen. Till last year, the canteen earned a gross profit of 20% of sales.
Effective 1 January 2018, selling prices were increased by 10%.
(vi) Details of some payments during 2018 are as follows:

Rs. 000
Canteen creditors 512
Salaries 285
Equipment 66
Electricity 263
(vii) Equipment acquired during the year is only 30% paid and the remaining amount is

Page 19 of 33
261
payable in February 2019.
(viii) Wages of canteen staff are paid on 5th of each month.
(ix) The club operates from a rented place. The rent is paid quarterly in advance on 1
March, 1 June, 1 September and 1 December. As per agreement, annual rent was
increased by Rs. 6,000 with effect from 1 September 2018.
(x) Balance of snooker tables as at 31 December 2017 represents the book value of 5
similar tables purchased in 2016. One of the tables was sold to a member for cash during
the year for Rs. 212,000.
(xi) Snooker tables are depreciated at 12.5% on straight line method while furniture &
equipment are depreciated at 20% using reducing balance method. Full year depreciation
is charged in the year of addition whereas no depreciation is charged in the year of
disposal.

Required:
(a) Prepare income and expenditure account for the year ended 31 December 2018. (12)
(b) Prepare statement of financial position as on 31 December 2018. (09)

Ans.1 Violin Family Club

(a) Income and expenditure account for the year ended 31 December 2018

Rs. in '000
Income
Subscription (W-1) 995
Gain on disposal of table 20
Profit from canteen 57
Life membership (W-2) 228
1,300
Expenditures
Rent 146
Salaries 285
Electricity 275
Depreciation – snooker tables 128
Depreciation – furniture & equipment 188
Subscription written off (Bad Debts) 45
(1,067)
Excess of income over expenditure 233

Canteen trading account for the year ended 31 December 2018


Rs. in '000
Sales 504×110÷80 693
Cost of goods sold
Opening stock 215
Purchases 536

Page 20 of 33
262
Closing stock (247)
504
Gross profit 189
Expenses
Wages 11×12 (132)
Profit from canteen 57

W-1: Subscription Rs. in '000


Opening arrears: Opening advance 2018 45
2016 15 Receipts (60+920+75) 1,055
2017 90 Write off (15+30) 45
Income balance (bal) 995 Closing arrears 30
Closing advance 75
1,175 1,175

W-2: Life membership Rs. in '000


Income
[(5+8+6)×120÷10] 228 Opening balance
(5×120×8÷10)+(
Closing balance 1,836 8×120×9÷10) 1,344
Receipt (6×120) 720
2,064 2,064

(b) Violin Family Club

Statement of financial position as on 31 December 2018 Rs. in '000


Assets
Non-current assets
Snooker table (960–192–128) 640
Furniture & equipment (720+220–188) 752
1,392

Current assets:
Canteen stock 247
Prepaid rent 25
Subscriptions in arrears 30
Bank (W-3) 1,094
1,396

2,788

Page 21 of 33
263
General funds

Opening balance (2,024–378)–1,344 (W-2) 302


Excess of income over expenditure 233
535
Non current liabilities:

Life membership fund [1,836(W.2) -228] (W-2) 1,608

Current Liabilities
Canteen creditors 142
Life membership fund 228

Accrued electricity 35
Subscription in advance (W-1) 75
Creditors for equipment (220–66) 154
Canteen wages payable 11
417

2,788

W-3: Bank/cash Rs. in '000


Subscriptions 1,055 Opening balance 181
Life membership (W-2) 720 Rent (36 + 36 + 37.5 + 37.5) 147
Sale proceeds from table 212 Salaries 285
Canteen receipts 693 Electricity 263
Canteen creditors 512
Canteen wages (11 x 12) 132
Equipment 66
Closing balance 1,094
2,680 2,680

Advance Rent Rs. in '000


1-1 b/f 24
(J.F 2018) Exp (bal.) 146
1-3 Bank 36 Or (24+36+36+37.5+37.5/3x1)
(M , A , M)
1-6 Bank 36
(J , J , A)
1-9 Bank
(S , A , N ) 37.5 c/d (J , F) 25
1-12 Bank 37.5 (37.5/3 X 2)

Page 22 of 33
264
Up to Last year:
36 x 4 = 144
From Sept 2018
Annual rent = 144 + 6 = 150
Quarterly Rent = 150 /4 = 37.5

Snooker Table Rs. in '000


Disposal
b/d 960 (960 / 5) 192
Dep 128

c/d (Bal.) 640

Furniture and Equipment Rs. in '000


Dep
b/d 720 (720 + 220 ) x 20% 188
Cash (66+66/30x70) 220
c/d (Bal.) 752

Electricity Payable Rs. in '000


Cash 263 b/d 23

Exp (Bal.) 275


c/d 35

Wages Payable Rs. in '000


Cash (11 x 12) 132 b/d 11
Exp (11 x 12) 132
c/d 11

Page 23 of 33
265
Creditor Rs. in '000
Cash 512 b/d 118

c/d 142 Purchase (bal.) 536

Disposal a/c Rs. in '000


Tables 192 Cash 212

Gain (Bal.) 20

Stock Rs. in '000


b/d 215 COS (BAL.) 504
Purchase 536
c/d 247

i. Working of Margin of 2018:


80 + 20 = 100 (up to last year)

Current year:
100 x 10% = 10
So, 100 + 10 = 110
Sales = 504 / 80 x 110 = 693

ii. Equipment:
66 / 30 x 100 = 220
Equipment 220
Bank 66
Payable 154
iii. Cost of snooker tables (for depreciation):

Let’s cost in 2016 is 100


Dep (12.5)
31-12-2016 87.5
Dep 12.5
31-12-2017 75.0

Page 24 of 33
266
960 / 75 x 100 = 1,280
[1,280 – (1,280 / 5)] = 1,024 x 12.5% = 128

Further practice

1. AB SPORTS AND SOCIAL CLUB


You have agreed to take over the role of bookkeeper for the AB sports and social club.
The summarised statement of financial position on 31 December 2014 as prepared by the previous bookkeeper
contained the following items.
Assets Rs.
Heating oil for clubhouse 1,000
Shop and cafe inventories 7,000
New sportswear, for sale, at cost 3,000
Used sportswear, for hire, at valuation 750
Equipment for groundsman
Cost 5,000
Depreciation 3,500 1,500
Subscriptions due 200
Bank
Current account 1,000
Deposit account 10,000

Capital and liabilities


Accumulated fund 23,150
Payables
Shop and cafe inventories 1,000
Sportswear 300

The bank account summary for the year to 31 December 2015 contained the following items.
Receipts Rs.
Subscriptions 11,000
Bankings
Shop and café 20,000
Sale of sportswear 5,000
Hire of sportswear 3,000
Interest on deposit account 800
39,800

Payments Rs.
Rent and repairs of clubhouse 6,000
Heating oil 4,000

Page 25 of 33
267
Sportswear 4,500
Grounds person 10,000
Shop and cafe purchases 9,000
Transfer to deposit account 6,000
39,500

You discover that the subscriptions due figure as at 31December 2014 was arrived at as follows.
Subscriptions unpaid for 2013 10
Subscriptions unpaid for 2014 230
Subscriptions paid for 2015 40

Corresponding figures at 31 December 2015 are:


Subscriptions unpaid for 2013 10
Subscriptions unpaid for 2014 20
Subscriptions unpaid for 2015 90

Subscriptions paid for 2016 200


Subscriptions due for more than 12 months should be written off with effect from 1 January 2015.
Asset balances at 31 December 2015 include:
Heating oil for club house 700
Shop and cafe inventories 5,000
New sportswear, for sale, at cost 4,000
Used sportswear, for hire, at valuation 1,000

Closing payables at 31 December 2015 are for:


shop and cafe inventories 800

Sportswear 450
heating oil for clubhouse 200

Two thirds of the sportswear purchases made in 2015 had been added to inventory of new sportswear in the
figures given in the list of assets above, and one third had been added directly to the inventory of used
sportswear for hire.

Half of the resulting 'new sportswear for sale at cost' at 31 December 2015 is actually over two years old. You
decide, with effect from 31 December 2015, to transfer these older items into the inventory of used sportswear,
at a valuation of 25% of their original cost.

No cash balances are held at 31 December 2014 or 31 December 2015. The equipment for the grounds person is
to be depreciated at 10% per annum, on cost.

Page 26 of 33
268
Required:
Prepare the income and expenditure account and statement of financial position for the AB sports club for 2015.
(23)

2. MONARCH SPORTS CLUB


The Monarch Sports Club has the following summary of its cash book for the year ended 30 June 2015:
Rs. Rs.
Opening bank balance 12,500
Receipts:
Subscriptions 18,000
Life membership fees 3,000
Competition receipts 7,500
Entrance fees 2,500
Equipment sold 1,000 32,000
44,500
Payments:
Transport to matches 3,700
Competition prizes 4,300
Coaching fees 2,100
Repairs to equipment 800
Purchase of new equipment 4,000
Purchase of sports pavilion 35,000
(49,900)
Closing balance (overdrawn) (5,400)

The following information is available regarding the financial position at the beginning and end of the accounting
year:
1 July 2014 30 June 2015
Rs. Rs.
Subscriptions in advance 1,100 900
Subscriptions in arrears 200 300
Coaching fees outstanding 150 450

Of the subscriptions outstanding at the beginning of the year, only half were eventually received.

The equipment sold during the year had a net book value of Rs.1,200 at 1 July 2014. Equipment is to be
depreciated at 20% per annum straight line. Life membership fees are taken to cover 10 years.

The treasurer insists that no depreciation needs to be charged on the sports pavilion, as buildings do not
decrease in value. He says that the last club of which he was treasurer did charge depreciation on its buildings
but that when the club came to replace them, there was still insufficient money in the bank to pay for the new
building.

Required: Prepare an income and expenditure account for the Monarch Sports Club for the year ended 30 June
2015. (10)

Page 27 of 33
269
3. LH SPORTS CLUB
The LH Sports Club opened on 1 May 2014 having purchased premises for Rs.80, 000 and furniture for Rs.18,000,
both financed by an interest-free loan from a member. The club secretary has produced the following income
and expenditure account for the year to 30 April 2015.
Income Rs. Rs.
Joining fees (89 members x Rs.200 each) 17,800
Annual subscriptions 12,000
Cafe profits 8,450
Dinner surplus (means profit) 830
Equipment hire receipts 1,750 40,830
Expenditure
Premises costs 10,990
Equipment costs 5,590
Secretary’s expenses 470
Bank charges 125 (17,175)
Surplus for the year 23,655

The income and expenditure account has been prepared after taking into account the following items at 30 April
2015:
□ cafe inventories Rs.1,400
□ payables for cafe supplies Rs.1,320
□ rates and insurances prepaid Rs.2,280

The following items have not been taken into account:


• the equipment costs figure includes Rs.4,000 for the purchase of equipment
• depreciation is to be provided as follows:
✓ at 2% on premises
✓ at 10% on furniture
✓ at 20% on equipment
• joining fees are to be spread over a five-year period
• the annual subscriptions figure includes Rs.960 paid in advance
• subscriptions outstanding at the end of the year, and expected to be collected, amount to Rs.300.

The bank balance at 30 April 2015 was Rs.21,295.

Required:
• Calculate the correct surplus for the year. (6)
• Prepare the statement of financial position at 30 April 2015. (8)

Page 28 of 33
270
Answers:
1. AB SPORTS AND SOCIAL CLUB

AB Sports and social club: Income and expenditure account


Rs. Rs.
Incomes
Subscriptions (W1) 10,720
Shop and cafe profit (W2) 9,200
Sale of sportswear (W3) 1,400
Hire of sportswear (W4) 1,700
Interest on deposit account 800 23,820

Rent of clubhouse 6,000


Expenditure:
Heating oil (1,000 + 4,000 + 200 - 700) 4,500
Grounds person 10,000
Bad debts (unpaid subscriptions = 10 +20) 30
Depreciation (5,000 x 10%) 500 (21,030)
Net surplus 2,790

AB Sports and Social Club statement of financial position as at 31 December 2014


Non-current assets Rs. Rs.
Equipment for grounds person
Cost 5,000
Depreciation (3,500 + 500) 4,000
1,000
Current assets
Heating oil 700
Shop and cafe inventories 5,000
New sportswear (4,000 – 2,000) 2,000
Hire sportswear (1,000 + 500) 1,500
Subscriptions due 90
Bank
Current account (1,000 + 39,800 – 39,500) 1,300
Deposit account (10,000 + 6,000) 16,000 26,590
27,590
Capital and liabilities
Accumulated fund b/f 23,150
Surplus for year 2,790 25,940
Current liabilities
Shop and café 800
Sportswear 450
Heating oil 200
Subscriptions in advance 200 1,650
27,590

Page 29 of 33
271
Workings
Inventory of heating oil
Rs.(000) Rs.(000)
b/d 1,000 Cost of sale 4,500

Purchases 4,200 c/d 700

1,100 1,100

Trade payables (heating oil)


Rs.(000) Rs.(000)
Cash 4,000 b/d -
purchase (bal) 4200
c/d 200

77 77

Subscriptions account
Rs.(000) Rs.(000)
b/d (10+230) 240 Bal. b/f 40
Income (bal fig) 10,720 Bank 11,000
Bad debts (10+20) 30
c/d 200 Bal. c/f 90
11,160 11,160

Shop and cafe results Rs. Rs.


Sales 20,000
Opening inventory 7,000
Purchases (9,000 + 800 - 1,000) 8,800
Closing inventory (5,000) (10,800)
Profit (gross) 9,200
Sale of sportswear Rs. Rs.
Sales 5,000
Opening inventory 3,000
Purchases (4,500 + 450 - 300) x 2/3 3,100
Closing inventory (4,000) 2,100
Profit (gross) 2,900
Loss on sportswear transferred (W.1) (1,500)
Profit 1,400
Hire of sportswear Rs. Rs.
Rentals 3,000
Opening balance 750
Additions of cost (4,500 + 450 - 300) x 1/3 1,550
Closing inventory at valuation (1,000) 1,300

Page 30 of 33
272
Surplus 1,700
(W.1)
Old sportswear 500
Loss 1,500
New sports wear 2,000
(4,000 x ½ = 2,000)
2. MONARCH SPORTS CLUB
Monarch Sports Club: Income and expenditure account year ended 30 June 2015
Income Rs. Rs.
Annual subscriptions (W1) 18,400
Life membership (3,000 x 10%) 300
Entrance fees 2,500
Surplus from competitions 7,500
28,700
Expenditure
Transport 3,700
Competition prizes 4,300
Coaching fees (2,100 - 150 + 450) 2,400
Repairs 800
Bad debts 100
Loss on disposal of equipment (W3) 200
Depreciation (W4) 800 (12,300)
Surplus for the year 16,400

Workings
(W1)
Subscriptions account
Rs. Rs.
Balance b/d (in arrears) 200 Balance b/d (in advance) 1,100
Income (bal) 18,400 Cash
Balance c/d (in advance) 900 Bad debts 100
Balance c/d (in arrears) 300
19,500 19,500

(W3) Sale of equipment

Disposal account
Rs. Rs.
Assets 1,200 Cash 1,000
Loss to I & E a/c 200

1,200 1,200
(W4) Depreciation
20% x 4,000 = 800
Equipment account

Page 31 of 33
273
Rs. Rs.
b/d 1,200 Disposal 1,200
Cash 4,000 Depreciation 800
c/d 3,200

5,200 5,200

3. LH SPORTS CLUB
(a) Surplus for the year
Rs. Rs.
Surplus per draft income and expenditure account 23,655
Add capital expenditure 4,000 Cr.
Deduct depreciation
Premises (80,000 x 20%) 1,600 Dr.
Furniture (18,000 x 10%) 1,800 Dr.
Equipment (4,000 x 20%) 800 Dr.

Less 80% joining fee [17,800 /5 x 4] 14,240 Dr.


Less subscriptions in advance 960 Dr.
Add: subscription outstanding 300 Cr.
New surplus for year 8,555
(b) LH Sports Club: Statement of financial position as at 30 April 2015
Rs. Rs.
Assets
Non-current assets
Premises (80,000 – 1,600) 78,400
Furniture (18,000 – 1,800) 16,200
Equipment (4,000 - 800) 3,200 97,800
Current assets
Inventory 1,400
Subscriptions in arrears 300
Prepaid rates and insurance 2,280
Bank 21,295 25,275
Total Assets 123,075

Capital and liabilities


Accumulated fund at 1 May 2014 -
Surplus for year 8,555
Accumulated fund at 30 April 2015 8,555
Joining fees (17,800 – 3,560 -3,560) 10,680
Loan from member(80,000+18,000) 98,000
Current liabilities

Page 32 of 33
274
Payables 1,320
Joining fees (17,800/5) 10,680
Subscriptions in advance 960
Total Capital and liabilities 123,075
Working
Non-current assets Cost Depreciation Net
Rs. Rs. Rs.
Premises 80,000 (1,600) 78,400
Furniture 18,000 (1,800) 16,200
Equipment 4,000 (800) 3,200
102,000 4,200 97,800

Page 33 of 33
275
Accounting for Not-for-Profit Organisations:
Introduction
Not-for-Profit Organisations (NPOs) are organisations organized and operated exclusively for social, educational,
professional, religious, health, charitable or any other not-for-profit purpose. An NPO's members, contributors and
other resource providers do not, in such capacity, receive any financial return directly from the NPO. (means no
dividend)
NPOs may be:
• companies formed under Section 42 of Companies Act, 2017;
• trusts formed under Trust Act, 1882;
• societies formed under the Societies Registration Act, 1860; or
• any other recognisable form of organisation giving value to the groups of people.

The financial objective of a profit-oriented entity is to make profit and maximise shareholders’ wealth while
financial objective of NPO is to provide its services effectively by achieving value for money. NPO applies or intends
to apply its profits, if any, or other income in promoting its objects, and prohibits the distribution of surplus to its
members, sponsors, promoters, etc. (in the form of dividends or drawings).

NPOs have income which they raise and costs which must be paid just like other organisations and although profit
is not their objective but they have to account for their income and costs. NPOs are accountable for their
effectiveness, economy and efficiency in utilising the funds. (to their members or sponsors)

Revenues of NPOs normally arise from donations, government grants and other contributions as well as from
membership fees, the sale of goods, the rendering of services or the use by others of NPO resources yielding rent,
interest, dividends.

Some accounting rules are as relevant to NPOs as to profit-oriented entities, for example, requirements relating to
inventory, non-current assets and recognition of revenue. However, some areas might be completely irrelevant, for
example, earnings per share.

Different terminology
NPOs use different accounting terminology from profit-oriented entities.
Profit-oriented entities Not-for-Profit Organisations
Statement of comprehensive income Statement of income and expenditure
Net profit Excess of income over expenditure or Surplus
Net loss Excess of expenditure over income or Deficit
Equity / Share capital and equity reserves Net assets / Accumulated fund / Accumulated
surplus / Accumulated deficit / Fund balance
Statement of changes in equity Statement of changes in net assets

Page 1 of 23
276
So far we have discussed and applied accounting concepts that are normally used by many small NPOs including
those which do not maintain proper double entry records. However, some NPOs are required to comply with
Accounting standards for non-profit organizations.

Accounting Standard for Not-for-Profit Organisations (ASNPO)


The Institute of Chartered Accountants of Pakistan (ICAP) issued the ‘Accounting Standard for Not-for-Profit
Organisations’ and as per Securities and Exchange Commission of Pakistan’s (SECP) directives, ASNPO is applicable
to associations not-for-profit registered under the Companies Act, 2017.

ASNPO is applicable to NPOs registered under the company law i.e. it is compulsory for such NPOs to comply with
requirements of ASNPO in addition to the requirements of applicable reporting framework e.g. IFRSs. In case any
requirement of ASNPO is/are inconsistent, the requirements of IFRSs shall prevail.

NPOs, other than companies, are also recommended to prepare financial statements in accordance with ASNPO.

Receipts and payments account

Illustration:

Receipts and payments account


Balance b/d X Donation to Dam Fund X
Subscriptions X Repairs X
Functions – ticket revenue X Telephone X
Sale of land X Extension of building X
Bank interest X Furniture X
Bequest X Electricity expenses X
Sundry income X Salary and wages X
Sundry expenses X
Balance c/d X
X X
Balance b/d X

Financial statements
Financial statements of NPO shall normally include:
• statement of financial position (or balance sheet)
• statement of income and expenditure (instead of statement of comprehensive income)
• statement of changes in net assets (instead of statement of changes in equity)
• statement of cash flows.
• Notes to financial statements and supporting schedules to which the financial statements are cross-referenced
are an integral part of such statements.

Page 2 of 23
277
Endowment Donation: it is cash or any other asset donated for the perpetual (continuous and never ending) benefit
of the non-profit organization. The donation is usually received with the requirement that the principal (original
investment) will remain intact and money earned from investing the principal will be used for the specific purposes
of the non-profit organization.

Most endowments are designed to provide a permanent source of income by keeping the original amount invested
and using the income for the purposes of NPO. An example of endowment is a donation to a university on the
condition that it is invested and that the investment income from the investment are used for scholarship of the
students.

Endowments are given to non-profit organizations with the intention that they be used to advance the mission of
the organization for long term.

RESTRICTIONS are stipulations that specify how resources must be used.


External restrictions are imposed from outside the organization, usually by the contributor of resources.
Internal restrictions are imposed in a formal manner by the organization itself, usually by a resolution of board of
trustees.
However, when we talk about restricted contributions we mean to say externally imposed restrictions only, means
if in a scenario there are internally restricted contributions we will simply consider them as unrestricted
contributions for the purpose of accounting (MCQ number 22).

Example: Types of funds


Question: Consider the following independent circumstances:
(a) A professional body of accountants (the NPO) sets-up a fund for financial support of deserving students. For
this purpose, Rs. 100 million have been allocated that will be invested and 80% of the investment income shall
be used for student support and 20% of investment income shall be added to fund investments. The fund
investments shall not be available for use by the NPO for its operations and the NPO shall preserve the
principal amount of fund.
(b) A healthcare NPO has raised money through special marketing drive in which overseas contributors deposited
$100 each in its ‘Save a life fund’ account. The contributions shall be used for the NPO’s routine operations
which focuses on providing life-saving drugs to patients who cannot afford the cost.
(c) An educational NPO has set-up a fund for development of new school in nearby rural area. The fund-raising
drive has been successful as many people have contributed for the cause. The fund-raising clearly stated that
the funds so raised shall only be used for construction and operations of school at that specific location.

Required:
Identify the type of above funds.

Answer:
(a) Endowment fund
(b) General fund (unrestricted)
(c) Restricted fund

Page 3 of 23
278
CONTRIBUTION REVENUE AND RECEIVABLE
Contribution
Contributions can come from many sources, including individuals, corporations, governments and other NPOs.
Contributions can be in cash or in kind. Contributions include contributions receivable that meet the criteria for
recognition in the financial statements (discussed later).

Definition: Contribution
A contribution is a non-reciprocal transfer to an NPO of cash or other assets (means cash or any other asset is
transferred from a third party with no expectation of payment in exchange) or a non-reciprocal settlement or
cancellation of its liabilities (e.g. a loan is waived off or creditor is not to be repaid unconditionally). Government
funding provided to an NPO is considered to be a contribution (however usual practice is to refer such fund as
grants).

Restrictions
Restrictions (explicit or implicit) on contributions may only be externally imposed (for accounting purposes)

Types of contribution
Restricted A restricted contribution is a contribution subject to externally imposed stipulations
contribution that specify the purpose for which the contributed asset is to be used.
Endowment An endowment contribution is a type of restricted contribution subject to externally
contribution imposed stipulations specifying that the resources contributed be maintained
permanently, although the constituent assets may change from time to time (means
e.g. cash is received and then building is constructed to earn rental income).
Unrestricted An unrestricted contribution is a contribution that is neither a restricted contribution
contribution nor an endowment contribution.

Example: Types of contributions


Question: Consider the following independent circumstances:
(a) A healthcare NPO received Rs. 10 million from wealthy individuals subject to the condition that this amount
shall only be used for acquisition of land for construction of a hospital in a specific village.
(b) A healthcare NPO received Rs. 25 million contribution from a wealthy individual in the year 2012. The sole
purpose of the amount is to support the NPO’s general operations in the year 2014 and 2015.
(c) An educational NPO received a plot of land from Mr. Salman subject to the condition that this land shall only
be used for construction of a primary education school to be run by that NPO. The fair value of this plot of land
is Rs. 12 million.
(d) An educational NPO received a plot of land from Mr. Jamal and it can be used to achieve general objectives of
that NPO. The fair value of this plot of land is Rs. 15 million.
(e) An educational NPO received Rs. 50 million from alumni (former) donors subject to the condition that the
principal balance shall be invested as per specified investment policy and NPO cannot use the principal balance
to fund operations. However, the NPO can utilise the investment earnings to pay for things such as academic
programs or building new school facilities.

Required:
Identify the type of contributions in above circumstances.

Page 4 of 23
279
Answer:
(a) Restricted contribution
(b) Restricted contribution in 2012 and 2013 then unrestricted in 2014 and 2015.
(c) Restricted contribution
(d) Unrestricted contribution
(e) Endowment contribution

Revenue recognition
Revenue from contributions is recognised by following either restricted fund method or deferral method (discussed
next). An NPO is required to select one method and apply it consistently over the periods and any change from one
method to the other shall be treated as change in accounting policy (as per IAS 8).

Contribution receivable

Recognition
A contribution receivable should be recognized as an asset when it meets the following criteria:
1. the amount to be received can be reasonably estimated; and
2. ultimate collection is reasonably assured.

Contribution receivable is different from accounts receivable in a business entity because the accounts receivables
are recognized after earning revenue and there is a legally enforceable right. However, there is no such legally
enforceable rights of contribution, as the NPO is not providing any goods or services against contributions.

Therefore, amount receivable should comparatively be more certain.

Pledge
A pledge is a promise to contribute cash or other assets to an NPO (like in function or in program). Similar to any
other contribution receivable, an uncollected pledge would only be recognized:
1. if it meets the above recognition criteria of contribution receivable.
2. there is reasonable assurance that the NPO will comply with conditions, if any, attached to the contribution (e.g
contribution is received if construction of DAM is started); and
3. contribution is not dependant on any contingent event outside NPO’s control (e.g. if Govt will also contribute
100 million).

Bequest/legacy
Bequests are often subject to considerable uncertainty surrounding both the timing of the receipt and the amount
that will actually be received. In many cases, the recognition criteria will not be satisfied and the bequest will not
be recognized until it is received.

Membership fee
Many NPOs receive membership fees. Such fees are considered fees for services when members receive services
having a value commensurate (in proportion to) with fees paid (e.g. to a cricket club or golf club). In other cases,
membership fees may be in substance contributions.

Page 5 of 23
280
An NPO would decide whether its membership fees are contributions or fees for services and account for them
accordingly on a consistent basis. Some membership fees have characteristics of both fees for services and
contributions. Such fees would be divided into the portion that relates to fees for services and the portion that is in
substance a contribution.

Example: Membership fee


Question: ABC Golf Club is members only club providing its members with sports facilities in the grounds owned and
maintained by it against annual subscription fee.

At 30 June 2012, the club had membership subscriptions in arrears amounting to Rs. 48,000,000 and had received
Rs. 12,000,000 in advance.

During the year to 30 June 2013, the club received Rs. 650,000,000 from its members. This amount includes:
▪ Rs. 26,000,000 received as donation from members (no conditions attached).
▪ Rs. 31,200,000 received for membership fee for the year to 30 June 2014.

At 30 June 2013, members owed Rs. 19,200,000 of subscriptions.


Half of the members who were in arrears at the end of the previous period still had not paid by 30 June 2013. It was
decided to write these amounts off.

Required:
How the revenue from above should be reported in financial statements of ABC Golf Club for the year ended on
30 June 2013?

Answer:
The donation of Rs. 26 million received shall be recognised as contribution revenue separately from fee for services
to members.

The subscription income (fee for services) may be calculated as follows:

Subscription Account
Rs. Rs.
b/d 48,000,000 b/d 12,000,000
subscription income (I&E) 600,000,000 Cash (Rs. 650m – 26m) 624,000,000
[fee for services]
Bad debts (Rs. 48m x 50%) 24,000,000
c/d 31,200,000 c/d 19,200,000
679,200,000 679,200,000

Government funding
Certain types of government funding are calculated and paid as if they were fees for services. However, because the
services being funded are provided to the NPO's community of service, and not directly to the government,
government funding is considered to be a contribution.

Page 6 of 23
281
Example: Government funding
Question: Mujahid Healthcare (MH) is a registered NPO. It has received government funding of Rs. 20 million for
which it has to provide vaccine (dosage and administration) for a viral disease to general public (8,000 dosages x Rs.
2,500 each) without taking any fee from them.

Required: Discuss the accounting treatment of above from perspective of MH.

Answer:
The amount of Rs. 20 million is being calculated on dosage basis (i.e. 8,000 dosages x Rs. 2,500) which might indicate
that Rs. 20 million should be recognised as fee-for-services in statement of income and expenditure.

However, since the service is not being provided to government but rather to MH’s community of service (i.e.
general public to whom MH provide healthcare services), the government funding of Rs. 20 million shall be
considered as contribution.

Further, since the purpose of government funding is specified, it shall be considered as restricted contribution.
In case NPO is required to provide goods or services directly to Government then the government funding should be
treated as fee income.

Contributed materials and services


A contribution of materials or services (e.g services of doctors by private hospitals to NPOs as a contribution rather
than cash) assets other than cash (e.g. food packets, medicines or clothing) would be measured at fair value only
when:
1. fair value can be reasonably estimated; and
2. when the materials and services are used in the normal course of the NPO's operations and would otherwise
have been purchased (if e.g NPO is not interested in advertisement but a channel has displayed an
advertisement out of its own choice it will not be recognized).

Often service contributions are not recorded because of record-keeping and valuation difficulties. For example, it
may be impractical to record the receipt of contributed services where the NPO depends heavily on the use of
volunteers to provide services (e.g. Hide collection volunteers throughout Pakistan). Where contributed materials
and services meet the criteria of fair value measurement, recording their value would provide useful information.
Contributed materials and services (e.g. cement bags or engineer provided by an engineering firm) that are part of
a constructed or developed capital asset would be recognized at fair value.

Inventories
Contribution of materials
When an NPO recognizes contributions of materials and goods, the cost of inventories shall reflect the fair value at
the date of contribution.

Inventories To be distributed at no charge or for a nominal charge (very small amount of money in comparison
with worth of an item e.g medicines)

Page 7 of 23
282
An NPO shall measure inventories at the lower of cost (fair value if contributed, purchase cost if purchased) and
current replacement cost (current purchase price) when they are held for:
1. distribution at no charge or for a nominal charge (MCQ 28); or
2. consumption in the production process of goods e.g somebody has donated chemicals to be used in production
of medicines (to be distributed) at no charge or for a nominal charge.

Example: Inventories

Question: Medicine-for-All is an NPO which provides medicine to communities living in underdeveloped areas at
nominal charge. It has following inventories:
Replacement
Cost NRV* Fair value
Item Type cost
Rupees
Panadol Received in kind Nil 6,000 26,000 28,000
Neubrol 24,000 4,000 24,500 25,000
Imodium 12,000 3,000 12,000 12,500
Purchased for cash
Motilium 15,000 2,500 14,700 15,200
Rijix 18,000 3,500 18,300 17,900
*provided at nominal charge [estimated selling price – estimated selling expenses]

Required:
Calculate the amount of inventory that should be presented in the statement of financial position of Medicine-for-
All from above data.
Answer:
Item Basis Rupees
Panadol Cost equal to fair value but replacement cost is lower[28,000 and
26,000
26,000]
Neubrol Cost (lower)[24,000 and 24,500] 24,000
Imodium Cost / replacement cost (equal) [12,000 & 12,000] 12,000
Motilium Replacement cost (lower) [15,000 & 14,700] 14,700
Rijix Cost (lower) [18,000 & 18,300] 18,000
Total 94,700

Collections
Definition: Collections
Collections are works of art(paintings), historical treasures (Kohinoor Diamond) or similar assets that are:
▪ held for public exhibition, education or research;
▪ protected, cared for and preserved; and
▪ subject to an organisational policy that requires any proceeds from their sale to be used to acquire other
items to be added to the collection or for the direct care of the existing collection.

Cost of collections:
1. Purchase price if acquired
2. Fair value if contributed (if not available then nominal value)
Plus directly attributable costs like PPE.

Page 8 of 23
283
Although items meeting the definition of a collection exhibit the characteristics of ‘assets’ they are excluded from
the definition of property, plant & equipment, and intangible assets. Collections are made up of items that are
often rare and unique. They have cultural and historical significance.

Although collections are usually held by museums or galleries, other NPOs may also have items that meet the
definition of a collection. For example, an NPO's library may include rare books which might be considered to be a
collection. The regular library materials, however, would not usually meet the definition of a collection.

NPOs holding collections act as custodians for the public interest. They undertake to protect and preserve the
collection for public exhibition, education or research. The existence of a policy requiring that any proceeds on the
sale of collection items be used to acquire additional items or for the direct care of the collection provides
evidence of the NPO's commitment to act as custodian of the collection.

Costs incurred in protecting and preserving collection is considered as repair and maintenance and therefore
recognise as expense.

Certain works of art and historical treasure not to be depreciated


Certain works of art and historical treasures may have lives that are so long as to be virtually unlimited. Works of
art and historical treasures in this category are those that have cultural, aesthetic (beauty), or historical value that is
worth preserving perpetually. In addition, the NPO must have the technological and financial ability to continue to
protect and preserve them. Works of art and historical treasures of this type would not be depreciated (if however,
carrying amount exceeds fair value then record a write down as impairment loss).

Property, plant and equipment


Definition: Tangible capital assets
Tangible capital assets are identifiable tangible assets that meet all of the following criteria:
▪ are held for use in the provision of services, for administrative purposes, for production of goods or for the
maintenance (cranes), repair, development or construction of other tangible capital assets;
▪ have been acquired, constructed or developed with the intention of being used on a continuing basis;
▪ are not intended for sale in the ordinary course of operations (not goods for sale); and
▪ are not held as part of a collection.

Measurement for contributed PPE:


A contributed asset (e.g. medical equipment) would be recognized at its fair value at the date of contribution.

When an estimate of fair value cannot reasonably be made, both the asset and the related contribution would be
recognized at nominal value (as in IAS 20).

A tangible capital asset purchased by an NPO at a value substantially below fair value would also be recognized at
its fair value with the difference between the consideration paid for the tangible capital asset and fair value reported
as a contribution.

A tangible moveable capital asset purchased from a grant may be recognised at carrying amount deducting the
grant. The grant is recognised in profit or loss over the life of the depreciable asset as a reduced depreciation
expense.

Page 9 of 23
284
If it is an asset granted for a specified period and the asset has to be returned at the end of the grant period, asset
shall be valued at fair value less present value of the estimated residual amount at the end of grant period(100,000
– 80,000).

Construction or development over time


The cost of PPE includes direct construction or development costs (such as materials and labour) and overhead costs
directly attributable to the construction or development activity. PPE which is developed or constructed by an NPO
might include contributed materials or labour, which would be recognized at fair value at the date of contribution.
Land

Land normally has an undeterminable life and would not be depreciated.


Unamortised deferred contributions related to PPE if it is not used:
When PPE no longer contribute to the NPO's ability to provide services, its carrying amount would be written down
to residual value, if any. A write-down would be necessary, for example, when the NPO no longer plans to use the
asset because it has been damaged or rendered obsolete.

When an asset's carrying amount is written down, a corresponding amount of any unamortized deferred
contributions related to the asset would be recognized as revenue, provided that all restrictions have been
complied with [MCQ 29].

Presentation of revenues and expenses — gross versus net


Revenues and expenses should be recognized and presented at their gross amounts (separately) and this
information may be presented in income and expenditure account or in the notes to the financial statements. The
determination of whether to report the revenues and expenses on a gross or net basis depends on the relative facts
and circumstances and requires significant judgment.

Example:
An NPO receives funding to undertake a specific research project. The NPO contracts at its own discretion with a
scientist to perform the research. The NPO would not have undertaken the research project had the funds not been
made available.

Required:
Whether the funding revenue and cost of scientist’s services be presented on gross basis or net basis?

Answer:
Although the NPO would not have undertaken the research project without the availability of the funding, the NPO
acts as the principal in contracting with the scientist. It specifies the details of the research to be carried out by the
scientist, and has discretion in selecting the scientist and in establishing the price to be paid. Thus, the expenses
incurred are obligations of the NPO. The funding revenue and cost of scientist services should be presented on
gross basis in statement of income and expenditure.

Example:
A research project is to be undertaken by a textile company, where there is a need for a trained person. An NPO
receives funding (reimbursement of salary) to allocate trained person to a textile company. The NPO allocates an
employee to textile company for the conduct of research. The NPO would be reimbursed for all the costs related to
that employee.

Page 10 of 23
285
Required:
Whether the reimbursement and employee-related costs be presented on gross basis or net basis?

Answer:
The NPO has an employee who is seconded (send) to a textile company to work under their direction and the NPO
is reimbursed for all of the costs related to that employee. As the NPO is the employer, they would report their
employee-related costs as expenses and would report the reimbursement of their costs as revenues on gross basis
in statement of income and expenditure.

Example:
An NPO engages in a number of fundraising activities, including a fundraising telethon, a telephone campaign, a
direct mail campaign, special events and a lottery. The NPO uses an outside fundraising consultant to conduct the
telethon and uses the NPO's own staff and volunteers in the telethon and the other activities. Funds solicited
(gathered) in each of the activities are raised in the name of the NPO.

Required: Whether the funds raised and related costs be presented on gross basis or net basis?

Answer:
Even though the NPO uses an outside fundraising consultant to conduct the telethon, the NPO is the principal in the
relationship with the donors as the funds are raised in its name. The NPO has discretion in selecting the outside
fundraiser, in establishing the fees to be paid and in determining the specifications of the telethon. The NPO also
has the credit risk if donors to the telethon do not pay according to their pledge. Thus, the NPO should recognize
the gross amounts fundraised in each of the activities as revenue of the NPO, and the total expenses of each
activity, including the fees charged by any outside consultant, as expenses of the NPO, separately.

Example:
An NPO is actively engaged in helping communities in flood affected area. A group of students organised a sports
event, announcing that the net proceeds of the event shall be given to the NPO.

Required:
Whether to report the revenue and costs of the event on gross basis or net basis?

Answer:
The NPO is not the principal in the fundraising event as it was not involved in organizing the event and did not bear
any risks in connection with it. The amount received by the NPO is a donation from the organizers of the event.
Neither the gross revenues nor the gross expenses of the event are recognized in the NPO's financial statements.
The net proceeds received are recognized as a contribution.

Conclusion: it means if the NPO is acting as a principal or the persons involved are of NPO’s employee then
recognise incomes and expenses on gross basis otherwise net.

Page 11 of 23
286
Definition: Fund accounting
Fund accounting comprises the collective accounting procedures resulting in a self-balancing set of accounts for
each fund established by legal, contractual or voluntary actions of an NPO. Elements of a fund can include assets,
liabilities, net assets, revenues and expenses (and gains and losses, where appropriate)[means each fund has all
the financial statement elements]

Net assets or fund balances may be internally or externally restricted. Internally restricted net assets or fund
balances are often referred to as reserves (or general fund).

Definition: Restrictions
Restrictions are stipulations imposed that specify how resources must be used. External restrictions are imposed
from outside the NPO, usually by the contributor of the resources. Internal restrictions are imposed in a formal
manner by the NPO itself, usually by resolution of the board of directors/council/board of trustees.

There are two methods of fund accounting:


1. Deferral method and
2. Restricted fund method (NOT EXAMINABLE)

1.Revenue recognition: deferral method


When an NPO uses fund accounting in their financial statements without following the restricted fund method,
contributions would be accounted for using the deferral method. The contributions and related income are
recognised as follows:
Contribution Recognition Accounting entries
A. Endowment Recognise as direct increases in Cash xxx
contributions net assets in the current period Endowment fund xxx
and excluded from revenue (In statement of
(MCQ 23). changes in net
(Endowment contributions will assets)
never be available to meet
expenses associated with the
organization operation.
Therefor an organization
following the deferral method
would exclude such contribution
from revenue available for
current expenses by recognising
them as direct increases in net
asset.)
B Restricted Recognise as revenue in current Cash xxx
(a) contributions for period. revenue xxx
expenses of (In I&E)
current period
(b) Restricted Defer as a liability and Cash Xxx
contributions for recognise as revenue in the Deferred revenue Xxx
(liability)

Page 12 of 23
287
Contribution Recognition Accounting entries
expenses of future same period(s) as the related Deferred revenue xxx
periods expenses are recognised. Revenue xxx
When the only restriction on a (when related
contribution is that it cannot be expense is
used until a particular future recognised)
period, the total amount of the
contribution would be
recognized as revenue in that
future period, whether or not it
has been spent.(e.g a
contribution for third future
period of operations)
(c) Restricted 1.In case of depreciable assets, Cash Xxx
contributions for defer as a liability and recognise Deferred revenue Xxx
the purchase of as revenue on the same basis as (liability)
capital assets the depreciation/amortisation Deferred revenue xxx
expense related to the acquired Revenue xxx
capital assets.

2.In case of non-depreciable Cash xxx


assets, recognise as direct Capital Asset fund xxx
increase in net assets (In statement of
(in this case, it is not possible to changes in net assets)
match the contribution with the
benefits provided since these
benefits are unlimited. Therefore
simply recognise as direct increase
in net assets)
Restricted 1.In case debt was incurred to Cash Xxx
contributions for fund expenses of future periods, Deferred revenue Xxx
the repayment of defer and recognise as revenue in (liability)
debt same period(s) as the related Deferred revenue xxx
expenses are recognised (MCQ Revenue xxx
No.24).
2.In case debt was incurred to Cash Xxx
fund the purchase of capital asset Deferred revenue Xxx
(depreciable), defer and recognise (liability)
as revenue on the same basis as Deferred revenue xxx
the depreciation/amortisation Revenue xxx
expense related to the acquired
capital assets(MCQ No.25).
3.In case debt was incurred to Cash xxx
fund the purchase of capital asset Capital Asset fund xxx

Page 13 of 23
288
(non-depreciable), recognise as (In statement of
direct increase in net assets. changes in net assets)
4.Otherwise, recognise as Cash xxx
revenue in income and revenue xxx
expenditure account(MCQ No.26) (In I&E)
Unrestricted Recognise as revenue in the Cash xxx
contributions current period revenue xxx
(In I&E)
Net investment 1.Externally restricted investment Cash xxx
income (includes income that must be added to Endowment fund xxx
revenue e.g rent principal resources held for (In statement of
income gains or endowment are recognised as changes in net assets)
losses on direct increase or decrease in net
investments. If assets. [MCQ 27]
amount is invested
in shares)

2.Other externally restricted Cash


investment income (similarly Deferred revenue
related expenses) are or In capital assets
recognised as per type of fund
restriction discussed above [ in B
and C ]

3.In case there is no external Cash


restriction, recognise in the revenue
statement of income and (In I&E)
expenditure.
Deferred contributions balances should be presented in the statement of financial position outside net
assets as liability
(E) Allocation of This Allocation shall be Unrestricted fund
Unrestricted fund reported as inter fund Restricted fund
transfer in statement of
changes in net assets

Page 14 of 23
289
Example: Professional Sports Club (I)
Question: The following information relates to Professional Sports Club (PSC), a Not-for-Profit Organisation.
Trial balance as at 30 June 2014
Dr. Cr.
Rs. m Rs. m
Total Funds as on 01 July 2013 (note 1) 1,715
Non-current assets (net) (note 6) 428
Investments (long term) 1,204
Short term bank loan 17
Prepaid and accrued expenses 8 11
Cash at bank 43
Fee-for-services 340
Fundraising in various tournaments (net proceeds) [organized by others 15
not the NPO otherwise funding and expenses should be presented
separately]
Contributions(note 2) 94
Government funding(note 3) 150
Investment income(note 4) 144
Salaries(note 8) 403
Rent and utilities (note 8) 354
Other expenses(note 8) 46
2,486 2,486
Additional information:
1. The composition of fund balances is as follows:
Rs. m
Fund for Supporting the Young-Talent (Externally imposed stipulation that
resources contributed be maintained permanently)[therefore endowment 50
fund]
Fund for expenses of gymnasium and training centre (Externally imposed
115
stipulation for specific use of resources)
Fund for acquiring a franchise in a popular league (Internally imposed
3
stipulation for specific use of resources)
Fund for general operations: no restrictions 1,547
1,715

. The details of contributions (same restrictions apply as are applicable to related fund) are as follows:
Rs. m
Contribution for ‘Supporting the Young-Talent’ [in statement of changes in
15 [para A]
net assets]
Contribution for expenses of gymnasium and training centre. 2 [para Bb]
Contribution to acquire freehold land (external restriction). However, the
12 [para B c 2]
land has not been acquired yet.
Contribution to repay loan that was taken to fund current year expenses
8 [para B (d)] 4
(therefore recognise as revenue).

Page 15 of 23
290
Contributions (unrestricted but PSC itself imposed restriction that Rs. 3
million will be allocated annually for acquiring franchise in a popular
57[ Para [C]]
league) (therefore recognise as revenue, however 3 million will be
transferred through statement of net assets)
94
3. The government funding was received to support PSC general operations for five years starting from 1 st January
2014.[para B(b)][treat it like government grant of IAS 20]

4. Investment income in trial balance include investment income of Rs. 6 million which is externally restricted to be
added to principal amount of resources for Young-Talent fund to be maintained permanently[para D 1 include in
endowment fund in statement of net assets]. There is no other restrictions on investment income [para D 3 include
in I & E].

5.Long term assets in the trial balance include freehold land of Rs. 20 million and collections of Rs. 8 million. These
collections represent items of such historic value that is worth preserving perpetually and PSC is committed to
protect and preserve them as part of its organisation policy.(collection are to be presented separately from other
non-current assets)

6.Non-current assets are depreciated at 20% reducing balance method. All the depreciation is allocated to general
operations.[in I&E]

7.As part of agreement with contributors of ‘Supporting the Young-Talent’, PSC is required to allocate Rs. 5 million
from unrestricted fund to the endowment fund, annually.[transfer through statement of changes in net assets]
8.The allocation of expenses is as follows:
Gymnasium
General
and training
operations
centre (para
(in I&E)
D 2)
Rs. m Rs. m
Salaries 370 33
Rent and utilities 325 29
Other expenses 40 6
735 68
Required:
Prepare the following (under deferral method) for PSC:
▪ Statement of income and expenditure for the year ended 30 June 2014.
▪ Statement of changes in net assets for the year ended 30 June 2014.
▪ Statement of financial position as at 30 June 2014 (single column).

Page 16 of 23
291
Answer:
Professional Sports Club
Statement of income and expenditure
For the year ended 30 June 2014
Income Rs. m
Fee-for-services 340
Fundraising proceeds 15
Contributions [8 + 57] 65
Government funding [150 / 5 years x 6/12] 15
Investment income [144 – 6] 138
Total 573
Expenditures
Expenses of gymnasium and training centre (68-68) 0
Salaries 370
Rent and utilities 325
Other expenses 40
Depreciation of long term assets [(428 – 8 – 20) x 20%] 80
Total (815)
Surplus / (Deficit) (242)

Professional Sports Club


Statement of changes in net assets (like statement of changes in equity)
For the year ended 30 June 2014
Internally Externally Externally
Unrestricted
restricted Restricted Restricted
Franchise Young-Talent Capital Total
General fund acquisition Endowment asset
fund fund

Balance 1 July 1,547 3 50 - 1,600


Surplus / (deficit) (242) - (242)
Contributions 15 12 27
Investment income 6 - 6
Internally imposed -
(3) 3 0
restrictions
Transfers (5) 5 - 0
Balance 30 June 1,297 6 76 12 1,391

Page 17 of 23
292
Professional Sports Club
Statement of financial position
As at 30 June 2014
Non-current assets Rs. M
Non-current assets [428 – 8 – 80] 340
Collections 8
Investments 1,204
1,552
Current assets
Prepaid expenses 8
Cash at bank 43
51
1,603
Fund balances / Net assets
Net assets: restricted for endowments 76
Net assets: internally restricted for franchise rights 6
General fund / Unrestricted net assets 1,297
Capital Asset fund / externally restricted 12
1,391

Non-current liabilities
Externally restricted fund for expenses for gymnasium and training centre (115+2-68) 49
Deferred grants/contributions Government funding [150 – 15 – 30] 105

Current liabilities
Deferred grants/contributions [ 150 / 5] (Govt funding current liability) 30
Short term bank loan 17
Accrued expenses 11
58
1,603
.

Page 18 of 23
293
Example:
Following is the trial balance of Chongtar International Hospital as on 31 December 2019:

Debit Credit
---- Rs. in million ----
Burns ward - capital work in progress 55.3
Cafeteria sales 24.4
Cash and bank balances 8.4
Donations for burns ward during the period [externally restricted][para B 75.1
c 1]
Expenses and gifts for ‘walk on diabetes day’ 2.6
Fees from patients 125.0
General donations 82.6
General fund 195.6
Inventory - cafeteria 4.7
Inventory - medicines 19.4
Inventory - hospital supplies 8.5
Medical equipment 185.4 64.2
Miscellaneous expenses 8.5
Other fixed assets 110.7 54.7
Payables 38.9
Purchases - cafeteria 16.4
Purchases - medicines 60.5
Purchases - hospital supplies 18.7
Receivables - panel corporates 31.4
Rent 19.6
Sponsorship for ‘walk on diabetes day’ 2.2
Salaries - administrative staff 24.0
Salaries - doctors and nursing staff 38.2
Short term investments 38.0
Utilities 12.4
662.7 662.7

Additional information:
(i) Cost of closing physical inventory of medicines and hospital supplies was Rs. 25.8 million and
Rs. 13.8 million respectively. Medicines costing Rs. 3.1 million were found expired. Medicines are only
used to treat the admitted patients and are not sold separately.
(ii) Year-end physical count of cafeteria inventory could not take place. Goods are sold in cafeteria at a
gross margin of 25% on sales.
(iii) Rent outstanding at year-end was Rs. 1.4 million.

Page 19 of 23
294
(iv) 15% of salaries of administrative staff and 10% of rent are related to cafeteria.
(v) Walk on diabetes day’ was organized in December 2019 [As it is the event conducted by the NPO,
which means income and expenditures should be presented separately]. Expenses relating to the
event amounting to Rs. 1.2 million were outstanding and unrecorded at year end.
(vi) Medical equipment having fair value of Rs. 36.8 million were received as donation on 01.01.2019.
These have been brought into use but have not been recorded in the books.(contributed depreciable
asset should be recognized at fair value and treat the contribution as income over the useful life of
asset.)
(vii) Depreciation is charged on all assets on reducing balance method at 15% per annum.

Required: Prepare the following (using deferral method):


(a) Income and expenditure account for the year ended 31 December 2019. (12)
(b) Statement of financial position as on 31 December 2019. (06)

Answer
Chongtar International Hospital
Income and Expenditure Account
For the year ended 31-12-2019
“Rs. in Millions”
Incomes:
Fees from Patients 125.0
General donation 82.6
Recognition of revenue of contribution of Medical equipment [36.8 x 15%] 5.52
Sponsorship for walk on diabetes day 2.2
Profit from Cafeteria (W- 1) 0.4
Total 215.72
Expenditures:
Loss of Medicines 3.1
Salaries of Administrative staff (24 - 3.6) 20.4
Salaries - Doctors and nursing staff 38.2
Medicines used (19.4 + 60.5 - 3.1 - 25.8) 51.0
Hospital supplies used (8.5 + 18.7 - 13.8) 13.4
Rent (19.6 + 1.4 - 2.1) 18.9
Walk on diabetes day (2.6 + 1.2) 3.8
Depreciation – Medical equipment 23.7
Depreciation – Other fixed assets 8.4
Utilities 12.4
Misc. expenses 8.5
Total 201.80
Surplus 13.92

Page 20 of 23
295
Workings:
1. Profit of Cafeteria:
Sales 24.4
Cost of Sales:
Opening stock 4.7
Purchases 16.4
Closing stock (Bal) 2.8 (18.3)
Gross Profit 6.1
Expenses:
Salaries (24 x 15%) (3.6)
Rent (19.6 + 1.4) x 10% (2.1)
Net Profit 0.4

Chongtar International Hospital


Statement of Financial Position
As on 31-12-2019
Non – Current Assets: “Rs. in Millions”
Medical equipment (185.4 + 36.8 - 64.2 - 23.7) 134.3
Other fixed assets (110.7 - 54.7 - 8.4) 47.6
Burn ward – CWIP 55.3
Current Assets:
Cafeteria stock 2.8
Medicine stock 25.8
Hospital supplies 13.8
Receivables 31.4
Short term investments 38.0
Cash and bank 8.4
Total 357.4
Fund and Liabilities:
General Fund[unrestricted] 195.6
Add: Surplus 13.92 209.52
Total
Non-current liabilities
Burn Ward Fund [restricted] 75.1
Deferred contributions [36.8 -5.52-(31.3 x 15%) Current portion 26.58
based on reducing balance method]
Current Liabilities:
Payable 38.9
Deferred contributions [31.3× 15%) 4.7
Rent payable 1.4
Expenses payable on walk on diabetes day 1.2
Total 357.4

Page 21 of 23
296
Workings:
i) Loss of Medicines 3.1
Purchase of medicines 3.1

ii) Cost of Sales of Cafeteria sales to get closing stock of Cafeteria:


Sales = 24.4
Margin = 25% on sales
24.4/100 x 75% = 18.3
Inventory
b/d 4.7 Cost of Sales 18.3
Purchases 16.4 c/d (bal) 2.8

iii) Rent expense 1.4


Rent payable 1.4
iv) Salary expense related to cafeteria
24 x 15% = 3.6
Rent expense related to cafeteria:
Given = 19.6
+ Recorded = 1.4
Total 21
x10% 2.1

Expenses related to walk on diabetes day:


Expenses on Diabetes day 1.2
Exp payable 1.2
Medical Equipment 36.8
Deferred contribution 36.8

Medical Equipment:
(185.4 – 64.2) x 15% = 18.18
+ 36.8 x 15% = 5.52
Total 23.7

vii) Other fixed assets (110.7 - 54.7) x 15% = 8.4

Recognition and measurement


Recognition criteria
The recognition criteria under ASNPO are as follows:
• the item has an appropriate basis of measurement and a reasonable estimate can be made of the amount
involved; and
• for items involving obtaining or giving up economic benefits, it is probable that such benefits will be obtained
or given up.

Page 22 of 23
297
The recognition criteria provide general guidance on when an item is recognized in the financial statements.
Professional judgment is required to consider the specific circumstances to identify whether any particular item is
recognized or not.
Revenue Revenues are generally recognized when performance is achieved and reasonable assurance
regarding measurement and collectability of the consideration exists.
Unrestricted Unrestricted contributions to NPO do not normally arise from the sale of goods or the
contributions rendering of services and, consequently, performance achievement is generally not relevant
to the recognition of unrestricted contributions; such revenues are generally recognized when
received or receivable.
Restricted Restricted contributions are recognized based on the nature of the related restriction.
contributions
Gains Gains are generally recognized when realized.
Expenses and Expenses and losses are generally recognized when an expenditure or previously recognized
losses asset does not have future economic benefit. Expenses are related to a period on the basis of
transactions or events occurring in that period or by allocation applying the matching
concept(like depreciation).

Measurement
Financial statements of NPOs are prepared primarily using the historical cost basis of measurement whereby
transactions and events are recognized in financial statements at the amount of cash or cash equivalents paid or
received or the fair value ascribed to transactions and events when they took place.

Financial statements are prepared with capital maintenance measured in financial terms [which means financial
capital maintenance (money terms) as in MCQ number 20] and with no adjustment being made for the effect on
capital of a change in the general purchasing power of the currency during the period.

Types of funds (Sources of Contributions):


The funds can be classified into following three categories:
1.Endowment fund
An endowment fund is a self-balancing set of accounts which reports the accumulation of endowment
contributions. Only endowment contributions and investment income subject to restrictions stipulating that it be
added to the principal amount of the endowment fund would be reported as revenue of the endowment fund.
Allocations of resources to the endowment fund that result from the imposition of internal restrictions are recorded
as inter-fund transfers (in statement of net assets)

2.Restricted fund
A restricted fund is a self-balancing set of accounts the elements of which are restricted or relate to the use of
restricted resources.
Allocations of resources that result from the imposition of internal restrictions are recorded as inter-fund transfers
to the restricted fund (in statement of net assets)

3.General fund / unrestricted fund


A general fund is a self-balancing set of accounts which reports all unrestricted revenue and restricted
contributions for which no corresponding restricted fund is presented. The fund balance represents net assets that
are not subject to externally imposed restrictions.

Page 23 of 23
298
Not For Profit Organisations
⯈ Example:
Ali has been very successful in business. When he was a young man, he very much enjoyed playing cricket and
has very fond memories of his days at the village cricket club.

He has donated Rs. 1,000,000 to the club to fund the building of a new club house which is under construction and
expected to be completed by the end of next year.

Required: How the above amount of Rs. 1 million should be recognised in the books of villagecricket club?

⯈ Answer:

This is restricted contribution for construction of a capital asset.


Initially, this amount shall be recognised as deferred contribution and presented as liability.
Subsequently, this shall be recognised as revenue on the same basis depreciation expense is
charged on the building of new club house.
⯈ Example:
A social club in a small town has managed to accumulate a significant balance on its accumulated fund over the years.

Its board of trustees have decided that the club should establish a fund to contribute to the school fees of children of
high promise from the town. Parents of such children would apply to the club for a grant of Rs. 50,000. A total Rs.
1,500,000 is to be set aside for this purpose.

Required: How to account for the above when the amount is set-aside and subsequently when the amount is actually
paid?

⯈ Answer:

This is not restricted contribution because there is no externally imposed stipulation rather an
internal restriction has been imposed by the NPO itself.
The transfer of Rs. 1,500,000 shall be presented in statement of changes in net assets (and not in
the statement of income and expenditure).

Debit General fund Rs. 1,500,000


Credit Special education fund Rs. 1,500,000
Subsequently, the amount paid will be recognised as reduction from the special fund.
Debit Special education fund Rs. 1,500,000
Credit Cash Rs. 1,500,000

Page 1 of 6

299
⯈ Example:
A member of cricket club donated Rs. 2 million for repayment of loan obtained by the club in order to finance its
general operations.

Required: How the above donation shall be recognised?

Answer
This is restricted contribution for repayment of debt. However, since the loan relates neither to expenses of future
periods nor to capital assets, the contribution shall be recognised as revenue in the current period

⯈ Example:
Jameel Mahtab Dispensary, an NPO, had capital asset of furniture at carrying value of Rs. 600,000 and related
unamortised deferred contribution balance of Rs. 400,000 in statement of financial position. At that date, the
furniture was destroyed by fire completely and is now worth nothing.

Required: Prepare accounting entries for the above issue.

Answer

The capital asset has been fully impaired and needs to be written off completely.
Debit Impairment loss (I&E) Rs. 600,000
Credit PPE (furniture) Rs. 600,000

In order to ensure application of matching concept, the unamortised deferred contribution shallbe recognised as
revenue.
Debit Deferred contribution Rs. 400,000
Credit Contribution revenue (I&E) Rs. 400,000

Page 2 of 6
300
PREPARATION OF FINANCIAL STATEMENTS
General [ASNPO: 3.9]
The accounting and approach for preparation of financial statements of an NPO is similar to general-purpose
financial statements of other entities except for the issues specifically addressed in ASNPO.
Financial statements of NPO shall normally include:
• statement of financial position (or balance sheet)
• statement of income and expenditure
• statement of changes in net assets
• statement of cash flows.

Notes to financial statements and supporting schedules to which the financial statements are cross-referenced are
an integral part of such statements; the same does not apply to information set out in other material attached to or
submitted with financial statements.

Statement of financial position [ASNPO: 4.18 & 4.25]


Information about the NPO's liquidity is presented by classifying current assets separately from non-current assets
and current liabilities separately from non-current liabilities. Cash and other assets subject to external restrictions
limiting their use to beyond one year from the date of the statement of financial position would be classified as non-
current assets.

Under the deferral method of accounting for contributions:


• endowment contributions are accumulated in the net assets balance; and
• internally restricted balances are reflected as appropriations of unrestricted net assets; and
• externally restricted contributions are accumulated in the statement of financial position as deferred
contributions.
A format of statement of financial position of an NPO is given below:
Not-for-Profit Organisation
Statement of financial position as at 31 December 20X2

20X2 20X1
Non-current assets Rs. 000 Rs. 000
STICKY NOTES

Capital assets (property, plant and equipment) 2,037 XX


Collections 80 XX
Investments 4,157 XX
6,274 XX
Current assets
Office supplies stock 55 XX
Prepaid expenses 58 XX
Subscription receivable 17 XX
Cash and cash equivalents 183 XX
313 XX
6,587 XXX

Page 3 of 6

301
20X2 20X1

Fund balances / Net assets


General fund / Unrestricted net assets 2,939 XX
Net assets: restricted for endowments 208 XX
Net assets: internally restricted for special projects 340 XX
3,487 XX
Non-current liabilities

AT A GLANCE
Deferred grants/contributions 1,800 XX
Loans 300 XX
2,100 XX
Current liabilities
Deferred grants/contributions 500 XX
Subscriptions received in advance 100 XX
Accrued expenses 400 XX
1,000 XX
6,587 XX

SPOTLIGHT
Statement of income and expenditure [ASNPO: 4.29, 13.9, 4.31 & 4.35]
Revenues and expenses should be recognized and presented at their gross amounts and this information may be
presented in the notes to the financial statements. NPO may classify expenses in the statement of income and
expenditure:
• by object (for example, salaries, rent, utilities);
• by function (for example, administrative, research, ancillary operations); or
• by program.

An NPO would classify its expenses in the manner that results in the most meaningful presentation in the

STICKY NOTES
circumstances. Whether the NPO prepares its budgets by function or object would be a factor to consider in deciding
which method of expense classification would be most appropriate for the NPO's financial statements.

The following approach is considered when attributing an expense among various operating functions:
• an expense that contributes directly to the output of one function is applied directly to that function, for
example, the cost of a staff member exclusively devoted to that function.
• an expense that contributes directly to the output of more than one function is attributed on a reasonable and
consistent basis to each function to which it applies (for example, the rent applicable to the space used for more
than one separately reported function, and the remuneration expense of an executive director of a small health
care NPO who, in addition to managing the NPO, provides direct health care services to clients of that NPO).

Page 4 of 6
302
The statement of income and expenditure should present:
• for each financial statement item, a total that includes all funds reported; and
• total excess or deficiency of revenues and gains over expenses and losses for the period.

A format of statement of income and expenditure of an NPO is given below:


Not-for-Profit Organisation
Statement of income and expenditure for the year ended 31 December 20X2

20X2 20X1

Income Rs. 000 Rs. 000

Fee-for-services 5,300 XX

Government grants 1,200 XX

Contributions 170 XX
AT A GLANCE

Fundraising events 300 XX

Profit from coffee bar / shop / canteen 50 XX

Investment income 31 XX

Other income 2 X

7,053 XXX

Expenditures

Salaries 3,070 XX

Rent 1,320 XX
SPOTLIGHT

Office supplies used 610 XX

Utilities 880 XX

Marketing and communications 422 XX

Amortisation of capital assets 153 XX

(6,455) (XXX)

Excess of income over expenditure i.e. surplus 598 XX


STICKY NOTES

Statement of changes in net assets [ASNPO: 4.37, 4.38, 4.9 & 4.12]
The statement of changes in net assets is presented in the similar way a statement of changes in equity is
prepared i.e. showing the movements in net assets during the year.

The statement of changes in net assets may be referred to as ‘the statement of changes in fund balances’ when
the NPO uses fund accounting in its financial statements.

Page 5 of 6
303
Inter-fund transfers should be presented in the statement of changes in net assets. Allocations of revenues and
expenses between funds that are made when the NPO first recognizes the revenue or expense are not consideredto
be transfers.

A format of statement of changes in net assets for an NPO is given below:

Not-for-Profit Organisation
Statement of changes in net assets for the year ended 31 December 20X2
Unrestricted / Endowment Special Total
General fund fund (internal) fund
Balance 1 January 2,386 150 315 2,851
Surplus 598 598
Endowment Contributions 20 20
Investment income 18 18
Transfer to special fund (25) 25 -
Transfer to endowment fund (20) 20 -
Balance 31 December 2,939 340 208 3,487

STICKY NOTES

Page 6 of 6
304
Question 1
Following amounts have been extracted from the financial statements of Lithops Limited:

2020 2019
----- Rs. in million -----
Sales 500 450
Cost of sales 378 300
Trade receivables 95 80
Trade payables 72 60
Inventory 93 75
Cash at bank 12 16

All sales and purchases are made on credit.

Required:
a) Calculate working capital cycle days for 2020. (Assume a 360 day year) (04)
b) Suggest four possible measures that can be taken to reduce working capital cycle days. (03)

Answer 1
(a) Working capital cycle days:

Trade A 𝐴𝑣𝑒𝑟𝑎𝑔𝑒 𝐷𝑒𝑏𝑡𝑜𝑟𝑠 87.5[(95 + 80) ÷ 2] ⇒ 63 days


𝑥 360 𝑥 360
receivables 𝐶𝑟𝑒𝑑𝑖𝑡 𝑆𝑎𝑙𝑒 500
collection period
Inventory holding B 𝐴𝑣𝑒𝑟𝑎𝑔𝑒 𝐼𝑛𝑣𝑒𝑛𝑡𝑜𝑟𝑦 84[(93 + 75) ÷ 2] ⇒ 80 days
𝑥 360 𝑥 360
period 𝑐𝑜𝑠𝑡 of sale 378
Trade payables C 𝐴𝑣𝑒𝑟𝑎𝑔𝑒 𝐶𝑟𝑒𝑑𝑖𝑡𝑜𝑟𝑠 66[(72 + 60) ÷ 2] ⇒ 60 days
𝑥 360 𝑥 360
payment period 𝐶𝑟𝑒𝑑𝑖𝑡 𝑃𝑢𝑟𝑐ℎ𝑎𝑠𝑒 (𝑤 − 1) 396
Working capital cycle =A+B−C = 63 + 80 ‒ 60 ⇒ 83 days

W-1: Calculation of purchases Rs. in million


Cost of goods sold 378
Add: Closing inventory 93
Less: Opening inventory (75)
Purchases (on credit) 396
(b) Measures to improve working capital cycle days:
▪ Give incentives to customers to pay on time
▪ Do not transact with customers who have a history of defaulting/late payments
▪ Automate the monitoring of accounts receivables
▪ Resolve disputes with customer as early as possible

Page 1 of 15
305
Q.2 The draft financial statements of Barbary Cement Limited (BCL) for the year ended 31 December
2020 include a plant having a carrying value of Rs. 400 million. Due to technological change, the remaining useful
life of the plant has been reduced to 4 years.

Following information has been gathered for impairment testing of the plant:
(i) Inflows from sale of product to be manufactured by the plant for the year 2021 are estimated at Rs. 200
million. These inflows are subject to 10% decrease in each subsequent year due to declining demand.
(ii) Outflows from operational cost for 2021 are estimated at Rs. 80 million. These outflows would increase by 5%
in each subsequent year despite decline in demand due to inflation and increase in plant’s wear and tear.
(iii) BCL’s net profit is subject to income tax of 20%.
(iv) Depreciation on plant is calculated using straight line method.
(v) The plant’s net disposal proceeds at the end of the useful life is estimated at Rs. 100 million.
(vi) Pre-tax and post-tax discount rates are 12% and 9.6% per annum respectively.
(vii) A technologically advanced plant with similar capacity can be purchased at Rs. 350 million. BCL has
received an offer to buy the existing plant for Rs. 250 million. BCL will have to incur shipping cost of Rs. 7
million, to dispatch the existing plant to the purchaser.

Required:
Compute the impairment loss to be recognized as at 31 December 2020. (07)

A.2

------ Rs. in million ------


Impairment loss:
Carrying value 400.0
Recoverable amount
Value in use (W-1) 333.6
Fair value less cost of sell (250‒7) 243.0
Higher of above (333.6)
66.4

W-1: Value in use 2021 2022 2023 2024 Total


--------------- Rs. in million ---------------
Inflows from sale 200 180 162 145.80
Operational cost (80) (84) (88.2) (92.61)
Disposal value - - - 100
Cash flows undiscounted 120 96 73.8 153.19
Discount factor @ 12% 0.8929 0.7972 0.7118 0.6355
PV of cash flows /value in use 107.2 76.5 52.5 97.4 333.6

Page 2 of 15
306
Q.4

An asset was purchased on 1 January 2017 for Rs. 100 million with useful life of 6 years and residual value
(i) of Rs. 10 million. On 1 January 2020, it is revalued to Rs. 120 millionwith remaining useful life of 3 years and
expected residual value of Rs. 15 million. How much depreciation will be charged for the year ended 31
December 2020?
(a) Rs. 15 million (b) Rs. 35 million
(c) Rs. 20 million (d) Rs. 25 million (01)
When items of property, plant and equipment are stated at revalued amounts, which ofthe following
(iii) disclosures shall be made?

(a) Any restrictions on the distribution of the revaluation surplus to shareholders


(b) The carrying amount of temporarily idle property, plant and equipment
(c) The gross carrying amount of any fully depreciated property, plant and equipmentthat is still in use

(d) All of the above (01)


(iv) Which of the following concepts measures profit in terms of an increase in theproductive
capacity of an entity?
(a) Physical capital maintenance
(b) Historical cost accounting
(c) Financial capital maintenance (money terms)
(d) Financial capital maintenance (real terms) (01)

) Which of the following should be included in the initial cost of investment property?
(a) Cost incurred on opening ceremony to celebrate completion of property
(b) Operating losses incurred before the property achieves the planned level ofoccupancy
(c) Abnormal waste of materials incurred in construction of property
(d) Property transfer taxes
(01)
(vi) An entity purchased an investment property on 1 January 2018 for Rs. 35 million. The property had an
estimated useful life of 35 years with no residual value. At 31 December 2020, the property had a fair
value of Rs. 42 million. On 1 January 2021, the property was sold for net proceeds of Rs. 40 million. Calculate
the profit or loss on disposal under both the cost and fair value models.
Cost model Fair value model
(a) Gain of Rs. 2 million Gain of Rs. 2 million
(b) Gain of Rs. 8 million Loss of Rs. 2 million
(c) Gain of Rs. 7 million Loss of Rs. 2 million
(d) Gain of Rs. 8 million Gain of Rs. 5 million

Page 3 of 15
307
Which of the following is not considered as transaction with owners with reference tostatement of changes in
equity?
(a) Issuance of shares at par (b) Issuance of shares at premium
(c) Profit for the year (d) Bonus issue of shares
(01)
(viii) Which two of the following factors could cause a company’s gross profit percentage onsales to be
above the expected level?
(b) Over-statement of closing inventories
(c) Sales were higher than expected
(d) Inclusion of disposal proceeds of non-current assets in sales
(e) Decrease in carriage charges borne by the company on goods sent to customers
(01)

A.4 (i) (b) Rs. 35 million


(iii) (a) Any restrictions on the distribution of the revaluation surplus to shareholders
(iv) (a) Physical capital maintenance
(v) (d) Property transfer taxes
(vi) (b) Gain of Rs. 8 million Loss of Rs. 2 million
(vii) (c) Profit for the year
(viii) (a) Over-statement of closing inventories
(c) Inclusion of disposal proceeds of non-current assets in sales

Q.6 Following are the extracts from the financial statements of Saguaro Limited (SL) for theyear ended 30
June 2021:

Statement of financial position as on 30 June 2021


Assets 2021 2020 Equity & liabilities 2021 2020
Rs. in million Rs. in million
Operating fixed assets 820 848 Share capital (Rs. 10 each) 700 500
Accumulated depreciation (300) (262) Share discount (40) -
Capital work in progress 84 - Retained earnings 220 315
Inventories 274 245 Long-term loans 175 210
Trade receivables 177 204 Trade payables 180 130
Insurance claim - 31 Accrued expenses 48 43
Advance to supplier 78 60 Current portion of long-
Cash and bank balances 193 112 term loans 43 40
1,326 1,238 1,326 1,238

Page 4 of 15
308
Statement of profit or loss for the year ended 30June
2021
Rs. in million
Sales 757
Cost of sales (485)
Gross profit 272
Operating expenses (310)
Gain on disposal of equipment 17
Loss before interest (21)

Other information:
(i) SL declared a final dividend of 10% on 30 September 2020 which was paid
in December 2020.
(ii) 20 million shares were issued in May 2021.
(iii) Insurance claim was related to plant and machinery destroyed in April 2020. The
plant had cost and book value of Rs. 63 million and Rs. 42 million respectively.
(iv) During the year, SL disposed of equipment having cost and net bookvalue of Rs.
75 million and Rs. 35 million respectively.
(v) Current portion of long-term loans include accrued interest of Rs. 5 million.
(2020: Rs. 1 million)
(vi) Trade payables include an amount of Rs. 14 million payables against capitalwork
in progress.

Required:
(a) Prepare SL’s statement of cash flows for the year ended 30 June 2021. (16)
(b) Calculate SL’s cash flow from operations for the year ended 30-06-2021 by
using direct method
(07)

A.6 Saguaro Limited


Statement of cash flows for the year ended 30 June 2021

Indirect method
Rs. in million
Cash flows from operating activities
Loss for the year (45)
Adjustments for:
Depreciation on property, plant and equipment 78
Interest expense 24
Gain on disposal of property, plant and equipment (17)
Profit before working capital changes 40
Changes in working capital:
Increase in inventory (29)

Page 5 of 15
309
Decrease in trade receivables 27
Increase in advance to supplier (18)
Increase in accrued expenses 5
Increase in trade payables 36
21
Cash generated from operations 61

Interest paid (20)


Net cash flows from operating activities 41

Cash flows from investing activities


Purchase of property, plant and equipment (47)
Proceeds from claim of machinery disposed last year 31
Payment for CWIP (70)
Proceeds from disposal of equipment 52
Net cash flows used in investing activities (34)

Cash flows from financing activities


Proceeds from issue of shares 160
Repayment of long term loan (36)
Dividend paid (50)
Net cash flows from financing activities 74
Net increase in cash and cash equivalents 81
Cash and cash equivalents at beginning of the year 112
Cash and cash equivalents at the end of the year 193

(b) Saguaro Limited Statement of Cash Flow For the year ended June 30,2021(Direct Method)
Receipt from customers (w-1) 784Payment to Suppliers (w-2) (496)Payment for exp (w-3) (227)Cash generated
from Operation61

Page 6 of 15
310
Page 7 of 15
311
Page 8 of 15
312
Q.7 Following information pertains to non-current assets of Bunny Ear Limited (BEL):

Land:
In January 2019, the government allotted a piece of land to BEL subject to the condition that BEL will establish a factory
building on it. The land was recorded at its fair value of Rs. 100 million.

Factory building:
On 1 March 2019, BEL started construction of the factory building. The construction work was completed on 30 June
2020. Payments related to the construction of the factory were as follows:

Description Date of payment Rs. in million


st
1 bill of contractor 1-Mar-2019 130
2nd bill of contractor 1-Aug-2019 190
rd
3 bill of contractor 1-Jan-2020 180
Last bill of contractor 1-Jul-2020 100
The project was financed through:
i. Government grant of Rs. 200 million received on 1 February 2019. Unused funds from government grant
were invested in a saving account @ 8% per annum.
ii. Withdrawals from the following existing running finance facilities obtained from Bank A and Bank B.

The relevant details are:


Bank A Bank B
Obtained on 1 January 2019 1 January 2020
Markup rate 12% 14%
-------- Rs. in million --------
Balance on 31 December 2019 250 -
Markup for 2019 22 -
Balance on 31 December 2020 350 200
Average balance during 2020 300 150
Markup for 2020 36 21

Manufacturing plant:
The manufacturing plant was purchased on 1 August 2020 at cost of Rs. 420 million.Rs. 240 million was
financed through an interest free loan from government. The loan will be forgiven if the plant is operated for atleast 4
years by BEL. Upon acquisition, there is a reasonable assurance that BEL will comply with this condition.

Other information:
• BEL uses cost model for subsequent measurement of property, plant and equipment.
• All government grants are recorded as deferred income and a part of it is transferred to income each year.
• Useful life of the factory building and manufacturing plant has been estimated at 25 years and 10 years respectively.

Page 9 of 15
313
Required:
Prepare relevant extracts (including comparative figures) from BEL’s statement of profit or loss for the year ended
31 December 2020 and statement of financial position as on that date. (Notes to the financial statements are not required.
Borrowing costs are to be calculated on the basis of number of months) (16)

A.7 Bunny Ear Limited


Extract from statement of profit and loss for the year ended 31 December 2020

2020 2019
------------Rs.in million--------------
Depreciation
• Factory building 12.5 -
• Manufacturing plant 17.5 -
Income from saving account (w-3) 3.66
Grant income
• Land 2.0 -
• Factory building 4.0
• Manufacturing plant 10.0
Interest expense (36+21-19),(22-6) 38.0 16.0

Extract from statement of financial position as on 31 December 2020

2020 2019
------------Rs.in million--------------
Non-current assets
Property plant and equipment
• Land 100.0 100.0
• Factory building [ (625(w1)-12.5(625/25×6/12)] 612.0
• Manufacturing plant (420-17.5(420/10×5/12)) 402.5
• Capital work in process-factory building (w-1) - 326.0

Non-current liabilities
Deferred government grant
• Land (100-2 (100/25×6/12)-4(100/25) 94 100.0
• Factory building (200-4 (200/25×6/12)-8(200/25) 188 200.0
• Manufacturing plant (240-10) (240/10×65/12)-24(240/10) 206

Current liabilities
Running finance 350+200 550.0 250.0
Deferred Govt grant (next year income)
Land 4 -
Factory building 8 -
Manufacturing plant 24 -

Page 10 of 15
314
W-1: cost of factory building Rs. In million
Payments in 2019 (130+190) 320.0
Borrowing cost capitalized in 2019 (w-2) 6.0
Balance as at 31 December 2019 326.0
Payments in 2020 (180+100) 280.0
Borrowing cost capitalized in 2019 (w-2) 19.0
625.0

W-2: Details of payment:


Funds from
Date Description Amounts Govt grant General
1-3-2019 1st bill 130 130 -
1-8-2019 2nd bill 180 70 120
1-1-2020 3rd bill 180 - 180

W-3: Borrowing cost to be capitalized:


2019:
120×12%×5*/12 =6 * August to December
Only loan from the bank A was available in 2019,so its rate is capitalization rate

2020:
120×12.67%×6*/12 =7.60 *January to June
180×12.67%×6*/12 =11.40 *January to June
To be capitalized in 2020 =19.00

Capitalization rate:
36 + 21
× 100 = 12.67%
300 + 150

W-4 Investment income from investment of govt grant :


200×8%×1*/12 =1.33 *February
70×8%×5*/12 =2.33 *March to July
= 3.66

Q.8 The accountant of Cereus Golf Club (CGC) was terminated on charges of fraud and you have been assigned the task of
preparing the accounts for the year ended 31 December 2020. You have found thatthe proper books had not been maintained.
The management of CGC has given you the following information:
(i) Cash and bank balances at 1 January 2020 amounted to Rs. 0.5 million and Rs. 2 million respectively. However,
as on 31 December 2020, there was no cash balance and Rs. 4.2 million in the bank.
(ii) The members are required to pay 3 years’ subscription in advance upon admission/renewal. Full year
subscription is charged from members joining during the year. Number of subscriptions received are as
under:

Page 11 of 15
315
Year No. of 3 years’
memberships subscriptionper
member
2018 100 Rs. 60,000
2019 140 Rs. 75,000
2020 160 Rs. 90,000

During 2020, 10 members were awarded membership on special permission but they had not paid the
subscription till year-end.
After year-end, 5 more members informed that they had paid the 3 years’ subscription amount in 2020. It was
found out that the amount was misappropriated by the accountant.
(iii) CGC had received a donation of Rs. 8 million in 2019 to meet the repair and maintenance expenditure of its
golf course. Out of total donation, the club has spent Rs. 2.2 million and Rs. 2.8 million in 2019 and 2020
respectively.
(iv) CGC started purchasing golf kits in 2020 for sales as well as for rent purposes. 20% of the purchases were
unpaid at year-end. Two third of the golf kit purchases made in 2020 had been added to inventory of golf kits for
sale and remaining had been added directly to golf kits forrent.
(v) Golf kits are sold for cash at cost plus 40%. Cost of closing inventory of golf kits for sale amounted to Rs. 1 million. It
was decided to transfer half of these kits into golf kits for rent at 30% of their original cost.
(vi) Some of the receipts and payments during the year were as follows:
Rupees
Rent of golf kits 650,000
Golf kits purchases 4,800,000
Annual insurances (paid till April 2021) 660,000
Salaries (including Rs. 350,000 for 2019) 2,800,000
Other expenses 2,320,000
(vii) CGC has a fidelity insurance policy and any cash deficiency upto a maximum of Rs. 2 million is
recoverable under the policy.
(viii) Fixed assets at 1 January 2020 had a book value of Rs. 25 million. All fixed assets are to bedepreciated at
15% per annum.

Required:
a. Prepare income and expenditure account for the year ended 31 December 2020. (11)
b. Prepare statement of financial position as on 31 December 2020. (09)

Page 12 of 15
316
A.8 Cereus Golf Club
(a) Income and expenditure account for the year ended 31 December 2020

Rs. in '000
Income:
Subscription income (W-1) 10,750
Rent on golf kits 650
Profit on sale of golf kit (4200-3000) 1200
12,600
Expenditures:
Insurance expense 660–220(660÷12×4) (440)
Salaries expense 2,800–350 (2,450)
Loss on golf kits transferred 500×0.7 (350)
Depreciation 3,750(25,000×15%) (3,750)
Other expenses (2,320)
Repair expenses for donation [2,800-2,800] -
Loss on misappropriation (450+4170-2000) (W-2) (2,620)
(11,930)
Excess of income over expenditure(surplus) 670

(b) Statement of financial position as on 31 December 2020


Rs. in '000
Non-current assets:
Fixed assets [25,000 – 3,750] 21,250
21,250
Current assets:
Golf kits 500
Insurance claim receivable 2,000
Golf kits on rent 2,150
Prepaid Insurance 220
Subscription in arrears (W-1) 300
Bank balance 4,200
9,370
30,620
General funds:
Opening balance (W-3) 12,350
Excess of income over expenditure(surplus) 670
13,020

Liabilities:
Repair and maintenance Donation in advance [8,000-2,200-2,800] 3,000
Creditors - golf kits 1,200

Page 13 of 15
317
Subscription in advance 13,400
30,620

Page 14 of 15
318
Accounting entry Golf kit for rent 150
Loss (I&E) 350
Golf kits for sale 500

Calculation of Sales: 3,000/100 x 140 = 4,200

Net profit on sales=4200-3000=120

(W-3): Opening general fund: Rs. in '000


Cash and bank balance 2,500
Fixed assets 25,000
Subscription in advance (9,000)
Accrued salaries (350)
Repair and maintenance donation fund (5,800)
12,350

Page 15 of 15
319
Spring 2022 [Far -1]
Q.1 Bulan Pakistan Limited (BPL) is planning to commence construction of a warehouse on1 January 2023 and
is expecting to complete it by 30 November 2023. The management wants to ascertain the borrowing costs
that can be included in the cost of warehouse. Relevant details in this respect are as follows:

(i) Expected payments related to the construction of the warehouse will be as follows:

Description Date of payment Rs. in million


st
1 bill of contractor 1-Feb-23 40
2nd bill of contractor 1-Apr-23 120
3rd bill of contractor 1-Sep-23 100
Last bill of contractor 1-Dec-23 90
350
(ii) The project can be financed through the following sources:
▪ Specific loan of Rs. 350 million at the rate of 16% per annum to be obtained on 1 January
2023. The principal will be payable in 5 equal annual instalments along with interest, from 1
January 2024.
▪ Withdrawals to be made from existing running finance facilities. These facilities will also be
used to finance other needs of BPL. Details of these facilities are as follows:

Limit Expected average


Name of bank balance for 2023 Interest rates
------ Rs. in million ------
Bank A 300 220 13.7%
Bank B 350 280 14.6%
(iii) The surplus funds available from the loan will be invested in a saving account at10% per
annum.
(iv) The construction work is expected to be suspended for the entire month of June 2023due to usual
monsoon rains.

Required:
Calculate the borrowing costs to be capitalised in the cost of warehouse in each of thefollowing
independent cases:
(a) if all the payments will be made from the specific loan only. (04)
(b) if all the payments will be made from running finance facilities only. (04)

Page 1 of 19
320
Q.2 Following information pertains to Dahl Limited (DL):

Summarised statement of financial position as at 31 December 2021


2021 2020 2021 2020
Rs. in million Rs. in million
Share capital 11.0 10.0 Property, plant and equipment 18.7 10.6
Retained earnings 32.9 33.8 Working capital other than cash 24.5 17.8
Revaluation surplus 4.0 - Cash 4.7 15.4
47.9 43.8 47.9 43.8

Additional information:
(i) Final dividend was paid in respect of year 2020 amounting to Rs. 3.4 million.
(ii) Additions to property, plant and equipment during the year amounted toRs. 14 million.
(iii) Tax expense for the year amounted to Rs. 2.4 million. Tax payable as at31 December
2021 amounted to Rs. 1 million (2020: Rs. 0.2 million)

Required:
Prepare DL’s statement of cash flows for the year ended 31 December 2021. (08)

Q.3 Following is the trial balance of Mahtab Welfare Hospital (MWH) as on 31 December 2021:

Debit Credit
---- Rs. in million ----
Capital work in progress – hospital building 335
Cash at bank 60
Closing inventory – medicines and supplies 14
Contributions received 281
General fund as at 1 January 2021 332
Medical equipment 320 100
Medicines and supplies used 76
Other expenditures 19
Payables 17
Research cost 33
Restricted fund as at 1 January 2021 180
Salaries 53
Total 910 910

Page 2 of 19
321
Additional information:
(i) The break-up of restricted fund balance is as follows:

Fund Description Rs. in million


Hospital building Contributions received for the construction of 120
fund hospital building.
As per the resolution of board of trustees,MWH is
Research fund required to allocate 20% of surplus 60
of each year to the research fund.

(ii) Contributions received include Rs. 55 million received for construction of hospital.
(iii) During the year, MWH also received construction materials having fair value ofRs. 65 million
for the hospital building which has not been recorded in books.
(iv) MWH has completed the construction of hospital building on 1 April 2021.
(v) Depreciation is to be charged as follows:

Hospital building 5% – straight line


Other fixed assets 10% – reducing balance

Required:
Prepare the following using deferral method:
(a) Statement of income and expenditure for the year ended 31 December 2021 (04)
(b) Statement of financial position as at 31 December 2021 (06)

Q.4 Both IAS 16 ‘Property, Plant and Equipment’ and IAS 40 ‘Investment Property’ deal with tangible non-
current assets of an entity. Discuss any four differences between IAS 16 and IAS 40. (06)

Q.5 The trial balance of Moon Mart (MM) did not agree as at 31 December 2021 and the shortage of Rs.
215,000 on the debit side was carried to suspense account. The financial statements prepared from the trial
balance showed net profit of Rs. 1,431,000.
During review, following matters were noted:

(i) A return outward of Rs. 18,000 was posted to the debit of return inward account ingeneral ledger.

(ii) A sales invoice of Rs. 42,000 was posted twice in sales ledger (means debtor account).
(iii) Balance of accumulated depreciation of equipment was brought forward a s Rs. 641,000
instead of Rs. 461,000 on 1 January 2021.
(iv) Following entries in cash book were not posted to general ledger:
a) Receipt of annual rent for the period ending 31 March 2022 amounting toRs. 336,000.

b) Payment of Rs. 220,000 for equipment purchased on 1 May 2021.


c) Cash purchases of Rs. 50,000.

Page 3 of 19
322
Additional information:

(i) After passing all the adjustments, the remaining amount of suspense account is to beconsidered as
loss from embezzlement.
(ii) MM uses periodic inventory method. Control accounts are not maintained for trade receivables and
payables. Equipment are depreciated at 15% using reducing balance method.

Required:
(a) Prepare suspense account. (04)
(b) Compute the corrected net profit. (04)

Q.6 Select the most appropriate answer(s) from the options available for each of the followingMultiple Choice
Questions.
(i) A plant has a carrying amount of Rs. 3.3 million as at 31 December 2021. Its fair value is Rs. 2.4 million
and costs of disposal are estimated at Rs. 0.1 million. Cash flows from the plant for the next 4 years
are estimated at Rs. 0.7 million per annum. It will be disposed of at the end of the 4th year for Rs. 0.6
million. Applicable discount rate is 10% per annum.

What is the approximate impairment loss on the plant to be recognized in the financialstatements for
the year ended 31 December 2021?

(a) Rs. 1 million (b) Rs. 2.6 million


(c) Rs. 0.7 million (d) Rs. 1.1 million (02)
(ii) The forgivable loan from government is accounted for as if there isno
reasonable assurance that the entity will meet the terms for forgiveness of loan.

(a) a liability (b) an income


(c) a government assistance (d) a government grant (01)

(iii) Which of the following statements is/are correct?


(I) Cash flows information cannot be manipulated easily, as compared to profit orloss because it is not
affected by different accounting policies.
(II) Cash flows information can be manipulated easily, as compared to profit or lossbecause it is affected by
different accounting estimates.
(a) Only (I) is correct (b) Only (II) is correct
(c) Both are correct (d) None is correct (01)
(iv) On 1 January 2019, a company purchased an asset for Rs. 5 million against which it received the government
grant of Rs. 0.5 million. The company deducted the grant from the cost of asset. It is the policy of the
company to depreciate such assets usingstraight line method over ten years. On 1 January 2021, the government
grant became repayable due to non-fulfilment of conditions. Repayment of grant will result in increasing:

Page 4 of 19
323
(a) carrying value by Rs. 0.5 million (b) carrying value by Rs. 0.4 million
(c) expense by Rs. 0.4 million (d) expense by Rs. 0.5 million (02)
(v) As per IAS 20 ‘Accounting for Government Grants and Disclosure of Government Assistance’, presenting the
whole grant as other income in the statement of comprehensive income or deducting it from a related expense,
is the correct treatment of:

(a) grant related to income


(b) forgivable loan expected to be received in next year
(c) government assistance in the form of free technical advice
(d) grant related to assets (01)
(vi) Which of the following statements is/are correct?

(I) The Conceptual Framework is not an IFRS and nothing in the ConceptualFramework overrides any
specific IFRS.
(II) One of the purpose of Conceptual Framework is to assist IASB to develop IFRSsthat are based on
consistent concepts.
(a) Only (I) is correct (b) Only (II) is correct
(c) Both are correct (d) None is correct (01)
(vii) Which of the following may be presented in both statement of comprehensive incomeand statement of cash
flows?

(a) Purchase of non-current assets (b) Issuance of shares


(c) Repayment of loan (d) Depreciation (01)
(viii) Which TWO of the following are internal sources of assessing whether there is anindication of
impairment?

(a) An expected decline in the asset’s market value


(b) An increase in interest rates
(c) Evidence that the asset is damaged
(d) Evidence that the entity’s performance is worse than expected (01)

Page 5 of 19
324
Q.7 Qamar Limited (QL) is in the business of consumer goods. Following are the summarized financial
statements of QL for 2021:

Statement of financial position as at 31 December 2021


Assets Rs. in million Equity and liabilities Rs. in million
Fixed assets 550 Share capital 600
Retained earnings 319
Current assets: Long-term loan 350
Inventory 440 Current liabilities:
Trade debtors 350 Trade creditors 150
Short term investment 160 Other payables 70
Cash and bank balances 39 Current maturity of loan 50
1,539 1,539

Statement of profit or loss for the year ended 31 December 2021


Rs. in million
Sales 2,150
Cost of goods sold (1,900)
Gross profit 250
Selling and administrative expenses (93)
Other income 40
Finance cost (35)
Net profit 162

Extracts from management reports submitted to the board of directors:

(i) Ratios for the year 2020:

Gross profit margin 9.5% Net profit margin 3.9%


Interest cover 2.4 times Inventory holding period 90.4 days
Return on non-current assets 16.8% Debtors turnover 7.3 times
Creditor payment period 55.1 days Acid test 0.9 times

(ii) Important financial and operating decisions taken during the year 2021:
▪ QL renewed a large contract with a customer. In the renewed contract, extendedcredit terms
were given to the customer.
▪ A major supplier agreed to reduce the prices by 10% on the condition of cash purchases
only. This reduction helped QL to avoid increase in prices of its products despite increase in
prices by competitors.
▪ Increasing working capital demands were met by making a share issue. A part ofthe proceeds
from the issue were also used to prepay a significant portion of thelong term loan.
▪ QL disposed of its main warehouse in the last month of the year at a gain of Rs. 25
million. The sale proceeds are temporarily invested in a short term investment.

Page 6 of 19
325
Required:
(a) Compute QL’s ratios for 2021 for comparison with 2020. (06)
(b) Keeping in view the financial and operating decisions extracted from management reports,
provide reasons for variation in the ratios computed in (a) above. (09)

Q.8 Chand Limited (CL) was incorporated on 1 January 2020 with an authorized share capital ofRs. 500 million
comprising of 50 million shares.
(i) Details of shares issued are as follows:
▪ On 1 March 2020, CL issued 20 million shares at Rs. 18 each.
▪ On 1 October 2020, CL issued 15% bonus shares. The market price per share immediately
before the announcement of bonus was Rs. 24 per share.
▪ On 1 September 2021, CL issued 40% right shares at a premium of Rs. 12.5 per
share. The market price per share immediately before the entitlement date was Rs. 33 per
share.
(ii) Following information has been extracted from CL’s draft financial statements:

2021 2020
Draft Audited
--- Rs. in million ---
Net profit 66 48
Revaluation surplus arising during the year - 20
Transfer of incremental depreciation 4 -

Final cash dividend - 10%


(iii) After the preparation of draft financial statements for the year ended 31 December
2021, it was discovered that installation cost of Rs. 12 million relating to a plant capitalized on 1
August 2020 was wrongly expensed out. The plant is subsequently measured using cost model and
is being depreciated @ 20% per annum on reducing balance method.
Required:
(a) Prepare CL’s statement of changes in equity for the year ended 31 December 2021
along with comparative figures. (Column for total is not required) (09)
(b) Compute CL’s basic and diluted earnings per share to be disclosed in the statement of
profit or loss for the years ended 31 December 2021 and 2020. (08)

Page 7 of 19
326
Q.9 Following information pertains to property, plant and equipment of Tsuki Limited (TL):

Office building Warehouse


Acquisition:
▪ Date of acquisition 1 July 2017 1 July 2018
▪ Cost (Rs. in million) 96 156
▪ Estimated useful life (in years) 16 12

Revalued amount:
▪ 1 January 2019 (Rs. in million) 116 138
▪ 1 January 2021 (Rs. in million) 80 143

Revised useful life on 1 January 2020 (in years) 9 14

Additional information:
(i) TL uses revaluation model for subsequent measurement and accounts for revaluationon net
replacement value method.
(ii) TL transfers maximum possible amount from the revaluation surplus to retainedearnings on
an annual basis.
(iii) The revalued amounts were determined by Sagheer Valuers (Private) Limited, anindependent
valuation company.
Required:
In accordance with IFRSs, prepare a note on ‘Property, plant and equipment’ (includingcomparative
information) for inclusion in TL’s financial statements for the year ended
31 December 2021. (Column for total is not required) (18)

A-1 Borrowing cost to be Capitalized


a) Specific borrowings:

Interest incurred on outstanding balance of loan:


350 x 16% x 10/12 [Feb to Nov] 46.67
Less: Investment Income:
[350 - 40] 310 x 10% x 2/12 [Feb, March] 5.17
7.92
[310 - 120] 190 x 10% x 5/12 [ April to August]
2.25
[190 - 100] 90 x 10% x 3/12 [Sep. to Nov.]

(15.34)

Borrowing cost to be Capitalized 31.33

b) General borrowings:

Expenditures x Capitalization Rate x Time period.

Page 8 of 19
327
40 x 14.2% x 10/12 [Feb to Nov] 4.73

120 x 14.2% x 8/12 [ April to Nov.] 11.36

100 x 14.2% x 3/12 [Sep. to Nov.] 3.55

Borrowing cost to be Capitalized 19.64

Workings:
W.1) capitalization rate:
[220 x 13.7% + 280 x 14.6%] / [220 + 80] x 100 = 14.2%

Page 9 of 19
328
A. 2

Dahl Limited
Statement of cash Flow
For the year ended 31 December 2021(Indirect Method)

Rs. in million

Cash flow from operating activities

Profit before tax ( W-7) 4.9


Adjustments for:

Depreciation charge (W-4) 9.9

Operating profit before working capital changes 14.8

Changes in working capital:

Working capital other than cash ( W-5 ) (7.5)

Cash generated from operations 7.3

Tax paid ( W-6 ) (1.6)

Net cash flow from operating activities 5.7

Cash flow from investing activities:

Purchase of PPE (14)

Net cash used in investing activities (14)

Net cash flow from financing activities:

Issuance of shares 1

Dividend paid (3.4)

Net cash from financing activities (2.4)

Net in cash and cash equivalent (10.7)

Cash and cash equivalent at beginning of year 15.4

Cash and cash equivalent at the end of year 4.7

Workings:
Cash and Cash Equivalent: 2021 2020
Cash 4.7 15.4
Total 4.7 15.4

Page 10 of 19
329
(W-1) Share capital (W-2) Retained earnings
b/d 10 Dividend 3.4 b/d 33.8

cash(bal) 1 PAT 2.5

c/d 11 c/d 32.9

(W-3) Rev. Surplus (W-4) PPE


b/d - b/d 10.6
Dep.(Bal) 9.9
PPE 4 cash 14

rev. surplus 4

c/d 4 (W-5) c/d 18.7


Workin
b/d 18
g b/d 0.2
(17.8 + 0.2)
Capital (W-6) Tax payable
Cash(bal) 1.6 Expense 2.4
Bal 7.5

c/d 25.5
(24.5 + 1)
c/d 1
(W – 7)
PAT = 2.5
Tax = 2.4
PBT 4.9

Page 11 of 19
330
A.3 Mahtab Welfare Hospital
(a) Statement of income and expenditure for the year ended 31 December 2021
Rs. in million
Income:
Contribution 281–55 226
Hospital contributions 240(120+55+65) ×5%×9/12 9
235
Expenditures:
Medicines used 76
Salaries 53
Other expense 19
Research cost 33-33 0
Depreciation – Hospital 400(335+65) ×5%×9/12 15
Depreciation – Other fixed assets (320–100) ×10% 22
(185)
Excess of income over expenditure 50
Mahtab Welfare Hospital
Statement of financial position as at 31 December 2021
Rs. in million
Non-current assets:
Hospital building-CWIP 335+65–15 385
Medical equipment 320–100–22 198
583
Current assets:
Medicines and supplies 14
Cash at bank 60
74
657
Funds:
General fund 332+50–10(50×20%) 372
Non-current liabilities:
Hospital deferred contribution 219
(120+55+65 =240-9-12(240x5%) current liability
Research fund 60–33+10 37
Current liabilities:
Payables 17
Hospital deferred contribution 12
657
Workings
w-1
Contributions 55
Hospital fund 55

w-2
Construction material 65
Hospital building Fund 65

Page 12 of 19
331
w-3
CWIP-Hospital building 65
Construction material 65

A.4
IAS 16 IAS 40
(i) Applicable to tangible assets held for use in Applicable to property held to earn rentalor for
business including owner occupied capital appreciation
property
(ii) Allows cost or revaluation model for Allows cost or fair value model for
subsequent remeasurement subsequent remeasurement
(iii) Change in fair value are taken to OCI Change in fair value are taken to P&L
and/or P&L only
(iv) Assets under revaluation model are Assets under fair value model are not
depreciated depreciated

A.6 (i) (c) Rs. 0.7 million (W-1)


(ii) (a) a liability
(iii) (a) Only (I) is correct
(iv) (b) the carrying value by Rs. 0.4 million (W-2)
(v) (a) Grant related to income
(vi) (c) Both are correct
(vii) (d) Depreciation
(viii) (c) Evidence that the asset is (d) Evidence that the entity’s performance is
damaged worse than expected

(W-1) :
C.A= 3.3
Fair value less cost to sell = 2.3
(2.4 – 0.1)
Value in use:
0.7[1 − (1 + 0.1) − 4/0.1) ] + 0.6(1 + 0.1)-4 = 2.63
Recoverable amount i.e. 263
Loss:
= 3.3 – 2.63
=0.7

Page 13 of 19
332
(W-2):
1.1.2019 Asset 5.0
Cash 5.0
1.1.209 Cash 0.5
Asset 0.5
31.12.2019 Depreciation 0.45
Acc.depreciation (5 – 0.5) ÷10 0.45
31.12.2020 Depreciation 0.45
Acc.depreciation 0.45
1.1.2021 Asset 0.5
Cash 0.5
31.12.2021 Depreciation 0.1
Acc.depreciation (0.5/10 x 2) 0.1

A.7 Qamar Limited


Ratio 2021 Reason
Gross profit margin 11.6% 2020: 9.5%
(250 ÷ 2150) ×100 Reduction in cost due to discount bymajor
supplier

Net profit margin 7.5% 2020: 3.9%


(162 ÷ 2150) ×100 ▪ Increase in gross profit margin / Reduction
in cost due to discount by major supplier
▪ Increase in sales volume achieved by not
increasing prices.
▪ Profit from disposal of warehouse
▪ Reduced interest expense due to
repayment of loan

Interest cover 5.6 times 2020: 2.4 times


[(162+35)÷35] ▪ Reduction in finance cost due to
repayment of loan
▪ Increase in overall profitability
Inventory holding 84.2 days 2020: 90.4 days
period (440÷1900)×365 Due to increase in sales volume achieved by
not increasing prices.

Return on non- 35.8% 2020: 16.8%


current assets (197÷550) × 100 ▪ Increase in overall profitability
▪ Disposal of main warehouse
(162 + 35)

Page 14 of 19
333
Debtors turnover 6.1 times 2020: 7.3 times
(2150÷350) Due to extended credit period offered to new
customer

Creditor payment 28.8 days 2020: 55.1 days


period (150÷1900)×365 Cash purchases from a major supplier to avail
discount

Acid test ratio 2.03 2020: 0.9 times


(549÷270) ▪ Due to funds generated from sale of
warehouse placed in short term
investments and share issue.
[440 + 350 + 160 + 39 – ▪ Decrease in payables due to cash
440] ÷ [150 + 70 + 50] purchases

Answer 8 (a):
Chand limited
Statement of changes in equity
For the year ended 31 December 2021
Rs. in millions
Share capital Share Retained earning Revaluation
premium Surplus
Balance as on 1.1.2020 - - - -
Issuance of shares 200 160 - -
(20 x 10) (20 x 8)
Bonus shares 30 (30)
(200 x 15%) = 30
Net profit-restated (W-1) (48 + 11) 59
Revaluation surplus 20
Balance as at 31-12-2020 -restated 230 130 59 20
Final dividend-2020 (230 x 10%) (23)
Right shares (W-2) 92 115
Net profit (W-1) (66 – 2.2) 63.8
Transfer Incremental depreciation 4 (4)
Balance as at 31-12-2021 322 245 103.8 16

Workings:
W-1: Correction of error:
For the year ended 31-12-2020
Plant 12
Expense 12

Depreciation 1
Accumulated Depreciation 1
(12 x 20% x 5/12) [Aug to Dec-2020]
Net effect on profit =12 - 1
=11 (Profit will increase)

Page 15 of 19
334
For the year ended 31-12-2021:
Depreciation 2.2
Accumulated depreciation 2.2
(12 – 1 = 11 x 0.2)
Net effect on Profit (Profit will decrease by 2.2)

W-2: Right Shares: 230/10


= 23
23 x 0.4 = 9.2

X 10 x 12.5
= 92 = 115 Cash 207
Share capital 92
Share premium 115
A.8 Part (b)
Calculation of Basic and Diluted EPS:
For 2020 [Originally reported]:
Basic and diluted : 48/19.17(W-1) = 2.50 per share
For 2021 :
2021
Basic and diluted 63.8/27.6 (W-2) = 2.31 per share
2020 [Restated]
Basic and Diluted 59/21.09 (W-2) = 2.80 per share
Workings:
1.Weighted average number of shares in 2020
Actual 2020 Weighted 2020
1-3 Issue for value 20 16.67
(20 x 10/12)
20 16.67
1-10 Issue for no value 3 2.5
(3/20 x 16.67)
31.12.2020 23 19.17

2. .Weighted average number of shares in 2021


Actual 2021 Current 2021 Prior 2020
b/d 23 23 19.17 (W-1)
1-9 Issue for value (W) 6.27 2.09 -
(6.27 x 4/12)
29.27 25.09 19.17
1-9 Issue for no value (W) 2.93 2.51 1.92
(2.93/29.27 x 25.09) (2.93/29.27 x 19.17)
31.12.2021 32.2 27.6 21.09

Working :
23 x 40% = 9.2 x 22.5/33
Issue for value = 6.27
Issue for no value [9.2 – 6.27] = 2.93

Page 16 of 19
335
A.8 Tuski Limited
Notes to the financial statement for the year ended 31 December 2021
1. Property, plant and equipment
2021 2020
Building Warehouse Building Warehouse
------------------------- Rs. in million ----------------------
Gross carrying amount:
Opening 116.00 138.00 116.00 138.00
Elimination (20) (21) -
-
Revaluation (16) 26 - -
Closing balance 80 143 116 138
Accumulated depreciation
Opening balance 20 21 8 12
Elimination (20) (21) - -

For the year 10.00 11.00 12.00 9.00


(80÷8) (143÷13) (108÷9) (126÷14)
Closing balance 10 11 20 21

Carrying amount 70 132 96 117

1.1

Building Warehouse
Measurement base Revaluation Revaluation
Useful life 9 years 14 years
Depreciation method Straight line Straight line

1.2 The last revaluation was performed on 1 January 2021 by Sagheer valuation services, anindependent
firm of valuers.
1.3 Had revaluations not been made, the carrying value of the buildings and warehouse as on31 December
2021 would have been Rs. 63 million and Rs. 117 million respectively.

Carrying value if cost model is used : Building Warehouse


----------- Rs. in million -----------
Book value on 31.12. 2020 (working) 72 126.75

Book value on 31 December 2021 (working) 63.00 117


(81×7÷9) (136.5×12÷14)

Disclosure of revaluation surplus (Office building):


2021 2020
Opening Bal. 24 27 (29 – 2)
Revaluation (16) -
Transfer of surplus (1) (3)
Closing Bal. 7 24
Disclosure of revaluation surplus (Warehouse):
2021 2020
Opening Bal. - -
Revaluation 16.25 -

Page 17 of 19
336
Transfer of surplus (1.25) -
Closing Bal. 15 -

Accounting entries:
Office building Dr. Cr.
1.7.2017 Office building 96
Cash 96
31.12.2017 Depreciation 3
Acc. depreciation (96/16 x 6/12) 3
31.12.2018 Depreciation 6
Acc. depreciation (96/16) 6
1.1.2019 Acc. depreciation 9
Office building (3 + 9) 9
1.1.2019 Office building 29
Rev. surplus 29
C.A (96 – 9) = 87
F.V = 116
Rev. Surplus = 29

31.12.2019 Depreciation 8
Acc. depreciation (116/14.5) 8
31.12.2019 Rev. surplus 2
Ret. Earnings (29/14.5) 2
1.1.2020 Revised useful life is 9 years.
31.12.2020 Depreciation 12
Acc. depreciation [(116 – 8)/9)] 12
31.12.2020 Rev. surplus 3
Ret. Earnings [(29 – 2)/9] 3
1.1.2021 Acc. depreciation 20
Office building (8 + 12) 20
1.1.2021 Rev. surplus 16
Office building 16
C.A (116 - 20) = 96
F.V = 80
Rev. Loss = 16
Remaining surplus before rev.loss was ( 29 – 5 = 24 ) and after rev.loss is 8 ( 24 –
16 ).
31.12.2021 Depreciation 10
Acc. depreciation (80/8) 10
31.12.2021 Rev. surplus 1
Ret. Earnings (8/8) 1

Ware house Dr. Cr.


1.7.2018 Ware house 156
Cash 156
31.12.2018 Depreciation 6.5
Acc. depreciation (156/12x 6/12) 6.5

1.1.2019 Acc. depreciation 6.5


Ware house 6.5
1.1.2019 Rev. Loss (P.L) 11.5

Page 18 of 19
337
Ware house 11.5
C.A (156 – 6.5) = 149.5
F.V = 138
Rev. loss =11.5
31.12.2019 Depreciation 12
Acc. depreciation (138/11.5) 12
1.1.2020 Revised life is 14 years
31.12.2020 Depreciation 9
Acc. depreciation (138 - 12)/14 9
1.1.2021 Acc. depreciation 21
Ware house (12 +9) 21
1.12021 Ware house 26
Reversal of loss (P.L) 9.75
Rev. surplus 16.25

C.A (138 – 12 – 9) 117


F.V 143
Rev. surplus 26

Reversal of loss : 11.5 Cr Rev. surplus


(26-9.75) = 16.25
Extra dep :
2019: (13-12) = 1 Dr
2020:(156-6.5 - 13) ÷ 14
= 9.75-9 =0.75 Dr
Reversal of loss =9.75 Cr
31.12.2021 Depreciation 11
Acc. depreciation (143/13) 11
31.12.2021 Rev. surplus 1.25
Ret. Earnings (16.25/13) 1.25

Carrying amount if cost model is used (Office Building):


1.7.2017: 96
1.12.2017: 96 ÷ 16 x 6/12 = 3 ; (96 – 3 = 93)
31.12.2018: 96 ÷ 16 =6 ; (93 – 6 = 87)
31.12.2019: 96 ÷ 16 = 6 ; ( 87 – 6 = 81)
1.1.2019: Remaining life is 9
31.12.2020: 81 ÷ 9 = 9 ; (81-9) = 72
31.12.2021: 81 ÷ 9 = 9 ; (72 – 9) = 63

Carrying amount if cost model is used (Warehouse):


1.7.2018: 156
31.12.2018: 156 ÷ 12 x 6/12 = 6.5 ; (156 – 6.5 = 149.5)
31.12.2019: 156 ÷ 12 = 13; (149.5 – 13 = 136.5)
1.1.2020: Remaining life is 14
31.12.2020: 136.5 ÷ 14 = 9.75 ; (136.5 – 9.75 = 126.75)
31.12.2021: 136.5 ÷ 14 = 9.75 ; (126.75 – 9.75 = 117)

Page 19 of 19
338
Section A

Q.1 Consider the following statements with reference to ‘Conceptual framework for financial
reporting’:

(i) Physical capital maintenance measures profit in terms of increase in the productivecapacity of an
entity.
(ii) In times of rising prices, profits will be overstated and assets will be understated when financial
statements are prepared on the basis of historical cost.
(iii) Income represents all increases in assets or decreases in liabilities that result in increasein equity.
(iv) To be a perfectly faithful representation, a depiction would have three characteristics. It would be
complete, relevant and verifiable.
(v) In value in use method, assets are measured at the amount that would be paid topurchase the
same or a similar asset currently.
(vi) Current cost and fair value are exit values.
(vii) Requirements of a standard overrides the requirements of conceptual framework.
(viii) Financial capital maintenance is likely to be the most relevant to investors as they areinterested in
maximizing the return on their investment and purchasing power.

Required:
Identify whether each of the above statements is TRUE or FALSE. Give reasons for statements
identified as FALSE.

(07)

Q.2 Discuss how the following should be dealt with in the current year’s financial statements of
relevant entities in accordance with IAS 20.

(a) Xero Limited (XL) received a government grant to setup a plant in an under-developedrural
area three years ago. One of the conditions of the grant was that XL will maintain a
minimum of 200 employees during the next five years. However, due to worsening
economic conditions, XL failed to maintain 200 employees and the full grant became
repayable immediately in the current year.

XL has been presenting the grant in statement of financial position by deducting the
grant in arriving at the carrying value of the plant.
(04)

(b) One Limited received a loan from government in the current year at an interest rate of 5%
per annum. The prevailing market interest rate is 12% per annum. The only condition
attached to the loan is that it should be used for acquisition of textile
machinery.

(03)

Page 1 of 22
339
Q.3 Oracle Family Club (OFC) was formed in January 2021. The following information is
available in respect of the first year of operations:

Receipt and payment account for the year ended 31 December 2021
Receipts Rs. in '000 Payments Rs. in '000
Subscriptions for: Salaries 640
▪ 2021 2,800 Rent 990
▪ 2022 1,360 Equipment 2,560
Joining fees 2,100 10% Fixed deposit 2,020
Canteen sales 720 Construction of building 1,500
Life-time memberships 1,840 Canteen purchases 700
Closing balance 410
8,820 8,820

Income and expenditure account for the year ended 31 December 2021
Expenditures Rs. in '000 Incomes Rs. in '000
Salaries 700 Subscription 3,450
Rent 760 Interest on fixed deposit 150
Depreciation of equipment 200 Life-time memberships 360
Surplus 2,330 Profit from canteen 30
3,990 3,990

Additional information:
(i) OFC also operates a canteen. All sales and purchases of canteen are made for cash.
(ii) Salary of canteen’s salesman amounted to Rs. 90,000 is included in payments.

Required:
Prepare OFC’s statement of financial position as on 31 December 2021.

(10)

Q.4 During the review of accounting records and financial statements for the year ended30
June 2022 of Tally Traders, following errors were highlighted:

(i) Sales included an outstanding balance of Rs. 500,000 for which a customer would needto
pay Rs. 485,000 only if payment is made within 30 days. The customer is expected to pay
within 30 days.
(ii) An item was included in closing inventory at its net realizable value of Rs. 490,000.
However, the item had a cost of Rs. 450,000.
Periodic inventory method is used to record the inventory transactions.
(iii) A sub-total of Rs. 234,000 was carried forward in the purchase account as Rs. 432,000.
(iv) A credit note issued to a customer of Rs. 128,000 was recorded as credit note received
from supplier.
(v) An office machine costing Rs. 3,540,000 with a carrying value of Rs. 2,040,000 as on1 July
2021 was disposed of on 28 February 2022 for Rs. 1,860,000. The sale proceeds were
credited to accumulated depreciation account and full year’s depreciation was provided
on the machine.

Page 3 of 22
340
Office machines are depreciated at 10% per annum using reducing balance method.

Required:
Prepare journal entries to correct the above errors. (Narrations are not required)

(08)

Q.5 On 1 March 2017, Zarmoney Limited imported an automatic plant for Rs. 130 million. The
commissioning of the plant was completed on 1 January 2018 at a cost of Rs. 10 million. The
economic life of the plant was estimated as 12 years and useful life of the plant was estimated as 8
years. The plant is being depreciated at 20% per annum using reducing balance method.

Due to declining demand for the product manufactured from this plant, an impairment test was
carried out at 31 December 2021. Following information has been gathered for impairment
testing of the plant:
(i) The current selling price of a similar plant in the local market is Rs. 50 million. The
present decommissioning cost of the plant is estimated at Rs. 2 million.
(ii) The plant’s net disposal proceeds at the end of the useful life is estimated at
Rs. 4 million.
(iii) The current market risk-free rate of interest is 8% per annum, however, an investor
would ask additional return of 2% for bearing the uncertainty inherent in such a plant.
(iv) A junior accountant has calculated following net cash flows from operating the plant:

Year 2022 2023 2024 2025


Net cash inflow (Rs. in million) 11 7 3 1

However, a review of accountant’s working has revealed the following:


▪ Depreciation of the plant has been included as an outflow in each year.
▪ Tax payments of Rs. 2 million has been included as an outflow in each year.
▪ Inflows from plant in 2022 include receipts from sale of existing inventory
amounting to Rs. 3 million.

Required:
Compute the impairment loss (if any) in the value of the plant to be recognised on
31 December 2021. (Show all necessary workings)

(08)

Q.6 Select the most appropriate answer(s) from the options available for each of the following Multiple
Choice Questions.
(i) Which of the following is NOT a limitation of ratio analysis?

(a) Use of different accounting policies and estimates


(b) Use of different formulas for calculating ratios
(c) Different inflation rates in different years
(d) Companies pursuing different strategies
(01)

Page 4 of 22
341
(ii) If the existing current ratio of a company is more than 1, what would be the impact ofa
credit purchase of inventory on the current ratio?

(a) Current ratio would increase


(b) Current ratio would decrease
(c) Current ratio would decrease but would remain higher than 1
(d) Current ratio would remain same
(01)

(iii) Which of the following statements are correct?

(I) Giving incentives to customer to pay on time would result in decrease in debtor’s
turnover in times.
(II) If all debtors pay their debts within the credit period, the average collection period
would be Nil.

(a) Only (I) is correct (b) Only (II) is correct


(c) Both are correct (d) None is correct (01)

(iv) Which TWO of the following would improve gearing ratio of a company?

(a) Issuance of shares at discount


(b) Repayment of a bank loan
(c) Issuance of bonus shares
(d) Disposal of a land at its carrying value (01)

(v) Which of the following changes would be considered as change in accounting policy?

(I) Changing the subsequent measurement model for property, plant and equipmentfrom cost
model to revaluation model.
(II) Changing the inventory valuation method from FIFO to Weighted average.

(a) Only (I) is change in accounting policy


(b) Only (II) is change in accounting policy
(c) Both are change in accounting policy
(d) None is change in accounting policy (01)

(vi) On 1 January 2021, a company borrowed Rs. 20 million @ 9% per annum for the purpose of
constructing an asset. The company started construction on 1 February 2021and paid Rs. 8 million on 1
March 2021 and Rs. 12 million on 1 July 2021. The assetwas ready to use on 1 September 2021. Surplus
funds were invested @ 6% per annum.

The borrowing cost that can be capitalized is:

(a) Rs. 660,000 (b) Rs. 710,000


(c) Rs. 900,000 (d) Rs. 1,050,000 (02)

Page 5 of 22
342
(vii) Which of the following statements are correct?

(I) Investment income on the temporary investment of unused funds of generalborrowings is


taken to profit or loss.
(II) Capitalisation of borrowing cost always commences as soon as construction of aqualifying asset
begins.

(a) Only (I) is correct (b) Only (II) is correct


(c) Both are correct (d) None is correct (01)

(viii) Which TWO of the following would be shown as a deduction from the column ofretained
earnings in statement of changes in equity?

(a) Transfer of incremental depreciation


(b) Issuance of shares at discount
(c) Cash dividend
(d) Transfer to general reserves (01)

(ix) Which TWO of the following situations would require prior year adjustment as perIAS 8?

(a) Changing the depreciation method from straight line basis to the reducing balancebasis in respect
of a building held for the last 10 years.
(b) Changing the measurement model for Investment property from cost model tofair value
model.
(c) Incorporating the effects of a material understatement found in last year closinginventories due
to incorrect formula in excel sheet.
(d) Adopting the requirements of IAS 20 for a government grant received by an entityfor the first
time. (01)

Page 6 of 22
343
Section B

Q.7 Following is the statement of financial position of Quicken Limited (QL) as at 30 June 2022:

2022 2021 2022 2021


Rs. in million Rs. in million
Share capital 480 400 Land and building 748 526
Revaluation surplus 135 - Vehicles 118 96
Retained earnings 337 325 Inventories 365 444
Long-term loan 335 460 Trade and other receivables 212 185
Trade and other payables 160 142 Cash and bank balances 73 111
Advance from customers 69 35
1,516 1,362 1,516 1,362

Additional information:
(i) During the year, land and building were revalued for the first time, resulting in a surplusof
Rs. 150 million and incremental depreciation of Rs. 15 million.
(ii) Depreciation on building charged to profit or loss amounted to Rs. 72 million.
(iii) During the year, vehicles having book value of Rs. 8 million were sold for Rs.
11 million received in cash. Further, sale proceeds of Rs. 6 million of another vehicle
(book value Rs. 7 million) disposed of in May 2021 were received inAugust 2021.
(iv) Vehicles costing Rs. 51 million were purchased during the year of which Rs. 12 million is
still unpaid.
(v) Interest on loan for the year amounted to Rs. 48 million of which Rs. 14 million was
capitalised in the cost of a building constructed during the year.
(vi) Following dividends were announced for the year ended 30 June 2022 and 2021:

2022 20% interim bonus shares and 15% final cash dividend
2021 5% interim bonus shares and 10% final cash dividend

Required:
Prepare QL’s statement of cash flows for the year ended 30 June 2022. (15)

Page 7 of 22
344
Q.8 Peach Tree Limited (PTL) was incorporated on 1 July 2020. Following information has been
extracted from its financial statements for the year ended 30 June 2022:

2022 2021
---- Rs. in million ----
Net profit 250 210
Revaluation surplus arising during the year 30 50
Total comprehensive income 280 260

Details of shares and bonds issued by PTL since incorporation are as follows:
(i) On 1 July 2020, 50 million ordinary shares having par value of Rs. 10 each were issued at
Rs. 14 each.
(ii) On 1 July 2020, 10 million 12% redeemable preference shares having par value of Rs.
50 each were issued at Rs. 64 each. Each preference share is convertible into3
ordinary shares after 5 years.
(iii) On 1 February 2021, further 20 million ordinary shares having par value of Rs. 10 each were
issued at prevailing market price of Rs. 16 each.
(iv) On 1 October 2021, 40% right shares were issued at a premium of Rs. 10 per share. The
market price per share immediately before the entitlement date was Rs. 30 per share.
(v) On 1 November 2021, 3 million convertible bonds having par value of Rs. 100 eachwere
issued. The bonds carry interest @ 10% per annum payable on 31 October eachyear.
Each bond is convertible into 7 ordinary shares after 3 years.

Required:
Compute basic and diluted earnings per share to be disclosed in PTL’s financial statements for the years
ended 30 June 2021 and 2022. (Show comparative figures) (15)

Q.9 Following information pertains to non-current assets of GnuCash Limited (GL):

(i) GL purchased a manufacturing plant for Rs. 340 million on 1 January 2021. On thatdate, the plant
had an estimated useful life and residual value of 13 years andRs. 60 million respectively. The
revalued amounts and residual value were as follows:

Revalued amount Residual value


----------- Rs. in million -----------
30 June 2021 304 54
30 June 2022 315 44

(ii) A warehouse owned by GL was given on rent on 1 January 2022. Previously, thewarehouse was
in use of GL.

The warehouse was acquired by GL on 1 July 2019 at a cost of Rs. 200 million and isbeing depreciated
@ 10% per annum on reducing balance method.

Page 8 of 22
345
Fair value of the warehouse on various dates are as follows:

Rs. in million
1 January 2022 206
30 June 2022 214

Rentals earned for the year ended 30 June 2022 amounted to Rs. 10 million out ofwhich Rs. 6
million is still outstanding.

(iii) GL acquired a property comprising of three similar showrooms at a total cost of Rs. 900 million on
1 October 2021. 40% of the cost of property is attributable to thevalue of land. Each of the showroom
can be leased out separately and has a useful life of 15 years with no residual value.

GL is using one showroom for its own products while the other showrooms were held to be leased out.
On 1 March 2022, the two showrooms were given on monthly rent of Rs. 4 million.

The fair value of each showroom is increasing by Rs. 3 million each month.

Other information:
Cost model is used for subsequent measurement of all property, plant and equipment
except for manufacturing plant for which revaluation model is used.
Maximum possible amount is transferred from the revaluation surplus to retainedearnings on an
annual basis.
Fair value model is used for subsequent measurement of all investment properties.

Required:
Prepare notes on ‘Property, Plant and Equipment’ and ‘Investment Property’, for inclusionin GL’s financial
statements for the year ended 30 June 2022.
(Comparative figures and column for total are not required) (20)

Page 9 of 22
346
Suggested Solution
Answer # 1

(i) True
(ii) True
(iii) False
Income does not include those increase in equity which are relating to contributions from holders of
equity claims.
(iv) False
The three characteristics are complete, neutral and free from error.
(v) False
Value in use is the present value of the cash flows, or other economic benefits that anentity expects
to derive from the use of an asset and from its ultimate disposal.
(vi) False
Current cost is an entry value while fair value is an exit value.
(vii) True
(viii) True

Answer # 2
a)
• When a government grant becomes repayable it is accounted for as a change in accounting estimate.
• As the grant was presented as deduction from related plant, its repayment would
be recognized by increasing the carrying value of the plant
• The cumulative additional depreciation that would have been recognized in profit or loss to date.
• Also, the circumstances giving rise to repayment of the grant might indicate the
possible impairment of the new carrying amount of the plant.
b)
• The benefit of the government loan at a below market rate of interest is treated as a government grant.
The loan shall be recognised and measured as per IFRS 9.
• Government grant should be recorded as the difference between the initial carryingamount of the loan
and the proceeds received.
• As the primary condition for the loan is acquisition of textile machinery, the grantshould be considered as
grant related to asset and should be recognized in profit or loss over the life of the machinery.
• The grant may be presented in the statement of financial position by setting up thegrant as deferred
income or by deducting the grant in arriving at the carrying value of the machinery

Page 10 of 22
347
Answer # 3:
Oracle Family Club
Statement of Financial Position
As on 31-Dec-2021
Rs. in '000
Non-Current Assets:
Equipment (2,560–200) 2,360
Capital work in progress - Building 1,500
3,860
Current Assets:
Canteen inventory 100
Subscription in arrears 650
Prepaid rent 230
Fixed Deposit 2,020
Interest receivable 150
Cash in hand 410
3,560
Total Assets 7,420
Equity and Liabilities
Opening Fund -
Surplus 2330
Joining Fee 2100
Life membership 1480
Liabilities:
Salaries payable ((700+90)–640) 150
Subscription in advance 1,360
7,420

Page 11 of 22
348
Working:

(W-1) Equipment (W-2) Salaries Payable (W-3) Prepaid Rent


b/d - Cash 640 b/d - b/d -
Cash 2560 Exp. 790
Dep. 200 (700 + 90) Cash 990 Exp. 760
c/d 150
c/d 2360 c/d 230

(W-4) Subscription A/c


b/d - b/d -
(W-5) Life Membership (W-6) Canteen Stock Cash 4160
b/d - b/d - (2800 +1360)
Income 360 COS 600
Cash 1840 Purchases700 c/d1360* c/d 680

c/d 1480 c/d 100


* 1360 related to 2022

(W-7)
Sales 720
Cost of sales
Opening -
Purchases 700
Closing stock(bal.) (100)
600
G. P 120
Salary (90)
Net profit 30

Page 12 of 22
349
Answer # 4
Tally Traders
Correcting Entries For the year ended 30-06-2022
Debit Credit
S. # Description
----- Rs. in '000 -----
(i) Sales 15
Receivables 15
(ii) Cost of goods sold (490–450) 40
Closing inventory 40
(iii) Suspense (W-1) (432–234) 198
Purchases 198
(iv) Sales return (W-2) 128
Purchase return 128
Accounts Payables 128
Account Receivables 128
(v) Depreciation Expense (W-3) 118
Accumulated depreciation 118
Accumulated Depreciation (3,540–2,040) +136 1636
Loss on disposal (2,040–136)–1,860 44
Sale proceeds 1860
Machine 3,540

W-1:
Entry in the books:
1- Purchase 432
Cash 432

Entry should be:


2- Purchase 234
Cash 234

Correcting Entry
3- Suspense Account 198
Purchase 198

W-2:
Entry in the books
1- Payable 128
Purchase return 128
Entry should be:
2- Sale return 128
Receivable 128

Page 13 of 22
350
Correcting Entry
3- Sale return 128
Purchase return 128
Payable a/c 128
Receivable a/c 128

W-3: Computation Rs. in '000


Correct depreciation 2,040×10%×8÷12 136
Depreciation charged (2,040–1,860) ×10% (18)
118

Answer # 5
Zarmoney Limited
Computation of impairment of plant
As on 31-12-2021
Carrying amount [140(130+10) x 0.8 x 0.8 x 0.8 x 0.8] 57.3
Recoverable amount (W-2) 52.6
Impairment loss (4.7)
Working:
W-1
Recoverable amount:
Higher of:
Fair value less cost to sell (50-2) 48
Value in use (W-3) 52.6
52.6

W-2: Value in use 2022 2023 2024 2025


--------------------- Rs. in million ---------------------
Net cash inflow 11.0 7.0 3.0 1.0
Depreciation 11.5 9.2 7.4 5.9
Income tax 2.0 2.0 2.0 2.0
Sale of inventory (3.0) - - -
Disposal value - - - 4.0
Cash flows undiscounted 21.5 18.2 12.4 12.9
Discount factor @ 10% (8+2) 0.909 0.826 0.751 0.683
PV of cash flows /value in use 19.5 15.0 9.3 8.8 52.6

Page 14 of 22
351
Answer # 6

(i) (b) Use of different formulas for calculating ratios


(ii) (c) Current ratio would decrease but would remain higher than 1
(iii) (d) None is correct
(iv) (a) Issuance of shares at discount
(b) Repayment of a bank loan
(v) (c) Both are change in accounting policy
(vi) (a) Rs. 660,000
(vii) (a) Only (I) is correct
(viii) (c) Cash dividend
(d) Transfer to general reserves
(ix) (b) Changing the measurement model for Investment property from cost model to
fair value model.
(c) Incorporating the effects of a material understatement found in last year closing
inventories due to incorrect formula in excel sheet.

Page 15 of 22
352
Answer # 7
Quicken Limited
Statement of cash flows
For the year ended 30 June 2022
Rs. in million
Cash flow from operating activities
Profit before tax 117
Adjustments for:
Depreciation expense (72 + 21) 93
Interest expense (48–14) 34
Gain on disposal of vehicle (11–8) (3)
Operating profit before working capital changes 241
Changes in working capital:
Decrease in inventory 79
Increase in trade and other receivables (33)
Increase in trade and other payables 6
Increase in advances from customers 34
86
Cash generated from operations 327
Interest paid (48)
Net cash flows from operating activities 279
Cash flow from investing activities
Purchase of Land and Building (130)
Purchase of Vehicles (51–12) (39)
Sale proceeds of vehicle (11+6) 17
Net cash used in investing activities (152)
Cash flow from financing activities
Repayment of loan (125)
Dividend paid (40)
Net cash flow from financing activities (165)
Net decrease in cash and cash equivalents (38)
Cash and cash equivalents at beginning of year 111
Cash and cash equivalents at end of year 73
Working:
Cash and cash equivalents 2022 2021
Cash and bank 73 111
73 111

Page 16 of 22
353
w-1: Retained earning A/c w-2: Revaluation surplus A/c
Dividend 40 b/d 325 Retained earnings 15 b/d -

(400 x 10%) Rev. Surplus 15 c/d 135 land 150

Profit (bal.) 117

Dividend 80 W-3 Inventory


(400 x 20%) Bal. 79
b/d 444
c/d 337 c/d 365

W-4 Vehicle W-6 Share capital


b/d 96 Disposal 8 b/d 400
vehicle 51 Dep(bal.) 21 Dividend 80
c/d 118 (400 x 20%)
c/d 480

W-5 Land & building W-7 long term loan A/c


b/d 526 Dep 72 Cash(bal.) 125 b/d 460

Revaluation 150

Cash(bal.) 130 c/d 335

Interest 14 c/d 748

W-8 Trade & other Receivable w-9: Trade & another Payable A/c
b/d 185 b/d 142
Bal. 33 Vehicle 12
Cash received 6 Bal. 6

c/d 212
c/d 160
w-10: Advance from customer A/c
b/d 35
Bal. 34

c/d 69

Page 17 of 22
354
Answer # 8
Peach Tree limited

Basic and diluted EPS for the year ended 30.06.2021

⇒ 210
Basic EPS = Rs. 3.60/share
58.33 (W-1)

⇒ 210+60 (W.2) 270


Diluted EPS = = Rs. 3.06/share
58.33+30 (W.2) 88.33

W.1)
Actual Current (weighted)
1-7-2020 b/d - -
1-7-2020 issued for value 50 50

50 50
1-2-2021 issue for value 20 8.33
(20 x 5/12)
70 58.33

W.2)
Preference shares remained outstanding for full year in 2021:
Increase in earnings [10 x 50 x 12%] 60
Increase in shares [10 x 3] 30
Incremental EPS [60/30] 2 per share
These are potentially dilutive since incremental EPS is less than basic EPS.

EPS for the year ended 30.06.2022:


2022 2021 (Restated)
Basic EPS 250/92.84 (W.3) 210/64.43(W.3)
2.69 per share 3.26 per share
Diluted EPS [250+20+60]/[92.84+14+30] [210+6]/[64.47+30]
2.41 per share (W.4) 2.86 per share

(W.3)
Actual Current (weighted) Prior (Adjusted)
1-7-2021 b/d 70 70 58.33
1-7-2020 issued for value 18.67 14 -
(18.67 x 9/12)
88.67 84 58.33
1-2-2021 issue for no value 9.33 8.84 6.14
(9.33/88.67 x 84) (9.33/8867 x 58.33)
98.00 92.84 64.47

70 x 40% = 28 x 20/30 = 18.67 [for value]

Page 18 of 22
355
28-1867 = 9.33 [for no value]

W.4)

Ranking in order of dilution Ranking:


Increase in
Convertible redeemable earnings/Increase in 60 (10 x 50 x 2.0
preference shares shares 12%)/30 (10 x 30) Dilutive 2
Increase in 20 (3 x 100 x 10% x
earnings/Increase in 8/12)/14 (3 x 7 x 1.43
Convertible bonds shares 8/12) Dilutive 1

Step 2 Testing whether dilutive or


not
Basic earnings 250 2.69
Basic number of shares 92.84
Adjust for:

1.
convertible bonds 250+20 2.52
92.84+14 Dilutive

2.
convertible bonds & 250+20+60 2.41
convertible preference shares 92.84+14+30
Dilutive

Page 19 of 22
356
Answer # 9
Gnu Cash Limited (GL)
Notes to the financial statements
For the year ended 30 June 2022
Property, plant and equipment Rs. In Million
Manufacturing plant Warehouse Showroom
Land Building
Gross carrying amount
Balance as on 1-7-2021 304.0 200.0 - -
Addition 120.0 180.0
Disposal - - - -
Elimination (20) (46.1) - -
Revaluation (23 + 8) 31 52.1 - -
Reclassification or transfer - (206.0) - -
to investment property
Balance as on 30-6-2022 315.0 - 120.0 180.0

Accumulated depreciation
Balance as on 1-7-2022 - 38 - -
Depreciation 20 8.1 - 9
Elimination (20) (46.1) - -
Disposal - - - -
Balance as on 30.06.2022 - - - 9
carrying amount 315.0 - 120.0 171.0
Measurement basis Revaluation Model Cost model Cost model Cost Model
Useful life/depreciation rate 13 years 10% - 15 years
Depreciation method Straight line Reducing - Straight line
balance

Revaluation related disclosures


1. The last revaluation of manufacturing plant was performed on 30 June 2022.
2. The revaluation was performed by an independent valuer.
3. The carrying amount of the plant at cost model as on 31 December 2022 would have been Rs. 307million.
W-1: Book value of plant under cost basis Rs. in million
Cost as on 1 January 2021 340
Depreciation for 2021 (340–60) ÷13 × (6÷12) (11)
Carrying amount as on 30-6-2021 329
Depreciation for 2022 (329–54) ÷12.5 (22)
Book value as on 30 June 2022 307

Movement in revaluation surplus:


Manufacturing plant Warehouse
Balance as on 1-7-2021 - -
Revaluation 8 52.1
Transfer of surplus - (52.1)
Balance as on 30-6-2022 8 46.89

Page 20 of 22
357
Investment property Warehouse Showroom
------ Rs. in million ------
Gross carrying amount - -
Additions (900×2/3) - 600.0
Transfer from property, plant and equipment 206.0 -
Fair value gain 8 54.0
(214–206) (3×2×9)
Balance as on 30-6-2022 214.0 654.0
Measurement basis Fair value Fair value

1. The rental income for the year ended 30 June 2022 is Rs. 26(10+4×4) million.
2. The revaluation was performed by an independent firm of valuers.

Working:
W-1 Journal entries:

Plant
1-1-2021
Plant 340
cash 340
30-6-2021
Dep 10.77
Acc. Dep 10.77
(340 -60) ÷13 x 6/12
Acc. Dep 10.77
Plant 10.77
Rev. loss (W-1.1) 25
Plant 25

Page 21 of 22
358
W-1.1

Carrying Amount (340 – 10.77) 329


Fair Value 304
Rev. loss 25
30-6-2022
Dep 20
(304 - 54) ÷ 12.5
Acc. dep 20
Acc. Dep 20
Plant 20
Plant 31
Reversal of loss 23
Rev. Surplus 8
W-2
C.A (304 – 20) 284
F. V 315
31

Reversal of loss 25 Cr. Rev. Surplus =


8 Cr.Extra dep (W-3) 2 Dr.

Reversal of loss(p/l) 23 Cr.


W-3:
Depreciation in 2022 if no revaluation
((340 - 10.77) – 54) ÷ 12.5 = 22
Less changed in 2022 = (20)
2
Warehouse
1-7-2019
Warehouse (PPE) 200
Cash 200
30-6-2020
Dep (200 x 10%) 20
Acc. Dep 20
30-06-2021
Dep (200 – 20) x 10% 18
Acc dep 18
31-12-2021
Dep (200 – 20 - 18) x 10% x 6/12 8.1
Acc dep 8.1

Page 22 of 22
359
1-1-2022
Change in use from IAS-16 to IAS-40
Acc. Dep (20 + 18 + 8.1) 46.1
Warehouse 46.1
Warehouse (W-3.1) 52.1
Surplus 52.1

(W-3.1)

WDV (200 – 46.1) 153.9


Fair Value 206
R. Surplus 52.1

30-6-2022
Investment property(W-3.2) 8
F.V Gain 8
R. Surplus (52.1 x 10%) 5.21
Retained earning 5.21
Investment property 206
Warehouse 206

W-3.2
Carrying Amount 206
Fair value 214
Fair Value Gain 8

Two Showroom (Investment Property)


1-10-2021
Investment property (900 ÷ 3 x 2) 600
Cash 600
30-6-2022
Investment property (3 x 2 x 9) 54
F.V gain 54

One Showroom (PPE)


1-10-2021
Land (900 ÷ 3 x 40%) 120
Building (900 ÷ 3 x 60%) 180
Cash 300
30-6-2022
Depreciation (180 ÷ 15 x 9/12) 9
Acc. Dep 9

Page 23 of 22
360

You might also like